Você está na página 1de 158

PBM EMPLOYEES ASSOCIATION VS.

rally cannot be cancelled because it has already been agreed upon in the
meeting. Pagcu explained further that the demonstration has nothing to do
PHILIPPINE BLOOMING MILLS with the Company because the union has no quarrel or dispute with
Management;

G.R. No. L-31195 June 5, 1973


6. That Management, thru Atty. C.S. de Leon, Company personnel manager,
informed PBMEO that the demonstration is an inalienable right of the union
PHILIPPINE BLOOMING MILLS EMPLOYMENT ORGANIZATION, NICANOR guaranteed by the Constitution but emphasized, however, that any
TOLENTINO, FLORENCIO, PADRIGANO RUFINO, ROXAS MARIANO DE LEON, demonstration for that matter should not unduly prejudice the normal
ASENCION PACIENTE, BONIFACIO VACUNA, BENJAMIN PAGCU and RODULFO operation of the Company. For which reason, the Company, thru Atty. C.S.
MUNSOD, petitioners, de Leon warned the PBMEO representatives that workers who belong to the
vs. first and regular shifts, who without previous leave of absence approved by
PHILIPPINE BLOOMING MILLS CO., INC. and COURT OF INDUSTRIAL the Company, particularly , the officers present who are the organizers of
RELATIONS, respondents. the demonstration, who shall fail to report for work the following morning
(March 4, 1969) shall be dismissed, because such failure is a violation of
the existing CBA and, therefore, would be amounting to an illegal strike;
MAKASIAR, J.:

7. That at about 5:00 P.M. on March 3, 1969, another meeting was


The petitioner Philippine Blooming Mills Employees Organization (hereinafter
convoked Company represented by Atty. C.S. de Leon, Jr. The Union panel
referred to as PBMEO) is a legitimate labor union composed of the employees of the
was composed of: Nicanor Tolentino, Rodolfo Munsod, Benjamin Pagcu and
respondent Philippine Blooming Mills Co., Inc., and petitioners Nicanor Tolentino,
Florencio Padrigano. In this afternoon meeting of March 3, 1969, Company
Florencio Padrigano, Rufino Roxas, Mariano de Leon, Asencion Paciente, Bonifacio
reiterated and appealed to the PBMEO representatives that while all
Vacuna, Benjamin Pagcu and Rodulfo Munsod are officers and members of the
workers may join the Malacaang demonstration, the workers for the first
petitioner Union.
and regular shift of March 4, 1969 should be excused from joining the
demonstration and should report for work; and thus utilize the workers in
Petitioners claim that on March 1, 1969, they decided to stage a mass the 2nd and 3rd shifts in order not to violate the provisions of the CBA,
demonstration at Malacaang on March 4, 1969, in protest against alleged abuses of particularly Article XXIV: NO LOCKOUT NO STRIKE'. All those who will not
the Pasig police, to be participated in by the workers in the first shift (from 6 A.M. to follow this warning of the Company shall be dismiss; De Leon reiterated the
2 P.M.) as well as those in the regular second and third shifts (from 7 A.M. to 4 P.M. Company's warning that the officers shall be primarily liable being the
and from 8 A.M. to 5 P.M., respectively); and that they informed the respondent organizers of the mass demonstration. The union panel countered that it
Company of their proposed demonstration. was rather too late to change their plans inasmuch as the Malacaang
demonstration will be held the following morning; and
The questioned order dated September 15, 1969, of Associate Judge Joaquin M.
Salvador of the respondent Court reproduced the following stipulation of facts of the 8. That a certain Mr. Wilfredo Ariston, adviser of PBMEO sent a cablegram to
parties parties the Company which was received 9:50 A.M., March 4, 1969, the contents of
which are as follows: 'REITERATING REQUEST EXCUSE DAY SHIFT
EMPLOYEES JOINING DEMONSTRATION MARCH 4, 1969.' (Pars. 3-8, Annex
3. That on March 2, 1969 complainant company learned of the projected
"F", pp. 42-43, rec.)
mass demonstration at Malacaang in protest against alleged abuses of the
Pasig Police Department to be participated by the first shift (6:00 AM-2:00
PM) workers as well as those working in the regular shifts (7:00 A.M. to 4:00 Because the petitioners and their members numbering about 400 proceeded with
PM and 8:00 AM to 5:00 PM) in the morning of March 4, 1969; the demonstration despite the pleas of the respondent Company that the first shift
workers should not be required to participate in the demonstration and that the
workers in the second and third shifts should be utilized for the demonstration from
4. That a meeting was called by the Company on March 3, 1969 at about
6 A.M. to 2 P.M. on March 4, 1969, respondent Company prior notice of the mass
11:00 A.M. at the Company's canteen, and those present were: for the
demonstration on March 4, 1969, with the respondent Court, a charge against
Company: (1) Mr. Arthur L. Ang (2) Atty. S. de Leon, Jr., (3) and all
petitioners and other employees who composed the first shift, charging them with a
department and section heads. For the PBMEO (1) Florencio Padrigano, (2)
"violation of Section 4(a)-6 in relation to Sections 13 and 14, as well as Section 15,
Rufino Roxas, (3) Mariano de Leon, (4) Asencion Paciente, (5) Bonifacio
all of Republic Act No. 875, and of the CBA providing for 'No Strike and No Lockout.' "
Vacuna and (6) Benjamin Pagcu.
(Annex "A", pp. 19-20, rec.). The charge was accompanied by the joint affidavit of
Arthur L. Ang and Cesareo de Leon, Jr. (Annex "B", pp. 21-24, rec.). Thereafter, a
5. That the Company asked the union panel to confirm or deny said corresponding complaint was filed, dated April 18, 1969, by Acting Chief Prosecutor
projected mass demonstration at Malacaang on March 4, 1969. PBMEO Antonio T. Tirona and Acting Prosecutor Linda P. Ilagan (Annex "C", pp. 25-30, rec.)
thru Benjamin Pagcu who acted as spokesman of the union panel,
confirmed the planned demonstration and stated that the demonstration or

1
In their answer, dated May 9, 1969, herein petitioners claim that they did not violate On October 31, 1969, herein petitioners filed with the respondent court a petition for
the existing CBA because they gave the respondent Company prior notice of the relief from the order dated October 9, 1969, on the ground that their failure to file
mass demonstration on March 4, 1969; that the said mass demonstration was a their motion for reconsideration on time was due to excusable negligence and
valid exercise of their constitutional freedom of speech against the alleged abuses of honest mistake committed by the president of the petitioner Union and of the office
some Pasig policemen; and that their mass demonstration was not a declaration of clerk of their counsel, attaching thereto the affidavits of the said president and clerk
strike because it was not directed against the respondent firm (Annex "D", pp. 31- (Annexes "K", "K-1" and "K-2", rec.).
34, rec.)
Without waiting for any resolution on their petition for relief from the order dated
After considering the aforementioned stipulation of facts submitted by the parties, October 9, 1969, herein petitioners filed on November 3, 1969, with the Supreme
Judge Joaquin M. Salvador, in an order dated September 15, 1969, found herein Court, a notice of appeal (Annex "L", pp. 88-89, rec.).
petitioner PBMEO guilty of bargaining in bad faith and herein petitioners Florencio
Padrigano, Rufino Roxas, Mariano de Leon, Asencion Paciente, Bonifacio Vacuna,
I
Benjamin Pagcu, Nicanor Tolentino and Rodulfo Munsod as directly responsible for
perpetrating the said unfair labor practice and were, as a consequence, considered
to have lost their status as employees of the respondent Company (Annex "F", pp. There is need of briefly restating basic concepts and principles which underlie the
42-56, rec.) issues posed by the case at bar.

Herein petitioners claim that they received on September 23, 1969, the aforesaid (1) In a democracy, the preservation and enhancement of the dignity and worth of
order (p. 11, rec.); and that they filed on September 29, 1969, because September the human personality is the central core as well as the cardinal article of faith of our
28, 1969 fell on Sunday (p. 59, rec.), a motion for reconsideration of said order dated civilization. The inviolable character of man as an individual must be "protected to
September 15, 1969, on the ground that it is contrary to law and the evidence, as the largest possible extent in his thoughts and in his beliefs as the citadel of his
well as asked for ten (10) days within which to file their arguments pursuant to person." 2
Sections 15, 16 and 17 of the Rules of the CIR, as amended (Annex "G", pp. 57-60,
rec. )
(2) The Bill of Rights is designed to preserve the ideals of liberty, equality and
security "against the assaults of opportunism, the expediency of the passing hour,
In its opposition dated October 7, 1969, filed on October 11, 1969 (p. 63, rec.), the erosion of small encroachments, and the scorn and derision of those who have
respondent Company averred that herein petitioners received on September 22, no patience with general principles." 3
1969, the order dated September 17 (should be September 15), 1969; that under
Section 15 of the amended Rules of the Court of Industrial Relations, herein
In the pithy language of Mr. Justice Robert Jackson, the purpose of the Bill of Rights is
petitioners had five (5) days from September 22, 1969 or until September 27, 1969,
to withdraw "certain subjects from the vicissitudes of political controversy, to place
within which to file their motion for reconsideration; and that because their motion
them beyond the reach of majorities and officials, and toestablish them as legal
for reconsideration was two (2) days late, it should be accordingly dismissed,
principles to be applied by the courts. One's rights to life, liberty and property, to
invoking Bien vs. Castillo, 1 which held among others, that a motion for extension of
free speech, or free press, freedom of worship and assembly, and other fundamental
the five-day period for the filing of a motion for reconsideration should be filed
rights may not be submitted to a vote; they depend on the outcome of no
before the said five-day period elapses (Annex "M", pp. 61-64, rec.).
elections." 4 Laski proclaimed that "the happiness of the individual, not the well-
being of the State, was the criterion by which its behaviour was to be judged. His
Subsequently, herein petitioners filed on October 14, 1969 their written arguments interests, not its power, set the limits to the authority it was entitled to exercise." 5
dated October 11, 1969, in support of their motion for reconsideration (Annex "I",
pp. 65-73, rec.).
(3) The freedoms of expression and of assembly as well as the right to petition are
included among the immunities reserved by the sovereign people, in the rhetorical
In a resolution dated October 9, 1969, the respondent en banc dismissed the motion aphorism of Justice Holmes, to protect the ideas that we abhor or hate more than
for reconsideration of herein petitioners for being pro forma as it was filed beyond the ideas we cherish; or as Socrates insinuated, not only to protect the minority who
the reglementary period prescribed by its Rules (Annex "J", pp. 74-75, rec.), which want to talk, but also to benefit the majority who refuse to listen. 6 And as Justice
herein petitioners received on October 28, 196 (pp. 12 & 76, rec.). Douglas cogently stresses it, the liberties of one are the liberties of all; and the
liberties of one are not safe unless the liberties of all are protected. 7
At the bottom of the notice of the order dated October 9, 1969, which was released
on October 24, 1969 and addressed to the counsels of the parties (pp. 75-76, rec.), (4) The rights of free expression, free assembly and petition, are not only civil rights
appear the requirements of Sections 15, 16 and 17, as amended, of the Rules of the but also political rights essential to man's enjoyment of his life, to his happiness and
Court of Industrial Relations, that a motion for reconsideration shall be filed within to his full and complete fulfillment. Thru these freedoms the citizens can participate
five (5) days from receipt of its decision or order and that an appeal from the not merely in the periodic establishment of the government through their suffrage
decision, resolution or order of the C.I.R., sitting en banc, shall be perfected within but also in the administration of public affairs as well as in the discipline of abusive
ten (10) days from receipt thereof (p. 76, rec.). public officers. The citizen is accorded these rights so that he can appeal to the
appropriate governmental officers or agencies for redress and protection as well as
for the imposition of the lawful sanctions on erring public officers and employees.

2
(5) While the Bill of Rights also protects property rights, the primacy of human rights harassment of local police officers. It was to the interest herein private respondent
over property rights is recognized. 8 Because these freedoms are "delicate and firm to rally to the defense of, and take up the cudgels for, its employees, so that
vulnerable, as well as supremely precious in our society" and the "threat of they can report to work free from harassment, vexation or peril and as consequence
sanctions may deter their exercise almost as potently as the actual application of perform more efficiently their respective tasks enhance its productivity as well as
sanctions," they "need breathing space to survive," permitting government profits. Herein respondent employer did not even offer to intercede for its employees
regulation only "with narrow specificity." 9 with the local police. Was it securing peace for itself at the expenses of its workers?
Was it also intimidated by the local police or did it encourage the local police to
terrorize or vex its workers? Its failure to defend its own employees all the more
Property and property rights can be lost thru prescription; but human rights are
weakened the position of its laborers the alleged oppressive police who might have
imprescriptible. If human rights are extinguished by the passage of time, then the
been all the more emboldened thereby subject its lowly employees to further
Bill of Rights is a useless attempt to limit the power of government and ceases to be
indignities.
an efficacious shield against the tyranny of officials, of majorities, of the influential
and powerful, and of oligarchs political, economic or otherwise.
In seeking sanctuary behind their freedom of expression well as their right of
assembly and of petition against alleged persecution of local officialdom, the
In the hierarchy of civil liberties, the rights of free expression and of assembly
employees and laborers of herein private respondent firm were fighting for their very
occupy a preferred position as they are essential to the preservation and vitality of
survival, utilizing only the weapons afforded them by the Constitution the
our civil and political institutions; 10 and such priority "gives these liberties the
untrammelled enjoyment of their basic human rights. The pretension of their
sanctity and the sanction not permitting dubious intrusions." 11
employer that it would suffer loss or damage by reason of the absence of its
employees from 6 o'clock in the morning to 2 o'clock in the afternoon, is a plea for
The superiority of these freedoms over property rights is underscored by the fact the preservation merely of their property rights. Such apprehended loss or damage
that a mere reasonable or rational relation between the means employed by the law would not spell the difference between the life and death of the firm or its owners or
and its object or purpose that the law is neither arbitrary nor discriminatory nor its management. The employees' pathetic situation was a stark reality abused,
oppressive would suffice to validate a law which restricts or impairs property harassment and persecuted as they believed they were by the peace officers of the
rights. 12 On the other hand, a constitutional or valid infringement of human rights municipality. As above intimated, the condition in which the employees found
requires a more stringent criterion, namely existence of a grave and immediate themselves vis-a-vis the local police of Pasig, was a matter that vitally affected their
danger of a substantive evil which the State has the right to prevent. So it has been right to individual existence as well as that of their families. Material loss can be
stressed in the main opinion of Mr. Justice Fernando in Gonzales vs. Comelec and repaired or adequately compensated. The debasement of the human being broken in
reiterated by the writer of the opinion in Imbong vs. Ferrer. 13 It should be added morale and brutalized in spirit-can never be fully evaluated in monetary terms. The
that Mr. Justice Barredo in Gonzales vs. Comelec, supra, like Justices Douglas, Black wounds fester and the scars remain to humiliate him to his dying day, even as he
and Goldberg in N.Y. Times Co. vs. Sullivan, 14 believes that the freedoms of speech cries in anguish for retribution, denial of which is like rubbing salt on bruised tissues.
and of the press as well as of peaceful assembly and of petition for redress of
grievances are absolute when directed against public officials or "when exercised in
As heretofore stated, the primacy of human rights freedom of expression, of
relation to our right to choose the men and women by whom we shall be
peaceful assembly and of petition for redress of grievances over property rights
governed," 15 even as Mr. Justice Castro relies on the balancing-of-interests
has been sustained. 18 Emphatic reiteration of this basic tenet as a coveted boon
test. 16 Chief Justice Vinson is partial to the improbable danger rule formulated by
at once the shield and armor of the dignity and worth of the human personality, the
Chief Judge Learned Hand, viz. whether the gravity of the evil, discounted by its
all-consuming ideal of our enlightened civilization becomes Our duty, if freedom
improbability, justifies such invasion of free expression as is necessary to avoid the
and social justice have any meaning at all for him who toils so that capital can
danger. 17
produce economic goods that can generate happiness for all. To regard the
demonstration against police officers, not against the employer, as evidence of bad
II faith in collective bargaining and hence a violation of the collective bargaining
agreement and a cause for the dismissal from employment of the demonstrating
employees, stretches unduly the compass of the collective bargaining agreement, is
The respondent Court of Industrial Relations, after opining that the mass
"a potent means of inhibiting speech" and therefore inflicts a moral as well as mortal
demonstration was not a declaration of strike, concluded that by their "concerted act
wound on the constitutional guarantees of free expression, of peaceful assembly and
and the occurrence temporary stoppage of work," herein petitioners are guilty
of petition. 19
bargaining in bad faith and hence violated the collective bargaining agreement with
private respondent Philippine Blooming Mills Co., inc.. Set against and tested by
foregoing principles governing a democratic society, such conclusion cannot be The collective bargaining agreement which fixes the working shifts of the
sustained. The demonstration held petitioners on March 4, 1969 before Malacaang employees, according to the respondent Court Industrial Relations, in effect imposes
was against alleged abuses of some Pasig policemen, not against their employer, on the workers the "duty ... to observe regular working hours." The strain
herein private respondent firm, said demonstrate was purely and completely an construction of the Court of Industrial Relations that a stipulated working shifts deny
exercise of their freedom expression in general and of their right of assembly and the workers the right to stage mass demonstration against police abuses during
petition for redress of grievances in particular before appropriate governmental working hours, constitutes a virtual tyranny over the mind and life the workers and
agency, the Chief Executive, again the police officers of the municipality of Pasig. deserves severe condemnation. Renunciation of the freedom should not be
They exercise their civil and political rights for their mutual aid protection from what predicated on such a slender ground.
they believe were police excesses. As matter of fact, it was the duty of herein
private respondent firm to protect herein petitioner Union and its members fro the

3
The mass demonstration staged by the employees on March 4, 1969 could not have shield themselves against such alleged police indignities. The insistence on the part
been legally enjoined by any court, such an injunction would be trenching upon the of the respondent firm that the workers for the morning and regular shift should not
freedom expression of the workers, even if it legally appears to be illegal picketing or participate in the mass demonstration, under pain of dismissal, was as heretofore
strike. 20 The respondent Court of Industrial Relations in the case at bar concedes stated, "a potent means of inhibiting speech." 22
that the mass demonstration was not a declaration of a strike "as the same not
rooted in any industrial dispute although there is concerted act and the occurrence
Such a concerted action for their mutual help and protection deserves at least equal
of a temporary stoppage work." (Annex "F", p. 45, rec.).
protection as the concerted action of employees in giving publicity to a letter
complaint charging bank president with immorality, nepotism, favoritism an
The respondent firm claims that there was no need for all its employees to discrimination in the appointment and promotion of ban employees. 23 We further
participate in the demonstration and that they suggested to the Union that only the ruled in the Republic Savings Bank case, supra, that for the employees to come
first and regular shift from 6 A.M. to 2 P.M. should report for work in order that loss or within the protective mantle of Section 3 in relation to Section 4(a-1) on Republic Act
damage to the firm will be averted. This stand failed appreciate the sine qua non of No. 875, "it is not necessary that union activity be involved or that collective
an effective demonstration especially by a labor union, namely the complete unity of bargaining be contemplated," as long as the concerted activity is for the furtherance
the Union members as well as their total presence at the demonstration site in order of their interests. 24
to generate the maximum sympathy for the validity of their cause but also
immediately action on the part of the corresponding government agencies with
As stated clearly in the stipulation of facts embodied in the questioned order of
jurisdiction over the issues they raised against the local police. Circulation is one of
respondent Court dated September 15, 1969, the company, "while expressly
the aspects of freedom of expression. 21 If demonstrators are reduced by one-third,
acknowledging, that the demonstration is an inalienable right of the Union
then by that much the circulation of the issues raised by the demonstration is
guaranteed by the Constitution," nonetheless emphasized that "any demonstration
diminished. The more the participants, the more persons can be apprised of the
for that matter should not unduly prejudice the normal operation of the company"
purpose of the rally. Moreover, the absence of one-third of their members will be
and "warned the PBMEO representatives that workers who belong to the first and
regarded as a substantial indication of disunity in their ranks which will enervate
regular shifts, who without previous leave of absence approved by the Company,
their position and abet continued alleged police persecution. At any rate, the Union
particularly the officers present who are the organizers of the demonstration, who
notified the company two days in advance of their projected demonstration and the
shall fail to report for work the following morning (March 4, 1969) shall be dismissed,
company could have made arrangements to counteract or prevent whatever losses
because such failure is a violation of the existing CBA and, therefore, would be
it might sustain by reason of the absence of its workers for one day, especially in
amounting to an illegal strike (;)" (p. III, petitioner's brief). Such threat of dismissal
this case when the Union requested it to excuse only the day-shift employees who
tended to coerce the employees from joining the mass demonstration. However, the
will join the demonstration on March 4, 1969 which request the Union reiterated in
issues that the employees raised against the local police, were more important to
their telegram received by the company at 9:50 in the morning of March 4, 1969,
them because they had the courage to proceed with the demonstration, despite
the day of the mass demonstration (pp. 42-43, rec.). There was a lack of human
such threat of dismissal. The most that could happen to them was to lose a day's
understanding or compassion on the part of the firm in rejecting the request of the
wage by reason of their absence from work on the day of the demonstration. One
Union for excuse from work for the day shifts in order to carry out its mass
day's pay means much to a laborer, more especially if he has a family to support.
demonstration. And to regard as a ground for dismissal the mass demonstration held
Yet, they were willing to forego their one-day salary hoping that their demonstration
against the Pasig police, not against the company, is gross vindictiveness on the part
would bring about the desired relief from police abuses. But management was
of the employer, which is as unchristian as it is unconstitutional.
adamant in refusing to recognize the superior legitimacy of their right of free
speech, free assembly and the right to petition for redress.
III
Because the respondent company ostensibly did not find it necessary to demand
The respondent company is the one guilty of unfair labor practice. Because the from the workers proof of the truth of the alleged abuses inflicted on them by the
refusal on the part of the respondent firm to permit all its employees and workers to local police, it thereby concedes that the evidence of such abuses should properly be
join the mass demonstration against alleged police abuses and the subsequent submitted to the corresponding authorities having jurisdiction over their complaint
separation of the eight (8) petitioners from the service constituted an and to whom such complaint may be referred by the President of the Philippines for
unconstitutional restraint on the freedom of expression, freedom of assembly and proper investigation and action with a view to disciplining the local police officers
freedom petition for redress of grievances, the respondent firm committed an unfair involved.
labor practice defined in Section 4(a-1) in relation to Section 3 of Republic Act No.
875, otherwise known as the Industrial Peace Act. Section 3 of Republic Act No. 8
On the other hand, while the respondent Court of Industrial Relations found that the
guarantees to the employees the right "to engage in concert activities for ... mutual
demonstration "paralyzed to a large extent the operations of the complainant
aid or protection"; while Section 4(a-1) regards as an unfair labor practice for an
company," the respondent Court of Industrial Relations did not make any finding as
employer interfere with, restrain or coerce employees in the exercise their rights
to the fact of loss actually sustained by the firm. This significant circumstance can
guaranteed in Section Three."
only mean that the firm did not sustain any loss or damage. It did not present
evidence as to whether it lost expected profits for failure to comply with purchase
We repeat that the obvious purpose of the mass demonstration staged by the orders on that day; or that penalties were exacted from it by customers whose
workers of the respondent firm on March 4, 1969, was for their mutual aid and orders could not be filled that day of the demonstration; or that purchase orders
protection against alleged police abuses, denial of which was interference with or were cancelled by the customers by reason of its failure to deliver the materials
restraint on the right of the employees to engage in such common action to better ordered; or that its own equipment or materials or products were damaged due to

4
absence of its workers on March 4, 1969. On the contrary, the company saved a is not exhausted by the delivery of one speech, the printing of one article or the
sizable amount in the form of wages for its hundreds of workers, cost of fuel, water staging of one demonstration. It is a continuing immunity to be invoked and
and electric consumption that day. Such savings could have amply compensated for exercised when exigent and expedient whenever there are errors to be rectified,
unrealized profits or damages it might have sustained by reason of the absence of abuses to be denounced, inhumanities to be condemned. Otherwise these
its workers for only one day. guarantees in the Bill of Rights would be vitiated by rule on procedure prescribing
the period for appeal. The battle then would be reduced to a race for time. And in
such a contest between an employer and its laborer, the latter eventually loses
IV
because he cannot employ the best an dedicated counsel who can defend his
interest with the required diligence and zeal, bereft as he is of the financial
Apart from violating the constitutional guarantees of free speech and assembly as resources with which to pay for competent legal services. 28-a
well as the right to petition for redress of grievances of the employees, the dismissal
of the eight (8) leaders of the workers for proceeding with the demonstration and
VI
consequently being absent from work, constitutes a denial of social justice likewise
assured by the fundamental law to these lowly employees. Section 5 of Article II of
the Constitution imposes upon the State "the promotion of social justice to insure The Court of Industrial Relations rule prescribes that motion for reconsideration of its
the well-being and economic security of all of the people," which guarantee is order or writ should filed within five (5) days from notice thereof and that the
emphasized by the other directive in Section 6 of Article XIV of the Constitution that arguments in support of said motion shall be filed within ten (10) days from the date
"the State shall afford protection to labor ...". Respondent Court of Industrial of filing of such motion for reconsideration (Sec. 16). As above intimated, these rules
Relations as an agency of the State is under obligation at all times to give meaning of procedure were promulgated by the Court of Industrial Relations pursuant to a
and substance to these constitutional guarantees in favor of the working man; for legislative delegation. 29
otherwise these constitutional safeguards would be merely a lot of "meaningless
constitutional patter." Under the Industrial Peace Act, the Court of Industrial
The motion for reconsideration was filed on September 29, 1969, or seven (7) days
Relations is enjoined to effect the policy of the law "to eliminate the causes of
from notice on September 22, 1969 of the order dated September 15, 1969 or two
industrial unrest by encouraging and protecting the exercise by employees of their
(2) days late. Petitioners claim that they could have filed it on September 28, 1969,
right to self-organization for the purpose of collective bargaining and for the
but it was a Sunday.
promotion of their moral, social and economic well-being." It is most unfortunate in
the case at bar that respondent Court of Industrial Relations, the very governmental
agency designed therefor, failed to implement this policy and failed to keep faith Does the mere fact that the motion for reconsideration was filed two (2) days late
with its avowed mission its raison d'etre as ordained and directed by the defeat the rights of the petitioning employees? Or more directly and concretely,
Constitution. does the inadvertent omission to comply with a mere Court of Industrial Relations
procedural rule governing the period for filing a motion for reconsideration or appeal
in labor cases, promulgated pursuant to a legislative delegation, prevail over
V
constitutional rights? The answer should be obvious in the light of the aforecited
cases. To accord supremacy to the foregoing rules of the Court of Industrial Relations
It has been likewise established that a violation of a constitutional right divests the over basic human rights sheltered by the Constitution, is not only incompatible with
court of jurisdiction; and as a consequence its judgment is null and void and confers the basic tenet of constitutional government that the Constitution is superior to any
no rights. Relief from a criminal conviction secured at the sacrifice of constitutional statute or subordinate rules and regulations, but also does violence to natural
liberties, may be obtained through habeas corpus proceedings even long after the reason and logic. The dominance and superiority of the constitutional right over the
finality of the judgment. Thus, habeas corpus is the remedy to obtain the release of aforesaid Court of Industrial Relations procedural rule of necessity should be
an individual, who is convicted by final judgment through a forced confession, which affirmed. Such a Court of Industrial Relations rule as applied in this case does not
violated his constitutional right against self-incrimination; 25 or who is denied the implement or reinforce or strengthen the constitutional rights affected,' but instead
right to present evidence in his defense as a deprivation of his liberty without due constrict the same to the point of nullifying the enjoyment thereof by the petitioning
process of law, 26 even after the accused has already served sentence for twenty- employees. Said Court of Industrial Relations rule, promulgated as it was pursuant to
two years. 27 a mere legislative delegation, is unreasonable and therefore is beyond the authority
granted by the Constitution and the law. A period of five (5) days within which to file
a motion for reconsideration is too short, especially for the aggrieved workers, who
Both the respondents Court of Industrial Relations and private firm trenched upon
usually do not have the ready funds to meet the necessary expenses therefor. In
these constitutional immunities of petitioners. Both failed to accord preference to
case of the Court of Appeals and the Supreme Court, a period of fifteen (15) days
such rights and aggravated the inhumanity to which the aggrieved workers claimed
has been fixed for the filing of the motion for re hearing or reconsideration (See. 10,
they had been subjected by the municipal police. Having violated these basic human
Rule 51; Sec. 1, Rule 52; Sec. 1, Rule 56, Revised Rules of Court). The delay in the
rights of the laborers, the Court of Industrial Relations ousted itself of jurisdiction
filing of the motion for reconsideration could have been only one day if September
and the questioned orders it issued in the instant case are a nullity. Recognition and
28, 1969 was not a Sunday. This fact accentuates the unreasonableness of the Court
protection of such freedoms are imperative on all public offices including the
of Industrial are concerned.
courts 28 as well as private citizens and corporations, the exercise and enjoyment of
which must not be nullified by mere procedural rule promulgated by the Court
Industrial Relations exercising a purely delegate legislative power, when even a law It should be stressed here that the motion for reconsideration dated September 27,
enacted by Congress must yield to the untrammelled enjoyment of these human 1969, is based on the ground that the order sought to be reconsidered "is not in
rights. There is no time limit to the exercise of the freedoms. The right to enjoy them accordance with law, evidence and facts adduced during the hearing," and likewise

5
prays for an extension of ten (10) days within which to file arguments pursuant to are short of being jurisdiction nullities or excesses, this Court would still be
Sections 15, 16 and 17 of the Rules of the Court of Industrial Relations (Annex "G", on firm legal grounds should it choose to reverse said decision here and
pp. 57-60, rec.); although the arguments were actually filed by the herein petitioners now even if such errors can be considered as mere mistakes of judgment or
on October 14, 1969 (Annex "I", pp. 70-73, rec.), long after the 10-day period only as faults in the exercise of jurisdiction, so as to avoid the unnecessary
required for the filing of such supporting arguments counted from the filing of the return of this case to the lower court for the sole purpose of pursuing the
motion for reconsideration. Herein petitioners received only on October 28, 1969 the ordinary course of an appeal. (Emphasis supplied). 30-d
resolution dated October 9, 1969 dismissing the motion for reconsideration for
being pro forma since it was filed beyond the reglementary period (Annex "J", pp.
Insistence on the application of the questioned Court industrial Relations rule in this
74-75, rec.)
particular case at bar would an unreasoning adherence to "Procedural niceties"
which denies justice to the herein laborers, whose basic human freedoms, including
It is true that We ruled in several cases that where a motion to reconsider is filed out the right to survive, must be according supremacy over the property rights of their
of time, or where the arguments in suppf such motion are filed beyond the 10 day employer firm which has been given a full hearing on this case, especially when, as
reglementary period provided for by the Court of Industrial Relations rules, the order in the case at bar, no actual material damage has be demonstrated as having been
or decision subject of 29-a reconsideration becomes final and unappealable. But in all inflicted on its property rights.
these cases, the constitutional rights of free expression, free assembly and petition
were not involved.
If We can disregard our own rules when justice requires it, obedience to the
Constitution renders more imperative the suspension of a Court of Industrial
It is a procedural rule that generally all causes of action and defenses presently Relations rule that clash with the human rights sanctioned and shielded with
available must be specifically raised in the complaint or answer; so that any cause of resolution concern by the specific guarantees outlined in the organic law. It should
action or defense not raised in such pleadings, is deemed waived. However, a be stressed that the application in the instant case Section 15 of the Court of
constitutional issue can be raised any time, even for the first time on appeal, if it Industrial Relations rules relied upon by herein respondent firm is unreasonable and
appears that the determination of the constitutional issue is necessary to a decision therefore such application becomes unconstitutional as it subverts the human rights
of the case, the very lis mota of the case without the resolution of which no final and of petitioning labor union and workers in the light of the peculiar facts and
complete determination of the dispute can be made. 30 It is thus seen that a circumstances revealed by the record.
procedural rule of Congress or of the Supreme Court gives way to a constitutional
right. In the instant case, the procedural rule of the Court of Industrial Relations, a
The suspension of the application of Section 15 of the Court of Industrial Relations
creature of Congress, must likewise yield to the constitutional rights invoked by
rules with reference to the case at is also authorized by Section 20 of
herein petitioners even before the institution of the unfair labor practice charged
Commonwealth Act No. 103, the C.I.R. charter, which enjoins the Court of Industrial
against them and in their defense to the said charge.
Relations to "act according to justice and equity and substantial merits of the case,
without regard to technicalities or legal forms ..."
In the case at bar, enforcement of the basic human freedoms sheltered no less by
the organic law, is a most compelling reason to deny application of a Court of
On several occasions, We emphasized this doctrine which was re-stated by Mr.
Industrial Relations rule which impinges on such human rights.30-a
Justice Barredo, speaking for the Court, in the 1970 case of Kapisanan, etc. vs.
Hamilton, etc., et. al., 30-e thus:
It is an accepted principle that the Supreme Court has the inherent power to
"suspend its own rules or to except a particular case from its operation, whenever
As to the point that the evidence being offered by the petitioners in the
the purposes of justice require." 30-b Mr. Justice Barredo in his concurring opinion
motion for new trial is not "newly discovered," as such term is understood
in Estrada vs. Sto. Domingo. 30-c reiterated this principle and added that
in the rules of procedure for the ordinary courts, We hold that such criterion
is not binding upon the Court of Industrial Relations. Under Section 20 of
Under this authority, this Court is enabled to cove with all situations without Commonwealth Act No. 103, 'The Court of Industrial Relations shall adopt
concerning itself about procedural niceties that do not square with the its, rules or procedure and shall have such other powers as generally
need to do justice, in any case, without further loss of time, provided that pertain to a court of justice: Provided, however, That in the hearing,
the right of the parties to a full day in court is not substantially impaired. investigation and determination of any question or controversy and in
Thus, this Court may treat an appeal as a certiorari and vice-versa. In other exercising any duties and power under this Act, the Court shall act
words, when all the material facts are spread in the records before Us, and according to justice and equity and substantial merits of the case, without
all the parties have been duly heard, it matters little that the error of the regard to technicalities or legal forms and shall not be bound by any
court a quo is of judgment or of jurisdiction. We can then and there render technical rules of legal evidence but may inform its mind in such manner as
the appropriate judgment. Is within the contemplation of this doctrine that it may deem just and equitable.' By this provision the industrial court is
as it is perfectly legal and within the power of this Court to strike down in disengaged from the rigidity of the technicalities applicable to ordinary
an appeal acts without or in excess of jurisdiction or committed with grave courts. Said court is not even restricted to the specific relief demanded by
abuse of discretion, it cannot be beyond the admit of its authority, in the parties but may issue such orders as may be deemed necessary or
appropriate cases, to reverse in a certain proceed in any error of judgment expedient for the purpose of settling the dispute or dispelling any doubts
of a court a quo which cannot be exactly categorized as a flaw of that may give rise to future disputes. (Ang Tibay v. C.I.R., G.R. No. 46496,
jurisdiction. If there can be any doubt, which I do not entertain, on whether Feb. 17, 1940; Manila Trading & Supply Co. v. Phil. Labor, 71 Phil. 124.) For
or not the errors this Court has found in the decision of the Court of Appeals these reasons, We believe that this provision is ample enough to have

6
enabled the respondent court to consider whether or not its previous ruling them by the firm (pp. 16-20, respondent's Brief; Annexes "A", "B" and "C", pp. 20-30,
that petitioners constitute a minority was founded on fact, without regard to rec.). Counsel for respondent firm insinuates that not all the 400 or so employee
the technical meaning of newly discovered evidence. ... (Alonso v. Villamor, participated in the demonstration, for which reason only the Union and its thirteen
16 Phil. 315; Chua Kiong v. Whitaker, 46 Phil. 578). (emphasis supplied.) (13) officers were specifically named in the unfair labor practice charge (p. 20,
respondent's brief). If that were so, then many, if not all, of the morning and regular
shifts reported for work on March 4, 1969 and that, as a consequence, the firm
To apply Section 15 of the Court of Industrial Relations rules with "pedantic rigor" in
continued in operation that day and did not sustain any damage.
the instant case is to rule in effect that the poor workers, who can ill-afford an alert
competent lawyer, can no longer seek the sanctuary of human freedoms secured to
them by the fundamental law, simply because their counsel erroneously believing The appropriate penalty if it deserves any penalty at all should have been
that he received a copy of the decision on September 23, 1969, instead of simply to charge said one-day absence against their vacation or sick leave. But to
September 22, 1969 - filed his motion for reconsideration September 29, 1969, dismiss the eight (8) leaders of the petitioner Union is a most cruel penalty, since as
which practically is only one day late considering that September 28, 1969 was a aforestated the Union leaders depend on their wages for their daily sustenance as
Sunday. well as that of their respective families aside from the fact that it is a lethal blow to
unionism, while at the same time strengthening the oppressive hand of the petty
tyrants in the localities.
Many a time, this Court deviated from procedure technicalities when they ceased to
be instruments of justice, for the attainment of which such rules have been devised.
Summarizing the jurisprudence on this score, Mr. Justice Fernando, speaking for a Mr. Justice Douglas articulated this pointed reminder:
unanimous Court in Palma vs. Oreta, 30-f Stated:
The challenge to our liberties comes frequently not from those who
As was so aptly expressed by Justice Moreland in Alonso v. Villamor (16 Phil. consciously seek to destroy our system of Government, but from men of
315 [1910]. The Villamor decision was cited with approval in Register of goodwill good men who allow their proper concerns to blind them to the
Deeds v. Phil. Nat. Bank, 84 Phil. 600 [1949]; Potenciano v. Court of fact that what they propose to accomplish involves an impairment of liberty.
Appeals, 104 Phil. 156 [1958] and Uy v. Uy, 14243, June 30, 1961, 2 SCRA
675.), decided as far back as 1910, "technicality. when it deserts its proper-
... The Motives of these men are often commendable. What we must
office as an aid to justice and becomes its great hindrance and chief
remember, however, is thatpreservation of liberties does not depend on
enemy, deserves scant consideration from courts." (Ibid., p, 322.) To that
motives. A suppression of liberty has the same effect whether the suppress
norm, this Court has remained committed. The late Justice Recto in Blanco
or be a reformer or an outlaw. The only protection against misguided zeal is
v. Bernabe, (63 Phil. 124 [1936]) was of a similar mind. For him the
a constant alertness of the infractions of the guarantees of
interpretation of procedural rule should never "sacrifice the ends justice."
liberty contained in our Constitution. Each surrender of liberty to the
While "procedural laws are no other than technicalities" view them in their
demands of the moment makes easier another, larger surrender. The battle
entirety, 'they were adopted not as ends themselves for the compliance
over the Bill of Rights is a never ending one.
with which courts have organized and function, but as means conducive to
the realization the administration of the law and of justice (Ibid., p.,128).
We have remained steadfastly opposed, in the highly rhetorical language ... The liberties of any person are the liberties of all of us.
Justice Felix, to "a sacrifice of substantial rights of a litigant in altar of
sophisticated technicalities with impairment of the sacred principles of
... In short, the Liberties of none are safe unless the liberties of all are
justice." (Potenciano v. Court of Appeals, 104 Phil. 156, 161 [1958]). As
protected.
succinctly put by Justice Makalintal, they "should give way to the realities of
the situation." (Urbayan v. Caltex, L-15379, Aug. 31, 1962, 5 SCRA 1016,
1019). In the latest decision in point promulgated in 1968, (Udan v. Amon, ... But even if we should sense no danger to our own liberties, even if we
(1968, 23 SCRA citing McEntee v. Manotok, L-14968, Oct. 27, 1961, 3 SCRA feel secure because we belong to a group that is important and respected,
272.) Justice Zaldivar was partial to an earlier formulation of Justice we must recognize that our Bill of Rights is a code of fair play for the less
Labrador that rules of procedure "are not to be applied in a very rigid, fortunate that we in all honor and good conscience must be observe. 31
technical sense"; but are intended "to help secure substantial justice."
(Ibid., p. 843) ... 30-g
The case at bar is worse.

Even if the questioned Court of Industrial Relations orders and rule were to be given
Management has shown not only lack of good-will or good intention, but a complete
effect, the dismissal or termination of the employment of the petitioning eight (8)
lack of sympathetic understanding of the plight of its laborers who claim that they
leaders of the Union is harsh for a one-day absence from work. The respondent
are being subjected to indignities by the local police, It was more expedient for the
Court itself recognized the severity of such a sanction when it did not include the
firm to conserve its income or profits than to assist its employees in their fight for
dismissal of the other 393 employees who are members of the same Union and who
their freedoms and security against alleged petty tyrannies of local police officers.
participated in the demonstration against the Pasig police. As a matter of fact, upon
This is sheer opportunism. Such opportunism and expediency resorted to by the
the intercession of the Secretary of Labor, the Union members who are not officers,
respondent company assaulted the immunities and welfare of its employees. It was
were not dismissed and only the Union itself and its thirteen (13) officers were
pure and implement selfishness, if not greed.
specifically named as respondents in the unfair labor practice charge filed against

7
Of happy relevance is the 1967 case of Republic Savings Bank vs. C.I.R., 32 where WHEREFORE, judgement is hereby rendered:
the petitioner Bank dismissed eight (8) employees for having written and published
"a patently libelous letter ... to the Bank president demanding his resignation on the
(1) setting aside as null and void the orders of the respondent Court of Industrial
grounds of immorality, nepotism in the appointment and favoritism as well as
Relations dated September 15 and October 9, 1969; and
discrimination in the promotion of bank employees." Therein, thru Mr. Justice Castro,
We ruled:
(2) directing the re instatement of the herein eight (8) petitioners, with full back pay
from the date of their separation from the service until re instated, minus one day's
It will avail the Bank none to gloat over this admission of the respondents.
pay and whatever earnings they might have realized from other sources during their
Assuming that the latter acted in their individual capacities when they
separation from the service.
wrote the letter-charge they were nonetheless protected for they were
engaged in concerted activity, in the exercise of their right of self
organization that includes concerted activity for mutual aid and protection, With costs against private respondent Philippine Blooming Company, Inc.
(Section 3 of the Industrial Peace Act ...) This is the view of some members
of this Court. For, as has been aptly stated, the joining in protests or
Zaldivar, Castro, Fernando and Esguerra, JJ., concur.
demands, even by a small group of employees, if in furtherance of their
interests as such, is a concerted activity protected by the Industrial Peace
Act. It is not necessary that union activity be involved or that collective Makalintal, C.J, took no part.
bargaining be contemplated. (Annot., 6 A.L.R. 2d 416 [1949]).

xxx xxx xxx

Instead of stifling criticism, the Bank should have allowed the respondents
to air their grievances.

xxx xxx xxx

The Bank defends its action by invoking its right to discipline for what it
calls the respondents' libel in giving undue publicity to their letter-charge.
To be sure, the right of self-organization of employees is not unlimited
(Republic Aviation Corp. vs. NLRB 324 U.S. 793 [1945]), as the right of the
employer to discharge for cause (Philippine Education Co. v. Union of Phil.
Educ. Employees, L-13773, April 29, 1960) is undenied. The Industrial Peace
Act does not touch the normal exercise of the right of the employer to
select his employees or to discharge them. It is directed solely against the
abuse of that right by interfering with the countervailing right of self
organization (Phelps Dodge Corp. v. NLRB 313 U.S. 177 [1941])...

xxx xxx xxx

In the final sum and substance, this Court is in unanimity that the Bank's
conduct, identified as an interference with the employees' right of self-
organization or as a retaliatory action, and/or as a refusal to bargain
collectively, constituted an unfair labor practice within the meaning and
intendment of section 4(a) of the Industrial Peace Act. (Emphasis
supplied.) 33

If free expression was accorded recognition and protection to fortify labor unionism
in the Republic Savings case, supra, where the complaint assailed the morality and
integrity of the bank president no less, such recognition and protection for free
speech, free assembly and right to petition are rendered all the more justifiable and
more imperative in the case at bar, where the mass demonstration was not against
the company nor any of its officers.

8
BUCK V. BELL, 274 U.S. 200 (1927) the County. The Circuit Court may consider the record of the board and the evidence
before it and such other admissible evidence as may be offered, and may affirm,
Syllabus revise, or reverse the order of the board and enter such order as it deems just.
Finally any party may apply to the Supreme Court of Appeals, which, if it grants the
1. The Virginia statute providing for the sexual sterilization of inmates of institutions appeal, is to hear the case upon the record of the trialin the Circuit Court, and may
supported by the State who shall be found to be afflicted with an hereditary form of enter such order as it thinks the Circuit Court should have entered. There can be no
insanity or imbecility, is within the power of the State under the Fourteenth doubt that, so far as procedure is concerned, the rights of the patient are most
Amendment. P. 274 U. S. 207. carefully considered, and, as every step in this case was taken in scrupulous
compliance with the statute and after months of observation, there is no doubt that,
2. Failure to extend the provision to persons outside the institutions named does not in that respect, the plaintiff in error has had due process of law.
render it obnoxious to the Equal Protection Clause.
The attack is not upon the procedure, but upon the substantive law. It seems to be
ERROR to a judgment of the Supreme Court of Appeals of the State of Virginia which contended that in no circumstances could such an order be justified. It certainly is
affirmed a judgment ordering the Superintendent of the State Colony of Epileptics contended that the order cannot be justified upon the existing grounds. The
and Feeble Minded to perform the operation of salpingectomy on Carrie Buck, the judgment finds the facts that have been recited, and that Carrie Buck "is the
plaintiff in error. probable potential parent of socially inadequate offspring, likewise afflicted, that she
Mr. JUSTICE HOLMES delivered the opinion of the Court. may be sexually sterilized without detriment to her general health, and that her
welfare and that of society will be promoted by her sterilization," and thereupon
This is a writ of error to review a judgment of the Supreme Court of Appeals of the makes the order. In view of the general declarations of the legislature and the
State of Virginia affirming a judgment of the Circuit Court of Amherst County by specific findings of the Court, obviously we cannot say as matter of law that the
which the defendant in error, the superintendent of the State Colony for Epileptics grounds do not exist, and, if they exist, they justify the result. We have seen more
and Feeble Minded, was ordered to perform the operation of salpingectomy upon than once that the public welfare may call upon the best citizens for their lives. It
Carrie Buck, the plaintiff in error, for the purpose of making her sterile. The case would be strange if it could not call upon those who already sap the strength of the
comes here upon the contention that the statute authorizing the judgment is void State for these lesser sacrifices, often not felt to be such by those concerned, in
under the Fourteenth Amendment as denying to the plaintiff in error due process of order to prevent our being swamped with incompetence. It is better for all the world
law and the equal protection of the laws. if, instead of waiting to execute degenerate offspring for crime or to let them starve
for their imbecility, society can prevent those who are manifestly unfit from
Carrie Buck is a feeble minded white woman who was committed to the State Colony continuing their kind. The principle that sustains compulsory vaccination is broad
above mentioned in due form. She is the daughter of a feeble minded mother in the enough to cover cutting the Fallopian tubes. Jacobson v. Massachusetts, 197 U. S.
same institution, and the mother of an illegitimate feeble minded child. She was 11. Three generations of imbeciles are enough.
eighteen years old at the time of the trial of her case in the Circuit Court, in the
latter part of 1924. An Act of Virginia, approved March 20, 1924, recites that the But, it is said, however it might be if this reasoning were applied generally, it fails
health of the patient and the welfare of society may be promoted in certain cases by when it is confined to the small number who are in the institutions named and is not
the sterilization of mental defectives, under careful safeguard, &c.; that the applied to the multitudes outside. It is the usual last resort of constitutional
sterilization may be effected in males by vasectomy and in females by arguments to point out shortcomings of this sort. But the answer is that the law does
salpingectomy, without serious pain or substantial danger to life; that the all that is needed when it does all that it can, indicates a policy, applies it to all
Commonwealth is supporting in various institutions many defective persons who, if within the lines, and seeks to bring within the lines all similarly situated so far and so
now discharged, would become a menace, but, if incapable of procreating, might be fast as its means allow. Of course, so far as the operations enable those who
discharged with safety and become self-supporting with benefit to themselves and otherwise must be kept confined to be returned to the world, and thus open the
to society, and that experience has shown that heredity plays an important part in asylum to others, the equality aimed at will be more nearly reached.
the transmission of insanity, imbecility, &c. The statute then enacts that, whenever
the superintendent of certain institutions, including the above-named State Colony, Judgment affirmed.
shall be of opinion that it is for the best interests of the patients and of society that MR. JUSTICE BUTLER dissents.
an inmate under his care should be sexually sterilized, he may have the operation
performed upon any patient afflicted with hereditary forms of insanity, imbecility,
&c., on complying with the very careful provisions by which the act protects the
patients from possible abuse.
The superintendent first presents a petition to the special board of directors of his
hospital or colony, stating the facts and the grounds for his opinion, verified by
affidavit. Notice of the petition and of the time and place of the hearing in the
institution is to be served upon the inmate, and also upon his guardian, and if there
is no guardian, the superintendent is to apply to the Circuit Court of the County to
appoint one. If the inmate is a minor, notice also is to be given to his parents, if any,
with a copy of the petition. The board is to see to it that the inmate may attend the
hearings if desired by him or his guardian. The evidence is all to be reduced to
writing, and, after the board has made its order for or against the operation, the
superintendent, or the inmate, or his guardian, may appeal to the Circuit Court of

9
RUBI VS. PROVINCIAL BOARD OF MINDORO "Whereas the provincial governor is of the opinion that the sitio of
Tigbao on Lake Naujan is a place most convenient for the
G.R. No. L-14078 March 7, 1919 Mangyanes to live on, Now, therefore be it

RUBI, ET AL. (manguianes), plaintiffs, "Resolved, that under section 2077 of the Administrative Code, 800
vs. hectares of public land in the sitio of Tigbao on Naujan Lake be selected as
THE PROVINCIAL BOARD OF MINDORO, defendant. a site for the permanent settlement of Mangyanes in Mindoro subject to the
approval of the Honorable Secretary of the Interior, and
MALCOLM, J.:
"Resolved further, That Mangyans may only solicit homesteads on this
In one of the cases which denote a landmark in American Constitutional History reservation providing that said homestead applications are previously
(Worcester vs. Georgia [1832], 6 Pet., 515), Chief Justice Marshall, the first luminary recommended by the provincial governor."
of American jurisprudence, began his opinion (relating to the status of an Indian)
with words which, with a slight change in phraseology, can be made to introduce the 2. That said resolution No. 25 (series 1917) of the provincial board of
present opinion This cause, in every point of view in which it can be placed, is of Mindoro was approved by the Secretary of the Interior of February 21, 1917.
the deepest interest. The legislative power of state, the controlling power of the 3. That on December 4, 1917, the provincial governor of Mindoro issued
constitution and laws, the rights if they have any, the political existence of a people, executive order No. 2 which says:
the personal liberty of a citizen, are all involved in the subject now to be considered.
"Whereas the provincial board, by Resolution No. 25, current
To imitate still further the opinion of the Chief Justice, we adopt his outline and series, has selected a site in the sitio of Tigbao on Naujan Lake for
proceed first, to introduce the facts and the issues, next to give a history of the so the permanent settlement of Mangyanes in Mindoro.
called "non-Christians," next to compare the status of the "non-Christians" with that
of the American Indians, and, lastly, to resolve the constitutional questions "Whereas said resolution has been duly approve by the Honorable,
presented. the Secretary of the Interior, on February 21, 1917.

I. INTRODUCTION. "Now, therefore, I, Juan Morente, jr., provincial governor of


Mindoro, pursuant to the provisions of section 2145 of the revised
This is an application for habeas corpus in favor of Rubi and other Manguianes of the Administrative Code, do hereby direct that all the Mangyans in the
Province of Mindoro. It is alleged that the Maguianes are being illegally deprived of townships of Naujan and Pola and the Mangyans east of the Baco
their liberty by the provincial officials of that province. Rubi and his companions are River including those in the districts of Dulangan and Rubi's place
said to be held on the reservation established at Tigbao, Mindoro, against their will, in Calapan, to take up their habitation on the site of Tigbao,
and one Dabalos is said to be held under the custody of the provincial sheriff in the Naujan Lake, not later than December 31, 1917.
prison at Calapan for having run away form the reservation.
"Any Mangyan who shall refuse to comply with this order shall
The return of the Solicitor-General alleges: upon conviction be imprisoned not exceed in sixty days, in
1. That on February 1, 1917, the provincial board of Mindoro adopted accordance with section 2759 of the revised Administrative Code."
resolution No. 25 which is as follows: 4. That the resolution of the provincial board of Mindoro copied in
The provincial governor, Hon. Juan Morente, Jr., presented the following paragraph 1 and the executive order of the governor of the same province
resolution: copied in paragraph 3, were necessary measures for the protection of the
Mangyanes of Mindoro as well as the protection of public forests in which
"Whereas several attempts and schemes have been made for the they roam, and to introduce civilized customs among them.
advancement of the non-Christian people of Mindoro, which were
all a failure, 5. That Rubi and those living in his rancheria have not fixed their dwelling
within the reservation of Tigbao and are liable to be punished in accordance
"Whereas it has been found out and proved that unless some other with section 2759 of Act No. 2711.
measure is taken for the Mangyan work of this province, no
successful result will be obtained toward educating these people. 6. That the undersigned has not information that Doroteo Dabalos is being
detained by the sheriff of Mindoro but if he is so detained it must be by
"Whereas it is deemed necessary to obliged them to live in one virtue of the provisions of articles Nos. 2145 and 2759 of Act No. 2711.
place in order to make a permanent settlement,
It thus appears that the provincial governor of Mindoro and the provincial board
"Whereas the provincial governor of any province in which non- thereof directed the Manguianes in question to take up their habitation in Tigbao, a
Christian inhabitants are found is authorized, when such a course site on the shore of Lake Naujan, selected by the provincial governor and approved
is deemed necessary in the interest of law and order, to direct by the provincial board. The action was taken in accordance with section 2145 of the
such inhabitants to take up their habitation on sites on unoccupied Administrative Code of 1917, and was duly approved by the Secretary of the Interior
public lands to be selected by him and approved by the provincial as required by said action. Petitioners, however, challenge the validity of this section
board. of the Administrative Code. This, therefore, becomes the paramount question which
the court is called upon the decide.
Section 2145 of the Administrative Code of 1917 reads as follows:

10
SEC. 2145. Establishment of non-Christina upon sites selected by provincial they cannot profit from the aid of our ministers and from that which gives
governor. With the prior approval of the Department Head, the provincial rise to those human necessities which men are obliged to give one another.
governor of any province in which non-Christian inhabitants are found is Having realized that convenience of this resolution, our kings, our
authorized, when such a course is deemed necessary in the interest of law predecessors, by different orders, have entrusted and ordered the viceroys,
and order, to direct such inhabitants to take up their habitation on sites on presidents, and governors to execute with great care and moderation the
unoccupied public lands to be selected by him an approved by the concentration of the indios intoreducciones; and to deal with their doctrine
provincial board. with such forbearance and gentleness, without causing inconveniences, so
that those who would not presently settle and who would see the good
In connection with the above-quoted provisions, there should be noted section 2759 treatment and the protection of those already in settlements would, of their
of the same Code, which read as follows: own accord, present themselves, and it is ordained that they be not
SEC. 2759. Refusal of a non-Christian to take up appointed habitation. required to pay taxes more than what is ordered. Because the above has
Any non-Christian who shall refuse to comply with the directions lawfully been executed in the greater part of our Indies, we hereby order and
given by a provincial governor, pursuant to section two thousand one decree that the same be complied with in all the remaining parts of the
hundred and forty-five of this Code, to take up habitation upon a site Indies, and the encomederos shall entreat compliance thereof in the
designated by said governor shall upon conviction be imprisonment for a manner and form prescribed by the laws of this title.
period not exceeding sixty days. xxx xxx xxx
The substance of what is now found in said section 2145 is not new to Philippine law. LAW VIII.
The genealogical tree of this section, if we may be permitted to use such
terminology, would read: Section 2077, Administrative Code of 1916; section 62, Act Philip II at the Pardo, on December 1, 1573. Philip III at Madrid, October 10,
No. 1397; section 2 of various special provincial laws, notably of Act No. 547, 1618.
specifically relating to the Manguianes; section 69, Act No. 387.
THE "REDUCCTIONES" BE MADE IN ACCORDANCE WITH THE CONDITIONS OF THIS
Section 2145 and its antecedent laws make use of the term "non-Christians." This LAW.
word, as will later be disclosed, is also found in varying forms in other laws of the
Philippine Islands. In order to put the phrase in its proper category, and in order to The places wherein the pueblos and reducciones shall be formed should
understand the policy of the Government of the Philippine Islands with reference to have the facilities of waters. lands, and mountains, ingress and egress,
the uncivilized elements of the Islands, it is well first of all to set down a skeleton husbandry and passageway of one league long, wherein the indios can
history of the attitude assumed by the authorities towards these "non-Christians," have their live stock that they may not be mixed with those of the
with particular regard for the legislation on the subject. Spaniards.

II. HISTORY. LAW IX.

A. BEFORE ACQUISITION OF THE PHILIPPINE BY THE UNITED STATES. Philip II at Toledo, on February 19, 1956.

The most important of the laws of the Indies having reference to the subject at hand THAT THE "INDIOS" IN "REDUCCIONES" BE NOT DEPRIVED OF THE LANDS
are compiled in Book VI, Title III, in the following language. PREVIOUSLY HELD BY THEM.

LAW I. With more good-will and promptness, the indios shall be concentrated
in reducciones. Provided they shall not be deprived of the lands and
The Emperor Charles and the Prince, the governor, at Cigales, on March 21, granaries which they may have in the places left by them. We hereby order
1551. Philip II at Toledo, on February 19, 1560. In the forest of Segovia on that no change shall be made in this respect, and that they be allowed to
September 13, 1565. In the Escorial on November 10, 1568. Ordinance 149 retain the lands held by them previously so that they may cultivate them
of the poblaciones of 1573. In San Lorenzo, on May 20, 1578, and profit therefrom.
THAT THE "INDIOS" BE REDUCED INTO "POBLACIONES" COMMUNITIES). xxx xxx xxx
In order that the indios may be instructed in the Sacred Catholic Faith and LAW XIII.
the evangelical law, and in order that they may forget the blunders of their
ancient rites and ceremonies to the end that they may live in harmony and THE SAME AS ABOVE.
in a civilized manner, it has always been endeavored, with great care and THAT THE "REDUCCIONES" BE NOT REMOVED WITHOUT ORDER OF THE KING,
special attention, to use all the means most convenient to the attainment VICEROY, OR COURT.
of these purposes. To carry out this work with success, our Council of the
Indies and other religious persons met at various times; the prelates of new No governor, or magistrate, or alcalde mayor, or any other court, has the
Spain assembled by order of Emperor Charles V of glorious memory in the right to alter or to remove thepueblos or the reducciones once constituted
year one thousand five hundred and forty-six all of which meetings were and founded, without our express order or that of the viceroy, president, or
actuated with a desire to serve God an our Kingdom. At these meetings it the royal district court, provided, however, that the encomenderos, priests,
was resolved that indios be made to live in communities, and not to live in or indios request such a change or consent to it by offering or giving
places divided and separated from one another by sierras and mountains, information to that en. And, because these claims are often made for
wherein they are deprived of all spiritual and temporal benefits and wherein private interests and not for those of the indios, we hereby order that this

11
law be always complied with, otherwise the change will be considered lack of all the nations which enable them to grasp the moral and material
fraudulently obtained. The penalty of one thousand pesos shall be imposed advantages that may be acquired in those towns under the protection and
upon the judge or encomendero who should violate this law. vigilance afforded them by the same laws.
LAW XV. It is equally highly depressive to our national honor to tolerate any longer
the separation and isolation of the non-Christian races from the social life of
Philip III at Madrid, on October 10, 1618. the civilized and Christian towns; to allow any longer the commission of
THAT THERE BE MAYORS AND ALDERMEN IN THE "REDUCTIONES," WHO SHALL BE depredations, precisely in the Island of Luzon wherein is located the seat of
"INDIOS." the representative of the Government of the, metropolis.

We order that in each town and reduccion there be a mayor, who should be It is but just to admit the fact that all the governments have occupied
an indio of the same reduccion; if there be more than eighty houses, there themselves with this most important question, and that much has been
should be two mayors and two aldermen, also indios; and, even if the town heretofore accomplished with the help and self-denial of the missionary
be a big one, there should, nevertheless, be more than two mayors and fathers who have even sacrificed their lives to the end that those
four aldermen, If there be less than eighty indios but not less than forty, degenerate races might be brought to the principles of Christianity, but the
there should be not more than one mayor and one alderman, who should means and the preaching employed to allure them have been insufficient to
annually elect nine others, in the presence of the priests , as is the practice complete the work undertaken. Neither have the punishments imposed
in town inhabited by Spaniards and indios. been sufficient in certain cases and in those which have not been guarded
against, thus giving and customs of isolation.
LAW XXI.
As it is impossible to consent to the continuation of such a lamentable state
Philip II, in Madrid, On May 2, 1563, and on November 25, 1578. At Tomar, of things, taking into account the prestige which the country demands and
on May 8, 1581. At Madrid, on January 10, 1589. Philip III, at Todesillas, on the inevitable duty which every government has in enforcing respect and
July 12, 1600. Philip IV, at Madrid, on October 1 and December 17, 1646. obedience to the national laws on the part of all who reside within the
For this law and the one following, see Law I, Tit. 4, Book 7. territory under its control, I have proceeded in the premises by giving the
most careful study of this serious question which involves important
THAT IN THE TOWNS OF THE "INDIOS," THERE SHALL LIVE NO SPANIARDS, NEGROES, interests for civilization, from the moral and material as well as the political
"MESTIZOS," AND MULATTOES.
standpoints. After hearing the illustrious opinions of all the local authorities,
We hereby prohibit and forbid Spaniards, negroes, mulattores, ecclesiastics, and missionaries of the provinces of Northern Luzon, and also
or mestizos to live to live in the reduccionesand towns and towns of after finding the unanimous conformity of the meeting held with the
the indios, because it has been found that some Spaniards who deal, trade, Archbishop of Manila, the Bishops of Jaro and Cebu, and the provincial
live, and associate with the indios are men of troublesome nature, of dirty prelates of the orders of the Dominicans, Agustinians, Recoletos,
ways of living; robbers, gamblers, and vicious and useless men; and, to Franciscans, and Jesuits as also of the meeting of the Council of Authorities,
avoid the wrongs done them, the indios would leave their towns and held for the object so indicated, I have arrived at an intimate conviction of
provinces; and the negroes, mestizos, and mulattoes, besides maltreating the inevitable necessity of proceeding in a practical manner for the
them and utilizing their services, contaminate them with their bad customs, submission of the said pagan and isolated races, as well as of the manner
idleness, and also some of their blunders and vices which may corrupt and and the only form of accomplishing such a task.
pervert the goal which we desire to reach with regard to their salvation,
For the reasons above stated and for the purpose of carrying out these
increase, and tranquillity. We hereby order the imposition of grave penalties objects, I hereby promulgate the following:
upon the commission of the acts above-mentioned which should not be
tolerated in the towns, and that the viceroys, presidents, governors, and DECREE.
courts take great care in executing the law within their powers and avail
themselves of the cooperation of the ministers who are truly honest. As 1. All the indian inhabitants (indios) of the Islands of Luzon are, from this
regards the mestizos and Indian and Chinese half-breeds (zambaigos), who date, to be governed by the common law, save those exceptions prescribed
are children of indiasand born among them, and who are to inherit their in this decree which are bases upon the differences of instructions, of the
houses and haciendas, they all not be affected by this law, it appearing to customs, and of the necessities of the different pagan races which occupy a
be a harsh thing to separate them from their parents. (Law of the Indies, part of its territory.
vol. 2, pp. 228, 229, 230, 231.) 2. The diverse rules which should be promulgated for each of these races
A clear exposition of the purposes of the Spanish government, in its efforts to which may be divided into three classes; one, which comprises those which
improve the condition of the less advanced inhabitants of the Islands by live isolated and roaming about without forming a town nor a home;
concentrating them in "reducciones," is found in the Decree of the Governor-General another, made up of those subdued pagans who have not as yet entered
of the Philippine Islands of January 14, 1881, reading as follows: completely the social life; and the third, of those mountain and rebellious
pagans shall be published in their respective dialects, and the officials,
It is a legal principle as well as a national right that every inhabitant of a priests, and missionaries of the provinces wherein they are found are
territory recognized as an integral part of a nation should respect and obey hereby entrusted in the work of having these races learn these rules. These
the laws in force therein; while, on other hand, it is the duty to conscience rules shall have executive character, beginning with the first day of next
and to humanity for all governments to civilize those backward races that April, and, as to their compliance, they must be observed in the manner
might exist in the nation, and which living in the obscurity of ignorance, prescribed below.

12
3. The provincial authorities in conjunction with the priests shall proceed, the ones who elect such officials under the direct charge of the authorities
from now on, with all the means which their zeal may suggest to them, to of the province or district.
the taking of the census of the inhabitants of the towns or settlement
already subdued, and shall adopt the necessary regulations for the 10. The races indicated in the preceding article, who voluntarily admit the
appointment of local authorities, if there be none as yet; for the advantages offered, shall, in return, have the obligation of constituting their
construction of courts and schools, and for the opening or fixing up of new towns, of constructing their town hall, schools, and country roads
means of communication, endeavoring, as regards the administrative which place them in communication with one another and with the
organization of the said towns or settlements, that this be finished before Christians; provided, the location of these towns be distant from their actual
the first day of next July, so that at the beginning of the fiscal year they residences, when the latter do not have the good conditions of location and
shall have the same rights and obligations which affect the remaining cultivations, and provided further the putting of families in a place so
towns of the archipelago, with the only exception that in the first two years selected by them be authorized in the towns already constituted.
they shall not be obliged to render personal services other than those 11. The armed force shall proceed to the prosecution and punishment of
previously indicated. the tribes, that, disregarding the peace, protection, and advantages offered
4. So long as these subdued towns or settlements are located infertile lands them, continue in their rebellious attitude on the first of next April,
appropriate for cultivation, the inhabitants thereof shall not be obliged to committing from now on the crimes and vexations against the Christian
move their dwelling-houses; and only in case of absolute necessity shall a towns; and for the this purposes, the Captain General's Office shall proceed
new residence be fixed for them, choosing for this purpose the place most with the organization of the divisions of the Army which, in conjunction with
convenient for them and which prejudices the least their interest; and, in the rural guards (cuadrilleros), shall have to enter the territory of such
either of these cases, an effort must be made to establish their homes with tribes. On the expiration of the term, they shall destroy their dwelling-
the reach of the sound of the bell. houses, labors, and implements, and confiscate their products and cattle.
Such a punishment shall necessarily be repeated twice a year, and for this
5. For the protection and defense of these new towns, there shall be purpose the military headquarters shall immediately order a detachment of
established an armed force composed precisely of native Christian, the the military staff to study the zones where such operations shall take place
organization and service of which shall be determined in a regulations and everything conducive to the successful accomplishment of the same.
based upon that of the abolished Tercios de Policia (division of the Guardia
Civil). 12. The chiefs of provinces, priests, and missioners, local authorities, and
other subordinates to my authorities, local authorities, and other
6. The authorities shall see to it that the inhabitants of the new towns subordinates to may authority, civil as well as military authorities, shall give
understand all the rights and duties affecting them and the liberty which the most effective aid and cooperation to the said forces in all that is within
they have as to where and now they shall till their lands and sell the the attributes and the scope of the authority of each.
products thereof, with the only exception of the tobacco which shall be
bought by the Hacienda at the same price and conditions allowed other 13. With respect to the reduccion of the pagan races found in some of the
producers, and with the prohibition against these new towns as well as the provinces in the southern part of the Archipelago, which I intend to visit, the
others from engaging in commerce of any other transaction with the preceding provisions shall conveniently be applied to them.
rebellious indios, the violation of which shall be punished with deportation. 14. There shall be created, under my presidency as Governor-General, Vice-
7. In order to properly carry out this express prohibition, the limits of the Royal Patron, a council or permanent commission which shall attend to and
territory of the rebellious indios shall be fixed; and whoever should go decide all the questions relative to the application of the foregoing
beyond the said limits shall be detained and assigned governmentally regulations that may be brought to it for consultations by the chiefs of
wherever convenient. provinces and priests and missionaries.

8. For the purpose of assisting in the conversion of the pagans into the 15. The secondary provisions which may be necessary, as a complement to
fraternity of the Catholic Church, all by this fact along be exempt for eight the foregoing, in brining about due compliance with this decree, shall be
years from rendering personal labor. promulgated by the respective official centers within their respective
jurisdictions. (Gaceta de Manila, No. 15) (Diccionario de la Administracion,
9. The authorities shall offer in the name of the State to the races not vol. 7, pp. 128-134.)
subdued (aetas and mountains igorrots the following advantages in returns
for their voluntary submission: to live in towns; unity among their families; B. AFTER ACQUISITON OF THE PHILIPPINES BY THE UNITED STATES.
concession of good lands and the right to cultivate them in the manner they Ever since the acquisition of the Philippine Islands by the United States, the question
wish and in the way them deem most productive; support during a year, as to the best method for dealing with the primitive inhabitants has been a
and clothes upon effecting submission; respect for their habits and customs perplexing one.
in so far as the same are not opposed to natural law; freedom to decide of
their own accord as to whether they want to be Christians or not; the 1. Organic law.
establishment of missions and families of recognized honesty who shall
The first order of an organic character after the inauguration of the American
teach, direct, protect, and give them security and trust them; the purchase
Government in the Philippines was President McKinley's Instructions to the
or facility of the sale of their harvests; the exemption from contributions
Commission of April 7, 1900, later expressly approved and ratified by section 1 of
and tributes for ten years and from the quintas (a kind of tax) for twenty
the Philippine Bill, the Act of Congress of July 1, 1902. Portions of these instructions
years; and lastly, that those who are governed by the local authorities as

13
have remained undisturbed by subsequent congressional legislation. One paragraph Vizcaya, Pangasinan, Paragua (Palawan), Tarlac, Tayabas, and Zambales. As an
of particular interest should here be quoted, namely: example of these laws, because referring to the Manguianes, we insert Act No. 547:
In dealing with the uncivilized tribes of the Islands, the Commission should No. 547. AN ACT PROVIDING FOR THE ESTABLISHMENT OF
adopt the same course followed by Congress in permitting the tribes of our LOCAL CIVIL GOVERNMENTS FOR THE MANGUIANES IN THE
North American Indians to maintain their tribal organization and PROVINCE OF MINDORO.
government and under which many of these tribes are now living in peace
and contentment, surrounded by civilization to which they are unable or By authority of the United States, be it enacted by the Philippine
unwilling to conform. Such tribal governments should, however, be Commission, that:
subjected to wise and firm regulation; and, without undue or petty SECTION 1. Whereas the Manguianes of the Provinces of Mindoro have not
interference, constant and active effort should be exercised to prevent progressed sufficiently in civilization to make it practicable to bring them
barbarous practices and introduce civilized customs. under any form of municipal government, the provincial governor is
Next comes the Philippine Bill, the Act of Congress of July 1, 1902, in the nature of authorized, subject to the approval of the Secretary of the Interior, in
an Organic Act for the Philippines. The purpose of section 7 of the Philippine Bill was dealing with these Manguianes to appoint officers from among them, to fix
to provide for a legislative body and, with this end in view, to name the prerequisites their designations and badges of office, and to prescribe their powers and
for the organization of the Philippine Assembly. The Philippine Legislature, composed duties: Provided, That the powers and duties thus prescribed shall not be in
of the Philippine Commission and the Philippine Assembly, was to have jurisdiction excess of those conferred upon township officers by Act Numbered Three
over the Christian portion of the Islands. The Philippine Commission was to retain hundred and eighty-seven entitled "An Act providing for the establishment
exclusive jurisdiction of that part of said Islands inhabited by Moros or other non- of local civil Governments in the townships and settlements of Nueva
Christian tribes. Vizcaya."

The latest Act of Congress, nearest to a Constitution for the Philippines, is the Act of SEC. 2. Subject to the approval of the Secretary of the Interior, the
Congress of August 29, 1916, commonly known as the Jones Law. This transferred provincial governor is further authorized, when he deems such a course
the exclusive legislative jurisdiction and authority theretofore exercised by the necessary in the interest of law and order, to direct such Manguianes to
Philippine Commission, to the Philippine Legislature (sec. 12). It divided the take up their habitation on sites on unoccupied public lands to be selected
Philippine Islands into twelve senatorial districts, the twelfth district to be composed by him and approved by the provincial board. Manguianes who refuse to
of the Mountain Province, Baguio, Nueva Vizcaya, and the Department of Mindanao comply with such directions shall upon conviction be imprisonment for a
and Sulu. The Governor-General of the Philippine Islands was authorized to appoint period not exceeding sixty days.
senators and representatives for the territory which, at the time of the passage of SEC. 3. The constant aim of the governor shall be to aid the Manguianes of
the Jones Law, was not represented in the Philippine Assembly, that is, for the his province to acquire the knowledge and experience necessary for
twelfth district (sec. 16). The law establish a bureau to be known as the "Bureau of successful local popular government, and his supervision and control over
non-Christian Tribes" which shall have general supervision over the public affairs of them shall be exercised to this end, an to the end that law and order and
the inhabitants which are represented in the Legislature by appointed senators and individual freedom shall be maintained.
representatives( sec. 22).
SEC. 4. When in the opinion of the provincial board of Mindoro any
Philippine organic law may, therefore, be said to recognized a dividing line between settlement of Manguianes has advanced sufficiently to make such a course
the territory not inhabited by Moros or other non-Christian tribes, and the territory practicable, it may be organized under the provisions of sections one to
which Moros or other non-Christian tribes, and the territory which is inhabited by sixty-seven, inclusive, of Act Numbered three hundred and eighty-seven, as
Moros or other non-Christian tribes. a township, and the geographical limits of such township shall be fixed by
2. Statute law. the provincial board.

Local governments in the Philippines have been provided for by various acts of the SEC. 5. The public good requiring the speedy enactment of this bill, the
Philippine Commission and Legislature. The most notable are Acts Nos. 48 and 49 passage of the same is hereby expedited in accordance with section two of
concerning the Province of Benguet and the Igorots; Act NO. 82, the Municipal 'An Act prescribing the order of procedure by the Commission in the
Code; ;Act no. 83, the Provincial Government Act; Act No. 183, the Character of the enactment of laws,' passed September twenty-sixth, nineteen hundred.
city of Manila; Act No. 7887, providing for the organization and government of the SEC. 6. This Act shall take effect on its passage.
Moro Province; Act No. 1396, the Special Provincial Government Act; Act No. 1397,
the Township Government Act; Act No. 1667, relating to the organization of Enacted, December 4, 1902.
settlements; Act No. 1963, the Baguio charger; and Act No. 2408, the Organic Act of
All of these special laws, with the exception of Act No. 1306, were repealed by Act
the Department of Mindanao and Sulu. The major portion of these laws have been
No. 1396 and 1397. The last named Act incorporated and embodied the provisions in
carried forward into the Administrative Codes of 1916 an d1917.
general language. In turn, Act No. 1397 was repealed by the Administrative Code of
Of more particular interest are certain special laws concerning the government of 1916. The two Administrative Codes retained the provisions in questions.
the primitive peoples. Beginning with Act No. 387, sections 68-71, enacted on April
These different laws, if they of the non-Christian inhabitants of the Philippines and a
9, 1902, by the United States Philippine Commission, having reference to the
settled and consistent practice with reference to the methods to be followed for their
Province of Nueva Vizcaya, Acts Nos. 4111, 422, 445, 500, 547, 548, 549, 550, 579,
advancement.
753, 855, 1113, 1145, 4568, 1306 were enacted for the provinces of Abra, Antique,
Bataan, Ilocos Norte, Ilocos Sur, Isabela. Lepanto-Bontoc, Mindoro, Misamis, Nueva C. TERMINOLOGY.

14
The terms made use of by these laws, organic and statutory, are found in varying It is well-known that within the specially organized provinces, there live persons
forms. some of who are Christians and some of whom are not Christians. In fact, the law
specifically recognizes this. ( Sec. 2422, Administrative Code of 1917, etc.)
"Uncivilized tribes" is the denomination in President McKinley's instruction to the
Commission. If the religious conception is not satisfactory, so against the geographical conception
is likewise inadquate. The reason it that the motive of the law relates not to a
The most commonly accepted usage has sanctioned the term "non-Christian tribes." particular people, because of their religion, or to a particular province because of its
These words are to be found in section 7 of the Philippine Bill and in section 22 of location, but the whole intent of the law is predicated n the civilization or lack of
the Jones Law. They are also to be found in Act No. 253 of the Philippines civilization of the inhabitants.
Commission, establishing a Bureau of non-Christian Tribes and in Act No. 2674 of the
Philippine Legislature, carried forward into sections 701-705 of the Administrative At most, "non-Christian" is an awkward and unsatisfactory word. Apologetic words
Code of 1917, reestablishing this Bureau. Among other laws which contain the usually introduce the term. "The so-called non-Christian" is a favorite expression.
phrase, there can be mentioned Acts Nos. 127, 128, 387, 547, 548, 549, 550, 1397, The Secretary of the Interior who for so many years had these people under his
1639, and 2551. jurisdiction, recognizing the difficulty of selecting an exact designation, speaks of the
"backward Philippine peoples, commonly known as the 'non-Christian tribes."' (See
"Non-Christian people," "non-Christian inhabitants," and "non-Christian Filipinos" Hearings before the Committee on the Philippines, United States Senate, Sixty-third
have been the favorite nomenclature, in lieu of the unpopular word "tribes," since Congress, third session on H.R. 18459, An Act to declare the purpose of the People of
the coming into being of a Filipinized legislature. These terms can be found in the United States as to the future political status of the Philippine Islands and to
sections 2076, 2077, 2390, 2394, Administrative Code of 1916; sections 701-705, provide a more autonomous government for the Islands, pp. 346, 351; letter of the
2145, 2422, 2426, Administrative Code of 1917; and in Acts Nos. 2404, 2435, 2444, Secretary of the Interior of June 30, 1906, circulated by the Executive Secretary.)
2674 of the Philippine Legislatures, as well as in Act No. 1667 of the Philippine
Commission. The idea that the term "non-Christian" is intended to relate to degree of civilization,
is substantiated by reference to legislative, judicial, and executive authority.
The Administrative Code specifically provides that the term "non-Christian" shall
include Mohammedans and pagans. (Sec. 2576, Administrative Code of 1917; sec. The legislative intent is borne out by Acts Nos. 48, 253, 387, 1667, and 2674, and
2561, Administrative Code of 1916, taken from Act No. 2408, sec. 3.) sections 701 et seq, and sections 2422 et seq, of the Administrative Code of 1917.
For instance, Act No. 253 charged the Bureau of non-Christian tribes to conduct
D. MEANING OF TERM "NON-CHRISTIAN." "systematic investigations with reference to non-Christian tribes . . . with special
If we were to follow the literal meaning of the word "non-Christian," it would of view to determining the most practicable means for bringing about their
course result in giving to it a religious signification. Obviously, Christian would be advancement in civilization and material property prosperity."
those who profess the Christian religion, and non-Christians, would be those who do As authority of a judicial nature is the decision of the Supreme Court in the case of
not profess the Christian religion. In partial corroboration of this view, there could United States vs. Tubban [Kalinga] ([1915], 29, Phil., 434). The question here arose
also be cited section 2576 of the last Administrative Code and certain well-known as to the effect of a tribal marriage in connection with article 423 of the Penal code
authorities, as Zuiga, "Estadismo de las Islas Filipinas," Professor Ferdinand concerning the husband who surprises his wife in the act of adultery. In discussing
Blumentritt, "Philippine Tribes and Languages," and Dr. N. M. Saleeby, "The Origin of the point, the court makes use of the following language:
Malayan Filipinos." (See Blair & Robertson, "The Philippine Islands," 1493-1898, vol.
III, p. 300, note; Craig-Benitez, "Philippine Progress prior to 1898," vol. I. p. 107.) . . . we are not advised of any provision of law which recognizes as legal a
tribal marriage of so-called non-Christians or members of uncivilized tribes,
Not content with the apparent definition of the word, we shall investigate further to celebrated within that province without compliance with the requisites
ascertain what is its true meaning. prescribed by General Orders no. 68. . . . We hold also that the fact that the
In one sense, the word can have a geographical signification. This is plainly to be accused is shown to bea member of an uncivilized tribe, of a low order of
seen by the provisions of many laws. Thus, according to the Philippine Bill, the intelligence, uncultured and uneducated, should be taken into consideration
authority of the Philippine Assembly was recognized in the "territory" of the Islands as a second marked extenuating circumstance.
not inhabited by Moros or other non-Christian tribes. Again, the Jones Law confers Of much more moment is the uniform construction of execution officials who have
similar recognition in the authorization of the twelfth senatorial district for the been called upon to interpret and enforce the law. The official who, as a member of
"territory not now represented in the Philippine Assembly." The Philippines the Philippine Commission, drafted much of the legislation relating to the so-called
Legislature has, time and again, adopted acts making certain other acts applicable Christians and who had these people under his authority, was the former Secretary
to that "part" of the Philippine Islands inhabited by Moros or other non-Christian of the Interior. Under date of June 30, 1906, this official addressed a letter to all
tribes. governor of provinces, organized under the Special Provincial Government Act, a
Section 2145, is found in article XII of the Provincial Law of the Administrative Code. letter which later received recognition by the Governor-General and was circulated
The first section of this article, preceding section 2145, makes the provisions of the by the Executive Secretary, reading as follows:
article applicable only in specially organized provinces. The specially organized Sir: Within the past few months, the question has arisen as to whether
provinces are the Mountain Province, Nueva Vizcaya, Mindoro, Batanes, and people who were originally non-Christian but have recently been baptized
Palawan. These are the provinces to which the Philippine Legislature has never seen or who are children of persons who have been recently baptized are, for the
fit to give all the powers of local self-government. They do not, however, exactly purposes of Act 1396 and 1397, to be considered Christian or non-
coincide with the portion of the Philippines which is not granted popular Christians.
representation. Nevertheless, it is still a geographical description.

15
It has been extremely difficult, in framing legislation for the tribes in these associated with the mountain tribes, either as a member thereof or as a
islands which are not advanced far in civilization, to hit upon any suitable recruit. So far, this question has not come up as to whether a Christian,
designation which will fit all cases. The number of individual tribes is so maintaining his religious belief, but throwing his lot and living with a non-
great that it is almost out of the question to enumerate all of them in an Christian tribe, would or would not be subject to the cedula tax. On one
Act. It was finally decided to adopt the designation 'non-Christians' as the occasion a prominent Hebrew of Manila claimed to this office that he was
one most satisfactory, but the real purpose of the Commission was not so exempt from the cedula tax, inasmuch as he was not a Christian. This
much to legislate for people having any particular religious belief as for Office, however, continued to collect cedula taxes from all the Jews, East
those lacking sufficient advancement so that they could, to their own Indians, Arabs, Chinamen, etc., residing in Manila. Quite a large proportion
advantage, be brought under the Provincial Government Act and the of the cedula taxes paid in this city are paid by men belonging to the
Municipal Code. nationalities mentioned. Chinamen, Arabs and other s are quite widely
scattered throughout the Islands, and a condition similar to that which exist
The mere act of baptism does not, of course, in itself change the degree of in Manila also exists in most of the large provincial towns. Cedula taxes are
civilization to which the person baptized has attained at the time the act of therefore being collected by this Office in all parts of these Islands on the
baptism is performed. For practical purposes, therefore, you will give the broad ground that civilized people are subject to such taxes, and non-
member of so-called "wild tribes" of your province the benefit of the doubt civilized people preserving their tribal relations are not subject thereto.
even though they may recently have embraced Christianity.
(Sgd.) JNO. S. HORD,
The determining factor in deciding whether they are to be allowed to Collector of Internal Revenue.
remain under the jurisdiction of regularly organized municipalities or what
form of government shall be afforded to them should be the degree of On September 17, 1910, the Collector of Internal Revenue addressed circular letter
civilization to which they have attained and you are requested to govern No. 327, approved by the Secretary of Finance and Justice, to all provincial
yourself accordingly. treasurers. This letter in part reads:
I have discussed this matter with the Honorable, the Governor-General, who In view of the many questions that have been raised by provincial
concurs in the opinion above expressed and who will have the necessary treasurers regarding cedula taxes due from members of non-Christian tribes
instructions given to the governors of the provinces organized under the when they come in from the hills for the purposes of settling down and
Provincial Government Act. (Internal Revenue Manual, p. 214.) becoming members of the body politic of the Philippine Islands, the
following clarification of the laws governing such questions and digest of
The present Secretary of the Interior, in a memorandum furnished a member of this rulings thereunder is hereby published for the information of all concerned:
court, has the following to say on the subject:
Non-Christian inhabitants of the Philippine Islands are so classed, not by
As far as names are concerned the classification is indeed unfortunate, but reason of the fact that they do not profess Christianity, but because of their
while no other better classification has as yet been made the present uncivilized mode of life and low state of development. All inhabitants of the
classification should be allowed to stand . . . I believe the term carries the Philippine Islands classed as members of non-Christian tribes may be
same meaning as the expressed in the letter of the Secretary of the Interior divided into three classes in so far as the cedula tax law is concerned . . .
(of June 30, 1906, herein quoted). It is indicative of the degree of civilization
rather than of religious denomination, for the hold that it is indicative of Whenever any member of an non-Christian tribe leaves his wild and
religious denomination will make the law invalid as against that uncivilized mode of life, severs whatever tribal relations he may have had
Constitutional guaranty of religious freedom. and attaches himself civilized community, belonging a member of the body
politic, he thereby makes himself subject to precisely the same law that
Another official who was concerned with the status of the non-Christians, was the governs the other members of that community and from and after the date
Collector of Internal Revenue. The question arose for ruling relatives to the cedula when he so attaches himself to the community the same cedula and other
taxation of the Manobos and the Aetas. Thereupon, the view of the Secretary of the taxes are due from him as from other members thereof. If he comes in after
Interior was requested on the point, who, by return indorsement, agreed with the the expiration of the delinquency period the same rule should apply to him
interpretation of the Collector of Internal Revenue. This Construction of the Collector as to persons arriving from foreign countries or reaching the age of
of Internal Revenue can be found in circular letter No. 188 of the Bureau of Internal eighteen subsequent to the expiration of such period, and a regular class A,
Revenue, dated June 11, 1907, reading as follows (Internal Revenue Manual, p. 214): D, F, or H cedula, as the case may be, should be furnished him without
The internal revenue law exempts "members of non-Christian tribes" from penalty and without requiring him to pay the tax for former years.
the payment of cedula taxes. The Collector of Internal Revenue has In conclusion, it should be borne in mind that the prime factors in
interpreted this provision of law to mean not that persons who profess determining whether or not a man is subject to the regular cedula tax is not
some form of Christian worship are alone subject to the cedula tax, and the circumstance that he does or does not profess Christianity, nor even his
that all other person are exempt; he has interpreted it to mean that all maintenance of or failure to maintain tribal relations with some of the well
persons preserving tribal relations with the so-called non-Christian tribes known wild tribes, but his mode of life, degree of advancement in
are exempt from the cedula tax, and that all others, including Jews, civilization and connection or lack of connection with some civilized
Mohammedans, Confucians, Buddists, etc., are subject to said tax so long community. For this reason so called "Remontados" and "Montescos" will be
as they live in cities or towns, or in the country in a civilized condition. In classed by this office as members of non-Christian tribes in so far as the
other words, it is not so much a matter of a man's form of religious worship application of the Internal Revenue Law is concerned, since, even though
or profession that decides whether or not he is subject to the cedula tax; it they belong to no well recognized tribe, their mode of life, degree of
is more dependent on whether he is living in a civilized manner or is

16
advancement and so forth are practically the same as those of the Igorrots The Official Census of 1903, in the portion written by no less an authority than De.
and members of other recognized non-Christina tribes. David P. Barrows, then "Chief of the Bureau of non-Christian Tribes," divides the
population in the Christian or Civilized Tribes, and non-Christian or Wild Tribes.
Very respectfully, (Census of the Philippine Islands [1903], vol. 1, pp. 411 et seq). The present Director
(Sgd.) ELLIS CROMWELL, of the Census, Hon. Ignacio Villamor, writes that the classification likely to be used in
Collector of Internal Revenue, the Census now being taken is: "Filipinos and Primitive Filipinos." In a Pronouncing
Gazetteer and Geographical Dictionary of the Philippine Islands, prepared in the
Approved: Bureau of Insular Affairs, War Department, a sub-division under the title non-
(Sgd.) GREGORIO ARANETA, Christian tribes is, "Physical and Political Characteristics of the non-Christian Tribes,"
Secretary of Finance and Justice. which sufficiently shows that the terms refers to culture and not to religion.
The two circular above quoted have since been repealed by Bureau of Internal In resume, therefore, the Legislature and the Judiciary, inferentially, and different
Revenue Regulations No. 1, promulgated by Venancio Concepcion, Acting Collector executive officials, specifically, join in the proposition that the term "non-Christian"
of Internal Revenue, and approved on April 16, 1915, by Honorable Victorino Mapa, refers, not to religious belief, but, in a way , to geographical area, and, more directly,
Secretary of Finance and Justice. Section 30 of the regulations is practically a to natives of the Philippine Islands of a law grade of civilization, usually living in
transcript of Circular Letter No. 327. tribal relationship apart from settled communities.
The subject has come before the Attorney-General for consideration. The Chief of E. THE MANGUIANES.
Constabulary request the opinion of the Attorney-General as to the status of a non-
Christian who has been baptized by a minister of the Gospel. The precise questions The so-called non-Christians are in various state approaching civilization. The
were these: "Does he remain non-Christian or is he entitled to the privileges of a Philippine Census of 1903 divided them into four classes. Of the third class, are the
Christian? By purchasing intoxicating liquors, does he commit an infraction of the Manguianes (or Mangyans) of Mindoro.
law and does the person selling same lay himself liable under the provision of Act Of the derivation of the name "Manguian" Dr. T. H. Pardo de Tavera in his Etimilogia
No. 1639?" The opinion of Attorney-General Avancea, after quoting the same de los nombres de Rozas de Filipinas, says:
authorities hereinbefore set out, concludes:
In Tagalog, Bicol, and Visaya, Manguian signifies "savage," "mountainer,"
In conformity with the above quoted constructions, it is probable that is "pagan," "negro." It may be that the use of this word is applicable to a great
probable that the person in question remains a non-Christian, so that, in number of Filipinos, but nevertheless it has been applied only to certain
purchasing intoxicating liquors both he and the person selling the same inhabitants of Mindoro. Even in primitive times without doubt this name
make themselves liable to prosecution under the provisions of Act No. was given to those of that island who bear it to-day, but its employed in
1639. At least, I advise you that these should be the constructions place three Filipino languages shows that the radical ngian had in all these
upon the law until a court shall hold otherwise. languages a sense to-day forgotten. In Pampango this ending still exists
Solicitor-General Paredes in his brief in this case says: and signifies "ancient," from which we can deduce that the name was
applied to men considered to be the ancient inhabitants, and that these
With respect to the meaning which the phrase non-Christian inhabitants has men were pushed back into the interior by the modern invaders, in whose
in the provisions of the Administrative code which we are studying, we language they were called the "ancients."
submit that said phrase does not have its natural meaning which would
include all non-Christian inhabitants of the Islands, whether Filipino or The Manguianes are very low in culture. They have considerable Negrito blood and
strangers, civilized or uncivilized, but simply refers to those uncivilized have not advanced beyond the Negritos in civilization. They are a peaceful, timid,
members of the non-Christian tribes of the Philippines who, living without primitive, semi-nomadic people. They number approximately 15,000. The
home or fixed residence, roam in the mountains, beyond the reach of law manguianes have shown no desire for community life, and, as indicated in the
and order . . . preamble to Act No. 547, have not progressed sufficiently in civilization to make it
practicable to bring them under any form of municipal government. (See Census of
The Philippine Commission in denominating in its laws that portion of the the Philippine (Islands [1903], vol. I, pp. 22, 23, 460.)
inhabitants of the Philippines which live in tribes as non-Christian tribes, as
distinguished from the common Filipinos which carry on a social and III. COMPARATIVE THE AMERICAN INDIANS.
civilized life, did not intended to establish a distinction based on the Reference was made in the Presidents' instructions to the Commission to the policy
religious beliefs of the individual, but, without dwelling on the difficulties adopted by the United States for the Indian Tribes. The methods followed by the
which later would be occasioned by the phrase, adopted the expression Government of the Philippines Islands in its dealings with the so-called non-Christian
which the Spanish legislation employed to designate the uncivilized portion people is said, on argument, to be practically identical with that followed by the
of the inhabitants of the Philippines. United States Government in its dealings with the Indian tribes. Valuable lessons, it
The phrase 'non-Christian inhabitants' used in the provisions of articles is insisted, can be derived by an investigation of the American-Indian policy.
2077 and 2741 of Act No. 2657 (articles 2145 and 2759) should be From the beginning of the United States, and even before, the Indians have been
understood as equivalent to members of uncivilized tribes of the treated as "in a state of pupilage." The recognized relation between the Government
Philippines, not only because this is the evident intention of the law, but of the United States and the Indians may be described as that of guardian and ward.
because to give it its lateral meaning would make the law null and It is for the Congress to determine when and how the guardianship shall be
unconstitutional as making distinctions base the religion of the individual.

17
terminated. The Indians are always subject to the plenary authority of the United United States, because it has never been denied, and because it alone can
States. enforce its laws on all the tribes.
Chief Justice Marshall in his opinion in Worcester vs. Georgia, hereinbefore In the later case of United States vs. Sandoval ([1913], 231 U.S., 28) the question to
mentioned, tells how the Congress passed an Act in 1819 "for promoting those be considered was whether the status of the Pueblo Indians and their lands was such
humane designs of civilizing the neighboring Indians." After quoting the Act, the that Congress could prohibit the introduction of intoxicating liquor into those lands
opinion goes on "This act avowedly contemplates the preservation of the Indian notwithstanding the admission of New Mexico to statehood. The court looked to the
nations as an object sought by the United States, and proposes to effect this object reports of the different superintendent charged with guarding their interests and
by civilizing and converting them from hunters into agriculturists." founds that these Indians are dependent upon the fostering care and protection of
the government "like reservation Indians in general." Continuing, the court said "that
A leading case which discusses the status of the Indians is that of the United during the Spanish dominion, the Indians of the pueblos were treated as wards
States vs. Kagama ([1886], 118 U.S., 375). Reference is herein made to the clause of requiring special protection, where subjected to restraints and official supervisions in
the United States Constitution which gives Congress "power to regulate commerce the alienation of their property." And finally, we not the following: "Not only does the
with foreign nations, and among the several States, and with the Indian tribes." The Constitution expressly authorize Congress to regulate commerce with the Indians
court then proceeds to indicate a brief history of the position of the Indians in the tribes, but long-continued legislative and executive usage and an unbroken current
United States (a more extended account of which can be found in Marshall's opinion of judicial decisions have attributed to the United States as a superior and civilized
in Worcester vs. Georgia, supra), as follows: nation the power and the duty of exercising a fostering care and protection over all
The relation of the Indian tribes living within the borders of the United dependent Indian communities within its borders, whether within its original territory
States, both before and since the Revolution, to the people of the United or territory subsequently acquired, and whether within or without the limits of a
States, has always been an anomalous one and of a complex character. state."

Following the policy of the European Governments in the discovery of With reference to laws affecting the Indians, it has been held that it is not within the
American towards the Indians who were found here, the colonies before the power of the courts to overrule the judgment of Congress. For very good reason, the
Revolution and the States and the United States since, have recognized in subject has always been deemed political in nature, not subject to the jurisdiction of
the Indians a possessory right to the soil over which they roamed and the judicial department of the government. (Matter of Heff [1905], 197 U.S., 488;
hunted and established occasional villages. But they asserted an ultimate U.S.vs. Celestine [1909], 215 U.S., 278; U.S. vs. Sandoval, supra;
title in the land itself, by which the Indian tribes were forbidden to sell or Worcester vs. Georgia, supra; U.S. vs. Rogers [1846], 4 How., 567; the Cherokee
transfer it to other nations or peoples without the consent of this Tobacco [1871], 11 Wall, 616; Roff vs. Burney [1897], 168 U.S., 218; Thomas vs.Gay
paramount authority. When a tribe wished to dispose of its lands, or any [1898], 169 U.S.., 264; Lone Wolf vs. Hitchcock[1903], 187 U.S., 553;
part of it, or the State or the United States wished to purchase it, a treaty Wallace vs. Adams [1907], 204 U.S., 415; Conley vs. Bollinger [1910], 216 U.S., 84;
with the tribe was the only mode in which this could be done. The United Tiger vs. Western Invest. Co. [1911], 221 U.S., 286; U.S. vs. Lane [1913], 232 U.S..,
States recognized no right in private persons, or in other nations, to make 598; Cyr vs. Walker (1911], 29 Okla, 281; 35 L.R.A. [N. S.], 795.) Whenever,
such a purchase by treaty or otherwise. With the Indians themselves these therefore, the United States sets apart any public land as an Indian reservation, it
relation are equally difficult to define. They were, and always have been, has full authority to pass such laws and authorize such measures as may be
regarded as having a semi-independent position when they preserved their necessary to give to the Indians thereon full protection in their persons and property.
tribal relations; not as States, not as nation not a possessed of the fall (U.S. vs.Thomas [1894], 151 U.S., 577.)
attributes of sovereignty, but as a separate people, with the power of All this borne out by long-continued legislative and executive usage, and an
regulating their internal and social relations, and thus far not brought under unbroken line of judicial decisions.
the laws of the Union or of the State within whose limits they resided.
The only case which is even remotely in point and which, if followed literally, might
The opinion then continues: result in the issuance of habeas corpus, is that of United States vs. Crook ([1879],
It seems to us that this (effect of the law) is within the competency of Fed. Cas. No. 14891). This was a hearing upon return to a writ ofhabeas
Congress. These Indian tribes are the wards of the nation. The are corpus issued against Brigadier General George Crook at the relation of Standing
communities dependent on the United States. dependent largely for their Bear and other Indians, formerly belonging to the Ponca Tribe of Indians. The petition
daily food. Dependent for their political rights. They owe no allegiance to alleged in substance that the relators are Indians who have formerly belonged to the
the States, and receive from the no protection. Because of the local ill Ponca tribe of Indians, now located in the Indian Territory; that they had some time
feeling, the people of the States where they are found are often their previously withdrawn from the tribe, and completely severed their tribal relations
deadliest enemies. From their very weakness and helplessness, so largely therewith, and had adopted the general habits of the whites, and were then
due to the course of dealing of the Federal Government with them and the endeavoring to maintain themselves by their own exertions, and without aid or
treaties in which it has been promised, there arise the duty of protection, assistance from the general government; that whilst they were thus engaged, and
and with it the power. This has always been recognized by the Executive without being guilty of violating any of the laws of the United States, they were
and by Congress, and by this court, whenever the question has arisen . . . arrested and restrained of their liberty by order of the respondent, George Crook.
The power of the General Government over these remnants of race once The substance of the return to the writ was that the relators are individual members
powerful, now weak and diminished in numbers, is necessary to their of, and connected with, the Ponca tribe of Indians; that they had fled or escaped
protection, as well as to the safety of those among whom they dwell. it form a reservation situated some place within the limits of the Indian Territory had
must exist in that government, because it never has existed anywhere else, departed therefrom without permission from the Government; and, at the request of
because the theater of its exercise is within the geographical limits of the the Secretary of the Interior, the General of the Army had issued an order which
required the respondent to arrest and return the relators to their tribe in the Indian

18
Territory, and that, pursuant to the said order, he had caused the relators to be forceful reasons exists for the segregation as existed for the segregation of the
arrested on the Omaha Indian Territory. different Indian tribes in the United States.
The first question was whether an Indian can test the validity of an illegal IV. CONSTITUTIONAL QUESTIONS.
imprisonment by habeas corpus. The second question, of much greater importance,
related to the right of the Government to arrest and hold the relators for a time, for A. DELEGATION OF LEGISLATIVE POWER.
the purpose of being returned to the Indian Territory from which it was alleged the The first constitutional objection which confronts us is that the Legislature could not
Indian escaped. In discussing this question, the court reviewed the policy the delegate this power to provincial authorities. In so attempting, it is contended, the
Government had adopted in its dealing with the friendly tribe of Poncase. Then, Philippine Legislature has abdicated its authority and avoided its full responsibility.
continuing, the court said: "Laws passed for the government of the Indian country,
and for the purpose of regulating trade and intercourse with the Indian tribes, confer That the maxim of Constitutional Law forbidding the delegation of legislative power
upon certain officers of the Government almost unlimited power over the persons should be zealously protected, we agree. An understanding of the rule will, however,
who go upon the reservations without lawful authority . . . Whether such an disclose that it has not bee violated in his instance.
extensive discretionary power is wisely vested in the commissioner of Indian affairs
The rule has nowhere been better stated than in the early Ohio case decided by
or not , need not be questioned. It is enough to know that the power rightfully exists,
Judge Ranney, and since followed in a multitude of case, namely: "The true
and, where existing, the exercise of the power must be upheld." The decision
distinction therefore is between the delegation of power to make the law, which
concluded as follows:
necessarily involves a discretion as to what it shall be, and conferring an authority or
The reasoning advanced in support of my views, leads me to conclude: discretion as to its execution, to be exercised under and in pursuance of the law. The
first cannot be done; to the later no valid objection can be made." (Cincinnati, W. &
1. that an Indian is a 'person' within the meaning of the laws of the United Z. R. Co. vs. Comm'rs. Clinton County [1852], 1 Ohio S.t, 88.) Discretion, as held by
States, and has, therefore, the right to sue out a writ of habeas corpus in a Chief Justice Marshall in Wayman vs. Southard ([1825], 10 Wheat., 1) may be
federal court, or before a federal judge, in all cases where he may be committed by the Legislature to an executive department or official. The Legislature
confined or in custody under color of authority of the United States or may make decisions of executive departments of subordinate official thereof, to
where he is restrained of liberty in violation of the constitution or laws of whom t has committed the execution of certain acts, final on questions of fact.
the United States. (U.S.vs. Kinkead [1918], 248 Fed., 141.) The growing tendency in the decision is to
2. That General George Crook, the respondent, being commander of the give prominence to the "necessity" of the case.
military department of the Platte, has the custody of the relators, under Is not all this exactly what the Legislature has attempted to accomplish by the
color of authority of the United States, and in violation of the laws enactment of section 21454 of the Administrative Code? Has not the Legislature
therefore. merely conferred upon the provincial governor, with the approval of the provincial
3. That n rightful authority exists for removing by force any of the relators board and the Department Head, discretionary authority as to the execution of the
to the Indian Territory, as the respondent has been directed to do. law? Is not this "necessary"?

4. that the Indians possess the inherent right of expatriation, as well as the The case of West vs. Hitchock, ([1906], 205 U.S., 80) was a petition for mandamus to
more fortunate white race, and have the inalienable right to "life, liberty, require the Secretary of the Interior to approve the selection and taking of one
and the pursuit of happiness," so long as they obey the laws and do not hundred and sixty acres by the relator out of the lands ceded to the United States by
trespass on forbidden ground. And, the Wichita and affiliated bands of Indians. Section 463 of the United States Revised
Statutes provided: "The Commissioner of Indian Affairs shall, under the direction of
5. Being restrained of liberty under color of authority of the United States, the Secretary of the Interior, and agreeably to such regulations as the President may
and in violation of the laws thereof, the relators must be discharged from prescribe, have the management of all Indian affairs, and of all matters arising out
custody, and it is so ordered. to the Indian relations." Justice Holmes said: "We should hesitate a good deal,
especially in view of the long established practice of the Department, before saying
As far as the first point is concerned, the decision just quoted could be used as that this language was not broad enough to warrant a regulation obviously made for
authority to determine that Rubi, the Manguian petitioner, a Filipino, and a citizen of the welfare of the rather helpless people concerned. The power of Congress is not
the Philippine Islands, is a "person" within the meaning of theHabeas Corpus Act, doubted. The Indians have been treated as wards of the nation. Some such
and as such, entitled to sue out a writ in the Philippine courts. (See also In re Race supervision was necessary, and has been exercised. In the absence of special
Horse [1895], 70 Fed., 598.) We so decide. provisions naturally it would be exercised by the Indian Department." (See also as
As to the second point the facts in the Standing Bear case an the Rubi case are not corroborative authority, it any is needed, Union Bridge Co. vs. U.S. [1907], 204 U.S..,
exactly identical. But even admitting similarity of facts, yet it is known to all that 364, reviewing the previous decisions of the United States Supreme Court:
Indian reservations do exist in the United States, that Indians have been taken from U.S. vs. Lane [1914], 232 U.S., 598.)
different parts of the country and placed on these reservation, without any previous There is another aspect of the question, which once accepted, is decisive. An
consultation as to their own wishes, and that, when once so located, they have been exception to the general rule. sanctioned by immemorial practice, permits the
made to remain on the reservation for their own good and for the general good of central legislative body to delegate legislative powers to local authorities. The
the country. If any lesson can be drawn form the Indian policy of the United States, it Philippine Legislature has here conferred authority upon the Province of Mindoro, to
is that the determination of this policy is for the legislative and executive branches be exercised by the provincial governor and the provincial board.
of the government and that when once so decided upon, the courts should not
interfere to upset a carefully planned governmental system. Perhaps, just as may

19
Who but the provincial governor and the provincial board, as the official Liberty consists in the ability to do what one caught to desire and in not
representatives of the province, are better qualified to judge "when such as course is being forced to do what one ought not do desire. (Montesque, spirit of the
deemed necessary in the interest of law and order?" As officials charged with the Laws.)
administration of the province and the protection of its inhabitants, who but they are
better fitted to select sites which have the conditions most favorable for improving Even liberty itself, the greatest of all rights, is no unrestricted license to ac
the people who have the misfortune of being in a backward state? according to one's own will. It is only freedom from restraint under
conditions essential to the equal enjoyment of the same right by others.
Section 2145 of the Administrative Code of 1917 is not an unlawful delegation of (Field, J., in Crowley vs. Christensen [1890], 137 U.S., 86.)
legislative power by the Philippine Legislature to provincial official and a department
head. Liberty does not import "an absolute right in each person to be, at all times
and in all circumstances, wholly freed from restraint. There are manifold
B. RELIGIOUS DISCRIMINATION restraints to which every person is necessarily subject for the common
good. On any other basis, organized society could not exist with safety to
The attorney de officio, for petitioners, in a truly remarkable brief, submitted on its members. Society based on the rule that each one is a law unto himself
behalf of his unknown clients, says that "The statute is perfectly clear and would soon be confronted with disorder and anarchy. Real liberty for all
unambiguous. In limpid English, and in words as plain and unequivocal as language could not exist under the operation of a principle which recognizes the right
can express, it provides for the segregation of 'non-Christians' and none other." The of each individual person to use his own, whether in respect of his person or
inevitable result, them, is that the law "constitutes an attempt by the Legislature to his property, regardless of the injury that may be done to others . . . There
discriminate between individuals because of their religious beliefs, and is, is, of course, a sphere with which the individual may asserts the supremacy
consequently, unconstitutional." of his own will, and rightfully dispute the authority of any human
Counsel's premise once being conceded, his arguments is answerable the government especially of any free government existing under a written
Legislature must be understood to mean what it has plainly expressed; judicial Constitution to interfere with the exercise of that will. But it is equally
construction is then excluded; religious equality is demanded by the Organic Law; true that in very well-ordered society charged with the duty of conserving
the statute has violated this constitutional guaranty, and Q. E. D. is invalid. But, as the safety of its members, the rights of the individual in respect of his
hereinbefore stated, we do not feel free to discard the long continued meaning given liberty may at times, under the pressure of great dangers, be subjected to
to a common expression, especially as classification of inhabitants according to such restraint to be enforced by reasonable regulations, as the safety of the
religious belief leads the court to what it should avoid, the nullification of legislative general public may demand." (Harlan, J., In Jacobson vs. Massachusetts
action. We hold that the term "non-Christian" refers to natives of the Philippines [1905] 197 U.S., 11.)
Islands of a low grade of civilization, and that section 2145 of the Administrative Liberty is freedom to do right and never wrong; it is ever guided by reason
Code of 1917, does not discriminate between individuals an account of religious and the upright and honorable conscience of the individual. (Apolinario
differences. Mabini.)
C. LIBERTY; DUE PROCESS OF LAW; EQUAL PROTECTION OF THE LAWS. Civil Liberty may be said to mean that measure of freedom which may be enjoyed in
The third constitutional argument is grounded on those portions of the President's a civilized community, consistently with the peaceful enjoyment of like freedom in
instructions of to the Commission, the Philippine Bill, and the Jones Law, providing others. The right to Liberty guaranteed by the Constitution includes the right to exist
"That no law shall be enacted in said Islands which shall deprive any person of life, and the right to be free from arbitrary personal restraint or servitude. The term
liberty, or property without due process of law, or deny to any person therein the cannot be dwarfed into mere freedom from physical restraint of the person of the
equal protection of the laws." This constitutional limitation is derived from the citizen, but is deemed to embrace the right of man to enjoy the faculties with which
Fourteenth Amendment to the United States Constitution and these provisions, it he has been endowed by this Creator, subject only to such restraints as are
has been said "are universal in their application, to all persons within the territorial necessary for the common welfare. As enunciated in a long array of authorities
jurisdiction, without regard to any differences of race, of color, or of nationality." including epoch-making decisions of the United States Supreme Court, Liberty
(Yick Wo vs. Hopkins [1886], 118 U.S., 356.) The protection afforded the individual is includes the right of the citizens to be free to use his faculties in all lawful ways; to
then as much for the non-Christian as for the Christian. live an work where he will; to earn his livelihood by an lawful calling; to pursue any
avocations, an for that purpose. to enter into all contracts which may be proper,
The conception of civil liberty has been variously expressed thus: necessary, and essential to his carrying out these purposes to a successful
conclusion. The chief elements of the guaranty are the right to contract, the right to
Every man may claim the fullest liberty to exercise his faculties, compatible choose one's employment, the right to labor, and the right of locomotion.
with the possession of like liberty by every other. (Spencer, Social Statistics,
p. 94.) In general, it may be said that Liberty means the opportunity to do those things
which are ordinarily done by free men. (There can be noted Cummings vs. Missouri
Liberty is the creature of law, essentially different from that authorized
[1866], 4 Wall, 277; Wilkinson vs. Leland [1829], 2 Pet., 627; Williams vs. Fears
licentiousness that trespasses on right. That authorized licentiousness that [1900], 179 U.S., 274; Allgeyer vs. Louisiana [1896], 165, U.S., 578;
trespasses on right. It is a legal and a refined idea, the offspring of high
State vs. Kreutzberg [1902], 114 Wis., 530. See 6 R.C.L., 258, 261.)
civilization, which the savage never understood, and never can understand.
Liberty exists in proportion to wholesome restraint; the more restraint on One thought which runs through all these different conceptions of Liberty is plainly
others to keep off from us, the more liberty we have . . . that man is free apparent. It is this: "Liberty" as understood in democracies, is not license; it is
who is protected from injury. (II Webster's Works, p. 393.) "Liberty regulated by law." Implied in the term is restraint by law for the good of the
individual and for the greater good of the peace and order of society and the general

20
well-being. No man can do exactly as he pleases. Every man must renounce So much for an analysis of those constitutional provisions on which petitioners rely
unbridled license. The right of the individual is necessarily subject to reasonable for their freedom. Next must come a description of the police power under which the
restraint by general law for the common good. Whenever and wherever the natural State must act if section 2145 is to be held valid.
rights of citizen would, if exercises without restraint, deprive other citizens of rights
which are also and equally natural, such assumed rights must yield to the regulation E. THE POLICE POWER.
of law. The Liberty of the citizens may be restrained in the interest of the public Not attempting to phrase a definition of police power, all that it is necessary to note
health, or of the public order and safety, or otherwise within the proper scope of the at this moment is the farreaching scope of the power, that it has become almost
police power. (See Hall vs. Geiger-Jones [1916], 242 U.S., 539; Hardie-Tynes possible to limit its weep, and that among its purposes is the power to prescribe
Manufacturing Co. vs. Cruz [1914], 189 Al., 66.) regulations to promote the health, peace, morals, education, and good order of the
None of the rights of the citizen can be taken away except by due process of law. people, and to legislate so as to increase the industries of the State, develop its
Daniel Webster, in the course of the argument in the Dartmouth College Case before resources and add to is wealth and prosperity. (See Barbiervs. Connolly [1884], 113
the United States Supreme Court, since a classic in forensic literature, said that the U.S., 27.) What we are not interested in is the right of the government to restrain
meaning of "due process of law" is, that "every citizen shall hold his life, liberty, liberty by the exercise of the police power.
property, an immunities under the protection of the general rules which govern "The police power of the State," one court has said, . . . "is a power coextensive with
society." To constitute "due process of law," as has been often held, a judicial self-protection, and is not inaptly termed the 'law of overruling necessity.' It may be
proceeding is not always necessary. In some instances, even a hearing and notice said to be that inherent and plenary power in the State which enables it to prohibit
are not requisite a rule which is especially true where much must be left to the all things hurtful to the comfort, safety and welfare of society." (Lake View vs. Rose
discretion of the administrative officers in applying a law to particular cases. (See Hill Cemetery Co. [1873], 70 Ill., 191.) Carried onward by the current of legislation,
McGehee, Due Process of Law, p. 371.) Neither is due process a stationary and blind the judiciary rarely attempt to dam the on rushing power of legislative discretion,
sentinel of liberty. "Any legal proceeding enforced by public authority, whether provided the purposes of the law do not go beyond the great principles that mean
sanctioned by age and customs, or newly devised in the discretion of the legislative security for the public welfare or do not arbitrarily interfere with the right of the
power, in furtherance of the public good, which regards and preserves these individual.
principles of liberty and justice, must be held to be due process of law."
(Hurtado vs.California [1883], 110, U.S., 516.) "Due process of law" means The Government of the Philippine Islands has both on reason and authority the right
simply . . . "first, that there shall be a law prescribed in harmony with the general to exercise the sovereign police power in the promotion of the general welfare and
powers of the legislative department of the Government; second, that this law shall the public interest. "There can be not doubt that the exercise of the police power of
be reasonable in its operation; third, that it shall be enforced according to the the Philippine Government belongs to the Legislature and that this power is limited
regular methods of procedure prescribed; and fourth, that it shall be applicable alike only by the Acts of Congress and those fundamental principles which lie at the
to all the citizens of the state or to all of a class." (U.S. vs. Ling Su Fan [1908], 10 foundation of all republican forms of government." (Churchill and Tait vs. Rafferty
Phil., 104, affirmed on appeal to the United States Supreme Court. 1) "What is due [1915], 32 Phil., 580; U.S. vs. Pompeya [1915], 31 Phil., 245.)
process of law depends on circumstances. It varies with the subject-matter and
With the foregoing approximation of the applicable basic principles before us, before
necessities of the situation." (Moyer vs. Peablody [1909], 212 U. S., 82.)
finally deciding whether any constitutional provision has indeed been violated by
The pledge that no person shall be denied the equal protection of the laws is not section 2145 of the Administrative Code, we should endeavor to ascertain the
infringed by a statute which is applicable to all of a class. The classification must intention of the Legislature in enacting this section. If legally possible, such
have a reasonable basis and cannot be purely arbitrary in nature. legislative intention should be effectuated.

We break off with the foregoing statement, leaving the logical deductions to be F. LEGISLATIVE INTENT.
made later on.
The preamble of the resolution of the provincial board of Mindoro which set apart the
D. SLAVERY AND INVOLUNTARY SERVITUDE. Tigbao reservation, it will be remembered, assigned as reasons fort the action, the
following: (1) The failure of former attempts for the advancement of the non-
The fourth constitutional contention of petitioner relates to the Thirteen Amendment Christian people of the province; and (2) the only successfully method for educating
to the United States Constitution particularly as found in those portions of Philippine the Manguianes was to oblige them to live in a permanent settlement. The Solicitor-
Organic Law providing "That slavery shall not exist in said Islands; nor shall General adds the following; (3) The protection of the Manguianes; (4) the protection
involuntary servitude exist except as a punishment for crime whereof the party shall of the public forests in which they roam; (5) the necessity of introducing civilized
have been duly convicted." It is quite possible that the Thirteenth Amendment, since customs among the Manguianes.
reaching to "any place subject to" the "jurisdiction" of the United States, has force in
the Philippine. However this may be, the Philippine Legislature has, by adoption, The present Secretary of the Interior says of the Tigbao reservation and of the
with necessary modifications, of sections 268 to 271 inclusive of the United States motives for its selection, the following:
Criminal Code, prescribed the punishment for these crimes. Slavery and involuntary
To inform himself of the conditions of those Manguianes who were taken
servitude, together wit their corollary, peonage, all denote "a condition of enforced,
together to Tigbao, the Secretary of the Interior on June 10 to 13, 1918,
compulsory service of one to another." (Hodges vs. U.S. [1906], 203 U.S., 1.) The
made a trip to the place. There he found that the site selected is a good
term of broadest scope is possibly involuntary servitude. It has been applied to any
one; that creditable progress has been made in the clearing of forests,
servitude in fact involuntary, no matter under what form such servitude may have
construction of buildings, etc., that there appears to be encouraging
been disguised. (Bailey vs. Alabama [1910], 219 U.S., 219.)
reaction by the boys to the work of the school the requirements of which
they appear to meet with enthusiastic interest after the first weeks which

21
are necessarily a somewhat trying period for children wholly unaccustomed civilized inhabitants of the country. (See Report of the Department for
to orderly behaviour and habit of life. He also gathered the impression that 1917.)
the results obtained during the period of less than one year since the
beginning of the institution definitely justify its continuance and The fundamental objective of governmental policy is to establish friendly relations
development. with the so-called non-Christians, and to promote their educational, agricultural,
industrial, and economic development and advancement in civilization. (Note Acts
Of course, there were many who were protesting against that segregation. Nos. 2208, 2404, 2444.) Act No. 2674 in reestablishing the Bureau of non-Christian
Such was naturally to be expected. But the Secretary of the Interior, upon Tribes, defines the aim of the Government towards the non-Christian people in the
his return to Manila, made the following statement to the press: following unequivocal terms:
"It is not deemed wise to abandon the present policy over those It shall be the duty of the Bureau of non-Christian Tribes to continue the
who prefer to live a nomadic life and evade the influence of work for advancement and liberty in favor of the region inhabited by non-
civilization. The Government will follow its policy to organize them Christian Filipinos and foster by all adequate means and in a systematical,
into political communities and to educate their children with the rapid, and complete manner the moral, material, economic, social, and
object of making them useful citizens of this country. To permit political development of those regions, always having in view the aim of
them to live a wayfaring life will ultimately result in a burden to rendering permanent the mutual intelligence between, and complete fusion
the state and on account of their ignorance, they will commit of, all the Christian and non-Christian elements populating the provinces of
crimes and make depredation, or if not they will be subject to the Archipelago. (Sec. 3.)
involuntary servitude by those who may want to abuse them."
May the Manguianes not be considered, as are the Indians in the United States,
The Secretary of the Interior, who is the official charged with the supervision of all proper wards of the Filipino people? By the fostering care of a wise Government,
the non-Christian people, has adopted as the polaris of his administration "the may not these unfortunates advance in the "habits and arts of civilization?" Would it
advancement of the non-Christian elements of our population to equality and be advisable for the courts to intrude upon a plan, carefully formulated, and
unification with the highly civilized Christian inhabitants." This is carried on by the apparently working out for the ultimate good of these people?
adoption of the following measures:
In so far as the Manguianes themselves are concerned, the purpose of the
(a) Pursuance of the closer settlement policy whereby people of Government is evident. Here, we have on the Island of Mindoro, the Manguianes,
seminomadic race are induced to leave their wild habitat and settle in leading a nomadic life, making depredations on their more fortunate neighbors,
organized communities. uneducated in the ways of civilization, and doing nothing for the advancement of the
Philippine Islands. What the Government wished to do by bringing than into a
(b) The extension of the public school system and the system of public reservation was to gather together the children for educational purposes, and to
health throughout the regions inhabited by the non-Christian people. improve the health and morals was in fine, to begin the process of civilization. this
(c) The extention of public works throughout the Mohammedan regions to method was termed in Spanish times, "bringing under the bells." The same idea
facilitate their development and the extention of government control. adapted to the existing situation, has been followed with reference to the
Manguianes and other peoples of the same class, because it required, if they are to
(d) Construction of roads and trials between one place and another among be improved, that they be gathered together. On these few reservations there live
non-Christians, to promote social and commercial intercourse and maintain under restraint in some cases, and in other instances voluntarily, a few thousands of
amicable relations among them and with the Christian people. the uncivilized people. Segregation really constitutes protection for the manguianes.
(e) Pursuance of the development of natural economic resources, especially Theoretically, one may assert that all men are created free and equal. Practically, we
agriculture. know that the axiom is not precisely accurate. The Manguianes, for instance, are not
free, as civilized men are free, and they are not the equals of their more fortunate
( f ) The encouragement of immigration into, and of the investment of brothers. True, indeed, they are citizens, with many but not all the rights which
private capital in, the fertile regions of Mindanao and Sulu.
citizenship implies. And true, indeed, they are Filipinos. But just as surely, the
The Secretary adds: Manguianes are citizens of a low degree of intelligence, and Filipinos who are a drag
upon the progress of the State.
To attain the end desired, work of a civilizing influence have been continued
among the non-Christian people. These people are being taught and guided In so far as the relation of the Manguianes to the State is concerned, the purposes of
to improve their living conditions in order that they may fully appreciate the the Legislature in enacting the law, and of the executive branch in enforcing it, are
benefits of civilization. Those of them who are still given to nomadic habits again plain. Settlers in Mindoro must have their crops and persons protected from
are being persuaded to abandon their wild habitat and settle in organized predatory men, or they will leave the country. It is no argument to say that such
settlements. They are being made to understand that it is the purpose of crimes are punished by the Penal Code, because these penalties are imposed after
the Government to organize them politically into fixed and per manent commission of the offense and not before. If immigrants are to be encouraged to
communities, thus bringing them under the control of the Government, to develop the resources of the great Islands of Mindoro, and its, as yet, unproductive
aid them to live and work, protect them from involuntary servitude and regions, the Government must be in a position to guarantee peace and order.
abuse, educate their children, and show them the advantages of leading a Waste lands do not produce wealth. Waste people do not advance the interest of the
civilized life with their civilized brothers. In short, they are being impressed
State. Illiteracy and thriftlessness are not conducive to homogeneity. The State to
with the purposes and objectives of the Government of leading them to protect itself from destruction must prod on the laggard and the sluggard. The great
economic, social, and political equality, and unification with the more highly
law of overwhelming necessity is all convincing.

22
To quote again from the instructive memorandum of the Secretary of the Interior: Constitutional guaranty at this time will leave the Government without
recourse to pursue the works of civilizing them and making them useful
Living a nomadic and a wayfaring life and evading the influence of citizens. They will thus left in a permanent state of savagery and become a
civilization, they (the manguianes) are engaged in the works of destruction vulnerable point to attack by those who doubt, nay challenge, the ability of
burning and destroying the forests and making illegal caigins thereon. the nation to deal with our backward brothers.
Not bringing any benefit to the State but instead injuring and damaging its
interests, what will ultimately become of these people with the sort of The manguianes in question have been directed to live together at Tigbao.
liberty they wish to preserve and for which they are now fighting in court? There they are being taught and guided to improve their living conditions.
They will ultimately become a heavy burden to the State and on account of They are being made to understand that they object of the government is
their ignorance they will commit crimes and make depredations, or if not to organize them politically into fixed and permanent communities. They
they will be subjected to involuntary servitude by those who may want to are being aided to live and work. Their children are being educated in a
abuse them. school especially established for them. In short, everything is being done
from them in order that their advancement in civilization and material
There is no doubt in my mind that this people a right conception of liberty prosperity may be assured. Certainly their living together in Tigbao does
and does not practice liberty in a rightful way. They understand liberty as not make them slaves or put them in a condition compelled to do services
the right to do anything they will going from one place to another in the for another. They do not work for anybody but for themselves. There is,
mountains, burning and destroying forests and making illegal caigins therefore, no involuntary servitude.
thereon.
But they are compelled to live there and prohibited from emigrating to
Not knowing what true liberty is and not practising the same rightfully, how some other places under penalty of imprisonment. Attention in this
can they allege that they are being deprived thereof without due process of connection is invited to the fact that this people, living a nomadic and
law? wayfaring life, do not have permanent individual property. They move from
xxx xxx xxx one place to another as the conditions of living warrants, and the entire
space where they are roving about is the property of the nation, the greater
But does the Constitutional guaranty that 'no person shall be deprived of part being lands of public domain. Wandering from one place to another on
his liberty without due process of law' apply to a class of persons who do the public lands, why can not the government adopt a measure to
not have a correct idea of what liberty is and do not practise liberty in a concentrate them in a certain fixed place on the public lands, instead of
rightful way? permitting them to roam all over the entire territory? This measure is
necessary both in the interest of the public as owner of the lands about
To say that it does will mean to sanction and defend an erroneous idea of which they are roving and for the proper accomplishment of the purposes
such class of persons as to what liberty is. It will mean, in the case at bar,
and objectives of the government. For as people accustomed to nomadic
that the Government should not adopt any measures looking to the welfare habit, they will always long to return to the mountains and follow a
and advancement of the class of persons in question. It will mean that this
wayfaring life, and unless a penalty is provinced for, you can not make
people should be let along in the mountains and in a permanent state of them live together and the noble intention of the Government of organizing
savagery without even the remotest hope of coming to understand liberty
them politically will come to naught.
in its true and noble sense.
G. APPLICATION AND CONCLUSION.
In dealing with the backward population, like the Manguianes, the
Government has been placed in the alternative of either letting them alone Our exhaustive study should have left us in a position to answer specific objections
or guiding them in the path of civilization. The latter measure was adopted and to reach a general conclusion.
as the one more in accord with humanity and with national conscience.
In the first place, it is argued that the citizen has the right, generally speaking, to go
xxx xxx xxx where he pleases. Could be not, however, be kept away from certain localities ? To
furnish an example from the Indian legislation. The early Act of Congress of 1802 (2
The national legislation on the subject of non-Christian people has tended U.S. Stat. at L., p. 141) Indian reservation. Those citizens certainly did not possess
more and more towards the education and civilization of such people and
absolute freedom of locomotion. Again the same law provided for the apprehension
fitting them to be citizens. The progress of those people under the tutelage of marauding Indians. Without any doubt, this law and other similar were accepted
of the Government is indeed encouraging and the signs of the times point
and followed time and again without question.
to a day which is not far distant when they will become useful citizens. In
the light of what has already been accomplished which has been winning It is said that, if we hold this section to be constitutional, we leave this weak and
the gratitude of most of the backward people, shall we give up the noble defenseless people confined as in a prison at the mercy of unscrupulous official.
work simply because a certain element, believing that their personal What, it is asked, would be the remedy of any oppressed Manguian? The answer
interests would be injured by such a measure has come forward and would naturally be that the official into whose hands are given the enforcement of
challenged the authority of the Government to lead this people in the pat of the law would have little or not motive to oppress these people; on the contrary, the
civilization? Shall we, after expending sweat, treasure, and even blood only presumption would all be that they would endeavor to carry out the purposes of the
to redeem this people from the claws of ignorance and superstition, now law intelligently and patriotically. If, indeed, they did ill-treat any person thus
willingly retire because there has been erroneously invoked in their favor confined, there always exists the power of removal in the hands of superior officers,
that Constitutional guaranty that no person shall be deprived of his liberty and the courts are always open for a redress of grievances. When, however, only the
without due process of law? To allow them to successfully invoke that validity of the law is generally challenged and no particular case of oppression is

23
called to the attention of the courts, it would seems that the Judiciary should not benefit the Filipino people as a whole. The Manguianes, in order to fulfill this
unnecessarily hamper the Government in the accomplishment of its laudable governmental policy, must be confined for a time, as we have said, for their own
purpose. good and the good of the country.
The question is above all one of sociology. How far, consistently with freedom, may Most cautiously should the power of this court to overrule the judgment of the
the right and liberties of the individual members of society be subordinated to the Philippine Legislature, a coordinate branch, be exercised. The whole tendency of the
will of the Government? It is a question which has assailed the very existence of best considered case is toward non-interference on the part of the courts whenever
government from the beginning of time. Now purely an ethical or philosophical political ideas are the moving consideration. Justice Holmes, in one of the aphorisms
subject, nor now to be decided by force, it has been transferred to the peaceful for which he is justly famous, said that "constitutional law, like other mortal
forum of the Judiciary. In resolving such an issue, the Judiciary must realize that the contrivances, has to take some chances." (Blinn vs.Nelson [1911], 222 U.S., 1.) If in
very existence of government renders imperatives a power to restrain the individual the final decision of the many grave questions which this case presents, the courts
to some extent, dependent, of course, on the necessities of the class attempted to must take "a chance," it should be with a view to upholding the law, with a view to
be benefited. As to the particular degree to which the Legislature and the Executive the effectuation of the general governmental policy, and with a view to the court's
can go in interfering with the rights of the citizen, this is, and for a along time to performing its duty in no narrow and bigoted sense, but with that broad conception
come will be, impossible for the courts to determine. which will make the courts as progressive and effective a force as are the other
departments of the Government.
The doctrines of laissez faire and of unrestricted freedom of the individual, as
axioms of economics and political theory, are of the past. The modern period has We are of the opinion that action pursuant to section 2145 of the Administrative
shown as widespread belief in the amplest possible demonstration of governmental Code does not deprive a person of his liberty without due process of law and does
activity. The courts unfortunately have sometimes seemed to trial after the other not deny to him the equal protection of the laws, and that confinement in
two branches of the government in this progressive march. reservations in accordance with said section does not constitute slavery and
involuntary servitude. We are further of the opinion that section 2145 of the
Considered, therefore, purely as an exercise of the police power, the courts cannot Administrative Code is a legitimate exertion of the police power, somewhat
fairly say that the Legislature has exceeded its rightful authority. it is, indeed, an analogous to the Indian policy of the United States. Section 2145 of the
unusual exercise of that power. But a great malady requires an equally drastic Administrative Code of 1917 is constitutional.
remedy.
Petitioners are not unlawfully imprisoned or restrained of their liberty. Habeas corpus
Further, one cannot hold that the liberty of the citizen is unduly interfered without can, therefore, not issue. This is the true ruling of the court. Costs shall be taxes
when the degree of civilization of the Manguianes is considered. They are restrained against petitioners. So ordered.
for their own good and the general good of the Philippines. Nor can one say that due
process of law has not been followed. To go back to our definition of due process of Arellano, C.J., Torres and Avancea, JJ., concur.
law and equal protection of the law, there exists a law ; the law seems to be
reasonable; it is enforced according to the regular methods of procedure prescribed;
and it applies alike to all of a class.
As a point which has been left for the end of this decision and which, in case of
doubt, would lead to the determination that section 2145 is valid. it the attitude
which the courts should assume towards the settled policy of the Government. In a
late decision with which we are in full accord, Gambles vs. Vanderbilt University (200
Southwestern Reporter, 510) the Chief Justice of the Supreme Court of Tennessee
writes:
We can seen objection to the application of public policy as a ratio decidendi. Every
really new question that comes before the courts is, in the last analysis, determined
on that theory, when not determined by differentiation of the principle of a prior
case or line of cases, or by the aid of analogies furnished by such prior case. In
balancing conflicting solutions, that one is perceived to tip the scales which the
court believes will best promote the public welfare in its probable operation as a
general rule or principle. But public policy is not a thing inflexible. No court is wise
enough to forecast its influence in all possible contingencies. Distinctions must be
made from time to time as sound reason and a true sense of justice may dictate."
Our attempt at giving a brief history of the Philippines with reference to the so-called
non-Christians has been in vain, if we fail to realize that a consistent governmental
policy has been effective in the Philippines from early days to the present. The idea
to unify the people of the Philippines so that they may approach the highest
conception of nationality. If all are to be equal before the law, all must be
approximately equal in intelligence. If the Philippines is to be a rich and powerful
country, Mindoro must be populated, and its fertile regions must be developed. The
public policy of the Government of the Philippine Islands is shaped with a view to

24
CRESPO VS. PROVINCIAL BOARD The contention of the Provincial Board cannot stand alone in the absence of proof or
evidence to support it. Moreover, in the proceedings held on 15 February 1971,
G.R. No. L-33237 April 15, 1988 nothing therein can be gathered that, in issuing the assailed order, the written
explanation submitted by petitioner was taken into account. The assailed order was
GREGORIO T. CRESPO, in His Capacity as Mayor of Cabiao, Nueva issued mainly on the basis of the evidence presented ex parte by respondent
Ecija, petitioner, Wycoco.
vs. 9
PROVINCIAL BOARD OF NUEVA ECIJA and PEDRO T. WYCOCO, respondents. In Azul vs. Castro, this Court said:

PADILIA, J.: From the earliest inception of instutional government in our country, the
concepts of notice and hearing have been fundamental. A fair and
Petitioner was the elected Municipal Mayor of Cabiao, Nueva Ecija, in the local enlightened system of justice would be impossible without the right to
elections of 1967. On 25 January 1971, an administrative complaint was filed against notice and to be board. The emphasis on substantive due process and other
him by private respondent, Pedro T. Wycoco for harassment, abuse of authority and recent ramifications of the due process clause sometimes leads bench and
oppression. 1 As required, petitioner filed a written explanation as to why he should bar to overlook or forget that due process was initially concerned with fair
not be dealt with administrdatively, with the Provincial Board of Nueve Ecija, in procedure. Every law student early learns in law school definition submitted
accordance with Section 5, Republic Act No. 5185. 2 by counsel Mr. Webster in Trustees of Dartmouth College v. Woodward (4
Wheat. 518) that due process is the equivalent of law of the land which
On 15 February 1971, without notifying petitioner or his counsel, public respondent means "The general law; a law which hears before it condemns, which
Provincial Board conducted a hearing of the aforecited administrative case. During proceeding upon inquiry and renders judgment only after trial ... that every
the hearing, private respondent Pedro T. Wycoco was allowed to present evidence, citizen shall hold his life, liberty, property, and immunities under the
testimonial and documentary, ex parte, and on the basis of the evidence presented, protection of the general rules which govern society.
the respondent Provincial Board passed Resolution No. 51 preventively suspending
petitioner from his office as municipal mayor of Cabiao, Nueva Ecija. 3 A sporting opportunity to be heard and the rendition of judgment only after
a lawful hearing by a coldly neutral and impartial judge are essential
In this petition for certiorari, prohibition and injunction with prayer for preliminary elements of procedural due process.
injunction, petitioner seeks to annul and set aside Resolution No. 51 of public
respondent Provincial Board, preventively suspending him from office and to enjoin We had occasion to emphasize in Santiago v. Santos (63 SCRA 392), which,
public respondent from enforcing and/or implementing the order of preventive unlike the case before us now, was only a summary action for ejectment
suspension and from proceeding further with the administrative case. that:
According to petitioner, the order of preventive suspension embodied in Resolution In an adversary proceeding, fairness and prudence dictate that a
No. 51 issued by the Provincial Board is arbitrary, high-handed, atrocious, shocking judgment, based only on plaintiffs evidence adduced ex parte and
and grossly violative of Section 5 of Republic Act No. 5185 which requires a hearing rendered without hearing defendant's evidence, should be avoided
and investigation of the truth or falsity of charges before preventive suspension is as much as possible. In order that bias may not be imputed to the
allowed. In issuing the order of preventive suspension, the respondent Provincial judge, he should have the patience and circumspection to give the
Board, petitioner adds, has grossly violated the fundamental and elementary opposing party a chance to present his evidence even if he thinks
principles of due process. 4 that the oppositor's proof might not be adequate to overthrow the
case for the plaintiff. A display of petulance and impatience in the
On 3 May 1971, this Court issued a preliminary injunction. 5 We agree with the conduct of the trial is a norm of conduct which is inconsistent with
petitioner that he was denied due process by respondent Provincial Board. the "cold neutrality of an impartial judge". 10
6
In Callanta vs. Carnation Philippines, Inc. this Court held: The petition, however, has become moot and academic. Records do not show that in
It is a principle in American jurisprudence which, undoubtedly, is well- the last local elections held on 18 January 1988, petitioner was elected to any public
recognized in this jurisdiction that one's employment, profession, trade or office.
calling is a "property right," and the wrongful interference therewith is an WHEREFORE, the petition is DISMISSED. The preliminary injunction issued by this
actionable wrong. The right is considered to be property within the Court on 3 May 1971 is LIFTED. No costs.
protection of a constitutional guaranty of due process of law. 7
SO ORDERED.
Undoubtedly, the order of preventive suspension was issued without giving the
petitioner a chance to be heard. To controvert the claim of petitioner that he was not Yap, Melencio-Herrera, Paras and Sarmiento, JJ., concur.
fully notified of the scheduled hearing, respondent Provincial Board, in its
Memorandum, contends that "Atty. Bernardo M. Abesamis, counsel for the petitioner
mayor made known by a request in writing, sent to the Secretary of the Provincial
Board his desire to be given opportunity to argue the explanation of the said
petitioner mayor at the usual time of the respondent Board's meeting, but
unfortunately, inspire of the time allowed for the counsel for the petitioner mayor to
appear as requested by him, he failed to appeal." 8

25
PEDRO VS. PROVINCIAL BOARD OF RIZAL or obstructing the operation and exploitation of the Bighani cockpit, which at that
time was completed and ready to be thrown open to the public.
G.R. No. 34163 September 18, 1931 On August 26, 1927, the Court of first Instance of Manila rendered judgment
GREGORIO PEDRO, petitioner-appellant, absolving the defendants from the complaint, which was affirmed by this court on
vs. October 15, 1928. (Company "Bighani" vs. 53 Phil., 886.)
THE PROVINCIAL BOARD OF RIZAL, ET AL., respondents-appellees. On September 18, 1927, the municipal council of Caloocan enacted Ordinance No.
VILLA-REAL, J.: 34, providing in the first section, among other things, that outside the barrios of
Loma, Talipapa, and Novaliches, where only one cockpit might be established,
This case is before us by virtue of the appeal taken by the petitioner Gregorio Pedro cockpits might be established at a distance of not less than 1,500 meters from
from the judgment of the Court of First Instance of Rizal dismissing his action for the another licensed cockpit, public schoolhouse, or any hospital or charitable institution
annulment of an ordinance, with costs against him. existing within the municipal radius.
In support of his appeal, the appellant assigns the following alleged errors as As a result of the general election held on June 5, 1928, in the municipality of
committed by the trial court in its judgment, to wit: Caloocan, Rizal, the municipal council, formerly comprising Pablo Pablo, as
president, Blas Bernardino, as vice-president, and Severino Paganiban, Diego Justo,
1. The lower court erred in holding that Ordinance No. 36, series of 1928, Esteban Sanchez, Patricio Galuran, Raymundo Andres, Emiliano Samson, Vicente
approved by the acting councilors, is valid and legal. Sevilla, Lucas Pascual, Placido C. del Mundo, Delfin Rodriguez, Jorge Nadurata,
2. The lower court erred in denying the petitioner an acquired right, Anacleto Victoria, Emilio Acab, and Mateo Austria, as councilors, was substituted by
notwithstanding Ordinance No. 35 and the permit giving him by the another comprising the newly elected Dominador Aquino, as president, Diego Justo,
president in accordance therewith. as vice-president, and Blas Bernardino, Flaviano de Jesus, Pedro Galang, Celestino C.
Celosa, Nicolas Carpio, Lucas Pascual, Basilio Biglang-awa, and Lucas Bustamante,
3. The lower court erred in holding that the opening, maintenance, and as councilors, who were inducted into office on October 16th of that year.
operation of the Galas cockpit is injurious to the consumptive patients of
the Santol Sanatorium. On December 21, 1928, the plaintiff herein, Gregorio Pedro, acquired by absolute
sale all the rights and interests of the "Sociedad Bighani" in the cockpit bearing its
4. The lower court erred in abstaining from making any ruling regarding the name. (Exhibit M.)
legality of the action taken by the provincial board, suspending the effects
of Ordinance No. 35 of the municipal council of Caloocan, and in finally On the same date, December 21, 1928, said plaintiff, Gregorio Pedro, addressed a
disapproving it, according to the resolutions enacted by it and numbered communication to the municipal council of Caloocan soliciting a permit to open,
1135, series of 1928, and 154, series of 1929. operate, maintain, and exploit said cockpit for a period of four years, binding himself
to observe to the letter all municipal ordinances on cockpits. (Exhibit A.)
5. The lower court erred in dismissing this case and in not declaring
permanent the injunction sought, and in not sentencing the plaintiffs On December 26, 1928, the municipal council of Caloocan passed resolution No. 202
[respondents] jointly and severally to pay the damages claimed in the approving Ordinance No. 35, series of 1928, amending section 1 of Ordinance No.
complaint. 34, series of 1927, providing, among other things that only one cockpit could be
established in each of the barrios of Galas, Loma, Talipapa, and Novaliches, and any
The following relevant facts are necessary for the decision of the question raised by other place outside said barrios, provided, in the latter case, said cockpits are at a
the instant appeal: distance of not less than 1,000 meters from another licensed cockpit, and 500
meters from any hospital or charitable institution within the municipality of
On May 8, 1926, there was organized in the municipality of Caloocan, Province of Caloocan. (Exhibit C.)
Rizal, an association for the construction and exploitation of cockpits, called "La
Sociedad Bighani." On the same date, December 26, 1928, the municipal councilors of Caloocan, Blas
Bernardino, Flaviano de Jesus, and Pedro Galang, signed and forwarded to the
On May 22, 1926, Eugenio Tansioco, the president of the association, applied to the provincial governor of Rizal an accusation against Dominador Aquino, the municipal
municipal president of Caloocan and obtain a permit to construct a building of strong president, and the other councilors who approved Ordinance No. 35, series of 1928,
materials at Galas, in said municipality, to be used as cockpit, upon payment of the alleging that they had been bribed to vote in favor of that ordinance. (Exhibit 4.)
proper fees. (Exhibit 1.)
The provincial governor endorsed the accusation to the provincial board of Rizal,
While the construction was under way, Pablo, then president of Caloocan, addressed which through resolution No. 1110 dated December 27, 1928, ordered the
a communication to Eugenio Tansioco on June 15, 1926, warning him that the site of temporary suspension of the members denounced pending the administrative
the building was not the one designated by the chief of police, and that it was within investigation of the accusation. By virtue of said resolution No. 1110 of the provincial
the radius of 1,500 meters from the hospital of the Philippine Antituberculosis board of Rizal, and using one of the powers conferred upon him by law, the
Society in Santol, in direct contravention of Ordinance No. 15, series of 1926, provincial governor of Rizal, Eligio Naval, suspended the municipal president and the
enacted on May, 1926. denounced members from their respective offices on December 28, 1928. (Exhibits
The permit having been annulled, and the payments theretofore made forfeited, the 5 to 5-E.)
"Sociedad Bighani" filed civil case No. 30537 in the Court of First Instance of Manila On the same date, December 28, 1928, between 9 and 10 o'clock in the morning,
on September 21, 1926, against said Pablo Pablo, as municipal president of the appellant Gregorio Pedro paid into the municipal treasury the sum of P2,050 as a
Caloocan, et al., for a preliminary injunction requiring them to refrain from impeding

26
license fee on his cockpit for the first quarter of the year 1929, and the proper Caloocan, to the public, for Ordinance No. 35, series of 1928, under which a permit
receipt (Exhibit L), and the permit (Exhibit D), were issued to him authorizing him to had been given him to open and exploit his aforesaid cockpit had been disapproved
operate, maintain, exploit, and open to the public a day cockpit in the barrio of by the provincial board of Rizal in its resolution No. 154, series of 1928, as a result of
Galas, Caloocan, Rizal, for a period of four years. which the aforementioned ordinance became null and void.
On December 29, 1928, the municipal council ad interim in Caloocan, passed The first question to decide in this appeal is that raised in the first assignment of
resolution No. 9, series of 1928, approving Ordinance No. 36, series of 1928, error, to wit, whether Ordinance No. 36, series of 1928, approved by the temporary
suspending the effects of resolution No. 202 of the suspended council, approving councilors, is valid.
Ordinance No. 35, series of 1928, while a special committee created by the same
ordinance investigated the expediency of permitting the exploitation and opening of The appellant argues for the nullity of Ordinance No. 36, series of 1928, approved on
the Galas cockpit at the site applied for by the proprietor, Gregorio Pedro. (Exhibit 6.) December 29, 1928, by the temporary councilors appointed by the provincial
governor of Rizal, Eligio Naval, on the ground that (1) it impairs the acquired rights
On the same date, December 29, 1928, the provincial board of Rizal passed of said appellant; (2) it was enacted on account of prejudice, because it was
resolution No. 1135 suspending the effects of resolution No. 202 of the municipal intended for a special and not a general purpose, namely to prevent, at any cost, the
council of Caloocan approving Ordinance No. 35, series of 1928, pending final opening, maintenance, and exploitation of the cockpit of the said petitioner-
decision on the validity of said ordinance by said board. (Exhibit H.) appellant; and (3) it provides for special committee composed of persons who are
not members of the council, vested them with powers which of their very nature,
On January 16, 1929, the Director of the Santol Tuberculosis Sanatorium addressed a cannot be delegated by said council to that committee.
communication to the temporary president of the municipal council of Caloocan,
Flaviano de Jesus, stating that a cockpit established in the barrio of Galas, owing to The petitioner-appellant contends that, having obtained the proper permit to
the noise and clamor of the crowd, would retard the recovery of the patients in said maintain, exploit, and open to the public the cockpit in question, having paid the
sanatorium, and would tend to increase the danger of spreading the disease among license fee and fulfilled all the requirements provided by Ordinance No. 35, series of
those visiting the cockpit. (Exhibit 11.) 1928, he has acquired a right which cannot be taken away from him by Ordinance
No. 36, series of 1928, which was subsequently approved. This court has already
On February 1, 1929, the Chief of the Executive Bureau confirmed the resolution of held that an ordinance regulating the functioning of cockpits does not create
the provincial board of Rizal holding the respondents in the administrative irrevocable rights and may be abrogated by another ordinance. (Vinco vs.
investigation mentioned above guilty of maladministration, and imposing upon each Municipality of Hinigaran, 41 Phil., 790; Joaquin vs. Herrera, 37 Phil., 705; 12 Corpus
of them a punishment of thirty days' suspension. (Exhibit 7.) Juris, 958, sec. 494; 37 Corpus Juris, 168.)
On the same date, February 1, 1929, following the decision of the Executive Bureau The petitioner-appellant also contends that said Ordinance No. 36 was passed due to
mentioned above, the provincial board of Rizal, through resolution No. 154, prejudice "because it was intended for a special and not a general purpose, namely
disapproved said resolution No. 202 of the municipal council of Caloocan, approving to prevent, at any cost, the opening, maintenance, and exploitation of the cockpit of
Ordinance No. 35, series of 1928. (Exhibit 1.) the said petitioner." The aforesaid Ordinance No. 36 was not approved for the
On February 2, 1929, the president of the third sanitary division of Rizal, acting upon purpose of injuring the petitioner, but to correct an irregularity consisting in the
the appellant's application filed on January 30, 1929, issued a certificate to the effect passage of Ordinance No. 35, which had been enacted to favor the said petitioner-
that after a proper inspection of the Galas cockpit, he had found it to be in good appellant. The "Sociedad Bighani," from which the herein petitioner-appellant
sanitary condition. acquired the ownership of the cockpit here in question, was denied a license to
operate it, because it had been constructed in violation of Ordinance No. 15, series
On February 7, 1929, Gregorio Pedro furnished a bond of P10,000 in favor of the of 1926, later amended by Ordinance No. 34, series of 1927. The "Sociedad Bighani"
municipality of Caloocan to secure the payment of the fees accruing during the instituted proceedings against the president and municipal council of Caloocan,
years from 1929 to 1932, which is the period included in the license issued to him Rizal, in civil case No. 30537 of the Court of First Instance of Manila, to prevent said
for the opening and operation of his cockpit in Galas, and this bond was accepted defendants from impeding the operation and exploitation of the Bighani cockpit, and
and approved by the respondent municipal president, Dominador Aquino, and the court decided in favor of said defendants, absolving them from the complaint on
certified by the provincial treasurer, Jose Villegas. (Exhibit E.) the ground among other reasons, that the Bighani cockpit had been constructed
within the prohibited distance from the Antitubercular Sanatorium of Santol, and that
On February 13, 1929, councilor Lucas Bustamante submitted a resolution at a decision was affirmed by this court on appeal. (Company "Bighani" vs. Pablo, supra.)
special session of the municipal council of Caloocan, whereby said council appealed
The cockpit in question now is the former Bighani cockpit mentioned above; it
to the Executive Bureau from the aforementioned resolution No. 154 of the occupies the same site; and the same hygienic reasons which prompted the
provincial board of Rizal, but the resolution did not pass owing to the lack of two-
enactment of Ordinance No. 15, amended by Ordinance No. 34, cited above, exist
thirds of the members necessary, with five members voting in favor and three now; therefore, when this was amended by Ordinance No. 35, reducing the distance
against it.
between a cockpit and any hospital, so that the Bighani cockpit would be beyond
On February 14, 1929, the appellant Gregorio Pedro sent the municipal president of said distance, the municipal council which amended it acted with partiality towards a
Caloocan a communication, informing him that having fulfilled all the requirements certain person, namely, the petitioner-appellant, to the prejudice of the patients in
of the law and the ordinances then in force, he would open his cockpit in Galas to the aforesaid sanatorium. According to Elliot in his work "Municipal Corporations,"
the public in the morning of February 17, 1929. (Exhibit J.) cited by said petitioner-appellant himself, said Ordinance No. 35 is void because it is
partial. (Elliot, Municipal Corporations, sec. 147; Dillon, Municipal Corporations, p.
On February 15, 1929, the respondent municipal president of Caloocan addressed a 915).
communication to the appellant Gregorio Pedro informing him that under no
circumstance could said president permit the appellant to open his cockpit in Galas,

27
Ordinance No. 36, which seeks to correct said irregularity, suspended the effects of
said Ordinance No. 35, impliedly reestablishing Ordinance No. 34, is therefore valid.
The other reason given by the petitioner-appellant to show that Ordinance No. 36, is
void is that the municipal council in approving it delegated its legislative powers to a
special sanitary committee.
Section 2 of Ordinance No. 36, series of 1928, provides as follows:
SEC. 2. A committee is hereby provided for, to be composed of the
president of the third sanitary division of Caloocan, Rizal, a practising
physician residing in this municipality, and a member of the municipal
council, whose duty it shall be to make the necessary investigation to
determine whether or not the exploitation of the cockpit in the barrio of
Galas for which Gregorio Pedro has applied for a permit, would be injurious
to any public or private interest. This special committee shall make such
investigation and submit a report in due form to this municipal council
within the shortest time possible for its definite action.
The municipal council of Caloocan pro tempore therefore does not delegate by that
ordinance to the special committee thereby created any legislative function, but only
entrusts to it the study of the effect of the operation and exploitation of the cockpit
under consideration upon public and private interests, in order to determine whether
or not the license should issue. Informational work of this nature, owing to its
technical character, may be entrusted to technical committees. (12 Corpus Juris,
846.)
Having arrived at the conclusion that Ordinance No. 36 is valid and that the
petitioner-appellant has acquired no irrevocable right by virtue of the license
granted him under Ordinance No. 35, approved to favor him, which is therefore void,
we need not discuss the other assignments of error by the petitioner-appellant.
Wherefore, we are of opinion and so hold: (1) That a license authorizing the
operation and exploitation of a cockpit is not property of which the holder may not
be deprived without due process of law, but a mere privilege which may be revoked
when the public interests so require; (2) that the work entrusted by a municipal
council to a special sanitary committee to make a study of the sanitary effects upon
the neighborhood of the establishment of a cockpit, is not legislative in character,
but only informational, and may be delegated; and (3) that an ordinance, approved
by a municipal council duly constituted, which suspends the effects of another which
had been enacted to favor the grantee of a cockpit license, is valid and legal.
By virtue whereof, finding no error in the judgment appealed from, it is hereby
affirmed, with costs against the appellant. So ordered.
Avancea, C.J., Johnson, Street, Malcolm, Villamor, Ostrand, Romualdez, and
Imperial, JJ., concur.

28
YEE SUE KOY V. ALMEDA, 70 PHIL. 141 more of the articles in question should be introduced as evidence for the prosecution
in criminal case No. 11591, entitled "People of the Philippines vs. Yee Fock (alias Yee
(1940) Sue Koy), Y. Tip and A. Sing," be likewise commanded to refrain from admitting the
same.
G.R. No. 47021 June 25, 1940
The petition is grounded on the propositions (1) that the search warrant issued on
YEE SUE KOY and YEE TIP, ET AL., petitioners, May 2, 1938, by the justice of the peace of Sagay and the seizure accomplished
vs. thereunder are illegal, because the warrant was issued three days ahead of the
MARIANO G. ALMEDA and JOSE ESTRADA, ET AL., respondents. application therefor and of the affidavit of the respondent Jose Estrada which is
insufficient in itself to justify the issuance of a search warrant, and because the
LAUREL, J.: issuance of said warrant manifestly contravenes the mandatory provisions both of
In response to a sworn application of Mariano G. Almeda, chief agent of the Anti- section 1, paragraph 3, of Article III of the Constitution and of 97 of General Orders
Usury Board, dated May 5, 1938, the justice of the peace of Sagay, Occidental No. 58, and (2) that the seizure of the aforesaid articles by means of a search
Negros, after taking the testimony of applicant's witness, Jose Estrada, special agent warrant for the purpose of using them evidence in the criminal case against the
of the Anti-Usury Board, issued on the same date a search warrant commanding any petitioners, is unconstitutional because the warrant thereby becomes unreasonable
peace officer to search during day time the store and premises occupied by Sam and amounts to a violation of the constitutional prohibition against compelling the
Sing & Co., situated at Sagay, Occidental Negros, as well as the person of said Sam accused to testify against themselves..
Sing & Co., and to seize the documents, notebooks, lists, receipts and promissory In their answers the respondents deny that the articles in question were seized by
notes being used by said Sam Sing & Co. in connection with their activities of the Anti-Usury Board to provide itself with evidence in the criminal prosecution
lending money at usurious rates of interest in violation of law, or such as may be against the petitioners, and allege that the seizure of said articles was an incident of
found, and to bring them forthwith before the aforesaid justice of the peace of the Government's duty of apprehending violations of the Usury Law, in connection
Sagay. On the same date, May 5, 1938, at 10:30 a. m. search was accordingly made with which the agents of the Anti-Usury Board are authorized, under section 4 of Act
by Mariano G. Almeda, Jose the Philippine Army, who seized certain receipt books, No. 4109 in relation to Act No. 4168, to examine the documents, papers and articles
vales or promissory notes, chits, notebooks, journal book, and collection list seized from the petitioners; that the search warrant complied of is valid and legal;
belonging to Sam Sing & Co. and enumerated in the inventory receipt issued by that, granting the existence of any irregularity in the issuance of said warrant, the
Mariano G. Almeda to owner of the documents, papers and articles seized same has been waived the return of the articles in question because the same
Immediately after the search and seizure thus effected, Mariano G. Almeda filed a constitute of corpus delicti or are pertinent or relevant thereto.
return with the justice of the peace of Sagay together with a request that the office
of the Anti-Usury Board be allowed to retain possession of the article seized for The petitioners contend that the search warrant herein complained of is illegal
examination, pursuant to section 4 of Act 4109, which request was granted. The first because it was issued three days before the application therefor and the supporting
unsuccessfully effort exerted by Sam Sing & Co. with a view to recovering the affidavit were signed by Mariano G. Almeda and Jose Estrada respectively. This
articles seized, was when their attorney, Godofredo P. Escalona, under date of March contention finds no support in the record before us. In the letter of March 4, 1939,
4, 1939, addressed a letter to the Executive Officer of the Anti-Usury requesting the written by the attorney for Sam Sing &. Co. to the Executive Officer of the Anti-Usury
return of said articles, on ground that the search warrant and seizure of May 5, 1938 Board, requesting the return of the articles seized, reference was made to the search
were illegal, only to receive the reply the request "cannot be complied with until warrant and seizure "of May 5, 1938." (Annex F of the petition) In the Court of First
after have served the purpose for which they were seized" and that "the return of Instance of Occidental Negros, praying for the return of the aforesaid articles, the
the papers must be with the consent and knowledge of the court which issued the search warrant was again referred to as having been issued on "May 5, 1938."
search warrant." Thereafter, under date of March 11, 1939, the same attorney filed a (Annex H of the petition.) It follows, therefore, that there is truth in the allegation of
motion with the Court of First Instance of Occidental Negros praying that the search the respondents that although the original order on which the warrant was issued
warrant issued on May 5, 1938 by the justice of the peace of Sagay and the seizure was prepared on May 2, 1938, when the justice of the peace signed the order for
effected thereunder be declared illegal and set aside and that the articles in search warrant, he placed the date "May 5, 1938."
question be ordered returned to Sam Sing & Co., which motion was denied in the
The criticism of the petitioners that the search warrant in question was not issued in
order dated July 24, 1939. A similar motion was presented to the justice of the peace
accordance with the formalities prescribed by section 1, paragraph, 3, of Article III of
of Sagay on October 27, 1939 but was denied the next day, October 28, 1939,
the Constitution and of section 97 of General Orders No. 58, is unfounded. On the
Meanwhile, an information dated September 30, 1939 had been filed in the Court of
contrary, we are satisfied that strict observance of such formalities was followed.
First Instance of Occidental Negros, charging Yee Fock alias Yee Sue Koy, Y. Tip and A.
The applicant Mariano G. Almeda, in his application, swore that "he made his own
Sing, managers of Sam Sing & Co., with a violation of Act No. 2655, the case being
personal investigation and ascertained that Sam Sing & Co. is lending money
docketed as No. 11591. Before this criminal case could be tried, the present petition
without license, charging usurious rate of interest and is keeping, utilizing and
was filed in this court on November 6, 1939, in which the petitioners pray that the
concealing in the store and premises occupied by it situated at Sagay, Occidental
search warrant of May 2, 1938 and the seizure of May 5, 1938 of the articles
Negros, documents, notebooks, lists, receipts, promissory notes, and book of
described in annex "D" of the petition be declared illegal and set aside; that the
accounts and records, all of which are being used by it in connection with its
respondents Mariano G. Almeda and Jose S. Estrada, as agent of the Anti-Usury
activities lending money rate interest in violation of the Usury Law." In turn, the
Board, the ordered and directed to return to the petitioners the articles listed in said
witness Jose Estrada, in his testimony before the justice of the peace of Sagay,
annex "D" of the petition; that pending these proceedings the provincial fiscal of
swore that he knew that Sam Sing & Co. was lending money without license and
Occidental Negros be commanded to refrain from using said articles as evidence in
charging usurious rate of interest, because he personally investigated the victims
criminal case No. 11591 which was set for trial; on November 13, 1939; that the
who had secured loans from said Sam Sing & Co. and were charged usurious rate of
respondent Judge of the Court of First Instance of Occidental Negros, in case all or
interest; that he knew that the said Sam Sing & Co. was keeping and using books of

29
accounts and records containing its transactions relative to its activities as money to the petitioners cannot be ordered. (People vs. Judge of the Court of First Instance
lender and the entries of the interest paid by its debtors, because he saw that said of Batangas et al., G. R. No. 46361, resolution of February 14, 1939, citing 56 C. J.
Sam Sing & Co. make entries and records of their debts and the interest paid 1166, 1250 and 1251; Uy Kheytin vs. Villareal, 42 Phil., 886; Peoplevs. Rubio, 57
thereon. As both Mariano G. Almeda and Jose Estrada swore that they had personal Phil., 384; People vs. Malasugui, 34 Off. Gaz., 2163, 2165.)
knowledge, their affidavits were sufficient for, thereunder, they could be held liable
for perjury if the facts would turn out to be not as they were stated under oath. The petition is dismissed, with costs against the petitioner. So ordered.
(Alvarez vs. Court of First Instance of Tayabas, et al., 35 Off. Gaz., 1183; Avancea, C.J., Imperial and Diaz, JJ., concur.
People vs. Sy Juco, 37 Off. Gaz., 508; Rodriguez vs. Villamiel, 37 Off. Gaz., 2406.) Moran, J., concurs in the result.
That the existence of probable cause has been determined by the justice of the
peace of Sagay before issuing the search warrant complained of, is shown by the
following statement in the warrant itself, to wit: "After examination under oath of the
complainant, Mariano G. Almeda, Chief Agent of the Anti-Usury Board, Department
of Justice and Special Agent of the Philippine Army, Manila, and the witness he
presented, . . . and this Court, finding that there is just and probable cause to believe
as it does believe, that the above described articles, relating to the activities of said
Sam Sing & Co. of lending money at usurious rate of interest, are being utilized and
kept and concealed at its store and premise occupied by said Sam Sing & Co., all in
violation of law." The description of the articles seized, given in the search warrant,
is likewise sufficient. Where, by the nature of the goods seized, their description
must be rather general, it is not required that a technical description be given, as
this would mean that no warrant could issue. (Alvarez vs. Court of First Instance of
Tayabas et al., 35 Off. Gaz., 1183, citing People vs.Rubio, 57 Phil., 384; and
People vs. Kahn, 256, Ill. App., 415.) Neither can there objection to the fact the
objects seized from the petitioners were retained by the agents of the Anti-Usury
Board, instead of being turned over to the justice of the peace of Sagay, for the
reason that the custody of said agents is the custody of the issuing officer or court,
the retention having been approved by the latter. (Molo vs. Yatco et al., 35 Off. Gaz.,
1335.) .
But it is further contended that the articles seizes should be ordered returned to the
petitioners because the seizure is unconstitutional, having been made for the
purpose of using the articles as evidence in the criminal case against the petitioners.
While we reiterate the rule that the seizure of books and documents by means of a
search warrant, for the purpose of using them as evidence in a criminal case against
the person whose possession they were found, is unconstitutional because it makes
the warrant unreasonable, and it is equivalent to a violation of the constitutional
provision prohibiting the compulsion of an accused to testify against himself
(Rodriguez et al. vs. Villamiel et al., 37 Off. Gaz., 2416, citing Uy Kheytin vs. Villa-
Real, 42 Phil., 886; Alvarez vs. Court of First Instance of Tayabas and Anti-Usury
Board, 35 Off. Gaz., 1183; Brady vs. U. S., 266 U.S. 620; Temparani vs. U. S., 299
Fed. 365; U. S. vs.Madden, 297 Fed. 679; Boyd vs. U. S. 116 U. S. 616; Carroll vs. U.
S., 267 U. S. 132), the said rule has no applicable force in the present case. While in
the cases of Rodriguez et al. vs. Villamiel et al., supra , and Alvarezvs. Court of First
Instance of Tayabas, supra , it appeared that the documents therein involved were in
fact seized for the purpose of discovering evidence to be used against the persons
from whom they were seized, in the case at bar this fact is not clear and is
furthermore denied. In the application for the issuance of the search warrant in
question, it was alleged that the articles seized were "being used by it (Sam Sing &
Co.) in connection with its activities of lending money at usurious rate of interest in
violation of the Usury Law," and it is now suggested (memoranda for respondents)
that the only object of the agents of the Anti-Usury Board in keeping the articles is to
prevent the petitioners from em plying them as a means of further violations of the
Usury Law. In this state of the record, without deciding the question whether the
petitioners will in fact use the articles in question, if returned, for illegal purposes,
we are not prepared to order the return prayed for by the petitioners. (Cf.
People vs. Rubio, 57 Phil., 384, 394-395.) If it be` true, furthermore, without,
however, deciding the point, that as alleged by the respondents the articles in
question constitute the corpus delicti of the violation of the Usury Law, their return

30
PASION SDA. DE GARCIA V. LOCSIN The right of the people to be secure in their persons, houses, papers, and
effects against unreasonable searches and seizures shall not be violated,
G.R. No. L-45950 June 20, 1938 and no warrants shall issue but upon probable cause, to be judge after
examination under oath or affirmation of the complainant and the
LEONA PASION VIUDA DE GARCIA, petitioner, witnesses he may produce, and particularly describing the place to be
vs. searched, and the persons or things to be seized.
DIEGO LOCSIN, Judge of First Instance of Tarlac,
FELIX IMPERIAL, Provincial Fiscal of Tarlac, and the ANTI-USURY Freedom from unreasonable searches and seizures is declared a popular right and
BOARD, respondents. for a search warrant to be valid, (1) it must be issued upon probable cause; (2) the
probable cause must be determined by the judge himself and not by the applicant or
LAUREL, J.: any other person; (3) in the determination of probable cause, the judge must
examine, under oath or affirmation, the complainant and such witnesses as the
This is a petition for mandamus presented to secure the annulment of a search latter may produce; and (4) the warrant issued must particularly describe the place
warrant and two orders of the respondent judge, and the restoration of certain to be searched and persons or things to be seized. These requirements are
documents alleged to have been illegally seized by an agent of the Anti-Usuary complemented by the Code of Criminal Procedure (G. O. No. 58), particularly with
Board. reference to the duration of the validity of the search warrant and the obligation of
It appears that on November 10, 1934, Mariano G. Almeda, an agent of the Anti- the officer seizing the property to deliver the same to the corresponding court (secs.
Usuary Board, obtained from the justice of the peace of Tarlac, Tarlac, a search 102-104). On more than one occasion, since the approval of the Constitution, we had
warrant(Exhibit B) commanding any officer of the law to search the person, house or emphasized the necessity of adherence to the constitutional requirements on this
store of the petitioner at Victoria, Tarlac, for "certain books, lists, chits, receipts, subject (Alvarez vs. Court of First Instance of Tayabas and Anti-Usury Board [1937],
documents and other papers relating to her activities as usurer." The search warrant 35 Off. Gaz., 1183; People vs. Sy Juco [1937], G.R. No. 41957; Rodriguez vs. Villamiel
was issued upon an affidavit given by the said Almeda "that he has and there (is) [1937], G.R. No. 44328; and Molo vs. Yatco [1936], 35 Off. Gaz., 1935) and we do not
just and probable cause to believe and he does believe that Leona Pasion de Garcia deem it necessary to reiterate what has been said or observed in these cases.
keeps and conceals in her house and store at Victoria, Tarlac, certain books, lists, In the instant case the existence of probable cause was determined not by the judge
chits, receipts, documents, and other papers relating to her activities as usurer, all of himself but by the applicant. All that the judge did was to accept as true the affidavit
which is contrary to the statute in such cases made and provided." On the same made by agent Almeda. He did not decide for himself. It does not appear that he
date, the said Mariano G. Almeda, accompanied by a captain of the Philippine examined the applicant and his witnesses, if any. Even accepting the description of
Constabulary, went to the office of the petitioner in Victoria, Tarlac and, after the properties to be seized to be sufficient and on the assumption that the receipt
showing the search warrant to the petitioner's bookkeeper, Alfredo Salas, and, issued is sufficiently detailed within the meaning of the law, the properties seized
without the presence of the petitioner who was ill and confined at the time, were not delivered to the court which issued the warrant, as required by law. (See,
proceeded with the execution thereof. Two packages of records and a locked filing secs. 95 and 104, G. O. No. 58.) instead, they were turned over to the respondent
cabinet containing several Papers and documents were seized by Almeda and a provincial fiscal and used by him in building up cases against the petitioner.
receipt therefor issued by him to Salas. The papers and documents seized were kept Considering that at the time the warrant was issued there was no case pending
for a considerable length of time by the Anti-Usury Board and thereafter were turned against the petitioner, the averment that the warrant was issued primarily for
over by it to the respondent fiscal who subsequently filed, in the Court of First exploration purposes is not without basis. The lower court is, therefore, correct in
Instance of Tarlac, six separate criminal cases against the herein petitioner for reaching the conclusion that the search warrant (Exhibit B) was illegally issued by
violation of the Anti-Usury Law. On several occasions, after seizure, the petitioner, the justice of the peace of Tarlac, Tarlac.
through counsel, demanded from the respondent Anti-Usury Board the return of the
documents seized. On January 7. and, by motion, on June 4, 1937, the legality of the The important question presented is whether upon the facts and under the
search warrant was challenged by counsel for the petitioner in the six criminal cases circumstances of the present case, there has been a waiver by the petitioner of her
and the devolution of the documents demanded. By resolution of October 5, 1937, constitutional immunity against unreasonable searches and seizures. While the
the respondent Judge of First Instance denied the petitioner's motion of June 4 for Solicitor-General admits that, in the light of decisions of this court, the search
the reason that though the search warrant was illegal, there was a waiver on the warrant was illegally issued, he maintains "(1) that the petitioner had waived her
part of the petitioner. "En el caso presente,"declared the respondent constitutional right by her acquiescence after the search and seizure, and (2) that
judge, "teniendo en cuenta que la acusada Por si o por medio de su representante, the application for the return of the documents illegally seized was made after an
no presento protests alguna contra el registro de autos, at verificarse el mismo, o unreasonable length of time after the date of seizure." Doubtless, the constitutional
despues de un tiempo rezonable, el juzgado declare que la citada con su silencio y immunity against unreasonable searches and seizures is a personal right which may
conducta, ha renunciado implicitanmente a su derecho a no ser sometido a un be waived. (People vs. Kagui Malasugui, 34 Off. Gaz., pp. 2163, 2164; 56 C.J., pp.
registro irrazonable, por lo que no le es pemitido quejarse despues, puesto que 1178, 1179; Cf. Rodriguez vs. Villamiel, supra.) The waiver may be either express or
cualquier defecto queha adolecido lo expedicion de la orden de registro y su implied (67 C.J., p. 304). No express waiver has been made in the case before us. It
ejecucion, ha quidado implilcitamente subsanado." A motion for reconsideration was is urged, however, that there has been a waiver by implication. It is well-settled that
presented but was denied by order of January 3, 1938. Petitioner registered her to constitute a waiver of a constitutional right, it must appear, first, that the right
exception. The resolution of October 5, 1937 and the order of January 3, 1938 are exists; secondly, that the persons involved had knowledge, either actual or
sought, together with the search warrant, Exhibit B, to be nullified in these constructive, of the existence of such right; and, lastly, that said person had an
proceedings. actual intention to relinquish the right. (67 C. J., 299.) It is true that the petitioner did
not object to the legality of the search when it was made. She could not have
Paragraph 3, section 1 of the bill of right of our Constitution provides as follows: objected because she was sick and was not present when the warrant was served

31
upon Alfredo Salas. Certainly, the constitutional immunity from unreasonable
searches and seizures, being a personal one, cannot be waived by anyone except
the person whose rights are invaded or one who is expressly authorized to do so in
his or her behalf. (56 C. J., p. 1183.) Of course, the petitioner came to know later of
the seizure of some of her papers and documents. But this was precisely the reason
why she sent her attorneys to the office of the Anti-Usuary Board to demand the
return of the documents seized. In any event, the failure on the part of the petitioner
and her bookkeeper to resist or object to the execution of the warrant does not
constitute an implied waiver of constitutional right. It is, as Judge Cooley observes,
but a submission to the authority of the law. (Const. Lim., 8th ed., Vol., I, p. 630.) As
the constitutional guaranty is not dependent upon any affirmative act of the citizen,
the courts do not place the citizen in the position of either contesting an officer's
authority by force, or waiving his constitutional rights; but instead they hold that a
peaceful submission to a search or seizure is not a consent or an invitation thereto,
but is merely a demonstration of regard for the supremacy of the law. (56 C.J., pp.
1180, 1181.)
As a general proposition, it may be admitted that waiver may be the result of a
failure to object within a reasonable time to a search and seizure illegally made. It
must be observed, however, that the petitioner, on several occasions, and prior to
the filing of criminal actions against her, had demanded verbally, through counsel,
the return by the Anti-Usuary Board of the properties seized. This is admitted by
Adolfo N. Feliciano, acting chief of the board, who said that the demand was refused
simply because no habiamos terminado con nuestra investigacion. (T.s.n., pp. 24-
25.) On July 7, 1936, counsel for the petitioner wrote a letter to the Anti-Usuary
Board demanding again the return of the documents withheld. And in connection
with the criminal cases pending against the petitioner, similar demands were made
on January 7, 1937 and on June 4, 1937. In the light of these circumstances, we find
that the petitioner did not waive her constitutional right. The delay in making
demand for the return of the documents seized is not such as to result in waiver by
implication.
In view of the foregoing, the writ prayed for is granted. The search warrant, Exhibit
B, is hereby declared void and of no effect; the orders of October 5, 1937 and
January 3, 1938 of the respondent judge are set aside; and the respondents Anti-
Usuary Board and the provincial fiscal of Tarlac or those acting in their behalf, are
hereby ordered to return and restore to the petitioner all the properties, documents,
papers and effects illegally seized from her, within forty-eight (48) hours from the
time this decision becomes final. Without costs. So ordered.
Avancea, C.J., Villa-Real, Abad Santos, Imperial, Diaz and Concepcion, JJ., concur.

32
BURGOS V. CHIEF OF STAFF suspend its rules or to except a particular case from its operation, whenever the
purposes of justice require it...".
G.R. No. L-64261 December 26, 1984 Respondents likewise urge dismissal of the petition on ground of laches.
JOSE BURGOS, SR., JOSE BURGOS, JR., BAYANI SORIANO and J. BURGOS Considerable stress is laid on the fact that while said search warrants were issued on
MEDIA SERVICES, INC.,petitioners, December 7, 1982, the instant petition impugning the same was filed only on June
vs. 16, 1983 or after the lapse of a period of more than six [6] months.
THE CHIEF OF STAFF, ARMED FORCES OF THE PHILIPPINES, THE CHIEF, Laches is failure or negligence for an unreasonable and unexplained length of time
PHILIPPINE CONSTABULARY, THE CHIEF LEGAL OFFICER, PRESIDENTIAL to do that which, by exercising due diligence, could or should have been done
SECURITY COMMAND, THE JUDGE ADVOCATE GENERAL, ET AL., respondents. earlier. It is negligence or omission to assert a right within a reasonable time,
ESCOLIN, J.: warranting a presumption that the party entitled to assert it either has abandoned it
or declined to assert it. 5
Assailed in this petition for certiorari prohibition and mandamus with preliminary
mandatory and prohibitory injunction is the validity of two [2] search warrants issued Petitioners, in their Consolidated Reply, explained the reason for the delay in the
on December 7, 1982 by respondent Judge Ernani Cruz-Pano, Executive Judge of the filing of the petition thus:
then Court of First Instance of Rizal [Quezon City], under which the premises known Respondents should not find fault, as they now do [p. 1, Answer, p. 3,
as No. 19, Road 3, Project 6, Quezon City, and 784 Units C & D, RMS Building, Manifestation] with the fact that the Petition was filed on June 16, 1983,
Quezon Avenue, Quezon City, business addresses of the "Metropolitan Mail" and "We more than half a year after the petitioners' premises had been raided.
Forum" newspapers, respectively, were searched, and office and printing machines,
equipment, paraphernalia, motor vehicles and other articles used in the printing, The climate of the times has given petitioners no other choice. If they had
publication and distribution of the said newspapers, as well as numerous papers, waited this long to bring their case to court, it was because they tried at
documents, books and other written literature alleged to be in the possession and first to exhaust other remedies. The events of the past eleven fill years had
control of petitioner Jose Burgos, Jr. publisher-editor of the "We Forum" newspaper, taught them that everything in this country, from release of public funds to
were seized. release of detained persons from custody, has become a matter of
executive benevolence or largesse
Petitioners further pray that a writ of preliminary mandatory and prohibitory
injunction be issued for the return of the seized articles, and that respondents, Hence, as soon as they could, petitioners, upon suggestion of persons close
"particularly the Chief Legal Officer, Presidential Security Command, the Judge to the President, like Fiscal Flaminiano, sent a letter to President Marcos,
Advocate General, AFP, the City Fiscal of Quezon City, their representatives, through counsel Antonio Coronet asking the return at least of the printing
assistants, subalterns, subordinates, substitute or successors" be enjoined from equipment and vehicles. And after such a letter had been sent, through Col.
using the articles thus seized as evidence against petitioner Jose Burgos, Jr. and the Balbino V. Diego, Chief Intelligence and Legal Officer of the Presidential
other accused in Criminal Case No. Q- 022782 of the Regional Trial Court of Quezon Security Command, they were further encouraged to hope that the latter
City, entitled People v. Jose Burgos, Jr. et al. 1 would yield the desired results.

In our Resolution dated June 21, 1983, respondents were required to answer the After waiting in vain for five [5] months, petitioners finally decided to come
petition. The plea for preliminary mandatory and prohibitory injunction was set for to Court. [pp. 123-124, Rollo]
hearing on June 28, 1983, later reset to July 7, 1983, on motion of the Solicitor
Although the reason given by petitioners may not be flattering to our judicial
General in behalf of respondents.
system, We find no ground to punish or chastise them for an error in judgment. On
At the hearing on July 7, 1983, the Solicitor General, while opposing petitioners' the contrary, the extrajudicial efforts exerted by petitioners quite evidently negate
prayer for a writ of preliminary mandatory injunction, manifested that respondents the presumption that they had abandoned their right to the possession of the seized
"will not use the aforementioned articles as evidence in the aforementioned case property, thereby refuting the charge of laches against them.
until final resolution of the legality of the seizure of the aforementioned
Respondents also submit the theory that since petitioner Jose Burgos, Jr. had used
articles. ..." 2 With this manifestation, the prayer for preliminary prohibitory
and marked as evidence some of the seized documents in Criminal Case No. Q-
injunction was rendered moot and academic.
022872, he is now estopped from challenging the validity of the search warrants. We
Respondents would have this Court dismiss the petition on the ground that do not follow the logic of respondents. These documents lawfully belong to
petitioners had come to this Court without having previously sought the quashal of petitioner Jose Burgos, Jr. and he can do whatever he pleases with them, within legal
the search warrants before respondent judge. Indeed, petitioners, before impugning bounds. The fact that he has used them as evidence does not and cannot in any way
the validity of the warrants before this Court, should have filed a motion to quash affect the validity or invalidity of the search warrants assailed in this petition.
said warrants in the court that issued them. 3 But this procedural flaw
Several and diverse reasons have been advanced by petitioners to nullify the search
notwithstanding, we take cognizance of this petition in view of the seriousness and
warrants in question.
urgency of the constitutional issues raised not to mention the public interest
generated by the search of the "We Forum" offices, which was televised in Channel 7 1. Petitioners fault respondent judge for his alleged failure to conduct an
and widely publicized in all metropolitan dailies. The existence of this special examination under oath or affirmation of the applicant and his witnesses, as
circumstance justifies this Court to exercise its inherent power to suspend its rules. mandated by the above-quoted constitutional provision as wen as Sec. 4, Rule 126
In the words of the revered Mr. Justice Abad Santos in the case of C. Vda. de of the Rules of Court . 6 This objection, however, may properly be considered moot
Ordoveza v. Raymundo, 4 "it is always in the power of the court [Supreme Court] to and academic, as petitioners themselves conceded during the hearing on August 9,

33
1983, that an examination had indeed been conducted by respondent judge of Col. the properties that may be seized is stolen property. Necessarily, stolen property
Abadilla and his witnesses. must be owned by one other than the person in whose possession it may be at the
time of the search and seizure. Ownership, therefore, is of no consequence, and it is
2. Search Warrants No. 20-82[a] and No. 20- 82[b] were used to search two distinct sufficient that the person against whom the warrant is directed has control or
places: No. 19, Road 3, Project 6, Quezon City and 784 Units C & D, RMS Building, possession of the property sought to be seized, as petitioner Jose Burgos, Jr. was
Quezon Avenue, Quezon City, respectively. Objection is interposed to the execution alleged to have in relation to the articles and property seized under the warrants.
of Search Warrant No. 20-82[b] at the latter address on the ground that the two
search warrants pinpointed only one place where petitioner Jose Burgos, Jr. was 4. Neither is there merit in petitioners' assertion that real properties were seized
allegedly keeping and concealing the articles listed therein, i.e., No. 19, Road 3, under the disputed warrants. Under Article 415[5] of the Civil Code of the
Project 6, Quezon City. This assertion is based on that portion of Search Warrant No. Philippines, "machinery, receptables, instruments or implements intended by the
20- 82[b] which states: owner of the tenement for an industry or works which may be carried on in a
building or on a piece of land and which tend directly to meet the needs of the said
Which have been used, and are being used as instruments and industry or works" are considered immovable property. In Davao Sawmill Co. v.
means of committing the crime of subversion penalized under P.D. Castillo 9 where this legal provision was invoked, this Court ruled that machinery
885 as amended and he is keeping and concealing the same at 19 which is movable by nature becomes immobilized when placed by the owner of the
Road 3, Project 6, Quezon City. tenement, property or plant, but not so when placed by a tenant, usufructuary, or
The defect pointed out is obviously a typographical error. Precisely, two search any other person having only a temporary right, unless such person acted as the
warrants were applied for and issued because the purpose and intent were to search agent of the owner.
two distinct premises. It would be quite absurd and illogical for respondent judge to In the case at bar, petitioners do not claim to be the owners of the land and/or
have issued two warrants intended for one and the same place. Besides, the building on which the machineries were placed. This being the case, the machineries
addresses of the places sought to be searched were specifically set forth in the in question, while in fact bolted to the ground remain movable property susceptible
application, and since it was Col. Abadilla himself who headed the team which to seizure under a search warrant.
executed the search warrants, the ambiguity that might have arisen by reason of the
typographical error is more apparent than real. The fact is that the place for which 5. The questioned search warrants were issued by respondent judge upon
Search Warrant No. 20- 82[b] was applied for was 728 Units C & D, RMS Building, application of Col. Rolando N. Abadilla Intelligence Officer of the P.C.
Quezon Avenue, Quezon City, which address appeared in the opening paragraph of Metrocom. 10 The application was accompanied by the Joint Affidavit of Alejandro M.
the said warrant. 7 Obviously this is the same place that respondent judge had in Gutierrez and Pedro U. Tango, 11 members of the Metrocom Intelligence and
mind when he issued Warrant No. 20-82 [b]. Security Group under Col. Abadilla which conducted a surveillance of the premises
prior to the filing of the application for the search warrants on December 7, 1982.
In the determination of whether a search warrant describes the premises to be
searched with sufficient particularity, it has been held "that the executing officer's It is contended by petitioners, however, that the abovementioned documents could
prior knowledge as to the place intended in the warrant is relevant. This would seem not have provided sufficient basis for the finding of a probable cause upon which a
to be especially true where the executing officer is the affiant on whose affidavit the warrant may validly issue in accordance with Section 3, Article IV of the 1973
warrant had issued, and when he knows that the judge who issued the warrant Constitution which provides:
intended the building described in the affidavit, And it has also been said that the
executing officer may look to the affidavit in the official court file to resolve an SEC. 3. ... and no search warrant or warrant of arrest shall issue except
ambiguity in the warrant as to the place to be searched." 8 upon probable cause to be determined by the judge, or such other
responsible officer as may be authorized by law, after examination under
3. Another ground relied upon to annul the search warrants is the fact that although oath or affirmation of the complainant and the witnesses he may produce,
the warrants were directed against Jose Burgos, Jr. alone, articles b belonging to his and particularly describing the place to be searched and the persons or
co-petitioners Jose Burgos, Sr., Bayani Soriano and the J. Burgos Media Services, Inc. things to be seized.
were seized.
We find petitioners' thesis impressed with merit. Probable cause for a search is
Section 2, Rule 126 of the Rules of Court, enumerates the personal properties that defined as such facts and circumstances which would lead a reasonably discreet and
may be seized under a search warrant, to wit: prudent man to believe that an offense has been committed and that the objects
sought in connection with the offense are in the place sought to be searched. And
Sec. 2. Personal Property to be seized. A search warrant may be issued when the search warrant applied for is directed against a newspaper publisher or
for the search and seizure of the following personal property: editor in connection with the publication of subversive materials, as in the case at
[a] Property subject of the offense; bar, the application and/or its supporting affidavits must contain a specification,
stating with particularity the alleged subversive material he has published or is
[b] Property stolen or embezzled and other proceeds or fruits of intending to publish. Mere generalization will not suffice. Thus, the broad statement
the offense; and in Col. Abadilla's application that petitioner "is in possession or has in his control
printing equipment and other paraphernalia, news publications and other documents
[c] Property used or intended to be used as the means of
which were used and are all continuously being used as a means of committing the
committing an offense. offense of subversion punishable under Presidential Decree 885, as
The above rule does not require that the property to be seized should be owned by amended ..." 12 is a mere conclusion of law and does not satisfy the requirements of
the person against whom the search warrant is directed. It may or may not be probable cause. Bereft of such particulars as would justify a finding of the existence
owned by him. In fact, under subsection [b] of the above-quoted Section 2, one of

34
of probable cause, said allegation cannot serve as basis for the issuance of a search description of the articles sought to be seized under the search warrants in question
warrant and it was a grave error for respondent judge to have done so. cannot be characterized differently.
Equally insufficient as basis for the determination of probable cause is the statement In the Stanford case, the U.S. Supreme Courts calls to mind a notable chapter in
contained in the joint affidavit of Alejandro M. Gutierrez and Pedro U. Tango, "that English history: the era of disaccord between the Tudor Government and the English
the evidence gathered and collated by our unit clearly shows that the premises Press, when "Officers of the Crown were given roving commissions to search where
above- mentioned and the articles and things above-described were used and are they pleased in order to suppress and destroy the literature of dissent both Catholic
continuously being used for subversive activities in conspiracy with, and to promote and Puritan Reference herein to such historical episode would not be relevant for it is
the objective of, illegal organizations such as the Light-a-Fire Movement, Movement not the policy of our government to suppress any newspaper or publication that
for Free Philippines, and April 6 Movement." 13 speaks with "the voice of non-conformity" but poses no clear and imminent danger
to state security.
In mandating that "no warrant shall issue except upon probable cause to be
determined by the judge, ... after examination under oath or affirmation of the As heretofore stated, the premises searched were the business and printing offices
complainant and the witnesses he may produce; 14 the Constitution requires no less of the "Metropolitan Mail" and the "We Forum newspapers. As a consequence of the
than personal knowledge by the complainant or his witnesses of the facts upon search and seizure, these premises were padlocked and sealed, with the further
which the issuance of a search warrant may be justified. In Alvarez v. Court of First result that the printing and publication of said newspapers were discontinued.
Instance, 15 this Court ruled that "the oath required must refer to the truth of the
facts within the personal knowledge of the petitioner or his witnesses, because the Such closure is in the nature of previous restraint or censorship abhorrent to the
purpose thereof is to convince the committing magistrate, not the individual making freedom of the press guaranteed under the fundamental law, 18 and constitutes a
the affidavit and seeking the issuance of the warrant, of the existence of probable virtual denial of petitioners' freedom to express themselves in print. This state of
cause." As couched, the quoted averment in said joint affidavit filed before being is patently anathematic to a democratic framework where a free, alert and
respondent judge hardly meets the test of sufficiency established by this Court in even militant press is essential for the political enlightenment and growth of the
Alvarez case. citizenry.

Another factor which makes the search warrants under consideration constitutionally Respondents would justify the continued sealing of the printing machines on the
objectionable is that they are in the nature of general warrants. The search warrants ground that they have been sequestered under Section 8 of Presidential Decree No.
describe the articles sought to be seized in this wise: 885, as amended, which authorizes "the sequestration of the property of any person,
natural or artificial, engaged in subversive activities against the government and its
1] All printing equipment, paraphernalia, paper, ink, photo (equipment, duly constituted authorities ... in accordance with implementing rules and
typewriters, cabinets, tables, communications/recording equipment, tape regulations as may be issued by the Secretary of National Defense." It is doubtful
recorders, dictaphone and the like used and/or connected in the printing of however, if sequestration could validly be effected in view of the absence of any
the "WE FORUM" newspaper and any and all documents communication, implementing rules and regulations promulgated by the Minister of National
letters and facsimile of prints related to the "WE FORUM" newspaper. Defense.
2] Subversive documents, pamphlets, leaflets, books, and other publication Besides, in the December 10, 1982 issue of the Daily Express, it was reported that
to promote the objectives and piurposes of the subversive organization no less than President Marcos himself denied the request of the military authorities
known as Movement for Free Philippines, Light-a-Fire Movement and April 6 to sequester the property seized from petitioners on December 7, 1982. Thus:
Movement; and,
The President denied a request flied by government prosecutors for
3] Motor vehicles used in the distribution/circulation of the "WE FORUM" sequestration of the WE FORUM newspaper and its printing presses,
and other subversive materials and propaganda, more particularly, according to Information Minister Gregorio S. Cendana.
1] Toyota-Corolla, colored yellow with Plate No. NKA 892; On the basis of court orders, government agents went to the We Forum
offices in Quezon City and took a detailed inventory of the equipment and
2] DATSUN pick-up colored white with Plate No. NKV 969 all materials in the premises.
3] A delivery truck with Plate No. NBS 524; Cendaa said that because of the denial the newspaper and its equipment
4] TOYOTA-TAMARAW, colored white with Plate No. PBP 665; and, remain at the disposal of the owners, subject to the discretion of the
court. 19
5] TOYOTA Hi-Lux, pick-up truck with Plate No. NGV 427 with
marking "Bagong Silang." That the property seized on December 7, 1982 had not been sequestered is further
confirmed by the reply of then Foreign Minister Carlos P. Romulo to the letter dated
In Stanford v. State of Texas 16 the search warrant which authorized the search for February 10, 1983 of U.S. Congressman Tony P. Hall addressed to President Marcos,
"books, records, pamphlets, cards, receipts, lists, memoranda, pictures, recordings expressing alarm over the "WE FORUM " case. 20 In this reply dated February 11,
and other written instruments concerning the Communist Party in Texas," was 1983, Minister Romulo stated:
declared void by the U.S. Supreme Court for being too general. In like manner,
directions to "seize any evidence in connectionwith the violation of SDC 13-3703 or 2. Contrary to reports, President Marcos turned down the recommendation
otherwise" have been held too general, and that portion of a search warrant which of our authorities to close the paper's printing facilities and confiscate the
authorized the seizure of any "paraphernalia which could be used to violate Sec. 54- equipment and materials it uses. 21
197 of the Connecticut General Statutes [the statute dealing with the crime of
conspiracy]" was held to be a general warrant, and therefore invalid. 17 The

35
IN VIEW OF THE FOREGOING, Search Warrants Nos. 20-82[a] and 20-82[b] issued by
respondent judge on December 7, 1982 are hereby declared null and void and are
accordingly set aside. The prayer for a writ of mandatory injunction for the return of
the seized articles is hereby granted and all articles seized thereunder are hereby
ordered released to petitioners. No costs.
SO ORDERED.
Fernando, C.J., Teehankee, Makasiar, Concepcion, Jr., Melencio-Herrera, Plana,
Relova, Gutierrez, Jr., De la Fuente and Cuevas, JJ., concur.
Aquino, J., took no part.

36
CORRO V. LISING , 137 SCRA 341 (1985) Hence, this petition for certiorari and mandamus, with application for preliminary
injunction and restraining order to enjoin respondent Regional Trial Court, National
G.R. No. L-69899 July 15, 1985 Capital Region, Branch 98 from proceeding with the trial of Criminal Case No. S3-Q-
29243, praying (a) that Search Warrant No. Q-00002 issued by respondent Judge
ROMMEL CORRO, petitioner, Esteban M. Lising be declared null and void ab initio and that a mandatory injunction
vs. be issued directing respondents City Fiscal's Office of Quezon City and Lt. Col. Berlin
HON. ESTEBAN LISING Presiding Judge, Regional Trial Court, Quezon City, Castillo and 1st Lt. Godofredo Ignacio jointly and severally to return immediately the
Branch XCV HON. REMIGIO ZARI Regional Trial Court, Quezon City, Branch documents/properties illegally seized from herein petitioner and that final injunction
98; CITY FISCAL'S OFFICE, Quezon City; LT. COL. BERLIN A. CASTILLO and be issued enjoining respondents City Fiscal's Office of Quezon City, Lt. Col. Castillo
1ST LT. GODOFREDO M. IGNACIO, respondents, and 1st Lt. Ignacio from utilizing said documents/properties as evidence in Criminal
Case No. 29243; and (b) that respondent PC-CIS officers Lt. Col. Berlin A. Castillo and
RELOVA, J.: lst Lt. Godofredo Ignacio be directed to reopen the padlocked office premises of the
On September 29, 1983, respondent Regional Trial Court judge Esteban Lising of Philippine Times at 610 Mezzanine Floor, Gochengco Building, T.M., Kalaw, Ermita,
Quezon City, upon application filed by Lt. Col. Berlin Castillo of the Philippine Manila.
Constabulary Criminal Investigation Service, issued Search Warrant No. Q-00002 In Our Resolution of February 19, 1985, respondents were required to file their
authorizing the search and seizure of comment. The plea for temporary restraining order was granted and respondents
1. Printed copies of Philippine Times; City Fiscal's Office of Quezon City, Lt. Col. Berlin Castillo and 1st Lt. Godofredo
Ignacio were enjoined from introducing as evidence for the state the
2. Manuscripts/drafts of articles for publication in the Philippine Times; documents/properties seized under Search Warrant No. Q-00002 in Criminal Cage
No. Q-29243 (Sedition case against petitioner), pending before the Regional Trial
3. Newspaper dummies of the Philippine Times; Court of Quezon City, Branch 98, effective immediately and continuing until further
4. Subversive documents, articles, printed matters, handbills, leaflets, banners; orders from the Court.

5. Typewriters, duplicating machines, mimeographing and tape recording machines, Respondents would have this Court dismiss the petition on the ground that (1) the
video machines and tapes present action is premature because petitioner should have filed a motion for
reconsideration of respondent Judge Lising's order of January 28, 1985; (2) probable
which have been used and are being used as instrument and means of committing cause exists justifying the issuance of a search warrant; (3) the articles seized were
the crime of inciting to sedition defined and penalized under Article 142 of the adequately described in the search warrant; (4) a search was conducted in an
Revised Penal Code, as amended by PD 1835 ... (p. 24, Rollo) orderly manner; (5) the padlocking of the searched premises was with the consent of
petitioner's wife; (6) the findings of the Agrava Board is irrelevant to the issue of the
On November 6, 1984, petitioner filed an urgent motion to recall warrant and to
validity of the search warrant; (7) press freedom is not an issue; and, (8) the petition
return documents/personal properties alleging among others that:
is barred by laches.
2. ... the properties seized are typewriters, duplicating machines, mimeographing
There is merit in the petition.
and tape recording machines, video machines and tapes which are not in any way,
inanimate or mute things as they are, connected with the offense of inciting to Respondents contend that petitioner should have filed a motion for reconsideration
sedition. of the order in question before coming to Us. This is not always so. When the
questions raised before the Supreme Court are the same as those which were
3. More so, documents or papers seized purporting to do the body of the crime has
squarely raised in and passed upon by the lower court, the filing of the motion for
been rendered moot and academic due to the findings of the Agrava Board that a
reconsideration in said court before certiorari can be instituted in the Supreme Court
military conspiracy was responsible for the slaying of the late Senator Benigno
is no longer a pre-requisite. As held in Bache & Co. (Phil.), Inc. vs. Ruiz, 37 SCRA 823,
Aquino, Jr. on August 21, 1983 at the Manila International Airport. The Agrava Board
(t)he rule requiring the filing of a motion for reconsideration before an application for
which has the exclusive jurisdiction to determine the facts and circumstances behind
a writ of certiorari can be entertained was never intended to be applied without
the killing had virtually affirmed by evidence testamentary and documentary the
considering the circumstances. The rule does not apply where, the deprivation of
fact that soldiers killed Benigno Aquino, Jr.
petitioners' fundamental right to due process taints the proceeding against them in
4. More so, the grave offense of libel, RTC, Q.C. Branch XCV has dismissed said case the court below not only with irregularity but also with nullity." Likewise, in Pajo, et
against the accused on all documents pertinent and more so as we repeat, rendered al. vs. Ago, et al., 108 Phil. 905 and in Gonzales vs. Court of Appeals, 3 SCRA 465,
moot and academic by the recent Agrava Report. (p. 27, Rollo) this Court ruled that "it is only when questions are raised for the first time before the
high court in a certiorari case that the writ shall not issue, unless the lower court had
On January 28, 1985, respondent Judge Lising denied the motion in a resolution, first been given an opportunity to pass upon the same." Further, in the case
pertinent portions of which state: of Matute vs. Court of Appeals, 26 SCRA 768, We held that "while as a matter of
policy a motion for reconsideration in the lower court has often been considered a
... The said articles presently form part of the evidence of the prosecution and they
condition sine qua non for the granting of a writ of certiorari, this rule does not apply
are not under the control of the prosecuting arm of the government. Under these
where the proceeding in which the error occurred is a patent nullity or where 'the
circumstances, the proper forum from which the petition to withdraw the articles
deprivation of petitioner's fundamental right to due process ... taints the proceeding
should be addressed, is the Office of the City Fiscal, Quezon City and not with this
against him in the court below not only with irregularity but with nullity (Luzon
Branch of the Court. It is to be further noted that it is not even with this Branch of
Surety Co. v. Marbella et al., L-16038, Sept. 30, 1960), or when special
the Court that the offense of inciting to sedition is pending. (p 29, Rollo)
circumstances warrant immediate and more direct action. ..." The records of this

37
petition clearly disclose that the issues herein raised have already been presented to 4. Four tape alleged speech of Mayor Climaco, two alleged speeches of Aquino
and passed upon by the court a quo. and a speech of one various artist;
Section 3, Article IV of the 1973 Constitution provides: 5. One bundle Dummies;
SEC. 3. ...no search warrant or warrant of arrest issue except upon probable cause to 6. Ten bundles of assorted copies of Philippine Times issued on different dates (Nos.
be determined by the judge, or such other responsible officer as may be authorized 6, 7, 8, 9, 10, 11, 12, 13, 14 & 15):
by law, after examination under oath or affirmation of the complainant and the
witnesses he may produce, and particularly describing the place to be searched and 7. One Typewriter Remington Brand Long Carriage with No. J-2479373;
the persons or things to be seized. 8. OneTypewriterAdler-short with No. 9003011;
and, Section 3, Rule 126 of the New Rules of Court, states that: 9. Three (3) bundles of Philippine Times latest issue for Baguio City (p. 26, Rollo)
SEC. 3. Requisites for issuing search warrant. A search warrant shall not issue but In Stonehill vs. Diokno, 20 SCRA 383, this Court held that search warrants
upon probable cause in connection with one specific offense to be determined by the authorizing the seizure of books of accounts and records "showing all the business
judge or justice of the peace after examination under oath or affirmation of the transactions" of certain persons, regardless of whether the transactions were legal
complainant and the witnesses he may produce, and particularly describing the or illegal, contravene the explicit comment of the Bill of Rights that the things to be
place to be searched and the persons or things to be seized. seized should be particularly described and defeat its major objective of eliminating
Probable cause may be defined as "such reasons, supported by facts and general warrants. In the case at bar, the search warrant issued by respondent judge
circumstances, as will warrant a cautious man in the belief that his actions, and the allowed seizure of printed copies of the Philippine Times, manuscripts/drafts of
means taken in prosecuting it, are legally just and proper (Burton vs. St. Paul, M & M. articles for publication, newspaper dummies, subversive documents, articles, etc.,
Ry. Co., 33 Minn. 189, cited in U.S. vs. Addison, 28 Phil. 566)." Thus, an application and even typewriters, duplicating machines, mimeographing and tape recording
for search warrant must state with particularly the alleged subversive materials machines. Thus, the language used is so all embracing as to include all conceivable
published or intended to be published by the publisher and editor of the Philippine records and equipment of petitioner regardless of whether they are legal or illegal.
Times, Rommel Corro. As We have stated in Burgos, Sr. vs. Chief of Staff of the The search warrant under consideration was in the nature of a general warrant
Armed Forces of the Philippines, 133 SCRA 800, "mere generalization will not which is constitutionally objectionable.
suffice." A search warrant should particularly describe the place to be searched and Respondents do not deny the fact that the business office of the "Philippine Times"
the things to be seized. "The evident purpose and intent of this requirement is to of which petitioner was the publisher-editor was padlocked and sealed. The
limit the things to be seized to those, and only those, particularly described in the consequence is, the printing and publication of said newspaper were discontinued. In
search warrant- to leave the officers of the law with no discretion regarding what Burgos, Sr. vs. Chief of Staff of the Armed Forces of the Philippines, supra, We held
articles they should seize, to the end that unreasonable searches and seizures may that "[sluch closure is in the nature of previous restraint or censorship abhorrent to
not be committed, that abuses may not be committed Bache & Co. Phil. Inc. vs, the freedom of the press guaranteed under the fundamental law, and constitutes a
Ruiz,supra)." The affidavit of Col. Castillo states that in several issues of the virtual denial of petitioners' freedom to express themselves in print. This state of
Philippine Times: being is patently anathematic to a democratic framework where a free, alert and
... we found that the said publication in fact foments distrust and hatred against the even militant press is essential for the political enlightenment and growth of the
government of the Philippines and its duly constituted authorities, defined and citizenry."
penalized by Article 142 of the Revised Penal Code as amended by Presidential Finally, respondents argue that while the search warrant was issued on September
Decree No. 1835; (p. 22, Rollo) 29, 1983 and was executed on the very same day, it was only on November 6, 1984,
and, the affidavit of Lt. Ignacio reads, among others or one (1) year, one (1) month and six (6) days when petitioner filed his motion for
the recall of the warrant and the return of the documents/personal properties.
... the said periodical published by Rommel Corro, contains articles tending to incite Having failed to act seasonably, respondents claim that petitioner is guilty of laches.
distrust and hatred for the Government of the Philippines or any of its duly
constituted authorities. (p. 23, Rollo) Laches is the failure or neglect, for an unreasonable and unexplained length of time,
to do that which by exercising due diligence, could or should have been done earlier.
The above statements are mere conclusions of law and will not satisfy the The negligence or omission to assert a right within a reasonable time, warranting a
requirements of probable cause. They can not serve as basis for the issuance of presumption that the party entitled to assert it either has abandoned it or declined
search warrant, absent of the existence of probable cause. In fact, as a consequence to assert it (Tijam vs. Sibonghanoy, L-21450, April 15, 1968, 23 SCRA 35).
of the search warrant issued, the items confiscated from the premises of the office of
the Philippine Times at 610 Mezzanine Floor, Gochengco Bldg., T.M. Kalaw, Ermita, In his petition, Corro alleged that on October 1, 1983, less than forty-two (42) hours
Manila were the following: after the military operatives shut down his newspaper on September 29, 1983, he
was invited by the Director-General PC/INP, and subsequently detained. Thereafter,
1. One bundle of assorted negative; he was charged with the crime of inciting to sedition before the City Fiscal's Office in
Quezon City, and on October 7, 1983, a preventive detention action was served
2. One bundle of assorted lay out; upon him. Consequently, he had to file a petition for habeas corpus. It was only on
3. Three folders of assorted articles/writings used by Philippine Times news and November 8, 1984 when this Court issued its Resolution in G.R. No. 68976, entitled:
other paraphernalias; In the Matter of the Petition for Habeas Corpus of Rommel Corro Angle Corro vs.
Minister Juan Ponce Enrile, et al., releasing Rommel Corro on recognizance of his
lawyers, Attys. Humberto B. Basco, Reynaldo Bagatsing and Edilberto Balce, In the

38
same month, November 1984, petitioner filed his motion to recall warrant and to
return the seized documents. When respondent judge denied the motion, he came
to Us.
Considering the above circumstances, the claim that petitioner had abandoned his
right to the possession of the seized properties is incorrect.
WHEREFORE, Search Warrant No. Q-00002 issued by the respondent judge on
September 29, 1983 is declared null and void and, accordingly, SET ASIDE.
The prayer for a writ of mandatory injunction for the return of the seized articles is
GRANTED and all properties seized thereunder are hereby ordered RELEASED to
petitioner. Further, respondents Lt. Col. Berlin A. Castillo and lst Lt. Godofredo M.
Ignacio are ordered to RE-OPEN the padlocked office premises of the Philippine
Times at 610 Mezzanine Floor, Gochengco Bldg., T.M. Kalaw, Ermita, Manila. SO
ORDERED.

39
BACHE & CO. (PHIL.) V. RUIZ agents nevertheless proceeded with their search which yielded six boxes of
documents.
[G.R. No. L-32409. February 27, 1971.] On March 3, 1970, petitioners filed a petition with the Court of First Instance of Rizal
praying that the search warrant be quashed, dissolved or recalled, that preliminary
BACHE & CO. (PHIL.), INC. and FREDERICK E. SEGGERMAN, Petitioners, v. prohibitory and mandatory writs of injunction be issued, that the search warrant be
HON. JUDGE VIVENCIO M. RUIZ, MISAEL P. VERA, in his capacity as declared null and void, and that the respondents be ordered to pay petitioners,
Commissioner of Internal Revenue, ARTURO LOGRONIO, RODOLFO DE jointly and severally, damages and attorneys fees. On March 18, 1970, the
LEON, GAVINO VELASQUEZ, MIMIR DELLOSA, NICANOR ALCORDO, JOHN respondents, thru the Solicitor General, filed an answer to the petition. After hearing,
DOE, JOHN DOE, JOHN DOE, and JOHN DOE, Respondents. the court, presided over by respondent Judge, issued on July 29, 1970, an order
DECISION dismissing the petition for dissolution of the search warrant. In the meantime, or on
April 16, 1970, the Bureau of Internal Revenue made tax assessments on petitioner
VILLAMOR, J.: corporation in the total sum of P2,594,729.97, partly, if not entirely, based on the
documents thus seized. Petitioners came to this Court.
This is an original action of certiorari, prohibition and mandamus, with prayer for a
writ of preliminary mandatory and prohibitory injunction. In their petition Bache & The petition should be granted for the following reasons:
Co. (Phil.), Inc., a corporation duly organized and existing under the laws of the
Philippines, and its President, Frederick E. Seggerman, pray this Court to declare null 1. Respondent Judge failed to personally examine the complainant and his witness.
and void Search Warrant No. 2-M-70 issued by respondent Judge on February 25, The pertinent provisions of the Constitution of the Philippines and of the Revised
1970; to order respondents to desist from enforcing the same and/or keeping the Rules of Court are:
documents, papers and effects seized by virtue thereof, as well as from enforcing
the tax assessments on petitioner corporation alleged by petitioners to have been "(3) The right of the people to be secure in their persons, houses, papers and effects
made on the basis of the said documents, papers and effects, and to order the against unreasonable searches and seizures shall not be violated, and no warrants
return of the latter to petitioners. We gave due course to the petition but did not shall issue but upon probable cause, to be determined by the judge after
issue the writ of preliminary injunction prayed for therein. examination under oath or affirmation of the complainant and the witnesses he may
produce, and particularly describing the place to be searched, and the persons or
The pertinent facts of this case, as gathered from record, are as follows: things to be seized." (Art. III, Sec. 1, Constitution.)
On February 24, 1970, respondent Misael P. Vera, Commissioner of Internal Revenue, "SEC. 3. Requisites for issuing search warrant. A search warrant shall not issue but
wrote a letter addressed to respondent Judge Vivencio M. Ruiz requesting the upon probable cause in connection with one specific offense to be determined by the
issuance of a search warrant against petitioners for violation of Section 46(a) of the judge or justice of the peace after examination under oath or affirmation of the
National Internal Revenue Code, in relation to all other pertinent provisions thereof, complainant and the witnesses he may produce, and particularly describing the
particularly Sections 53, 72, 73, 208 and 209, and authorizing Revenue Examiner place to be searched and the persons or things to be seized.
Rodolfo de Leon, one of herein respondents, to make and file the application for
search warrant which was attached to the letter. "No search warrant shall issue for more than one specific offense.

In the afternoon of the following day, February 25, 1970, respondent De Leon and his "SEC. 4. Examination of the applicant. The judge or justice of the peace must,
witness, respondent Arturo Logronio, went to the Court of First Instance of Rizal. before issuing the warrant, personally examine on oath or affirmation the
They brought with them the following papers: respondent Veras aforesaid letter- complainant and any witnesses he may produce and take their depositions in
request; an application for search warrant already filled up but still unsigned by writing, and attach them to the record, in addition to any affidavits presented to
respondent De Leon; an affidavit of respondent Logronio subscribed before him." (Rule 126, Revised Rules of Court.)
respondent De Leon; a deposition in printed form of respondent Logronio already
The examination of the complainant and the witnesses he may produce, required by
accomplished and signed by him but not yet subscribed; and a search warrant
Art. III, Sec. 1, par. 3, of the Constitution, and by Secs. 3 and 4, Rule 126 of the
already accomplished but still unsigned by respondent Judge.
Revised Rules of Court, should be conducted by the judge himself and not by others.
At that time respondent Judge was hearing a certain case; so, by means of a note, The phrase "which shall be determined by the judge after examination under oath or
he instructed his Deputy Clerk of Court to take the depositions of respondents De affirmation of the complainant and the witnesses he may produce," appearing in the
Leon and Logronio. After the session had adjourned, respondent Judge was informed said constitutional provision, was introduced by Delegate Francisco as an
that the depositions had already been taken. The stenographer, upon request of amendment to the draft submitted by the Sub-Committee of Seven. The following
respondent Judge, read to him her stenographic notes; and thereafter, respondent discussion in the Constitutional Convention (Laurel, Proceedings of the Philippine
Judge asked respondent Logronio to take the oath and warned him that if his Constitutional Convention, Vol. III, pp. 755-757) is enlightening:
deposition was found to be false and without legal basis, he could be charged for
"SR. ORENSE. Vamos a dejar compaero los piropos y vamos al grano.
perjury. Respondent Judge signed respondent de Leons application for search
warrant and respondent Logronios deposition, Search Warrant No. 2-M-70 was then En los casos de una necesidad de actuar inmediatamente para que no se frusten los
sign by respondent Judge and accordingly issued. fines de la justicia mediante el registro inmediato y la incautacion del cuerpo del
delito, no cree Su Seoria que causaria cierta demora el procedimiento apuntado en
Three days later, or on February 28, 1970, which was a Saturday, the BIR agents
su enmienda en tal forma que podria frustrar los fines de la justicia o si Su Seoria
served the search warrant petitioners at the offices of petitioner corporation on
encuentra un remedio para esto casos con el fin de compaginar los fines de la
Ayala Avenue, Makati, Rizal. Petitioners lawyers protested the search on the ground
justicia con los derechos del individuo en su persona, bienes etcetera, etcetera.
that no formal complaint or transcript of testimony was attached to the warrant. The

40
"SR. FRANCISCO. No puedo ver en la practica el caso hipottico que Su Seoria "A And after finishing reading the stenographic notes, the Honorable Judge
pregunta por la siguiente razon: el que solicita un mandamiento de registro tiene requested or instructed them, requested Mr. Logronio to raise his hand and warned
que hacerlo por escrito y ese escrito no aparecer en la Mesa del Juez sin que alguien him if his deposition will be found to be false and without legal basis, he can be
vaya el juez a presentar ese escrito o peticion de sucuestro. Esa persona que charged criminally for perjury. The Honorable Court told Mr. Logronio whether he
presenta el registro puede ser el mismo denunciante o alguna persona que solicita affirms the facts contained in his deposition and the affidavit executed before Mr.
dicho mandamiento de registro. Ahora toda la enmienda en esos casos consiste en Rodolfo de Leon.
que haya peticion de registro y el juez no se atendra solamente a sea peticion sino
que el juez examiner a ese denunciante y si tiene testigos tambin examiner a los "Q And thereafter?
testigos. "A And thereafter, he signed the deposition of Mr. Logronio.
"SR. ORENSE. No cree Su Seoria que el tomar le declaracion de ese denunciante "Q Who is this he?
por escrito siempre requeriria algun tiempo?.
"A The Honorable Judge.
"SR. FRANCISCO. Seria cuestio de un par de horas, pero por otro lado minimizamos
en todo lo posible las vejaciones injustas con la expedicion arbitraria de los "Q The deposition or the affidavit?
mandamientos de registro. Creo que entre dos males debemos escoger. el menor.
"A The affidavit, Your Honor."
x x x
Thereafter, respondent Judge signed the search warrant.
"MR. LAUREL. . . . The reason why we are in favor of this amendment is because we
The participation of respondent Judge in the proceedings which led to the issuance
are incorporating in our constitution something of a fundamental character. Now,
of Search Warrant No. 2-M-70 was thus limited to listening to the stenographers
before a judge could issue a search warrant, he must be under the obligation to
readings of her notes, to a few words of warning against the commission of perjury,
examine personally under oath the complainant and if he has any witness, the
and to administering the oath to the complainant and his witness. This cannot be
witnesses that he may produce . . ."
consider a personal examination. If there was an examination at all of the
The implementing rule in the Revised Rules of Court, Sec. 4, Rule 126, is more complainant and his witness, it was the one conducted by the Deputy Clerk of Court.
emphatic and candid, for it requires the judge, before issuing a search warrant, to But, as stated, the Constitution and the rules require a personal examination by the
"personally examine on oath or affirmation the complainant and any witnesses he judge. It was precisely on account of the intention of the delegates to the
may produce . . ." Constitutional Convention to make it a duty of the issuing judge to personally
examine the complainant and his witnesses that the question of how much time
Personal examination by the judge of the complainant and his witnesses is would be consumed by the judge in examining them came up before the
necessary to enable him to determine the existence or non-existence of a probable Convention, as can be seen from the record of the proceedings quoted above. The
cause, pursuant to Art. III, Sec. 1, par. 3, of the Constitution, and Sec. 3, Rule 126 of reading of the stenographic notes to respondent Judge did not constitute sufficient
the Revised Rules of Court, both of which prohibit the issuance of warrants except compliance with the constitutional mandate and the rule; for by that manner
"upon probable cause." The determination of whether or not a probable cause exists respondent Judge did not have the opportunity to observe the demeanor of the
calls for the exercise of judgment after a judicial appraisal of facts and should not be complainant and his witness, and to propound initial and follow-up questions which
allowed to be delegated in the absence of any rule to the contrary. the judicial mind, on account of its training, was in the best position to conceive.
In the case at bar, no personal examination at all was conducted by respondent These were important in arriving at a sound inference on the all-important question
Judge of the complainant (respondent De Leon) and his witness (respondent of whether or not there was probable cause.
Logronio). While it is true that the complainants application for search warrant and 2. The search warrant was issued for more than one specific offense.
the witness printed-form deposition were subscribed and sworn to before
respondent Judge, the latter did not ask either of the two any question the answer to Search Warrant No. 2-M-70 was issued for" [v]iolation of Sec. 46(a) of the National
which could possibly be the basis for determining whether or not there was probable Internal Revenue Code in relation to all other pertinent provisions thereof particularly
cause against herein petitioners. Indeed, the participants seem to have attached so Secs. 53, 72, 73, 208 and 209." The question is: Was the said search warrant issued
little significance to the matter that notes of the proceedings before respondent "in connection with one specific offense," as required by Sec. 3, Rule 126?
Judge were not even taken. At this juncture it may be well to recall the salient facts.
To arrive at the correct answer it is essential to examine closely the provisions of the
The transcript of stenographic notes (pp. 61-76, April 1, 1970, Annex J-2 of the
Tax Code referred to above. Thus we find the following:
Petition) taken at the hearing of this case in the court below shows that per
instruction of respondent Judge, Mr. Eleodoro V. Gonzales, Special Deputy Clerk of Sec. 46(a) requires the filing of income tax returns by corporations.
Court, took the depositions of the complainant and his witness, and that
stenographic notes thereof were taken by Mrs. Gaspar. At that time respondent Sec. 53 requires the withholding of income taxes at source.
Judge was at the sala hearing a case. After respondent Judge was through with the
Sec. 72 imposes surcharges for failure to render income tax returns and for
hearing, Deputy Clerk Gonzales, stenographer Gaspar, complainant De Leon and
rendering false and fraudulent returns.
witness Logronio went to respondent Judges chamber and informed the Judge that
they had finished the depositions. Respondent Judge then requested the Sec. 73 provides the penalty for failure to pay the income tax, to make a return or to
stenographer to read to him her stenographic notes. Special Deputy Clerk Gonzales supply the information required under the Tax Code.
testified as follows:
Sec. 208 penalizes" [a]ny person who distills, rectifies, repacks, compounds, or
manufactures any article subject to a specific tax, without having paid the privilege

41
tax therefore, or who aids or abets in the conduct of illicit distilling, rectifying, Books of accounts, financial records, vouchers, journals, correspondence, receipts,
compounding, or illicit manufacture of any article subject to specific tax . . .," and ledgers, portfolios, credit journals, typewriters, and other documents and/or paper
provides that in the case of a corporation, partnership, or association, the official showing all business transactions including disbursement receipts, balance sheets
and/or employee who caused the violation shall be responsible. and related profit and loss statements.

Sec. 209 penalizes the failure to make a return of receipts, sales, business, or gross "Thus, the warrants authorized the search for and seizure of records pertaining to all
value of output removed, or to pay the tax due thereon. business transactions of petitioners herein, regardless of whether the transactions
were legal or illegal. The warrants sanctioned the seizure of all records of the
The search warrant in question was issued for at least four distinct offenses under petitioners and the aforementioned corporations, whatever their nature, thus openly
the Tax Code. The first is the violation of Sec. 46(a), Sec. 72 and Sec. 73 (the filing of contravening the explicit command of our Bill of Rights that the things to be
income tax returns), which are interrelated. The second is the violation of Sec. 53 seized be particularly described as well as tending to defeat its major objective:
(withholding of income taxes at source). The third is the violation of Sec. 208 the elimination of general warrants."
(unlawful pursuit of business or occupation); and the fourth is the violation of Sec.
209 (failure to make a return of receipts, sales, business or gross value of output While the term "all business transactions" does not appear in Search Warrant No. 2-
actually removed or to pay the tax due thereon). Even in their classification the six M-70, the said warrant nevertheless tends to defeat the major objective of the Bill of
above-mentioned provisions are embraced in two different titles: Secs. 46(a), 53, 72 Rights, i.e., the elimination of general warrants, for the language used therein is so
and 73 are under Title II (Income Tax); while Secs. 208 and 209 are under all-embracing as to include all conceivable records of petitioner corporation, which, if
seized, could possibly render its business inoperative.
Title V (Privilege Tax on Business and Occupation).
In Uy Kheytin, Et. Al. v. Villareal, etc., Et Al., 42 Phil. 886, 896, this Court had
Respondents argue that Stonehill, Et. Al. v. Diokno, Et Al., L-19550, June 19, 1967 (20 occasion to explain the purpose of the requirement that the warrant should
SCRA 383), is not applicable, because there the search warrants were issued for particularly describe the place to be searched and the things to be seized, to wit:
"violation of Central Bank Laws, Internal Revenue (Code) and Revised Penal Code;"
whereas, here Search Warrant No 2-M-70 was issued for violation of only one code, ". . . Both the Jones Law (sec. 3) and General Orders No. 58 (sec. 97) specifically
i.e., the National Internal Revenue Code. The distinction more apparent than real, require that a search warrant should particularly describe the place to be searched
because it was precisely on account of the Stonehill incident, which occurred and the things to be seized. The evident purpose and intent of this requirement is to
sometime before the present Rules of Court took effect on January 1, 1964, that this limit the things to be seized to those, and only those, particularly described in the
Court amended the former rule by inserting therein the phrase "in connection with search warrant to leave the officers of the law with no discretion regarding what
one specific offense," and adding the sentence "No search warrant shall issue for articles they shall seize, to the end that unreasonable searches and seizures may
more than one specific offense," in what is now Sec. 3, Rule 126. Thus we said in not be made, that abuses may not be committed. That this is the correct
Stonehill: interpretation of this constitutional provision is borne out by American authorities."
"Such is the seriousness of the irregularities committed in connection with the The purpose as thus explained could, surely and effectively, be defeated under the
disputed search warrants, that this Court deemed it fit to amend Section 3 of Rule search warrant issued in this case.
122 of the former Rules of Court that a search warrant shall not issue but upon
probable cause in connection with one specific offense. Not satisfied with this A search warrant may be said to particularly describe the things to be seized when
qualification, the Court added thereto a paragraph, directing that no search warrant the description therein is as specific as the circumstances will ordinarily allow
shall issue for more than one specific offense." (People v. Rubio; 57 Phil. 384); or when the description expresses a conclusion of
fact not of law by which the warrant officer may be guided in making the
3. The search warrant does not particularly describe the things to be seized. search and seizure (idem., dissent of Abad Santos, J.,); or when the things described
are limited to those which bear direct relation to the offense for which the warrant is
The documents, papers and effects sought to be seized are described in Search being issued (Sec. 2, Rule 126, Revised Rules of Court). The herein search warrant
Warrant No. 2-M-70 in this manner: does not conform to any of the foregoing tests. If the articles desired to be seized
"Unregistered and private books of accounts (ledgers, journals, columnars, receipts have any direct relation to an offense committed, the applicant must necessarily
and disbursements books, customers ledgers); receipts for payments received; have some evidence, other than those articles, to prove the said offense; and the
certificates of stocks and securities; contracts, promissory notes and deeds of sale; articles subject of search and seizure should come in handy merely to strengthen
telex and coded messages; business communications, accounting and business such evidence. In this event, the description contained in the herein disputed
records; checks and check stubs; records of bank deposits and withdrawals; and warrant should have mentioned, at least, the dates, amounts, persons, and other
records of foreign remittances, covering the years 1966 to 1970." pertinent data regarding the receipts of payments, certificates of stocks and
securities, contracts, promissory notes, deeds of sale, messages and
The description does not meet the requirement in Art III, Sec. 1, of the Constitution, communications, checks, bank deposits and withdrawals, records of foreign
and of Sec. 3, Rule 126 of the Revised Rules of Court, that the warrant should remittances, among others, enumerated in the warrant.
particularly describe the things to be seized.
Respondents contend that certiorari does not lie because petitioners failed to file a
In Stonehill, this Court, speaking thru Mr. Chief Justice Roberto Concepcion, said: motion for reconsideration of respondent Judges order of July 29, 1970. The
contention is without merit. In the first place, when the questions raised before this
"The grave violation of the Constitution made in the application for the contested
Court are the same as those which were squarely raised in and passed upon by the
search warrants was compounded by the description therein made of the effects to court below, the filing of a motion for reconsideration in said court
be searched for and seized, to wit:
beforecertiorari can be instituted in this Court is no longer a prerequisite. (Pajo, etc.,

42
Et. Al. v. Ago, Et Al., 108 Phil., 905). In the second place, the rule requiring the filing The tax assessments referred to earlier in this opinion were, if not entirely as
of a motion for reconsideration before an application for a writ of certiorari can be claimed by petitioners at least partly as in effect admitted by respondents
entertained was never intended to be applied without considering the based on the documents seized by virtue of Search Warrant No. 2-M-70.
circumstances. (Matutina v. Buslon, Et Al., 109 Phil., 140.) In the case at bar time is Furthermore, the fact that the assessments were made some one and one-half
of the essence in view of the tax assessments sought to be enforced by respondent months after the search and seizure on February 25, 1970, is a strong indication that
officers of the Bureau of Internal Revenue against petitioner corporation, On account the documents thus seized served as basis for the assessments. Those assessments
of which immediate and more direct action becomes necessary. (Matute v. Court of should therefore not be enforced.
Appeals, Et Al., 26 SCRA 768.) Lastly, the rule does not apply where, as in this case,
the deprivation of petitioners fundamental right to due process taints the PREMISES CONSIDERED, the petition is granted. Accordingly, Search Warrant No. 2-
proceeding against them in the court below not only with irregularity but also with M-70 issued by respondent Judge is declared null and void; respondents are
nullity. (Matute v. Court of Appeals, Et Al., supra.) permanently enjoined from enforcing the said search warrant; the documents,
papers and effects seized thereunder are ordered to be returned to petitioners; and
It is next contended by respondents that a corporation is not entitled to protection respondent officials the Bureau of Internal Revenue and their representatives are
against unreasonable search and seizures. Again, we find no merit in the contention. permanently enjoined from enforcing the assessments mentioned in Annex "G" of
the present petition, as well as other assessments based on the documents, papers
"Although, for the reasons above stated, we are of the opinion that an officer of a and effects seized under the search warrant herein nullified, and from using the
corporation which is charged with a violation of a statute of the state of its creation, same against petitioners in any criminal or other proceeding. No pronouncement as
or of an act of Congress passed in the exercise of its constitutional powers, cannot to costs.
refuse to produce the books and papers of such corporation, we do not wish to be
understood as holding that a corporation is not entitled to immunity, under the 4th Concepcion, C.J., Dizon, Makalintal, Zaldivar, Fernando, Teehankee and Makasiar, JJ.,
Amendment, against unreasonable searches and seizures. A corporation is, after all, concur.
but an association of individuals under an assumed name and with a distinct legal
entity. In organizing itself as a collective body it waives no constitutional immunities Reyes, J.B.L., J., concurs with Mr. Justice Barredo.
appropriate to such body. Its property cannot be taken without compensation. It can
only be proceeded against by due process of law, and is protected, under the 14th Castro, J., concurs in the result.
Amendment, against unlawful discrimination . . ." (Hale v. Henkel, 201 U.S. 43, 50 L.
ed. 652.)
"In Linn v. United States, 163 C.C.A. 470, 251 Fed. 476, 480, it was thought that a
different rule applied to a corporation, the ground that it was not privileged from
producing its books and papers. But the rights of a corporation against unlawful
search and seizure are to be protected even if the same result might have been
achieved in a lawful way." (Silverthorne Lumber Company, Et. Al. v. United States of
America, 251 U.S. 385, 64 L. ed. 319.)
In Stonehill, Et. Al. v. Diokno, Et Al., supra, this Court impliedly recognized the right
of a corporation to object against unreasonable searches and seizures, thus:
"As regards the first group, we hold that petitioners herein have no cause of action
to assail the legality of the contested warrants and of the seizures made in
pursuance thereof, for the simple reason that said corporations have their respective
personalities, separate and distinct from the personality of herein petitioners,
regardless of the amount of shares of stock or the interest of each of them in said
corporations, whatever, the offices they hold therein may be. Indeed, it is well
settled that the legality of a seizure can be contested only by the party whose rights
have been impaired thereby, and that the objection to an unlawful search and
seizure is purely personal and cannot be availed of by third parties. Consequently,
petitioners herein may not validly object to the use in evidence against them of the
documents, papers and things seized from the offices and premises of the
corporations adverted to above, since the right to object to the admission of said
papers in evidence belongs exclusively to the corporations, to whom the seized
effects belong, and may not be invoked by the corporate officers in proceedings
against them in their individual capacity . . ."
In the Stonehill case only the officers of the various corporations in whose offices
documents, papers and effects were searched and seized were the petitioners. In
the case at bar, the corporation to whom the seized documents belong, and whose
rights have thereby been impaired, is itself a petitioner. On that score, petitioner
corporation here stands on a different footing from the corporations in Stonehill.

43
OLAES V. PEOPLE review of the trial court's findings of fact hes within the exclusive
appellate jurisdiction of the Court of Appeals.
G.R. Nos. 78347-49 November 9, 1987 We reiterate the rule here.
ADOLFO OLAES and LINDA M. CRUZ, petitioners, Even so, the Court has decided, without detracting from the validity of the above-
vs. cited observations., to deviate from the established procedure on this matter and to
PEOPLE OF THE PHILIPPINES and HON. JUDGE ALICIA L. SANTOS (In her categorically resolve the issues presented iii the case before us. The challenged
capacity as Presiding Judge of the Regional Trial Court of Olongapo City, orders are, indeed, interlocutory. Nevertheless, a restatement of the principles
Branch 73), respondents. governing such issues wilt it is expected, simplify the proceedings in the court.
CRUZ, J: below and speed up the disposition of the criminal case against the petitioners.

In this petition for certiorari and prohibition with preliminary injunction, the The petitioners claim that the search warrant issued by the respondent judge is
petitioners challenge the admission by the respondent judge of evidence seized by unconstitutional because it does not indicate the specific offense they are supposed
virtue of an allegedly invalid March warrant and of an extrajudicial confession taken to have committed. There is, therefore, according to them, no valid finding of
from them without according them the right to assistance of counsel. 1 They seek to probable cause as a justification for the issuance of the said warrant in conformity
restrain further proceedings in the criminal case against them for violation of the with the Bill of Rights. In support of this argument, they cite Stonehill v.
Dangerous Drugs Act (which we have suspended) 2 and ask that they be acquitted Diokno, 6 where Chief Justice Concepcion struck down the search warrants issued
with the setting aside of the questioned orders. therein for being based on the general allegation that the petitioners had committed
violations of "Central Bank Laws, Tariff and Customs Laws, Internal Revenue Code
The Solicitor General, in his Comment, suggests that the petition should be and Revised Penal Code." He declared:
dismissed as it is not alleged therein that the respondent judge has committed grave
abuse of discretion or acted without or in excess of jurisdiction. He adds that if any In other words, no specific offense had been alleged in said
reversible error has been committed, it may be corrected not in this petition but in applications. The averments thereof with respect to the offense
an ordinary appeal, which may not even be necessary if the petitioners are committed were abstract. As a consequence, it was impossible for
exonerated. 3 the judges who issued the warrants to have found the existence of
probable cause, for the same presupposes the introduction of
The petitioners, in their Reply, do not meet these arguments head-on, thus impliedly competent proof that the party against whom it is sought has
admitting the formal defect in their petition, but subject that technicalities should performed particular acts, or committed specific omissions,
yield to substantial questions in the interest of justice and to avoid unnecessarilyor violating a given provision of our criminal law.
protracted litigation. Their contention is that since there are important constitutional
issues involved, these questions should disposition of their case 4 be decided in this We have examined the search warrant issued in the instant case and find it does not
petition instead of having them debated and resolved first in the lower court in come under the structures of the Stonehill doctrine. In the case cited, there was a
acconce with the usual procedure, to the prejudice of the speedy bare reference to the laws in general, without any specification of the particular
sections thereof that were alleged to have been violated out of the hundreds of
We are not usually persuaded by this kind of argument, since procedural rules are prohibitions contained in such modifications. There is no similar ambiguity in the
intended precisely to insure an orderly administration of justice. Rights are best instant case.
established in accordance with the procedure laid down by the adjective law, which
is as binding on the parties as the substantive law since they are supposed to While it is true that the caption of the search warrant states that it is in connection
complement each other. The Solicitor General is obviously correct in faulting the with "Violation of RA 6425, otherwise known as the Dangerous Drugs Acts of 1972,"
petition and in contending that, besides being defective, it is not the proper remedy it is clearly recited in the text thereof that "There is probable cause to believe that
at this time. There is no disputing this stand. Adolfo Olaes alias "Debie" and alias "Baby" of No. 628 Comia St., Filtration, Sta. Rita,
Olongapo City, has in their possession and control and custody of marijuana dried
Worthy of note in this connection is The separate opinion of the present Chief Justice stalks/leaves/seeds/cigarettes and other regulated/prohibited and exempt narcotics
in Joseph v. Vilialuz, 5 where he declared that: preparations which is the subject of the offense stated above." 7 Although the
specific section of the Dangerous Drugs Act is not pinpointed, there is no question at
. . . the Court adheres to the settled rule that it will not overrule in all of the specific offense alleged to have been committed as a basis for the finding
a special civil action the trial court's interlocutory order denying a of probable cause. The search warrant also satisfies the requirement in the Bill of
motion to dismiss for failure or insufficiency of the prosecution's Rights of the particularity of the description to be made of the "place to be searched
evidence since it cannot review in such special civil action the and the persons or things to be seized."
prosution's evidence and decide here and now in advance that it
has or has not established beyond reasonable doubt the guilt of The petitioners also fault the admission of the extrajudicial confessions which they
the petitioners-accused. The orderly procedure prescribed by the had given without the assistance or advice of counsel and cite Section 20 of the Bill
Rules of Court is for the accused to present their evidence after of Rights of the 1973 Constitution providing that "any confession obtained in
which the trial court will on the basis of the evidence presented violation of this section shall be inadmissible in evidence."
before it by both the prosecution and the defense render its
judgment of conviction or acquittal. If the verdict be one of In the separate sworn statements taken from Adolfo Olaes and Linda Cruz on
acquittal, the case ends there. If it be a verdict of conviction, then September 24, 1982, 8 it appears that both petitioners were, before being examined,
appeal is the proper remedy - and such appeal in order to have a specifically informed of their right to the assistance of counsel, which would be
provided them by the investigating office at their request. Asked if they understood,

44
they said "Opo" and affixed their signatures opposite their answer. This was followed
by a statement entitled "Pagpapatunay" or Verification in which they saidinter
alia that they did not need the assistance of counsel ("Hindi ko na kailangan and
tulong ng isang manananggol.") which they also signed. It was only after these
preliminary precautions had been taken that the interrogation began and was
recorded in the sworn statement later introduced against them at their trial. There is
no claim that any force, violence, intimidation or threat or any means vitiating the
free wig was employed against them. Their only objection to the extrajudicial
confessions is that they were obtained without the assistance of counsel. They do
not aver in their petition that they were not apprised of their right to counsel or that
they were denied the assistance of counsel when they asked for it, or, indeed, that
they had asked for it.
Even so, their investigation did not conform to the requirements laid down in People
v. Galit, 9 where we declared:
At the time a person is arrested, it shall be the duty of the
arresting officer to inform him of the reason for the arrest and he
must be shown the warrant of arrest, if any, He shall be informed
of his constitutional rights to remain silent and to counsel, and that
any statement he might make could be used against him. The
person arrested shall have the right to communicate with his
lawyer, a relative, or anyone he chooses by the most expedient
means by telephone if possible or by letter or messenger. It
shall be the responsibility of the arresting officer to see to it that
this is accomplislied. No custodial investigation shall be conducted
unless it be in the presence of coursel engaged by the person
arrested, by any person on his behalf, or appointed by the court
upon petition either of the detainee himself or by anyone on his
behalf. The right to counsel may be waived but the waiver shall
not be valid unless made with the assistance of counsel. Any
statement obtained in violation of the procedure herein laid down,
whether exculpatory or inculpatory, in whole or in part, shall be
inadmissible in evidence.
These requirements were made even stricter under Article III, Section 12 of the 1987
Constitution, providing as follows:
Sec. 12. (1) Any person under investigation for the commission
offense shall have the right to be informed of his right to remain
silent and to have competent and independent counsel preferally
of his own choice. If the person cannot afford the services of
counsel, he must be provided with one. These rights cannot be
waived except in writing and in the presence of counsel.
(3) Any confession or admission obtained in violation of this or
Section 17 hereof shall be inadmissible in evidence against him.
Applying the above rules, we reach the conclusion that the extrajudicial confessions
should be declared inadmissible as evidence against the herein petitioners.
WHEREFORE, the petition is partly granted. The extrajudicial confessions are
excluded but the articles seized under the challenged search warrant may be
admitted in evidence. Our temporary restraining order of May 25, 1987, is lifted. No
costs.
SO ORDERED.
Teehankee, C.J., Narvasa, Paras and Gancayco, JJ., concur.

45
People v. Kagui Malasugui, 63 Phil. 221 (1936) circumstances above stated. Before he left his home between 4.30 and 5 o'clock
that morning, he had it full of bills because he was, as usual, on his way to purchase
G.R. No. L-44335 July 30, 1936 palay in Carcar as part of his work. The memorandum of amounts, Exhibit D, found
THE PEOPLE OF THE PHILIPPINE ISLANDS, plaintiff-appellee, in the pocketbook, Exhibit B, is in Tan Why's handwriting; and in the afternoon before
vs. his death he was given by Kaw Tin of the Mindanao Rice Industrial Company the sum
KAGUI MALASUGUI, defendant-appellant. of P150 to be invested in the purchase of palay, in addition to the P80 that remained
of the amount previously delivered to him.
Manuel Jose for appellant.
Office of the Solicitor-General Hilado for appellee. The appellant had to be searched after he had voluntarily produced the bracelets
Exhibit A and placed them on Lieutenant Jacaria's table, because, upon being asked
DIAZ, J.: if he had any thing, he tremblingly answered in the negative.
At about 5:30 o'clock in the morning of March 5, 1935, Tan Why, a Chinese The foregoing facts are inferred from the testimony of the government witnesses
merchant, a resident of Cotabato, was found lying on the ground, with several Alamada, Chua Sian, Kaw Tin, Yap Sea, Lieutenant Jacaria, and Police Sergeant
wounds in the head, on a path leading to the barrio of Carcar, Cotabato, and situated Urangut.
within the property of another Chinese named Yu Enching Sero. Tan Why received a
wound on the upper part of his forehead, which necessarily proved fatal because it Alamada testified that the deceased, shortly before he died, Kagui as his aggressor,
fractured his skull. He died as a result of this wound shortly afterward in the and the appellant was known by this name in Cotabato. He also testified that on the
Cotabato Hospital where he had been brought by an agent of authority with the aid morning in question, he saw the appellant, with a club in his hand, pass by the
of some laborers who had gone to the scene of the crime. house where he and the deceased lived. The club, Exhibit M, then with bloodstains,
was found near the place where Tan Why was wounded.
The death of Tan Why was imputed to the herein accused who was charged with the
crime of robbery with homicide. He was convicted of said crime and sentenced Chua Sian, an employee of the deceased, identified the pocketbook Exhibit B saying
to reclusion perpetua, to return the sum of P150 to the Mindanao Rice Industrial that it was the same that the deceased used to carry whenever he went to make
Company, and to indemnify the heirs of the deceased Tan Why in the sum of P1,000, purchases; that it was usually kept in a box at Tan Why's store; that the deceased in
with costs. The accused appealed therefrom and assigns five errors as committed by truth carried it when he left his store on the morning of March 5, 1935, to purchase
the lower court, which may be briefly summarized as follows: palay, and that it was then full of bills.

(1) In not having granted him a period of twenty-four hours to prepare his Kaw Tin, cashier of the Mindanao Rice Industrial Company, in turn, testified that on
defense; the night before the crime, he gave the deceased, at the latter's request the sum of
P150 to purchase palay, inasmuch as he was a buyer of said commodity for the
(2) In having denied his petition for the return of the articles taken from him company in Cotabato.
during the search of his person, without the corresponding judicial warrant;
About four meters from the place where Tan Why's body was found, there was a
(3) In having admitted Exhibits A, B, C, D, E, F, L and L-1, as evidence in the coconut tree with two dangling leaves, as if they were so arranged intentionally to
case; hide anybody who might post himself near the trunk. At the very place where the
tips of the leaves touched the ground, there were footprints presumably of
(4) In having denied his petition to dismiss for lack evidence, filed somebody who had posted who had posted himself there in ambush, without being
immediately after the fiscal had finished presenting his evidence; and
seen: the fresh footprints exactly the same size as the appellant's foot; and the said
(5) In having found him guilty of robbery with homicide, instead of bloodstained club was found very near the place. Such was the testimony of
acquitting him. Lieutenant Jacaria and Sergeant Eusebio de los Santos who inspected the scene of
the crime, particularly the latter who did so early in the morning and took said
When Tan Why was found on the morning in question, he was still alive and able to measurements with the aid of Exhibit E which is a part of a reed-grass leaf.
answer laconically "Kagui", when Moro Alamada, was among the first to approach
him, asked who had attacked him. The appellant was known by this name in The appellant testified at the trial that Lieutenant Jacaria and Sergeant Urangut had
Cotabato, whereupon Lieutenant A. Jacaria of the Constabulary ordered his forcibly and through intimidation taken from him the bracelets (Exhibit A), the
immediate arrest. The accused was arrested shortly after eight o'clock in the pocketbook (Exhibit B) and all the money which he carried (Exhibit C); and that, but
morning of the same day, and after he had been brought to Lieutenant Jacaria, who for the printing thereon, the identification card found in the pocketbook then was
had already been informed, that he had just redeemed two pairs of bracelets from blank and there was no memorandum of the kind of Exhibit D, in Tan Why's
some pawnshops of Cotabato and that he carried money, said lieutenant asked him handwriting, inside the pocketbook, thereby, insinuating that it was Lieutenant
for the bracelets and he then voluntarily and without protest produced what now Jacaria who typed or caused to be typewritten on the card Tan Why's name and
appear in the record as Exhibit A. He was later searched, without opposition or personal data and who placed Exhibit D in the pocketbook. There is nothing of record
protest on his part, and it was discovered that he also had the pocketbook (Exhibit to corroborate the appellant's imputation to said two officers; and it is unbelievable
B), containing P92 in bills (Exhibit C), Tan Why's identification card and a that they so acted because they were induced by no other motive than to comply
memorandum of amounts with some Chinese characters (Exhibit D). In one of the with their duties as agents of authority. The appellant permitted them to search his
pockets of his pants was found some change, making the total amount of money person and to take from him the articles in question to be used as evidence against
found in his possession P92.68. him in due time; at least, he neither made any objection nor even muttered a bit of
protest. Consequently, his contention that he was subjected to the rigor of an
Tan Why, the deceased, carried the pocketbook, Exhibit B, as he did on former unreasonable search to dispossess him of his effects without judicial warrant, and
occasions, a few hours before his body was found in the condition and under the that the court should have ordered their return to him when he so formally

46
requested before the trial, is unfounded. When one voluntarily submits to a search or When the search of the person detained or arrested and the seizure of the effects
consents to have it made of his person or premises, he is precluded from later found in his possession are incidental to an arrest made in conformity with the law,
complaining thereof. (Cooley, Constitutional Limitations, 8th ed., vol. I page 631.) they cannot be considered unreasonable, much less unlawful.(Weeks vs. United
The right to be secure from unreasonable search may, like every right, be waived States, 232 U. S., 652, citing favorably 1 Bishop, Crim. Proc., sec. 211; Wharton,
and such waiver may be made either expressly or impliedly. On the other hand, the Crim. Pl. & Pr., 8th ed., sec. 60; Dillon vs. O'Brien, 16 Cox, C. C., 245, Ir. L. R. 20 C. L.,
appellant was then charged with the crime, imputed to him by Tan Why before the 300; Moreno vs. Ago Chi, 12 Phil., 439; United States vs. Welsh, 247 Fed., 239;
latter's death, of having assaulted the "deceased; that he was then also known to be United States vs. Kraus, 270 Fed., 578, 582, par. 7; Garske vs. United States, 1 Fed.
carrying much money; and that a few moments before he was brought to Lieutenant [2d], 620; King vs. United States, 1 Fed. [2d], 931.) In this last case it was said:
Jacaria, and shortly after the assault on Tan Why, he was able to redeem two pairs of
bracelets from two persons to whom he had pledge them several months before. In these circumstances of search and seizure of defendant engaged in the
These are circumstances which undoubtedly warranted his arrest without a previous commission of a felony, and of which the officers had reliable information
judicial warrant, only upon a verbal order from said officer to Sergeant Urangut, or of and cause to believe, there is nothing unreasonable within the import of
the latter's own will, inasmuch as he had direct knowledge of the aggression that term in the Fourth Amendment. After the amendment, as before it, the
committed on the person of Tan Why, his violent death, the revelation made by Tan law necessarily sanctions arrest, search, and seizure of persons engaged in
Why before his death naming the appellant as the author of the of the aggression, commission of a crime, or reasonably believed to have committed a felony,
and the other circumstances already stated. This is so because under the law, without any paper warrant. This case is so plain that it suffices to say so.
members of Insular Police or Constabulary as well as those of the municipal police Anent an identical question, the Supreme Court of Virginia, in United
and of chartered cities like Manila and Baguio, and even of townships (secs. 848, States vs. Snyder, supra, said:
2463, 2564, 2165 and 2383 of the Revised Administrative Code) may make arrests
without judicial warrant, not only when a crime is committed or about to be To hold that no criminal can, in any case, be arrested and searched for the
committed in their presence but also when there is reason to believe or sufficient evidence and tokens of his crime without a warrant, would be to leave
ground to suspect that one has been committed and that it was committed by the society, to a large extent, at the mercy of the shrewdest, the most expert,
person arrested by them. (U. S. vs. Fortaleza, 12 Phil., 472; U. S. vs. Samonte, 16 and the most depraved of criminals, facilitating their escape in many
Phil., 516; U. S. vs. Batallones, 23 Phil., 46; U. S. vs. Santos, 36 Phil., 853.) An arrest instances.
made under said circumstances is not unlawful but perfectly justified; and the agent
The appellant contends that the lower court did not grant him even twenty-four
of authority who makes the arrest does not violate the provisions of article 269 of
hours to prepare his defense, thereby denying him the right afforded to every
the Revised Penal Code which defines and punishes unlawful arrest, nor infringe the
accused by section 30 of General Orders, No. 58. We have carefully examined the
constitutional precept relative to the inviolability of one's right to be secure in his
record and found nothing to convince us that said court had in truth refused to grant
person, house, papers, and effects against unreasonable search and seizures; as
him time for said purpose. To be entitled to said time as a matter of right, the
either provision of law permits, like the above cited sections of the Revised
interested party must expressly ask for it, inasmuch as the law so prescribes. The
Administrative Code, the making of arrests without judicial warrant, when there exist
truth, however, is that the appellant not only did not ask for it but, after his petition
reasonable motives therefor so that the person arrested may be brought to the
for the return of his effects to him had been denied, he forthwith asked, through his
corresponding authority. In fact such was the appellant's state and circumstances
counsel, to be arraigned and to proceed with the trial.
when he was searched and his alleged effects (Exhibits A, B, C and D with Tan Why's
identification card) were taken from him and, such being the case, it was proper, The appellant likewise contends that there was error in the admission of the
perfectly lawful, prudent and even necessary, in order to avoid any possible evidence Exhibits A, B, C, and D, alleging that they had been taken from him
"surprise or aggression from the appellant, in the search to be made and the effects through force and intimidation. The record shows that before proceeding with the
in question seized by the persons concerned, to be presented, as they were in truth trial in the lower court, the appellant asked for the return of said effects (Exhibits A,
presented to the competent authority. Section 105 of General Orders, No. 58 reads: B, C, and D) to him on the ground that they were unlawfully taken away from him.
Leaving aside the foregoing considerations, his testimony cannot prevail against nor
A person charged with a crime may be marched for dangerous weapons or
is it sufficient to counteract that of the government witnesses, Lieutenant Jacaria
anything which may be used as proof of the commission of the crime.
and Sergeant Urangut, who testified that when Lieutenant Jacaria asked him what
Article III, section 1, paragraph (3), of our Constitution is identical in all respects to other things he carried, after having voluntarily placed the two pairs of bracelets,
the Fourth Amendment of the Constitution of the United States; and said Exhibit A, on the table, and Sergeant Urangut felt his body, he did not show the least
constitutional precept has been interpreted as not prohibiting arrests, searches and opposition. It follows, therefore, that the lower court committed no error in accepting
seizures without judicial warrant, but only those that are unreasonable. as evidence Exhibits A, B, C and D, not only because the appellant did not object to
the taking thereof from him when searched, but also because the effects found in his
In United States vs. Snyder (278 Fed., 650), it was said: possession of a person detained or arrested are perfectly admissible as evidence
The Fourth Amendment, providing that "the right of the people to be secure against him, if they constitute the corpus delicti or are pertinent or relevant thereto.
in their persons, houses, papers, and effects against unreasonable search (Adamsvs. New York, 192 U.S., 585; 24 Sup. Ct., 372; 48 Law. ed., 575;
and seizures shall not be violated, and no warrant shall issue, but upon Weeks vs. United States, supra; People vs. Mayen, 205 Pac., 435.) It is certainly
probable cause, supported by oath or affirmation, and particularly repugnant to maintain the opposite view because it would amount to authorizing the
describing the place to be searched and the persons or things to be seized," return to the accused of the means of conviction seized from him, notwithstanding
contains no prohibition of arrest, search, or seizure without a warrant, but their being eloquent proofs of crime, for him to conceal, destroy or otherwise
only against "unreasonable" searches and seizures. dispose of, in order to assure his impunity.

47
The appellant attempted to prove that the money found in his possession had been
given to him, on different occasions, by the witnesses who testified in his favor, as
Kagui Guialal, Kagui Patak, Kakim, Akun or Amay Indo. Kagui Guialal, in truth,
testified that he had given the accused P90, two days before the latter's arrest;
Kagui Patak, P45, on two occasions, one month, and two weeks, respectively, also
prior to his arrest; and Kakim and Akun or Amay Indo, P22.50 and P20, seven and
five days, respectively, before the day of the crime. These four witnesses are all
relatives of the appellant; and if it were true that the latter had enough money
several days prior to the aggression and death of Tan Why, the natural thing to
assume is that he would have not redeemed the two bracelets Exhibit A, precisely
on the very morning in question. Furthermore, their testimony did not destroy the
unexplained finding in the appellant's possession, of the deceased Tan Why's
pocketbook, containing much money, and the latter's personal papers. In the
absence of an explanation of how one has come into the possession of stolen effects
belonging to a person wounded and treacherously killed, he must necessarily be
considered the author of the aggression and death of said person and of the robbery
committed on him (U.S. vs. Merin, 2 Phil., 88; U.S. vs.Divino, 18 Phil., 425).
The facts which we consider as having been fully established in view of the
foregoing considerations, constitute the crime of robbery with homicide defined and
punished with reclusion perpetua to death in article 249, subsection 1, of the
Revised Penal Code. Inasmuch as no aggravating circumstance had been proven and
the penalty imposed by the lower court is in accordance with law, taking into
consideration the rules prescribed in article 63 of said Code, the appealed judgment
is hereby affirmed, except that part thereof containing a pronouncement in favor of
the Mindanao Rice Industrial Company, a strange party to the case, which should be
entirely eliminated; and it is ordered that Exhibits B (pocketbook), C (money), D
(memorandum of amounts) and Tan Why's identification card be delivered to the
latter's heirs, with costs against the appellant. So ordered.
Avancea, C. J., Villa-Real, Abad Santos, Imperial, Recto, and Laurel, JJ., concur.

48
PEOPLE V. VELOSO, 48 PHIL. 169 (1925) In the municipal court of the City of Manila, the persons arrest in the raid were
accused of gambling. All of them were eventually acquitted in the Court of First
G.R. No. L-23051 October 20, 1925 Instance for lack of proof, with the sole exception of Veloso, who was found guilty of
maintaining a gambling house. This case reached the appellate court where the
THE PEOPLE OF THE PHILIPPINES ISLANDS, plaintiff-appellant, accused was finally sentenced to pay a fine of P500. (No. 22163. 1 )
vs.
JOSE MA. VELOSO, defendant-appellant. The foregoing are the principal facts taken mainly from the findings of the trial
judge, the Honorable Vicente Nepomuceno. Counsel for the appellant makes no
MALCOLM, J.: effort to impugn these findings, except that he stresses certain points as more
favorable to the case of his client. The defense, as previously indicated, is planted
This is an appeal from a judgment of the Court of First Instance of Manila finding the squarely on the contention that since the name of Veloso did not appear in the
accused, Jose Ma. Veloso, guilty of the crime of resistance of the agents of the search warrant, but instead the pseudonym John Doe was used, Veloso had a legal
authority, in violation of article 252 of the Penal Code, and sentencing him to four right to resist the police by force. The nature of this defense makes it advisable to
months and one day imprisonment, arresto mayor, with the accessory penalties, to set forth further facts, relating particularly to the search warrant, before passing to
pay a fine of P200, with the corresponding subsidiary imprisonment in case of the law.
insolvency, and to pay the costs. The errors assigned by counsel for the accused as
appellant, go to the proposition that the resistance of the police was justifiable on There are found in the record the application for search warrant, the affidavit for
account of the illegality of the John Doe search warrant. search warrant, and the search warrant. The application reads:
In May, 1923, the building located at No. 124 Calle Arzobispo, City of Manila, was UNITED STATES OF AMERICA
used by an organization known as the Parliamentary Club. Jose Ma. Veloso was at PHILIPPINE ISLANDS
that time a member of the House of Representative of the Philippine Legislature. He
was also the manager of the club. IN THE MUNICIPAL COURT OF THE CITY OF MANILA

The police of Manila had reliable information that the so-called Parliamentary Club THE PEOPLE OF THE PHILIPPINE ISLANDS, plaintiff, vs. JOHN DOE,
was nothing more than a gambling house. Indeed, on May 19, 1923, J. F. Townsend, Defendant.
the chief of the gambling squad, had been to the club and verified this fact. As a APPLICATION FOR (G)
result, on May 25, 1923, Detective Andres Geronimo of the secret service of the City SEARCH WARRANT
of Manila, applied for, and obtained a search warrant from Judge Garduo of the
municipal court. Thus provided, the police attempted to raid the Parliamentary Club Testimony taken before Hon. L. Garduo, Judge, Municipal Court, Manila.
a little after three in the afternoon of the date above- mentioned. They found the
Andres Geronimo, being duly sworn, testifies as follows:
doors to the premises closed and barred. Accordingly, one band of police including
policeman Rosacker, ascended a telephone pole, so as to enter a window of the Q. What is your name, residence and occupation? A. Andres
house. Other policemen, headed by Townsend, broke in the outer door. Geronimo, No. 47 Revellin, detective.
Once inside the Parliamentary Club, nearly fifty persons were apprehended by the Q. Are you the applicant of this search warrant? A. Yes, sir.
police. One of them was the defendant Veloso. Veloso asked Townsend what he
wanted, and the latter showed him the search warrant. Veloso read it and told Q. Do you know the premises situated at No. 124 Calle Arzobispo,
Townsend that he was Representative Veloso and not John Doe, and that the police District of W. C., City of Manila? A. Yes. sir.
had no right to search the house. Townsend answered that Veloso was considered as
Q. Do you know who occupies said premises? A. I do not know.
John Doe. As Veloso's pocket was bulging, as if it contained gambling utensils,
According to the best of my information the house is occupied by
Townsend required Veloso to show him the evidence of the game. About five minutes
John Doe.
was consumed in conversation between the policemen and the accused the
policemen insisting on searching Veloso, and Veloso insisting in his refusal to submit Q . What are your reasons for applying for this search warrant?
to the search. A. It has been reported to me by a person whom I consider to be
reliable that in said premises there are instruments and devices
At last the patience of the officers was exhausted. So policeman Rosacker took hold
used in gambling games, such as cards, dice, chips, lottery tickets,
of Veloso only to meet with his resistance. Veloso bit Rosacker in the right forearm,
lists of drawing and lists used in prohibited games kept. It has
and gave him a blow in another part of the body, which injured the policeman quite
been reported to me by a person whom I consider to be reliable
severely. Through the combined efforts of Townsend and Rosacker, Veloso was finally
that there are or there will be gambling conducted in said
laid down on the floor, and long sheets of paper, of reglas de monte, cards,
premises. The aforesaid premises are known as gambling house. I
cardboards, and chips were taken from his pockets.
have watched the foregoing premises and believed it to be a
All of the persons arrested were searched and then conducted to the patrol wagons. gambling house and a place where instruments and devices used
Veloso again refused to obey and shouted offensive epithets against the police in gambling games, such as cards, dice, chips, lottery tickets, lists
department. It was necessary for the policemen to conduct him downstairs. At the of drawing and lists used in prohibited games are kept.
door, Veloso resisted so tenaciously that three policemen were needed to place him
I, Andres Geronimo, being duly sworn, depose and say that I have read the
in the patrol wagon. 1awph!l.net
foregoing questions and answers and that I find the same to correct and
true to the best of my knowledge and belief.

49
(Sgd.) ANDRES GERONIMO the person of thing to be seized." (Section 97.) After the judge or justice shall have
examined on oath the complainant and any witnesses he may produce, and shall
Subscribed and sworn to before me this 25th day of May, 1923. have taken their depositions in writing (section 98), and after the judge or justice is
(Sgd.) L. GARDUO Judge, Municipal Court satisfied of the existence of facts upon which the application is based, or that there
is probable cause to believe that they exist, he must issue the warrant which must
The affidavit and the search warrant are so nearly alike that it will suffice to copy the be substantially in the following form:
search warrant alone. This document reads:
. . . You are, therefore, commanded, . . . to make immediate search on the
UNITED STATES OF AMERICA person of ............................, or in the house
PHILIPPINE ISLANDS situated ...................................... (describing it or any other place to be
searched with reasonable particularity, as the case may be) for the
IN THE MUNICIPAL COURT OF THE CITY OF MANILA
following property: . . . ." (Section 99.) It is finally provided that "a person
THE PEOPLE OF THE PHILIPPINE ISLANDS, Plaintiff, charged with a crime may be searched for dangerous weapons or anything
which may be used as proof of the commission of the crime. (Section 105).
vs.
A search warrant must conform strictly to the requirements of the constitutional and
JOHN DOE, Defendant. statutory provisions under which it is issued. Otherwise it has rightly been held,
must be absolutely legal, "for there is not a description of process known to the law,
SEARCH WARRANT (G)
the execution of which is more distressing to the citizen. Perhaps there is none which
The People of the Philippine Islands, to any member of the excites such intense feeling in consequence of its humiliating and degrading effect."
The warrant will always be construed strictly without, however, going the full length
Police Force of the City of Manila. of requiring technical accuracy. No presumptions of regularity are to be invoked in
GREETING: aid of the process when an officer undertakes to justify under it. (24 R. C. L., pp. 711,
et seq.; Reed vs. Rice [1829], 2 J. J. Marshall [Ky.] 44; 19 Am. Dec., 122; Smith vs.
Proof by affidavit having this day been made before me by Andres McDuffee [1914], 72 Ore., 276; Ann. Cas. 1916 D, 947.)
Geronimo that he has good reason to believe and does believe that John
Doe has illegally in his possession in the building occupied by him and The search warrant has been likened to a warrant of arrest. Although apprehending
which is under his control, namely in the building numbered 124 Calle that there are material differences between the two, in view of the paucity of
Arzobispo, City of Manila, Philippines Islands, certain devices and effects authority pertaining to John Doe search warrants we propose to take into
used in violation of the Gambling Law, to wit: money, cards, chips, reglas, consideration the authorities relied upon by the appellant, thus following the
pintas, tables and chairs and other utensils used in connection with the precedent of Uy Kheytin vs. Villareal ([1920], 42 Phil., 886), where the regularity of
game commonly known as monte and that the said John Doe keeps and the issuance of the search warrant was also questioned.
conceals said devices and effects with the illegal and criminal intention of In the lower court, and again in this court, the attorneys for the defense quoted from
using them in violation of the Gambling Law. Wharton's Criminal Procedure. In that text at pages 51, 52, 54, 55, and 56 of volume
Now therefore, you are hereby commanded that at any time in the day or 1 of the Tenth Edition, is found the following:
night within ten (10) days on or after this date to make a search on the Form and Sufficiency of Warrant. Technical accuracy is not required. . . .
person of said John Doe and in the house situated at No. 124 Calle
Arzobispo, City of Manila, Philippine Islands, in quest of the above described xxx xxx xxx
devices and effects and if you find the same or any part thereof, you are
Name and description of the accused should be inserted in the body of the
commanded to bring it forthwith before me as provided for by law.
warrant and where the name is unknown there must be such a description
Given under my hand, this 25th day of May, 1923. of the person accused as will enable the officer to identify him when found.

(Sgd.) L. GARDUO xxx xxx xxx


Judge, Municipal Court
Warrant for apprehension of unnamed party, or containing a wrong name
Coming now to the legal aspects of the case it is first worthy of mention that by for the party to be apprehended is void, except in those cases where it
reason of the Fourth Amendment to the United States Constitution and the eleventh contains a descriptio personae such as will enable the officer to identify the
and eighteenth paragraphs of the Philippine Bill of Rights, as found in the present accused.
Organic Act, the security of the dwelling and the person is guaranteed. The organic
xxx xxx xxx
act provides "that the right to be secured against unreasonable searches and
seizures shall not be violated." It further provides "that no warrant shall issue but John Doe' Warrants. It follows, on principle, from what has already been said
upon probable cause, supported by oath or affirmation and particularly describing regarding the essential requirements of warrants for the apprehension of
the place to be searched and the person or things to be seized." persons accused, and about blank warrants, that a warrant for the
apprehension of a person whose true name is unknown, by the name of
In the Philippine Code of Criminal Procedure are found provisions of the same import
"John Doe" or "Richard Roe," "whose other or true name in unknown," is
although naturally entering more into detail. It is therein provided, among other
void, without other and further descriptions of the person to be
things, that "a search warrant shall not issue except for probable cause and upon
apprehended, and such warrant will not justify the officer in acting under it.
application supported by oath particularly describing the place to be searched and

50
Such a warrant must, in addition, contain the best descriptio a right to resist by force, using no more than was necessary to resist the
personae possible to be obtained of the person or persons to be unlawful acts of the officer . . .
apprehended, and this description must be sufficient to indicate clearly the
proper person or persons upon whom the warrant is to be served; and The defendants, therefore, in resisting the officer in making an arrest under
should state his personal appearance and peculiarities, give his occupation the warrant in question, if they were guilty of no improper or excessive
and place of residence, and any other circumstances by means of which he force or violence, did not do an unlawful act by lawful means, or a lawful
can be identified. act by unlawful means, and so could not be convicted of the misdemeanor
of a riot, with which they are charged in the indictment.
Person apprehended in act of committing a crime, under a "John Doe"
warrant, on the other hand, the apprehension will not be illegal, or the Appellant's argument, as based on these authorities, runs something like this. The
officer liable, because under such circumstances it is not necessary that a law, constitutional and statutory, requires that the search warrant shall not issue
warrant should have been issued. unless the application "particularly" describe the person to be seized. A failure thus
to name the person is fatal to the validity of the search warrant. To justify search and
The authority most often cited to sustain the text, and quoted with approval by the arrest, the process must be legal. Illegal official action may be forcibly resisted.
United States Supreme Court, is the case of Commonwealth vs. Crotty ([1865], 10
Allen [Mass.], 403). It there appeared that one Peaslee had made a complaint to the For the prosecution, however, as the arguments are advanced by the Attorney-
police court Lee, charging that "John Doe or Richard Roe, whose other or true name General, and as the law was summarized by the trial judge, there is much to be said.
is to your complainant unknown," had committed an assault and battery upon him; Careful and logical reflection brings forth certain points of paramount force and
upon which complaint a warrant was issued against "John Doe or Richard Roe, whose exercising a decisive influence. We will now make mention of them by correlating the
other or true name is to your complainant unknown, named in the foregoing facts and the law.
complaint." Neither the complaint nor the warrant contained any further description In the first place, the affidavit for the search warrant and the search warrant itself
or means of identification of the person to be arrested. Crotty resisted the arrest described the building to be searched as "the building No. 124 Calle Arzobispo, City
upon the ground that the warrant was invalid. Mr. Chief Justice Bigelow, as the organ of Manila, Philippine Islands." This, without doubt, was a sufficient designation of the
of the Supreme Court of Massachusetts, said: premises to be searched. It is the prevailing rule that a description of a place to be
We cannot entertain a doubt that the warrant on which the officer searched is sufficient if the officer with the warrant can, with reasonable effort,
attempted to arrest one of the defendant at the time of the alleged riot was ascertain and identify the place intended. (Steele vs. U. S. [1925], U. S. Supreme
insufficient, illegal and void. It did not contain the name of the defendant, Court Advance Opinions 1924-1925; 69 Law. ed., 757). The police officers were
nor any description or designation by which he could be known and accordingly authorized to break down the door and enter the premises of the
identified as the person against whom it was issued. It was in effect a building occupied by the so-called Parliamentary Club. When inside, they then had
general warrant, upon which any other individual might as well have been the right to arrest the persons presumably engaged in a prohibited game, and to
arrested, as being included in the description, as the defendant himself. confiscate the evidence of the commission of the crime. It has been held that an
Such a warrant was contrary to elementary principles, and in direct officer making an arrest may take from the person arrested any money or property
violation of the constitutional right of the citizen, as set forth in the found upon his person, which was used in the commission of the crime or was the
Declaration of Rights, article 14, which declares that every subject has a fruit of the crime, or which may furnish the person arrested with the means of
right to be secure from all unreasonable searches and seizures of his committing violence or of escaping, or which may be used as evidence on the trial of
person, and that all warrants, therefore, are contrary to this right, if the the cause, but not otherwise. (Moreno vs. Ago Chi [1909], 12 Phil., 439.)
order in the warrant to a civil officer to arrest one or more suspected Proceeding along a different line of approach, it is undeniable that the application for
persons or to seize their property be not accompanied with a special the search warrant, the affidavit, and the search warrant failed to name Jose Ma.
designation of the persons or objects of search, arrest or seizure. This is in Veloso as the person to be seized. But the affidavit and the search warrant did state
fact only a declaration of an ancient common law right. It was always that "John Doe has illegally in his possession in the building occupied by him, and
necessary to express the name or give some description of a party to be which is under his control, namely, in the building numbered 124 Calle Arzobispo,
arrested on a warrant; and if one was granted with the name in blank, and City of Manila, Philippine Islands, certain devices and effects used in violation of the
without other designation of the person to be arrested, it was void. (1 Hale Gambling Law." Now, in this connection, it must not be forgotten that the Organic
P. C. 577. 2 Ib. 119. Foster, 312. 7 Dane Ab. 248. 1 Chit. Crim. Law, 39. Act requires a particular description of the place to be searched, and the person or
Mead vs. Haws, 7 Cow., 332, and cases cited.) things to be seized, and that the warrant in this case sufficiently described the place
This rule or principle does not prevent the issue and service of a warrant and the gambling apparatus, and, in addition, contained a description of the person
against a party whose name is unknown. In such case the best description to be seized. Under the authorities cited by the appellant, it is invariably recognized
possible of the person to be arrested is to be given in the warrant; but it that the warrant for the apprehension of an unnamed party is void, " except in those
must be sufficient to indicate clearly on whom it is to be served, by stating cases where it contains a description personae such as will enable the officer to
his occupation, his personal appearance and peculiarities, the place of his identify the accused." The description must be sufficient to indicate clearly the
residence, or other circumstances by which he can be identified. (1 Chit. proper person upon whom the warrant is to be served. As the search warrant stated
Crim. Law, 39, 40.) that John Doe had gambling apparatus in his possession in the building occupied by
him at No. 124 Calle Arzobispo, City of Manila, and as this John Doe was Jose Ma.
The warrant being defective and void on its face, the officer had no right to Veloso, the manager of the club, the police could identify John Doe as Jose Ma.
arrest the person on whom he attempted to serve it. He acted without Veloso without difficulty.
warrant and was a trespasser. The defendant whom he sought to arrest had

51
Again, it must be remembered that No. 124 Calle Arzobispo was supposed to be
used for club purposes. It was not the home of Veloso; not the place of abode of the
family, which the law carefully protects in all of its sanctity. It was a club partially
public in nature. It was, moreover, a camouflaged club with a high sounding name
calculated to mislead the police, but intended for nefarious practices. In a club of
such a character, unlike in the home, there would commonly be varying occupancy,
a number of John Does and Richard Roes whose names would be unknown to the
police.
It is also borne out by the authorities that, in defense of himself, any member of his
family or his dwelling, a man has a right to employ all necessary violence. But even
in the home, and much less so in a club or public place, the person sought to be
arrested or to be searched should use no more force than is necessary to repel the
unlawful act of the officers. To authorize resistance to the agents of the authority,
the illegality of the invasion must be clearly manifest. Here, there was possibly a
proper case for protest. There was no case for excessive violence to enforce the
defendant's idea of a debatable legal question. (Commonwealth vs. Crotty, supra;
People vs. Chan Fook [1921], 42 Phil., 230; 3 Groizard, Codigo Penal, pp. 456, 457.)
The trial judge deduced from the searched warrant that the accused Veloso was
sufficiently identified therein. Mention was made by his Honor of the code provision
relating to a complaint or information, permitting a fictitious name to be inserted in
the complaint or information, in lieu of the true name. The Attorney-General adds to
this the argument that the police were authorized to arrest without a warrant since a
crime was being committed. We find it unnecessary to comment on this contention.
John Doe search warrants should be the exception and not the rule. The police
should particularly describe the place to be searched and the person or things to be
seized, wherever and whenever it is feasible. The police should not be hindered in
the performance of their duties, which are difficult enough of performance under the
best of conditions, by superficial adherence to technicality or far fetched judicial
interference.
We agree with the trial judge and with the Attorney-General in their conclusions to
the effect that the search warrant was valid, and that the defendant has been
proved guilty beyond a reasonable doubt, of the crime of resistance of the agents of
the authority.
The information alleges that at the time of the commission of the crime, the accused
was a member of the House of Representatives. The trial court was led to consider
this allegation in relation with the facts as an aggravating circumstance, and to
sentence the accused accordingly. We doubt, however, that advantage was taken by
the offender of his public position when he resisted the officers of the law. The
offender did not necessarily make use of the prestige of his office as a means to
commit a crime. Undoubtedly, Jose Ma. Veloso, as Juan de la Cruz, would have
resisted the police just as stoutly, as the Honorable Jose Ma. Veloso did. The penalty,
accordingly, falls within the medium of that provided by the Penal Code.
Finding present no reversible error, agreeing in all respects with the findings of facts
as made by the trial judge, and concurring with the trial judge in his legal
conclusion, with one exception, it results that the judgment appealed from must be,
as it is hereby, affirmed, with the sole modification that the defendant and appellant
shall be sentenced to two months and one day imprisonment, arresto mayor, with
the costs of this instance against him. Let the corresponding order to carry this
judgment into effect issue.
Avancea, C.J., Street, Villamor, Ostrand, Johns, and Romualdez, JJ., concur.
Villa-Real, JJ., took no part.

52
NOLASCO V. CRUZ PANO, 139 SCRA 152 CPP/NPA and the National Democratic Front, including support money from foreign
and local sources intended to be used for rebellion. 1
(1985) 5. In connection with the search made at 12:00 N. of August 6th the following may
be stated:
G.R. No. L-69803 October 8, 1985
(a) TOLENTINO was a person then in charge of the premises. He was arrested by the
CYNTHIA D. NOLASCO, MILA AGUILAR-ROQUE and WILLIE C.
searching party presumably without a warrant of arrest.
TOLENTINO, petitioners,
vs. (b) The searching party seized 428 documents and written materials, 2 and
HON. ERNANI CRUZ PAO, Executive Judge, Regional Trial Court of Quezon additionally a portable typewriter, and 2 wooden boxes, making 431 items in all. 3
City; HON. ANTONIO P. SANTOS, Presiding Judge, Branch XLII, Metropolitan
Trial Court of Quezon City: HON. SERGIO F. APOSTOL, City Fiscal, Quezon (c) According to the Return, submitted in the SEARCH WARRANT CASE on August
City; HON. JUAN PONCE ENRILE, LT. GEN. FIDEL RAMOS and COL. JESUS 10th, 4 the search was made in the presence of Dra. Marciana Galang, owner of the
ALTUNA, respondents. premises, and of two (2) Barangay Tanods. No mention was made that TOLENTINO
was present. The list of the 428 articles and documents attached to the Return was
MELENCIO-HERRERA, J.: signed by the two Barangay Tanods, but not by Dra. Galang.
The facts before the Court in these Certiorari, Prohibition, and mandamus 6. (a) On August 10th, the three petitioners, AGUILAR-ROQUE, NOLASCO and
proceedings will be briefly stated. The three petitioners will be referred to through TOLENTINO, were charged before the Quezon City Fiscal's Office (the CITY FISCAL,
their surnames of NOLASCO, AGUILAR-ROQUE and TOLENTINO. for short) upon complaint filed by the CSG against petitioners for
"Subversion/Rebellion and/or Conspiracy to Commit Rebellion/Subversion."
1. Prior to August 6, 1984 (hereinafter to be referred to without the year), AGUILAR-
ROQUE was one of the accused of Rebellion in Criminal Case No. (b) On August 13th, the CITY FISCAL filed an Information for Violation of Presidential
MC-25-113 of Military Commission No. 25, both cases being entitled "People of the Decree No. 33 (Illegal Possession of Subversive Documents) against petitioners
Philippines vs. Jose Ma. Sison, et al." She was then still at large. before Branch 42 of the Metropolitan Trial Court of Quezon City (the SUBVERSIVE
DOCUMENTS CASE), respondent Judge Antonio P. Santos, presiding.
2. At 11:30 A.M. on August 6th, AGUILAR-ROQUE and NOLASCO were arrested by a
Constabulary Security Group (CSG) at the intersection of Mayon Street and P. Margall (c) On August 16th, CSG filed a Motion for Reconsideration with the CITY FISCAL,
Street, Quezon City. The stated time is an allegation of petitioners, not denied by praying that AGUILAR-ROQUE and NOLASCO be charged with Subversion. The Motion
respondents. The record does not disclose that a warrant of arrest had previously was denied on November 16th.
beeen issued against NOLASCO.
7. (a) On September 10th, the CSG submitted an Amended Return in the SEARCH
3. At 12:00 N. on August 6th, elements of the CSG searched the premises at 239-B WARRANT CASE praying, inter alia, that the CSG be allowed to retain the seized 431
Mayon Street, Quezon City. The stated time is an allegation of petitioners, not documents and articles, in connection with cases that are presently pending against
specifically denied by respondents. In their COMMENT, however, respondents have Mila Aguilar Roque before the Quezon City Fiscal's Office and the court. 5
alleged that the search was conducted "late on the same day"; that is late on august
6th. (b) On September 28th, petitioners were required by Judge Pano to comment on the
Amended Return, which AGUILAR-ROQUE did on October 18th, raising the issue of
4. On August 6th, at around 9:00 A.M., Lt. Col. Virgilio G. Saldajeno of the CSG, the inadmissibility of any evidence obtained pursuant to the Search Warrant.
applied for a Search Warrant from respondent Hon. Ernani Cruz Pao, Executive
Judge of the Regional Trial Court in Quezon City, to be served at No. 239-B Mayon (c) On December 13, 1984, Judge Pao admitted the Amended Return and ruled that
Street, Quezon City, determined tyo be the leased residence of AGUILAR-ROQUE, the seized documents "shall be subject to disposition of the tribunal trying the case
after almost a month of "round the clock surveillance" of the premises as a against respondent."
"suspected underground house of the CPP/NPA." AGUILAR-ROQUE has been long
8. (a) On December 12th, petitioners filed a Motion to Suppress in the SUBVERSIVE
wanted by the military for being a high ranking officer of the Communist Party of the
DOCUMENTS CASE, praying that such of the 431 items belonging to them be
Philippines, particularly connected with the MV Karagatan/Doa Andrea cases.
returned to them. It was claimed that the proceedings under the Search Warrant
In connection with the Search Warrant issued, the following may be stated: were unlawful. Judge Santos denied the Motion on January 7, 1985 on the ground
that the validity of the Search Warrant has to be litigated in the SEARCH WARRANT
(a) The Search Warrant was issued in proceedings entitled "People of the Philippines CASE. He was apparently not aware of the Order of Judge Pao of December 13th
vs. Mila Aguilar-Roque, Accused, Search Warrant No. 80- 84 for rebellion" (the issued in the SEARCH WARRANT CASE.
SEARCH WARRANT CASE). Judge Panos Court was Branch 88.
Hence, this Petition for Certiorari, Prohibition and mandamus to annul and set aside
(b) It does not appear from the records before us that an application in writing was the (1) Search Warrant issued by respondent RTC Judge Pao; (2) his Order admitting
submitted by Lt. Col. Saldajeno to Judge Pao. the Amended Return and granting the Motion to Retain Seized Items; and (3) Order
of respondent MTC Judge Santos denying petitioners' Motion to Suppress.
(c) According to the record, Lt. Col. Saldajeno and his witness S/A Dionicio A. Lapus,
were examined under oath by Judge Pao but only the deposition of S/A Lapus has This Court, on February 12, 1985, issued a Temporary Restraining Order enjoining
been submitted to us. The latter deposed that to his personal knowledge, there were the respondents or their duly authorized representatives from introducing evidence
kept in the premises to be searched records, documents and other papers of the obtained under the Search Warrant.

53
The PETITIONERS principally assert that the Search Warrant is void because it is a articles for publication, newspaper dummies subversive documents,
general warrant since it does not sufficiently describe with particularity the things articles, etc., and even typewriters, duplicating machines, mimeographing
subject of the search and seizure, and that probable cause has not been properly and tape recording machines. Thus, the language used is so all embracing
established for lack of searching questions propounded to the applicant's witness. as to include all conceivable records and equipment of petitioner regardless
The respondents, represented by the Solicitor General, contend otherwise, adding of whether they are legal or illegal. The search warrant under consideration
that the questions raised cannot be entertained in this present petition without was in the nature of a general warrant which is constitutionally
petitioners first moving for the quashal of the disputed Search Warrant with the objectionable. 8
issuing Judge.
The lack of particularization is also evident in the examination of the witness
We find merit in the Petition. presented by the applicant for Search Warrant.
Section 3, Article IV of the Constitution, guarantees the right of the people to be Q Mr. Dionicio Lapus, there is an application for search warrant filed by Lt.
secure in their persons, houses, papers and effects against unreasonable searches Col. Virgilio Saldajeno and the Court would like to know if you affirm the
and seizures of whatever nature and for any purpose. It also specifically provides truth of your answer in this deposition?
that no Search Warrant shall issue except upon probable cause to be determined by
the Judge or such other responsible officer as may be authorized by law, after (The deposition instead)
examination under oath or affirmation of the complainant and the witnesses he may A Yes, sir,
produce, and particularly describing the place to be searched and the things to be
seized. Q How long did it take you for the surveillance?

The disputed Search Warrant (No. 80-84) describes the personalities to be seized as A Almost a month, sir.
follows:
Q Are you a lawyer, Mr. Lapus?
Documents, papers and other records of the Communist Party of the
A No, Your Honor, but I was a student of law.
Phihppines/New Peoples Army and/or the National Democratic Front, such
as Minutes of the Party Meetings, Plans of these groups, Programs, List of Q So, you are more or less familiar with the requisites of the application for
possible supporters, subversive books and instructions, manuals not search warrant?
otherwise available to the public, and support money from foreign or local
sources. A Yes, Your Honor.

It is at once evident that the foregoing Search Warrant authorizes the seizure of Q How did you come to know of the person of Mila Aguilar-Roque?
personal properties vaguely described and not particularized. It is an all- embracing
A Because of our day and night surveillance, Your Honor, there were so
description which includes everything conceivable regarding the Communist Party of
many suspicious persons with documents.
the Philippines and the National Democratic Front. It does not specify what the
subversive books and instructions are; what the manuals not otherwise available to Q What kind of documents do you refer to?
the public contain to make them subversive or to enable them to be used for the
crime of rebellion. There is absent a definite guideline to the searching team as to A Documents related to the Communist Party of Philippines and New
what items might be lawfully seized thus giving the officers of the law discretion People's Army.
regarding what articles they should seize as, in fact, taken also were a portable Q What else?
typewriter and 2 wooden boxes. It is thus in the nature of a general warrant and
infringes on the constitutional mandate requiring particular description of the things A Conferences of the top ranking officials from the National Democratic
to be seized. In the recent rulings of this Court, search warrants of similar Front, Organization of the Communist Party of the Philippines ...
description were considered null and void for being too general. Thus:
Q And may include what else?
Subversive documents, pamphlets, leaflets, books, and other publications
to promote the objectives and purposes of the subversive organizations A Other papers and documents like Minutes of the Party Meetings, Plans of
known as Movement for Free Philippines. Light-a-Fire Movement and April 6 these groups, Programs, List of possible supporters, subversive books and
Movement. 6 instructions, manuals not otherwise available to the public and support
money from foreign and local sources. 9
The things to be seized under the warrant issued by respondent judge were
described as 'subversive documents, propaganda materials, FAs, printing The foregoing questions propounded by respondent Executive Judge to the
paraphernalia and all other subversive materials Such description hardly applicant's witness are not sufficiently searching to establish probable cause. The
provided a definite guideline to the search team as to what articles might "probable cause" required to justify the issuance of a search warrant comprehends
be lawfully seized thereunder. Said description is no different from if not such facts and circumstances as will induce a cautious man to rely upon them and
worse than, the description found in the search warrants in "Burgos, et al. act in pursuant thereof. 10 Of the 8 questions asked, the 1st, 2nd and 4th pertain to
v. the Chief of Staff"which this Court declared null and void for being too Identity. The 3rd and 5th are leading not searching questions. The 6th, 7th and 8th
general. 7 refer to the description of the personalities to be seized, which is Identical to that in
the Search Warrant and suffers from the same lack of particularity. The examination
In the case at bar, the search warrant issued by respondent judge allowed conducted was general in nature and merely repetitious of the deposition of said
the seizure of printed copies of the Philippine Times, manuscripts/drafts of

54
witness. Mere generalization will not suffice and does not satisfy the requirements of the Temporary Restraining Order enjoining respondent from introducing evidence
probable cause upon which a warrant may issue. 11 obtained pursuant to the Search Warrant in the Subversive Documents case hereby
made permanent, the, personalities seized may be retained by the Constabulary
Respondents claim, however, that the proper forum for questioning the illegality of a Security Group for possible introduction as evidence in Criminal Case No. SMC-1-1,
Search Warrant is with the Court that issued it instead of this original, independent pending before Special Military commission No. 1, without prejudice to petitioner
action to quash. The records show, however, that petitioners did raise that issue in Mila Aguilar-Roque objecting to their relevance and asking said Commission to return
the SEARCH WARRANT CASE in their Comment, dated October 18, 1984. In fact, they to her any and all irrelevant documents and articles.
already questioned the admissibility of the evidence obtained under the Search
Warrant, even during the inquest investigation on August 10, 1984. And in the SO ORDERED.
SUBVERSIVE DOCUMENTS CASE, they filed a Motion to Suppress on December 12,
1984 claiming that the proceedings under the Search Warrant were unlawful. Plana, Escolin Relova, Gutierrez, Jr., De la Fuente, Alampay and Patajo concur.
Substantially, therefore, while not denominated as a motion to quash, petitioners Makasiar, C.J., concurs in the result.
had questioned the legality of the Search Warrant.
Aquino, J.; took no part.
Parenthetically, it strikes the Court that the pendency of the SEARCH WARRANT CASE
and of the SUBVERSIVE DOCUMENTS CASE before two different Courts is not Concepcion Jr., J., reserves his vote.
conducive to an orderly administration of justice. It should be advisable that,
whenever a Search Warrant has been issued by one Court, or Branch, and a criminal
prosecution is initiated in another Court, or Branch, as a result of the service of the
Search Warrant, the SEARCH WARRANT CASE should be consolidated with the
criminal case for orderly procedure. The later criminal case is more substantial than
the Search Warrant proceeding, and the Presiding Judge in the criminal case should
have the right to act on petitions to exclude evidence unlawfully obtained.
Notwithstanding the irregular issuance of the Search Warrant and although,
ordinarily, the articles seized under an invalid search warrant should be returned,
they cannot be ordered returned in the case at bar to AGUILAR-ROQUE. Some
searches may be made without a warrant. Thus, Section 12, Rule 126, Rules of
Court, explicitly provides:
Section 12. Search without warrant of person arrested.A person charged
with an offense may be searched for dangerous weapons or anything which
may be used as proof of the commission of the offense.
The provision is declaratory in the sense that it is confined to the search, without a
search warrant, of a person who had been arrested. It is also a general rule that, as
an incident of an arrest, the place or premises where the arrest was made can also
be search without a search warrant. In this latter case, "the extent and
reasonableness of the search must be decided on its own facts and circumstances,
and it has been stated that, in the application of general rules, there is some
confusion in the decisions as to what constitutes the extent of the place or premises
which may be searched. 12 "What must be considered is the balancing of the
individual's right to privacy and the public's interest in the prevention of crime and
the apprehension of criminals." 13
Considering that AGUILAR-ROQUE has been charged with Rebellion, which is a crime
against public order; that the warrant for her arrest has not been served for a
considerable period of time; that she was arrested within the general vicinity of her
dwelling; and that the search of her dwelling was made within a half hour of her
arrest, we are of the opinion that in her respect, the search at No. 239-B Mayon
Street, Quezon City, did not need a search warrant; this, for possible effective results
in the interest of public order.
Such being the case, the personalities seized may be retained. by CSG, for possible
introduction as evidence in the Rebellion Case, leaving it to AGUILAR-ROQUE to
object to their relevance and to ask Special Military Commission No.1 to return to her
any and all irrelevant documents and articles.
WHEREFORE, while Search Warrant No. 80-84 issued on August 6, 1984 by
respondent Executive Judge Ernani Cruz Pao is hereby annulled and set aside, and

55
PAPA V. MAGO, 22 SCRA 857 (1968) assuming them to have been misdeclared and, undervalued, were not subject to
seizure under Section 2531 of the Tariff and Customs Code because Remedios Mago
G.R. No. L-27360 February 28, 1968 had bought them from another person without knowledge that they were imported
illegally; that the bales had not yet been opened, although Chief of Police Papa had
HON. RICARDO G. PAPA, as Chief of Police of Manila; HON. JUAN PONCE arranged with the Commissioner of Customs regarding the disposition of the goods,
ENRILE, as Commissioner of Customs; PEDRO PACIS, as Collector of and that unless restrained their constitutional rights would be violated and they
Customs of the Port of Manila; and MARTIN ALAGAO, as Patrolman of the would truly suffer irreparable injury. Hence, Remedios Mago and Valentin Lanopa
Manila Police Department, petitioners, prayed for the issuance of a restraining order, ex parte, enjoining the above-named
vs. police and customs authorities, or their agents, from opening the bales and
REMEDIOS MAGO and HILARION U. JARENCIO, as Presiding Judge of Branch examining the goods, and a writ of mandamus for the return of the goods and the
23, Court of First Instance of Manila, respondents. trucks, as well as a judgment for actual, moral and exemplary damages in their
favor.
ZALDIVAR, J.:
On November 10, 1966, respondent Judge Hilarion Jarencio issued an order ex
This is an original action for prohibition and certiorari, with preliminary parte restraining the respondents in Civil Case No. 67496 now petitioners in the
injunction filed by Ricardo Papa, Chief of Police of Manila; Juan once Enrile, instant case before this Court from opening the nine bales in question, and at the
Commissioner of Customs; Pedro Pacis, Collector of Customs of the Port of Manila; same time set the hearing of the petition for preliminary injunction on November 16,
and Martin Alagao, a patrolman of the Manila Police Department, against Remedios 1966. However, when the restraining order was received by herein petitioners, some
Mago and Hon. Hilarion Jarencio, Presiding Judge of Branch 23 of the Court of First bales had already been opened by the examiners of the Bureau of Customs in the
Instance of Manila, praying for the annulment of the order issued by respondent presence of officials of the Manila Police Department, an assistant city fiscal and a
Judge in Civil Case No. 67496 of the Court of First Instance of Manila under date of representative of herein respondent Remedios Mago.
March 7, 1967, which authorized the release under bond of certain goods which
were seized and held by petitioners in connection with the enforcement of the Tariff Under date of November 15, 1966, Remedios Mago filed an amended petition
and Customs Code, but which were claimed by respondent Remedios Mago, and to in Civil Case No. 67496, including as party defendants Collector of Customs Pedro
prohibit respondent Judge from further proceeding in any manner whatsoever in said Pacis of the Port of Manila and Lt. Martin Alagao of the Manila Police Department.
Civil Case No. 67496. Pending the determination of this case this Court issued a writ Herein petitioners (defendants below) filed, on November 24, 1966, their "Answer
of preliminary injunction restraining the respondent Judge from executing, enforcing with Opposition to the Issuance of a Writ of Preliminary Injunction", denying the
and/or implementing the questioned order in Civil Case No. 67496 and from alleged illegality of the seizure and detention of the goods and the trucks and of
proceeding with said case. their other actuations, and alleging special and affirmative defenses, to wit: that the
Court of First Instance of Manila had no jurisdiction to try the case; that the case fell
Petitioner Martin Alagao, head of the counter-intelligence unit of the Manila within the exclusive jurisdiction of the Court of Tax Appeals; that, assuming that the
Police Department, acting upon a reliable information received on November 3, 1966 court had jurisdiction over the case, the petition stated no cause of action in view of
to the effect that a certain shipment of personal effects, allegedly misdeclared and the failure of Remedios Mago to exhaust the administrative remedies provided for in
undervalued, would be released the following day from the customs zone of the port the Tariff and Customs Code; that the Bureau of Customs had not lost jurisdiction
of Manila and loaded on two trucks, and upon orders of petitioner Ricardo Papa, over the goods because the full duties and charges thereon had not been paid; that
Chief of Police of Manila and a duly deputized agent of the Bureau of Customs, the members of the Manila Police Department had the power to make the seizure;
conducted surveillance at gate No. 1 of the customs zone. When the trucks left gate that the seizure was not unreasonable; and the persons deputized under Section
No. 1 at about 4:30 in the afternoon of November 4, 1966, elements of the counter- 2203 (c) of the Tariff and Customs Code could effect search, seizures and arrests in
intelligence unit went after the trucks and intercepted them at the Agrifina Circle, inland places in connection with the enforcement of the said Code. In opposing the
Ermita, Manila. The load of the two trucks consisting of nine bales of goods, and the issuance of the writ of preliminary injunction, herein petitioners averred in the court
two trucks, were seized on instructions of the Chief of Police. Upon investigation, a below that the writ could not be granted for the reason that Remedios Mago was not
person claimed ownership of the goods and showed to the policemen a "Statement entitled to the main reliefs she prayed for; that the release of the goods, which were
and Receipts of Duties Collected in Informal Entry No. 147-5501", issued by the subject to seizure proceedings under the Tariff and Customs Code, would deprive the
Bureau of Customs in the name of a certain Bienvenido Naguit. Bureau of Customs of the authority to forfeit them; and that Remedios Mago and
Claiming to have been prejudiced by the seizure and detention of the two Valentin Lanopa would not suffer irreparable injury. Herein petitioners prayed the
trucks and their cargo, Remedios Mago and Valentin B. Lanopa filed with the Court of court below for the lifting of the restraining order, for the denial of the issuance of
First Instance of Manila a petition "for mandamus with restraining order or the writ of preliminary injunction, and for the dismissal of the case.
preliminary injunction, docketed as Civil Case No. 67496, alleging, among others, At the hearing on December 9, 1966, the lower Court, with the conformity of
that Remedios Mago was the owner of the goods seized, having purchased them the parties, ordered that an inventory of the goods be made by its clerk of court in
from the Sta. Monica Grocery in San Fernando, Pampanga; that she hired the trucks the presence of the representatives of the claimant of the goods, the Bureau of
owned by Valentin Lanopa to transport, the goods from said place to her residence Customs, and the Anti-Smuggling Center of the Manila Police Department. On
at 1657 Laon Laan St., Sampaloc, Manila; that the goods were seized by members of December 13, 1966, the above-named persons filed a "Compliance" itemizing the
the Manila Police Department without search warrant issued by a competent court; contents of the nine bales.
that anila Chief of Police Ricardo Papa denied the request of counsel for Remedios
Mago that the bales be not opened and the goods contained therein be not Herein respondent Remedios Mago, on December 23, 1966, filed an ex
examined; that then Customs Commissioner Jacinto Gavino had illegally assigned parte motion to release the goods, alleging that since the inventory of the goods
appraisers to examine the goods because the goods were no longer under the seized did not show any article of prohibited importation, the same should be
control and supervision of the Commissioner of Customs; that the goods, even released as per agreement of the patties upon her posting of the appropriate bond

56
that may be determined by the court. Herein petitioners filed their opposition to the to answer for whatever duties and taxes the said goods may still be liable; and (8)
motion, alleging that the court had no jurisdiction to order the release of the goods that the bond for the release of the goods was sufficient.
in view of the fact that the court had no jurisdiction over the case, and that most of
the goods, as shown in the inventory, were not declared and were, therefore, subject The principal issue in the instant case is whether or not, the respondent Judge
to forfeiture. A supplemental opposition was filed by herein petitioners on January had acted with jurisdiction in issuing the order of March 7, 1967 releasing the goods
19, 1967, alleging that on January 12, 1967 seizure proceedings against the goods in question.
had been instituted by the Collector of Customs of the Port of Manila, and the The Bureau of Customs has the duties, powers and jurisdiction, among others,
determination of all questions affecting the disposal of property proceeded against (1) to assess and collect all lawful revenues from imported articles, and all other
in seizure and forfeiture proceedings should thereby be left to the Collector of dues, fees, charges, fines and penalties, accruing under the tariff and customs laws;
Customs. On January 30, 1967, herein petitioners filed a manifestation that the (2) to prevent and suppress smuggling and other frauds upon the customs; and (3)
estimated duties, taxes and other charges due on the goods amounted to to enforce tariff and customs laws. 1 The goods in question were imported from
P95,772.00. On February 2, 1967, herein respondent Remedios Mago filed an urgent Hongkong, as shown in the "Statement and Receipts of Duties Collected on Informal
manifestation and reiteration of the motion for the release under bond of the goods. Entry". 2 As long as the importation has not been terminated the imported goods
On March 7, 1967, the respondent Judge issued an order releasing the goods remain under the jurisdiction of the Bureau of customs. Importation is deemed
to herein respondent Remedios Mago upon her filing of a bond in the amount of terminated only upon the payment of the duties, taxes and other charges upon the
P40,000.00, and on March 13, 1967, said respondent filed the corresponding bond. articles, or secured to be paid, at the port of entry and the legal permit for
withdrawal shall have been granted. 3 The payment of the duties, taxes, fees and
On March 13, 1967, herein petitioner Ricardo Papa, on his own behalf, filed a other charges must be in full. 4
motion for reconsideration of the order of the court releasing the goods under bond,
upon the ground that the Manila Police Department had been directed by the The record shows, by comparing the articles and duties stated in the aforesaid
Collector of Customs of the Port of Manila to hold the goods pending termination of "Statement and Receipts of Duties Collected on Informal Entry" with the
the seizure proceedings. manifestation of the Office of the Solicitor General 5 wherein it is stated that the
estimated duties, taxes and other charges on the goods subject of this case
Without waiting for the court's action on the motion for reconsideration, and amounted to P95,772.00 as evidenced by the report of the appraiser of the Bureau
alleging that they had no plain, speedy and adequate remedy in the ordinary course of Customs, that the duties, taxes and other charges had not been paid in full.
of law, herein petitioners filed the present action for prohibition and certiorari with Furthermore, a comparison of the goods on which duties had been assessed, as
preliminary injunction before this Court. In their petition petitioners alleged, among shown in the "Statement and Receipts of Duties Collected on Informal Entry" and the
others, that the respondent Judge acted without jurisdiction in ordering the release "compliance" itemizing the articles found in the bales upon examination and
to respondent Remedios Mago of the disputed goods, for the following reasons: (1) inventory, 6 shows that the quantity of the goods was underdeclared, presumably to
the Court of First Instance of Manila, presided by respondent Judge, had no avoid the payment of duties thereon. For example, Annex B (the statement and
jurisdiction over the case; (2) respondent Remedios Mago had no cause of action in receipts of duties collected) states that there were 40 pieces of ladies' sweaters,
Civil Case No. 67496 of the Court of First Instance of Manila due to her failure to whereas Annex H (the inventory contained in the "compliance") states that in bale
exhaust all administrative remedies before invoking judicial intervention; (3) the No. 1 alone there were 42 dozens and 1 piece of ladies' sweaters of assorted colors;
Government was not estopped by the negligent and/or illegal acts of its agent in not in Annex B, only 100 pieces of watch bands were assessed, but in Annex H, there
collecting the correct taxes; and (4) the bond fixed by respondent Judge for the were in bale No. 2, 209 dozens and 5 pieces of men's metal watch bands (white) and
release of the goods was grossly insufficient. 120 dozens of men's metal watch band (gold color), and in bale No. 7, 320 dozens of
men's metal watch bands (gold color); in Annex B, 20 dozens only of men's
In due time, the respondents filed their answer to the petition for prohibition handkerchief were declared, but in Annex H it appears that there were 224 dozens of
and certiorari in this case. In their answer, respondents alleged, among others: (1) said goods in bale No. 2, 120 dozens in bale No. 6, 380 dozens in bale No. 7, 220
that it was within the jurisdiction of the lower court presided by respondent Judge to dozens in bale No. 8, and another 200 dozens in bale No. 9. The articles contained in
hear and decide Civil Case No. 67496 and to issue the questioned order of March 7, the nine bales in question, were, therefore, subject to forfeiture under Section 2530,
1967, because said Civil Case No. 67496 was instituted long before seizure, and pars. e and m, (1), (3), (4), and (5) of the Tariff and Customs Code. And this Court
identification proceedings against the nine bales of goods in question were instituted has held that merchandise, the importation of which is effected contrary to law, is
by the Collector of Customs; (2) that petitioners could no longer go after the goods subject to forfeiture, 7 and that goods released contrary to law are subject to seizure
in question after the corresponding duties and taxes had been paid and said goods and forfeiture. 8
had left the customs premises and were no longer within the control of the Bureau of
Customs; (3) that respondent Remedios Mago was purchaser in good faith of the Even if it be granted, arguendo, that after the goods in question had been
goods in question so that those goods can not be the subject of seizure and brought out of the customs area the Bureau of Customs had lost jurisdiction over the
forfeiture proceedings; (4) that the seizure of the goods was affected by members of same, nevertheless, when said goods were intercepted at the Agrifina Circle on
the Manila Police Department at a place outside control of jurisdiction of the Bureau November 4, 1966 by members of the Manila Police Department, acting under
of Customs and affected without any search warrant or a warrant of seizure and directions and orders of their Chief, Ricardo C. Papa, who had been formally
detention; (5) that the warrant of seizure and detention subsequently issued by the deputized by the Commissioner of Customs, 9 the Bureau of Customs had regained
Collector of Customs is illegal and unconstitutional, it not being issued by a judge; jurisdiction and custody of the goods. Section 1206 of the Tariff and Customs Code
(6) that the seizing officers have no authority to seize the goods in question because imposes upon the Collector of Customs the duty to hold possession of all imported
they are not articles of prohibited importation; (7) that petitioners are estopped to articles upon which duties, taxes, and other charges have not been paid or secured
institute the present action because they had agreed before the respondent Judge to be paid, and to dispose of the same according to law. The goods in question,
that they would not interpose any objection to the release of the goods under bond therefore, were under the custody and at the disposal of the Bureau of Customs at
the time the petition for mandamus, docketed as Civil Case No. 67496, was filed in

57
the Court of First Instance of Manila on November 9, 1966. The Court of First The purpose of the seizure by the Customs bureau was to verify whether or
Instance of Manila, therefore, could not exercise jurisdiction over said goods even if not Custom duties and taxes were paid for their importation. Hence, on
the warrant of seizure and detention of the goods for the purposes of the seizure and December 23, 1964, Customs released 22 bales thereof, for the same were
forfeiture proceedings had not yet been issued by the Collector of Customs. found to have been released regularly from the Cebu Port (Petition Annex
"L"). As to goods imported illegally or released irregularly from Customs
The ruling in the case of "Alberto de Joya, et al. v. Hon. Gregorio Lantin, et al.," custody, these are subject to seizure under Section 2530 m. of the Tariff
G.R. No. L-24037, decided by this Court on April 27, 1967, is squarely applicable to and Customs Code (RA 1957).
the instant case. In the De Joya case, it appears that Francindy Commercial of Manila
bought from Ernerose Commercial of Cebu City 90 bales of assorted textiles and The Bureau of Customs has jurisdiction and power, among others to
rags, valued at P117,731.00, which had been imported and entered thru the port of collect revenues from imported articles, fines and penalties and suppress
Cebu. Ernerose Commercial shipped the goods to Manila on board an inter-island smuggling and other frauds on customs; and to enforce tariff and customs
vessel. When the goods where about to leave the customs premises in Manila, on laws (Sec. 602, Republic Act 1957).
October 6, 1964, the customs authorities held them for further verification, and upon
examination the goods were found to be different from the declaration in the cargo The goods in question are imported articles entered at the Port of
manifest of the carrying vessel. Francindy Commercial subsequently demanded from Cebu. Should they be found to have been released irregularly from Customs
the customs authorities the release of the goods, asserting that it is a purchaser in custody in Cebu City, they are subject to seizure and forfeiture, the
good faith of those goods; that a local purchaser was involved so the Bureau of proceedings for which comes within the jurisdiction of the Bureau of
Customs had no right to examine the goods; and that the goods came from a Customs pursuant to Republic Act 1937.
coastwise port. On October 26, 1964, Francindy Commercial filed in the Court of First Said proceeding should be followed; the owner of the goods may set
Instance of Manila a petition for mandamus against the Commissioner of Customs up defenses therein (Pacis v. Averia, L-22526, Nov. 20, 1966.) From the
and the Collector of Customs of the port of Manila to compel said customs decision of the Commissioner of Customs appeal lies to the Court of Tax
authorities to release the goods. Appeals, as provided in Sec. 2402 of Republic Act 1937 and Sec. 11 of
Francindy Commercial alleged in its petition for mandamus that the Bureau of Republic Act, 1125. To permit recourse to the Court of First Instance in
Customs had no jurisdiction over the goods because the same were not imported to cases of seizure of imported goods would in effect render ineffective the
the port of Manila; that it was not liable for duties and taxes because the transaction power of the Customs authorities under the Tariff and Customs Code and
was not an original importation; that the goods were not in the hands of the importer deprive the Court of Tax Appeals of one of its exclusive appellate
nor subject to importer's control, nor were the goods imported contrary to law with jurisdictions. As this Court has ruled in Pacis v. Averia, supra, Republic Acts
its (Francindy Commercial's) knowledge; and that the importation had been 1937 and 1125 vest jurisdiction over seizure and forfeiture proceedings
terminated. On November 12, 1964, the Collector of Customs of Manila issued a exclusively upon the Bureau of Customs and the Court of Tax Appeals. Such
warrant of seizure and identification against the goods. On December 3, 1964, the law being special in nature, while the Judiciary Act defining the jurisdiction
Commissioner of Customs and the Collector of Customs, as respondents in of Courts of First Instance is a general legislation, not to mention that the
the mandamus case, filed a motion to dismiss the petition on the grounds of lack of former are later enactments, the Court of First Instance should yield to the
jurisdiction, lack of cause of action, and in view of the pending seizure and forfeiture jurisdiction of the Customs authorities.
proceedings. The Court of First Instance held resolution on the motion to dismiss in It is the settled rule, therefore, that the Bureau of Customs acquires exclusive
abeyance pending decision on the merits. On December 14, 1964, the Court of First jurisdiction over imported goods, for the purposes of enforcement of the customs
Instance of Manila issued a preventive and mandatory injunction, on prayer by laws, from the moment the goods are actually in its possession or control, even if no
Francindy Commercial, upon a bond of P20,000.00. The Commissioner of Customs warrant of seizure or detention had previously been issued by the Collector of
and the Collector of Customs sought the lifting of the preliminary and mandatory Customs in connection with seizure and forfeiture proceedings. In the present case,
injunction, and the resolution of their motion to dismiss. The Court of First Instance the Bureau of Customs actually seized the goods in question on November 4, 1966,
of Manila, however, on January 12, 1965, ordered them to comply with the and so from that date the Bureau of Customs acquired jurisdiction over the goods for
preliminary and mandatory injunction, upon the filing by Francindy Commercial of an the purposes of the enforcement of the tariff and customs laws, to the exclusion of
additional bond of P50,000.00. Said customs authorities thereupon filed with this the regular courts. Much less then would the Court of First Instance of Manila have
Court, on January 14, 1965, a petition for certiorari and prohibition with preliminary jurisdiction over the goods in question after the Collector of Customs had issued the
injunction. In resolving the question raised in that case, this Court held: warrant of seizure and detention on January 12, 1967. 10And so, it cannot be said, as
This petition raises two related issues: first, has the Customs bureau respondents contend, that the issuance of said warrant was only an attempt to
jurisdiction to seize the goods and institute forfeiture proceedings against divest the respondent Judge of jurisdiction over the subject matter of the case. The
them? and (2) has the Court of First Instance jurisdiction to entertain the court presided by respondent Judge did not acquire jurisdiction over the goods in
petition for mandamus to compel the Customs authorities to release the question when the petition for mandamus was filed before it, and so there was no
goods? need of divesting it of jurisdiction. Not having acquired jurisdiction over the goods, it
follows that the Court of First Instance of Manila had no jurisdiction to issue the
Francindy Commercial contends that since the petition in the Court of questioned order of March 7, 1967 releasing said goods.
first Instance was filed (on October 26, 1964) ahead of the issuance of the
Customs warrant of seizure and forfeiture (on November 12, 1964),the Respondents also aver that petitioner Martin Alagao, an officer of the Manila
Customs bureau should yield the jurisdiction of the said court. Police Department, could not seize the goods in question without a search warrant.
This contention cannot be sustained. The Chief of the Manila Police Department,
The record shows, however, that the goods in question were actually Ricardo G. Papa, having been deputized in writing by the Commissioner of Customs,
seized on October 6, 1964, i.e., before Francindy Commercial sued in court. could, for the purposes of the enforcement of the customs and tariff laws, effect

58
searches, seizures, and arrests, 11 and it was his duty to make seizure, among others, this section was re-enacted in the 3d section of the Act of July 18, 1866,
of any cargo, articles or other movable property when the same may be subject to chap. 201, 14 Stat. at L. 178, and was thereafter embodied in the Revised
forfeiture or liable for any fine imposed under customs and tariff laws. 12 He could Statutes as 3061, Comp. Stat. 5763, 2 Fed. Stat. Anno. 2d ed. p. 1161.
lawfully open and examine any box, trunk, envelope or other container wherever Neither 3061 nor any of its earlier counterparts has ever been attacked as
found when he had reasonable cause to suspect the presence therein of dutiable unconstitutional. Indeed, that section was referred to and treated as
articles introduced into the Philippines contrary to law; and likewise to stop, search operative by this court in Von Cotzhausen v. Nazro, 107 U.S. 215, 219, 27 L.
and examine any vehicle, beast or person reasonably suspected of holding or ed. 540, 541, 2 Sup. Ct. Rep. 503. . . .
conveying such article as aforesaid. 13 It cannot be doubted, therefore, that petitioner
Ricardo G. Papa, Chief of Police of Manila, could lawfully effect the search and In the instant case, we note that petitioner Martin Alagao and his companion
seizure of the goods in question. The Tariff and Customs Code authorizes him to policemen did not have to make any search before they seized the two trucks and
demand assistance of any police officer to effect said search and seizure, and the their cargo. In their original petition, and amended petition, in the court below
latter has the legal duty to render said assistance. 14This was what happened Remedios Mago and Valentin Lanopa did not even allege that there was a
precisely in the case of Lt. Martin Alagao who, with his unit, made the search and search. 18 All that they complained of was,
seizure of the two trucks loaded with the nine bales of goods in question at the That while the trucks were on their way, they
Agrifina Circle. He was given authority by the Chief of Police to make the were intercepted without any search warrant near the Agrifina Circle and
interception of the cargo. 15 taken to the Manila Police Department, where they were detained.
Petitioner Martin Alagao and his companion policemen had authority to effect But even if there was a search, there is still authority to the effect that no
the seizure without any search warrant issued by a competent court. The Tariff and search warrant would be needed under the circumstances obtaining in the instant
Customs Code does not require said warrant in the instant case. The Code case. Thus, it has been held that:
authorizes persons having police authority under Section 2203 of the Tariff and
Customs Code to enter, pass through or search any land, inclosure, warehouse, store The guaranty of freedom from unreasonable searches and seizures is
or building, not being a dwelling house; and also to inspect, search and examine any construed as recognizing a necessary difference between a search of a
vessel or aircraft and any trunk, package, or envelope or any person on board, or to dwelling house or other structure in respect of which a search warrant may
stop and search and examine any vehicle, beast or person suspected of holding or readily be obtained and a search of a ship, motorboat, wagon, or
conveying any dutiable or prohibited article introduced into the Philippines contrary automobile for contraband goods, where it is not practicable to secure a
to law, without mentioning the need of a search warrant in said cases. 16 But in the warrant because the vehicle can be quickly moved out of the locality or
search of a dwelling house, the Code provides that said "dwelling house may be jurisdiction in which the warrant must be sought. (47 Am. Jur., pp. 513-514,
entered and searched only upon warrant issued by a judge or justice of the peace. . . citing Carroll v. United States, 267 U.S. 132, 69 L. ed., 543, 45 S. Ct., 280,
." 17 It is our considered view, therefor, that except in the case of the search of a 39 A.L.R., 790; People v. Case, 320 Mich., 379, 190 N.W., 389, 27 A.L.R.,
dwelling house, persons exercising police authority under the customs law may 686.)
effect search and seizure without a search warrant in the enforcement of customs
In the case of People v. Case (320 Mich., 379, 190 N.W., 389, 27 A.L.R., 686),
laws.
the question raised by defendant's counsel was whether an automobile truck or an
Our conclusion finds support in the case of Carroll v. United States, 39 A.L.R., automobile could be searched without search warrant or other process and the
790, 799, wherein the court, considering a legal provision similar to Section 2211 of goods therein seized used afterwards as evidence in a trial for violation of the
the Philippine Tariff and Customs Code, said as follows: prohibition laws of the State. Same counsel contended the negative, urging the
constitutional provision forbidding unreasonable searches and seizures. The Court
Thus contemporaneously with the adoption of the 4th Amendment, said:
we find in the first Congress, and in the following second and fourth
Congresses, a difference made as to the necessity for a search warrant . . . Neither our state nor the Federal Constitution directly prohibits
between goods subject to forfeiture, when concealed in a dwelling house of search and seizure without a warrant, as is sometimes asserted. Only
similar place, and like goods in course of transportation and concealed in a "unreasonable" search and seizure is forbidden. . . .
movable vessel, where readily they could be put out of reach of a search
. . . The question whether a seizure or a search is unreasonable in the
warrant. . . .
language of the Constitution is a judicial and not a legislative question; but
Again, by the 2d section of the Act of March 3, 1815 (3 Stat. at L.231, in determining whether a seizure is or is not unreasonable, all of the
232, chap. 94), it was made lawful for customs officers not only to board circumstances under which it is made must be looked to.
and search vessels within their own and adjoining districts, but also to stop,
The automobile is a swift and powerful vehicle of recent
search and examine any vehicle, beast or person on which or whom they
development, which has multiplied by quantity production and taken
should suspect there was merchandise which was subject to duty, or had
possession of our highways in battalions until the slower, animal-drawn
been introduced into the United States in any manner contrary to law,
vehicles, with their easily noted individuality, are rare. Constructed as
whether by the person in charge of the vehicle or beast or otherwise, and if
covered vehicles to standard form in immense quantities, and with a
they should find any goods, wares, or merchandise thereon, which they had
capacity for speed rivaling express trains, they furnish for successful
probably cause to believe had been so unlawfully brought into the country,
commission of crime a disguising means of silent approach and swift
to seize and secure the same, and the vehicle or beast as well, for trial and
escape unknown in the history of the world before their advent. The
forfeiture. This Act was renewed April 27, 1816 (3 Sta. at L. 315, chap. 100),
question of their police control and reasonable search on highways or other
for a year and expired. The Act of February 28, 1865, revived 2 of the Act
public places is a serious question far deeper and broader than their use in
of 1815, above described, chap. 67, 13 Stat. at L. 441. The substance of

59
so-called "bootleging" or "rum running," which is itself is no small matter. Presidential Decree No. 9, in relation to General Order No. 6, dated
While a possession in the sense of private ownership, they are but a vehicle September 22, 1972, and General Order No. 7, dated September 23, 1972,
constructed for travel and transportation on highways. Their active use is in relation further to Presidential Decree No. 885, and considering that the
not in homes or on private premises, the privacy of which the law especially firearm subject of this case was not used in the circumstances as embraced
guards from search and seizure without process. The baffling extent to in paragraph I thereof, applying the provision of indeterminate sentence
which they are successfully utilized to facilitate commission of crime of all law, accused Ruben Burgos is hereby sentenced to suffer an imprisonment
degrees, from those against morality, chastity, and decency, to robbery, of twenty (20) years of reclusion temporal maximum, as minimum penalty,
rape, burglary, and murder, is a matter of common knowledge. Upon that to reclusion perpetua, as maximum penalty, pursuant to sub-paragraph B,
problem a condition, and not a theory, confronts proper administration of of Presidential Decree No. 9, as aforementioned, with accessory penalties,
our criminal laws. Whether search of and seizure from an automobile upon as provided for by law.
a highway or other public place without a search warrant is unreasonable is
in its final analysis to be determined as a judicial question in view of all the As a result of this judgment, the subject firearm involved in this case
circumstances under which it is made. (Homemade revolver, caliber .38, Smith and Wesson, with Serial No.
8.69221) is hereby ordered confiscated in favor of the government, to be
Having declared that the seizure by the members of the Manila Police disposed of in accordance with law. Likewise, the subversive documents,
Department of the goods in question was in accordance with law and by that seizure leaflets and/or propaganda seized are ordered disposed of in accordance
the Bureau of Customs had acquired jurisdiction over the goods for the purpose of with law.
the enforcement of the customs and tariff laws, to the exclusion of the Court of First
Instance of Manila, We have thus resolved the principal and decisive issue in the The information charged the defendant-appellant with the crime of illegal possession
present case. We do not consider it necessary, for the purposes of this decision, to of firearm in furtherance of subversion in an information which reads as follows:
discuss the incidental issues raised by the parties in their pleadings. That in the afternoon of May 13, 1982 and thereabout at Tiguman, Digos,
WHEREFORE, judgment is hereby rendered, as follows: Davao del Sur, Philippines, within the jurisdiction of this Court, the above-
named accused with intent to possess and without the necessary license,
(a) Granting the writ of certiorari and prohibition prayed for by petitioners; permit or authority issued by the proper government agencies, did then
and there wilfully, unlawfully and feloniously keep, possess, carry and have
(b) Declaring null and void, for having been issued without jurisdiction, the in his possession, control and custody one (1) homemade revolver, caliber .
order of respondent Judge Hilarion U. Jarencio, dated March 7, 1967, in Civil Code No. 38, make Smith and Wesson, with Serial No. 8.69221, which firearm was
67496 of the Court of First Instance of Manila; issued to and used by the accused at Tiguman, Digos, Davao del Sur, his
(c) Declaring permanent the preliminary injunction issued by this Court on area of operations by one Alias Commander Pol for the New People's Army
March 31, 1967 restraining respondent Judge from executing, enforcing and/or (NPA), a subversive organization organized for the purpose of overthrowing
implementing his order of March 7, 1967 in Civil Case No. 67496 of the Court of First the Government of the Republic of the Philippines through lawless and
Instance of Manila, and from proceeding in any manner in said case; violent means, of which the accused had knowledge, and which firearm was
used by the accused in the performance of his subversive tasks such as the
(d) Ordering the dismissal of Civil Case No. 67496 of the Court of First Instance recruitment of New Members to the NPA and collection of contributions
of Manila; and1wph1.t from the members.
(e) Ordering the private respondent, Remedios Mago, to pay the costs. CONTRARY TO LAW.
It is so ordered. The evidence for the prosecution is summarized in the decision of the lower court as
follows:
Concepcion, C.J., Reyes, J.B.L., Dizon, Makalintal, Bengzon, J.P., Sanchez, Castro,
Angeles and Fernando, JJ., concur.1wph1.t xxx xxx xxx

PEOPLE V. BURGOS, 144 SCRA 1 (1986) . . . Through the testimony of Pat. Pepito Bioco, and Sgt. Romeo Taroy, it
appears that by virtue of an intelligent information obtained by the
G.R. No. L-68955 September 4, 1986 Constabulary and INP units, stationed at Digos, Davao del Sur, on May 12,
1982, one Cesar Masamlok personally and voluntarily surre0ndered to the
PEOPLE OF THE PHILIPPINES, plaintiff-appellee, authorities at about 9:00 o'clock A.M. at Digos, Davao del Sur Constabulary
vs. Headquarters, stating that he was forcibly recruited by accused Ruben
RUBEN BURGOS y TITO, defendant-appellant. Burgos as member of the NPA, threatening him with the use of firearm
against his life, if he refused.
GUTIERREZ, JR., J.:
Along with his recruitment, accused was asked to contribute one (1) chopa
This is an appeal from the decision of the Regional Trial Court of Davao del Sur, 11 th of rice and one peso (P1.00) per month, as his contribution to the NPA TSN,
Judicial Region, Digos, Davao del Sur convicting defendant- appellant Ruben Burgos page 5, Hearing-October 14, 1982).
y Tito of The crime of Illegal Possession of Firearms in Furtherance of Subversion.
The dispositive portion of the decision reads: Immediately, upon receipt of said information, a joint team of PC-INP units,
composed of fifteen (15) members, headed by Captain Melchesideck
WHEREFORE, finding the guilt of accused Ruben Burgos sufficiently Bargio, (PC), on the following day, May 13, 1982, was dispatched at
established beyond reasonable doubt, of the offense charges , pursuant to

60
Tiguman; Davao del Sur, to arrest accused Ruben Burgos. The team left the accused and attended the seminar, Those present in the seminar were:
headquarter at 1:30 P.M., and arrived at Tiguman, at more or less 2:00 accused Ruben Burgos, Antonio Burgos, Oscar Gomez, Landrino Burgos,
o'clock PM where through the help of Pedro Burgos, brother of accused, the alias Pedipol and one alias Jamper.
team was able to locate accused, who was plowing his field. (TSN, pages 6-
7, Hearing-October 14, 1982). The first speaker was accused Ruben Burgos, who said very distinctly that
he is an NPA together with his companions, to assure the unity of the
Right in the house of accused, the latter was caned by the team and Pat. civilian. That he encouraged the group to overthrow the government,
Bioco asked accused about his firearm, as reported by Cesar Masamlok. At emphasizing that those who attended the seminar were already members
first accused denied possession of said firearm but later, upon question of the NPA, and if they reveal to the authorities, they will be killed.
profounded by Sgt. Alejandro Buncalan with the wife of the accused, the
latter pointed to a place below their house where a gun was buried in the Accused, while talking, showed to the audience pamphlets and documents,
ground. (TSN, page 8, Hearing-October 14, 1982). then finally shouted, the NPA will be victorious. Masamlok likewise Identified
the pamphlets as those marked as Exh. exhibits "B", "C", and "D" for the
Pat. Bioco then verified the place pointed by accused's wife and dug the prosecution. (TSN, pages 75, 76 and 77, Hearing-January 4, 1983)
grounds, after which he recovered the firearm, Caliber .38 revolver, marked
as Exhibit "A" for the prosecution. Other speakers in said meeting were Pedipol, Jamper and Oscar Gomez,
who likewise expounded their own opinions about the NPA. It was also
After the recovery of the firearm, accused likewise pointed to the team, announced in said seminar that a certain Tonio Burgos, will be responsible
subversive documents which he allegedly kept in a stock pile of qqqcogon for the collection of the contribution from the members. (TSN, pages 78-79,
at a distance of three (3) meters apart from his house. Then Sgt. Taroy Hearing- January 4, 1983)
accordingly verified beneath said cogon grass and likewise recovered
documents consisting of notebook colored maroon with spiral bound, On May 12, 1982, however, Cesar Masamlok surrendered to Captain Bargio
Exhibit "B" for the prosecution; a pamphlet consisting of eight (8) leaves, of the Provincial Headquarters of the Philippine Constabulary, Digos, Davao
including the front and back covers entitled Ang Bayan, Pahayagan ng del Sur.
Partido Komunista ng Pilipinas, Pinapatnubayan ng Marxismo, Leninismo Assistant Provincial Fiscal Panfilo Lovitos was presented t prove that on May
Kaisipang Mao qqqZedong dated December 31, 1980, marked as Exhibit 19, 1982, he administered the subscription of th extra-judicial confession of
"C", and another pamphlet Asdang Pamantalaang Masa sa Habagatang accused Ruben Burgos, marked as Exhibit "E " for the prosecution,
Mindanao, March and April 1981 issue, consisting of ten (10) pages, marked consisting of five (5) pages.
as Exhibit "D" for the prosecution.
Appearing voluntarily in said office, for the subscription of his confession,
Accused, when confronted with the firearm Exhibit "A", after its recovery, Fiscal Lovitos, realizing that accused was not represented by counsel,
readily admitted the same as issued to him by Nestor Jimenez, otherwise requested the services of Atty. Anyog, whose office is adjacent to the
known as a certain Alias Pedipol, allegedly team leader of the sparrow unit Fiscal's Office, to assist accused in the subscription of his extra-judicial
of New People's Army, responsible in the liquidation of target personalities, statement.
opposed to NPA Ideological movement, an example was the killing of the
late Mayor Llanos and Barangay Captain of Tienda Aplaya Digos, Davao del Atty. Anyog assisted accused in the reading of his confession from English
Sur. (TSN, pages 1-16, Hearing-October 14,1982). to Visayan language, resulting to the deletion of question No. 19 of the
document, by an inserted certification of Atty. Anyog and signature of
To prove accused's subversive activities, Cesar Masamlok, a former NPA accused, indicating his having understood, the allegations of his extra-
convert was presented, who declared that on March 7, 1972, in his former judicial statement.
residence at Tiguman Digos, Davao del Sur, accused Ruben Burgos,
accompanied by his companions Landrino Burgos, Oscar Gomez and Fiscal Lovitos, before accused signed his statement, explained to him his
Antonio Burgos, went to his house at about 5:00 o'clock P.M. and called him constitutional rights to remain silent, right to counsel and right to answer
downstair. Thereupon, accused told Masamlok, their purpose was to ask any question propounded or not.
rice and one (1) peso from him, as his contribution to their companions, the
With the aid of Atty. Anyog, accused signed his confession in the presence
NPA of which he is now a member. (TSN, pages 70, 71, 72, Hearing-January
of Atty. Anyog and Fiscal Lovitos, without the presence of military
4, 1983).
authorities, who escorted the accused, but were sent outside the cubicle of
Accused and his companions told Masamlok, he has to join their group Fiscal Lovitos while waiting for the accused. (TSN, pages 36-40, nearing
otherwise, he and his family will be killed. He was also warned not to reveal November 15, 1982)
anything with the government authorities. Because of the threat to his life
Finally, in order to prove illegal possession by accused of the subject
and family, Cesar Masamlok joined the group. Accused then told him, he
firearm, Sgt. Epifanio Comabig in-charge of firearms and explosives, NCO
should attend a seminar scheduled on April 19, 1982. Along with this
Headquarter, Philippine Constabulary, Digos, Davao del Sur, was presented
invitation, accused pulled gut from his waistline a .38 caliber revolver which
and testified, that among the lists of firearm holders in Davao del Sur,
Masamlok really saw, being only about two (2) meters away from accused,
nothing was listed in the name of accused Ruben Burgos, neither was his
which make him easily Identified said firearm, as that marked as Exhibit "A"
name included among the lists of persons who applied for the licensing of
for the prosecution. (TSN, pages 72, 73, and 74, Hearing-January 4, 1983).
the firearm under Presidential Decree No. 1745.
On April 19, 1982, as previously invited, Masamlok, accompanied by his
father, Matuguil Masamlok, Isabel Ilan and Ayok Ides went to the house of

61
After the above-testimony the prosecution formally closed its case and categorically, that the above-questions embraced in the numbers allegedly
offered its exhibits, which were all admitted in evidence, despite objection stated in the extrajudicial confession of accused, involving her to such NPA
interposed by counsel for accused, which was accordingly overruled. personalities, as Jamper, Pol, Anthony, etc., were not true because on the
date referred on April 28, 1982, none of the persons mentioned came to her
On the other hand, the defendant-appellant's version of the case against him is house for treatment, neither did she meet the accused nor able to talk with
stated in the decision as follows: him. (TSN, pages 118- 121, Hearing-May 18, 1983)
From his farm, the military personnel, whom he said he cannot recognize, She, however, admitted being familiar with one Oscar Gomez, and that she
brought him to the PC Barracks at Digos, Davao del Sur, and arrived there was personally charged with subversion in the Office of the Provincial
at about 3:00 o'clock, on the same date. At about 8:00 o'clock P.M., in the Commander, Philippine Constabulary, Digos, Davao del Sur, but said charge
evening, he was investigated by soldiers, whom he cannot Identify because was dismissed without reaching the Court. She likewise stated that her son,
they were wearing a civilian attire. (TSN, page 14 1, Hearing-June 15, 1983) Rogelio Arellano, was likewise charged for subversion filed in the Municipal
The investigation was conducted in the PC barracks, where he was detained Trial Court of Digos, Davao del Sur, but was likewise dismissed for lack of
with respect to the subject firearm, which the investigator, wished him to sufficient evidence to sustain his conviction. (TSN, pages 121-122, in
admit but accused denied its ownership. Because of his refusal accused relation to her cross-examination, Hearing-May 18, 1983)
was mauled, hitting him on the left and right side of his body which To support accused's denial of the charge against him, Barangay Captain of
rendered him unconscious. Accused in an atmosphere of tersed solemnity, Tiguman, Digos, Davao del Sur, Salvador qqqGalaraga was presented, who
crying and with emotional attachment, described in detail how he was declared, he was not personally aware of any subversive activities of
tortured and the ordeals he was subjected. accused, being his neighbor and member of his barrio. On the contrary, he
He said, after recovery of his consciousness, he was again confronted with can personally attest to his good character and reputation, as a law abiding
subject firearm, Exhibit "A", for him to admit and when he repeatedly citizen of his barrio, being a carpenter and farmer thereat. (TSl pages 128-
refused to accept as his own firearm, he was subjected to further prolong 129, Hearing-May 18, 1983)
(sic) torture and physical agony. Accused said, his eyes were covered with He however, admitted in cross-examination, that there were a lot of arrests
wet black cloth with pungent effect on his eyes. He was undressed, with made by the authorities in his barrio involving subversive activities but they
only blindfold, pungent water poured in his body and over his private parts, were released and were not formally charged in Court because they publicly
making his entire body, particularly his penis and testicle, terribly irritating took their oath of allegiance with the government. (TSN, pages 133-134, in
with pungent pain. relation to page 136, Hearing-May 18, 1983)
All along, he was investigated to obtain his admission, The process of Finally, to support accused's denial of the subject firearm, his wife, Urbana
beating, mauling, pain and/or ordeal was repeatedly done in similar cycle, Burgos, was presented and who testified that the subject firearm was left in
from May 13 and 14, 1982. intercepted only whenever he fell unconscious their house by Cesar Masamlok and one Pedipol on May 10, 1982. It was
and again repeated after recovery of his senses, night time, when the two left the gun, alleging that it was not in order, and
Finally on May 15, 1982, after undergoing the same torture and physical that they will leave it behind, temporarily for them to claim it later. They
ordeal he was seriously warned, if he will still adamantly refuse to accept were the ones who buried it. She said, her husband, the accused, was not in
ownership of the subject firearm, he will be salvaged, and no longer able to their house at that time and that she did not inform him about said firearm
bear any further the pain and agony, accused admitted ownership of neither did she report the matter to the authorities, for fear of the life of her
subject firearm. husband. (TSN, page 24, November 22, 1983)

After his admission, the mauling and torture stopped, but accused was On cross-examination, she said, even if Masamlok during the recovery of
made to sign his affidavit marked as Exhibit "E" for the prosecution, the firearm, was wearing a mask, she can still Identify him. (TSN, page 6,
consisting of five (5) pages, including the certification of the administering Hearing-November 22, 1983)
officer, (TSN, pages 141-148, Hearing-June 15, 1983) After the above-testimony, accused through counsel formally rested his
In addition to how he described the torture inflicted on him, accused, by case in support of accused's through counsel manifestation for the
way of explanation and commentary in details, and going one by one, the demurrer to evidence of the prosecution, or in the alternative for violation
allegations and/or contents of his alleged extrajudicial statement, attributed merely of simple illegal possession of firearm, 'under the Revised
his answers to those questions involuntarily made only because of fear, Administrative Code, as amended by Republic Act No. 4, reflected in the
threat and intimidation of his person and family, as a result of unbearable manifestation of counsel for accused. (TSN, pages 113-114, Hearing-May
excruciating pain he was subjected by an investigator, who, unfortunately 18, 1983)
he cannot Identify and was able to obtain his admission of the subject Accused-appellant Ruben Burgos now raises the following assignments of error, to
firearm, by force and violence exerted over his person. wit:
To support denial of accused of being involved in any subversive activities, I THE TRIAL COURT ERRED IN HOLDING THAT (SIC) THE ARREST OF
and also to support his denial to the truth of his alleged extra-judicial ACCUSED-APPELLANT WITHOUT VALID WARRANT TO BE LAWFUL.
confession, particularly questions Nos. 35, 38, 41, 42, 43, 44, 45, 46 and
47, along with qqqs answers to those questions, involving Honorata
Arellano ahas Inday Arellano, said Honorata Arellano appeared and declared

62
II THE TRIAL COURT ERRED IN HOLDING THE SEARCH IN THE HOUSE OF warrant. Rule 113, Section 6 * of the Rules of Court, provides the exceptions as
ACCUSED-APPELLANT FOR FIREARM WITHOUT VALID WARRANT TO BE follows:
LAWFUL.
a) When the person to be arrested has committed, is actually committing, or is
III THE TRIAL COURT ERRED IN HOLDING ACCUSED-APPELLANT GUILTY about to commit an offense in his presence;
BEYOND REASONABLE DOUBT FOR VIOLATION OF P.D. No. 9 IN RELATION
TO GENERAL ORDERS NOS. 6 AND 7 b) When an offense has in fact been committed, and he has reasonable ground to
believe that the person to be arrested has committed it;
Was the arrest of Ruben Burgos lawful? Were the search of his house and the
subsequent confiscation of a firearm and documents allegedly found therein c) When the person to be arrested is a prisoner who has escaped from a penal
conducted in a lawful and valid manner? Does the evidence sustaining the crime establishment or place where he is serving final judgment or temporarily confined
charged meet the test of proving guilt beyond reasonable doubt? while his case is pending or has escaped while being transferred from one
confinement to another.
The records of the case disclose that when the police authorities went to the house
of Ruben Burgos for the purpose of arresting him upon information given by Cesar The Court stated that even if there was no warrant for the arrest of Burgos, the fact
Masamlok that the accused allegedly recruited him to join the New People's Army that "the authorities received an urgent report of accused's involvement in
(NPA), they did not have any warrant of arrest or search warrant with them (TSN, p. subversive activities from a reliable source (report of Cesar Masamlok) the
25, October 14, 1982; and TSN, p. 61, November 15, 1982). circumstances of his arrest, even without judicial warrant, is lawfully within the
ambit of Section 6-A of Rule 113 of the Rules of Court and applicable jurisprudence
Article IV, Section 3 of the Constitution provides: on the matter."
The right of the people to be secure in their persons, houses, papers, and If the arrest is valid, the consequent search and seizure of the firearm and the
effects against unreasonable searches and seizures of whatever nature and alleged subversive documents would become an incident to a lawful arrest as
for any purpose shall not be violated, and no search warrant or warrant of provided by Rule 126, Section 12, which states:
arrest shall issue except upon probable cause to be determined by the
judge, or such other responsible officer as may be authorized by law, after A person charged with an offense may be searched for dangerous weapons
examination under oath or affirmation of the complainant and the or anything which may be used as proof of the commission of the offense.
witnesses he may produce, and particularly describing the place to be The conclusions reached by the trial court are erroneous.
searched, and the persons or things to be seized.
Under Section 6(a) of Rule 113, the officer arresting a person who has just
The constitutional provision is a safeguard against wanton and unreasonable committed, is committing, or is about to commit an offense must
invasion of the privacy and liberty of a citizen as to his person, papers and effects. have personal knowledge of that fact. The offense must also be committed in his
This Court explained in Villanueva vs. Querubin (48 SCRA 345) why this right is so presence or within his view. (Sayo v. Chief of Police, 80 Phil. 859).
important:
There is no such personal knowledge in this case. Whatever knowledge was
It is deference to one's personality that lies at the core of this right, but it possessed by the arresting officers, it came in its entirety from the information
could be also looked upon as a recognition of a constitutionally protected furnished by Cesar Masamlok. The location of the firearm was given by the
area, primarily one's home, but not necessarily thereto confined. (Cf. Hoffa appellant's wife.
v. United States, 385 US 293 [19661) What is sought to be guarded is a
man's prerogative to choose who is allowed entry to his residence. In that At the time of the appellant's arrest, he was not in actual possession of any firearm
haven of refuge, his individuality can assert itself not only in the choice of or subversive document. Neither was he committing any act which could be
who shall be welcome but likewise in the kind of objects he wants around described as subversive. He was, in fact, plowing his field at the time of the arrest.
him. There the state, however powerful, does not as such have access
The right of a person to be secure against any unreasonable seizure of his body and
except under the circumstances above noted, for in the traditional
any deprivation of his liberty is a most basic and fundamental one. The statute or
formulation, his house, however humble, is his castle. Thus is outlawed any
rule which allows exceptions to the requirement of warrants of arrest is strictly
unwarranted intrusion by government, which is called upon to refrain from
construed. Any exception must clearly fall within the situations when securing a
any invasion of his dwelling and to respect the privacies of his life, (Cf.
warrant would be absurd or is manifestly unnecessary as provided by the Rule. We
Schmerber v. California, 384 US 757 [1966], Brennan, J. and Boyd v. United
cannot liberally construe the rule on arrests without warrant or extend its application
States, 116 US 616, 630 [1886]). In the same vein, Landynski in his
beyond the cases specifically provided by law. To do so would infringe upon personal
authoritative work (Search and Seizure and the Supreme Court [1966],
liberty and set back a basic right so often violated and so deserving of full
could fitly characterize this constitutional right as the embodiment of a
protection.
'spiritual concept: the belief that to value the privacy of home and person
and to afford its constitutional protection against the long reach of The Solicitor General is of the persuasion that the arrest may still be considered
government is no legs than to value human dignity, and that his privacy lawful under Section 6(b) using the test of reasonableness. He submits that. the
must not be disturbed except in case of overriding social need, and then information given by Cesar Masamlok was sufficient to induce a reasonable ground
only under stringent procedural safeguards.' (Ibid, p. 47). that a crime has been committed and that the accused is probably guilty thereof.
The trial court justified the arrest of the accused-appelant without any warrant as In arrests without a warrant under Section 6(b), however, it is not enough that there
falling under one of the instances when arrests may be validly made without a is reasonable ground to believe that the person to be arrested has committed a
crime. A crime must in fact or actually have been committed first. That a crime has

63
actually been committed is an essential precondition. It is not enough to suspect A None Sir.
that a crime may have been committed. The fact of the commission of the offense
must be undisputed. The test of reasonable ground applies only to the identity of the Q Neither were you armed with a search warrant?
perpetrator. A No Sir.
In this case, the accused was arrested on the sole basis of Masamlok's verbal report. Q As a matter of fact, Burgos was not present in his house when you went
Masamlok led the authorities to suspect that the accused had committed a crime. there?
They were still fishing for evidence of a crime not yet ascertained. The subsequent
recovery of the subject firearm on the basis of information from the lips of a A But he was twenty meters away from his house.
frightened wife cannot make the arrest lawful, If an arrest without warrant is
Q Ruben Burgos was then plowing his field?
unlawful at the moment it is made, generally nothing that happened or is discovered
afterwards can make it lawful. The fruit of a poisoned tree is necessarily also tainted. A Yes Sir.
More important, we find no compelling reason for the haste with which the arresting Q When you called for Ruben Burgos you interviewed him?
officers sought to arrest the accused. We fail to see why they failed to first go
through the process of obtaining a warrant of arrest, if indeed they had reasonable A Yes Sir.
ground to believe that the accused had truly committed a crime. There is no
Q And that you told him that Masamlok implicated him?
showing that there was a real apprehension that the accused was on the verge of
flight or escape. Likewise, there is no showing that the whereabouts of the accused A No Sir.
were unknown,
Q What did you tell him?
The basis for the action taken by the arresting officer was the verbal report made by
Masamlok who was not required to subscribe his allegations under oath. There was A That we received information that you have a firearm, you surrender that
no compulsion for him to state truthfully his charges under pain of criminal firearm, first he denied but when Sgt. Buncalan interviewed his wife, his
prosecution. (TSN, p. 24, October 14, 1982). Consequently, the need to go through wife told him that it is buried, I dug the firearm which was wrapped with a
the process of securing a search warrant and a warrant of arrest becomes even cellophane.
more clear. The arrest of the accused while he was plowing his field is illegal. The Q In your interview of Burgos you did not remind him of his rights under the
arrest being unlawful, the search and seizure which transpired afterwards could not constitution considering that he was purposely under arrest?
likewise be deemed legal as being mere incidents to a valid arrest.
A I did not.
Neither can it be presumed that there was a waiver, or that consent was given by
the accused to be searched simply because he failed to object. To constitute a Q As a matter of fact, he denied that he has ever a gun?
waiver, it must appear first that the right exists; secondly, that the person involved
had knowledge, actual or constructive, of the existence of such a right; and lastly, A Yes Sir.
that said person had an actual intention to relinquish the right (Pasion Vda. de Q As a matter of fact, the gun was not in his possession?
Garcia v. Locsin, 65 Phil. 689). The fact that the accused failed to object to the entry
into his house does not amount to a permission to make a search therein (Magoncia A It was buried down in his horse.
v. Palacio, 80 Phil. 770). As pointed out by Justice Laurel in the case of Pasion Vda.
de Garcia V. Locsin (supra) Q As a matter of fact, Burgos did not point to where it was buried?

xxx xxx xxx A Yes Sir.

. . . As the constitutional guaranty is not dependent upon any affirmative (TSN, pp. 25-26, Hearing-October 14, 1982)
act of the citizen, the courts do not place the citizen in the position of either Considering that the questioned firearm and the alleged subversive documents were
contesting an officer's authority by force, or waiving his constitutional obtained in violation of the accused's constitutional rights against unreasonable
rights; but instead they hold that a peaceful submission to a search or searches and seizures, it follows that they are inadmissible as evidence.
seizure is not a consent or an invitation thereto, but is merely a
demonstration of regard for the supremacy of the law. (56 C.J., pp. 1180, There is another aspect of this case.
1181).
In proving ownership of the questioned firearm and alleged subversive documents,
We apply the rule that: "courts indulge every reasonable presumption against the prosecution presented the two arresting officers who testified that the accused
waiver of fundamental constitutional rights and that we do not presume readily admitted ownership of the gun after qqqs wife pointed to the place where it
acquiescence in the loss of fundamental rights." (Johnson v. Zerbst 304 U.S. 458). was buried. The officers stated that it was the accused himself who voluntarily
pointed to the place where the alleged subversive documents were hidden.
That the accused-appellant was not apprised of any of his constitutional rights at the
time of his arrest is evident from the records: Assuming this to be true, it should be recalled that the accused was never informed
of his constitutional rights at the time of his arrest. So that when the accused
A CALAMBA: allegedly admitted ownership of the gun and pointed to the location of the
Q When you went to the area to arrest Ruben Burgos, you were not armed subversive documents after questioning, the admissions were obtained in violation
with an arrest warrant?

64
of the constitutional right against self-incrimination under Sec. 20 of Art. IV of the Bill of Ternura cannot be considered as proceeding from a totally unbiased
of Rights winch provides: source. . . .
No person shall be compelled to be a witness against himself. Any person In the instant case, Masamlok's testimony was totally uncorroborated. Considering
under investigation for the commission of an offense shall have the right to that Masamlok surrendered to the military certainly his fate depended on how
remain silent and to counsel, and to be informed of such right.. . . eagerly he cooperated with the authorities. Otherwise, he would also be charged
with subversion. The trade-off appears to be his membership in the Civil Home
The Constitution itself mandates that any evidence obtained in violation of this right Defense Force. (TSN, p. 83, January 4, 1983). Masamlok may be considered as an
is inadmissible in evidence. Consequently, the testimonies of the arresting officers interested witness. It can not be said that his testimony is free from the opportunity
as to the admissions made by the appellant cannot be used against him. and temptation to be exaggerated and even fabricated for it was intended to secure
The trial court validly rejected the extra-judicial confession of the accused as his freedom.
inadmissible in evidence. The court stated that the appellant's having been Despite the fact that there were other persons present during the alleged NPA
exhaustively subjected to physical terror, violence, and third degree measures may seminar of April 19, 1982 i.e., Masamlok's father ,Matuguil Masamlok, Isabel Ilan and
not have been supported by reliable evidence but the failure to present the Ayok Ides (TSN, p. 74, January 4, 1983) who could have corroborated Cesar
investigator who conducted the investigation gives rise to the "provocative Masamlok's testimony that the accused used the gun in furtherance of subversive
presumption" that indeed torture and physical violence may have been committed activities or actually engaged in subversive acts, the prosecution never presented
as stated. any other witness.
The accused-appellant was not accorded his constitutional right to be assisted by This Court is, therefore, constrained to rule that the evidence presented by the
counsel during the custodial interrogation. The lower court correctly pointed out that prosecution is insufficient to prove the guilt of the accused beyond reasonable
the securing of counsel, Atty. Anyog, to help the accused when he subscribed under doubt.
oath to his statement at the Fiscal's Office was too late. It could have no palliative
effect. It cannot cure the absence of counsel at the time of the custodial As held in the case of People vs. Baia (34 SCRA 347):
investigation when the extrajudicial statement was being taken.
It is evident that once again, reliance can be placed on People v.
With the extra-judicial confession, the firearm, and the alleged subversive Dramayo (42 SCRA 59), where after stressing that accusation is not,
documents inadmissible in evidence against the accused-appellant, the only according to the fundamental law, synonymous with guilt, it was made
remaining proof to sustain the charge of Illegal Possession of Firearm in Furtherance clear: 'Only if the judge below and the appellate tribunal could arrive at a
of Subversion is the testimony of Cesar Masamlok. conclusion that the crime had been committed precisely by the person on
trial under such an exacting test should the sentence be one of conviction.
We find the testimony of Masamlok inadequate to convict Burgos beyond reasonable It is thus required that every circumstance favoring his innocence be duly
doubt. It is true that the trial court found Masamlok's testimony credible and taken into account. The proof against him must survive the test of reason;
convincing. However, we are not necessarily bound by the credibility which the trial the strongest suspicion must not be permitted to sway judgment. The
court attaches to a particular witness. As stated in People vs.. Cabrera (100 SCRA conscience must be satisfied that on the defendant could be laid the
424): responsibility for the offense charged; that not only did he perpetrate the
xxx xxx xxx act but that it amounted to a crime. What is required then is moral
certainty.' (Ibid, 64. Cf. People v. Alvarez, 55 SCRA 81; People v. Joven, 64
. . .Time and again we have stated that when it comes to question of SCRA 126; People vs. Ramirez, 69 SCRA 144; People vs. Godov 72 SCRA 69;
credibility the findings of the trial court are entitled to great respect upon People v. Lopez, 74 SCRA 205; People v. Poblador, 76 SCRA 634; People v.
appeal for the obvious reason th+at it was able to observe the demeanor, Quiazon, 78 SCRA 513; People v. Nazareno, 80 SCRA 484; People vs.
actuations and deportment of the witnesses during the trial. But we have Gabilan 115 SCRA 1; People v. Gabiana, 117 SCRA 260; and People vs.
also said that this rule is not absolute for otherwise there would be no Ibanga 124 SCRA 697).
reversals of convictions upon appeal. We must reject the findings of the trial
court where the record discloses circumstances of weight and substance We are aware of the serious problems faced by the military in Davao del Sur where
which were not properly appreciated by the trial court. there appears to be a well-organized plan to overthrow the Government through
armed struggle and replace it with an alien system based on a foreign ideology. The
The situation under which Cesar Masamlok testified is analogous to that found open defiance against duly constituted authorities has resulted in unfortunate levels
in People vs. Capadocia (17 SCRA 98 1): of violence and human suffering publicized all over the country and abroad. Even as
we reiterate the need for all freedom loving citizens to assist the military authorities
. . . The case against appellant is built on Ternura's testimony, and the issue
in their legitimate efforts to maintain peace and national security, we must also
hinges on how much credence can be accorded to him. The first remember the dictum in Morales vs. Enrile (1 21 SCRA 538, 569) when this Court
consideration is that said testimony stands uncorroborated. Ternura was the
stated:
only witness who testified on the mimeographing incident. . . .
While the government should continue to repel the communists, the
xxx xxx xxx
subversives, the rebels, and the lawless with an the means at its command,
. . .He was a confessed Huk under detention at the time. He knew his fate it should always be remembered that whatever action is taken must always
depended upon how much he cooperated with the authorities, who were be within the framework of our Constitution and our laws.
then engaged in a vigorous anti-dissident campaign. As in the case of
Rodrigo de Jesus, whose testimony We discounted for the same reason, that

65
Violations of human rights do not help in overcoming a rebellion. A cavalier attitude
towards constitutional liberties and protections will only fan the increase of
subversive activities instead of containing and suppressing them.
WHEREFORE, the judgment of conviction rendered by the trial court is REVERSED
and SET ASIDE. The accused-appellant is hereby ACQUITTED, on grounds of
reasonable doubt, of the crime with which he has been charged.
The subject firearm involved in this case (homemade revolver, caliber .38, Smith and
Wesson, with Serial No. 8.69221) and the alleged subversive documents are ordered
disposed of in accordance with law.
Cost de oficio.
SO ORDERED.
Feria (Chairman), Fernan, Alampay and Paras, JJ., concur.

66
MIRANDA V. ARIZONA, 384 U.S. 436 (1966) (h) The warnings required and the waiver needed are, in the absence of a fully
effective equivalent, prerequisites to the admissibility of any statement, inculpatory
or exculpatory, made by a defendant. Pp. 476-477.
Syllabus 2. The limitations on the interrogation process required for the protection of the
In each of these cases, the defendant, while in police custody, was questioned by individual's constitutional rights should not cause an undue interference with a
police officers, detectives, or a prosecuting attorney in a room in which he was cut proper system of law enforcement, as demonstrated by the procedures of the FBI
off from the outside world. None of the defendants was given a full and effective and the safeguards afforded in other jurisdictions. Pp. 479-491.
warning of his rights at the outset of the interrogation process. In all four cases, the 3. In each of these cases, the statements were obtained under circumstances that
questioning elicited oral admissions, and, in three of them, signed statements as did not meet constitutional standards for protection of the privilege against self-
well, which were admitted at their trials. All defendants were convicted, and all incrimination. Pp. 491-499.
convictions, except in No. 584, were affirmed on appeal.
98 Ariz. 18, 401 P.2d 721; 15 N.Y.2d 970, 207 N.E.2d 527; 16 N.Y.2d 614, 209 N.E.2d
Held: 110; 342 F.2d 684, reversed; 62 Cal.2d 571, 400 P.2d 97, affirmed.
1. The prosecution may not use statements, whether exculpatory or inculpatory,
stemming from questioning initiated by law enforcement officers after a person has CERTIORARI TO THE SUPREME COURT OF ARIZONA
been taken into custody or otherwise deprived of his freedom of action in any
significant way, unless it demonstrates the use of procedural safeguards effective to
secure the Fifth Amendment's privilege against self-incrimination. Pp. 444-491. MR. CHIEF JUSTICE WARREN delivered the opinion of the Court.

(a) The atmosphere and environment of incommunicado interrogation as it exists


today is inherently intimidating, and works to undermine the privilege against self- The cases before us raise questions which go to the roots of our concepts of
incrimination. Unless adequate preventive measures are taken to dispel the American criminal jurisprudence: the restraints society must observe consistent with
compulsion inherent in custodial surroundings, no statement obtained from the the Federal Constitution in prosecuting individuals for crime. More specifically, we
defendant can truly be the product of his free choice. Pp. 445-458. deal with the admissibility of statements obtained from an individual who is
subjected to custodial police interrogation and the necessity for procedures which
(b) The privilege against self-incrimination, which has had a long and expansive assure that the individual is accorded his privilege under the Fifth Amendment to the
historical development, is the essential mainstay of our adversary system, and Constitution not to be compelled to incriminate himself.
guarantees to the individual the "right to remain silent unless he chooses to speak in
the unfettered exercise of his own will," during a period of custodial interrogation as
We dealt with certain phases of this problem recently in Escobedo v. Illinois, 378 U.
well as in the courts or during the course of other official investigations. Pp. 458-465.
S. 478 (1964). There, as in the four cases before us, law enforcement officials took
(c) The decision in Escobedo v. Illinois, 378 U. S. 478, stressed the need for the defendant into custody and interrogated him in a police station for the purpose
protective devices to make the process of police interrogation conform to the of obtaining a confession. The police did not effectively advise him of his right to
dictates of the privilege. Pp. 465-466. remain silent or of his right to consult with his attorney. Rather, they confronted him
with an alleged accomplice who accused him of having perpetrated a murder. When
(d) In the absence of other effective measures, the following procedures to the defendant denied the accusation and said "I didn't shoot Manuel, you did it,"
safeguard the Fifth Amendment privilege must be observed: the person in custody they handcuffed him and took him to an interrogation room. There, while handcuffed
must, prior to interrogation, be clearly informed that he has the right to remain and standing, he was questioned for four hours until he confessed. During this
silent, and that anything he says will be used against him in court; he must be interrogation, the police denied his request to speak to his attorney, and they
clearly informed that he has the right to consult with a lawyer and to have the prevented his retained attorney, who had come to the police station, from consulting
lawyer with him during interrogation, and that, if he is indigent, a lawyer will be with him. At his trial, the State, over his objection, introduced the confession against
appointed to represent him. Pp. 467-473. him. We held that the statements thus made were constitutionally inadmissible.
(e) If the individual indicates, prior to or during questioning, that he wishes to remain
silent, the interrogation must cease; if he states that he wants an attorney, the This case has been the subject of judicial interpretation and spirited legal debate
questioning must cease until an attorney is present. Pp. 473-474. since it was decided two years ago. Both state and federal courts, in assessing its
implications, have arrived at varying conclusions. [Footnote 1] A wealth of scholarly
(f) Where an interrogation is conducted without the presence of an attorney and a material has been written tracing its ramifications and underpinnings. [Footnote 2]
statement is taken, a heavy burden rests on the Government to demonstrate that Police and prosecutor have speculated on its range and desirability. [Footnote 3] We
the defendant knowingly and intelligently waived his right to counsel. P. 475. granted certiorari in these cases, 382 U.S. 924, 925, 937, in order further to explore
(g) Where the individual answers some questions during in-custody interrogation, he some facets of the problems thus exposed of applying the privilege against self-
has not waived his privilege, and may invoke his right to remain silent thereafter. Pp. incrimination to in-custody interrogation, and to give concrete constitutional
475-476. guidelines for law enforcement agencies and courts to follow.

We start here, as we did in Escobedo, with the premise that our holding is not an
innovation in our jurisprudence, but is an application of principles long recognized

67
and applied in other settings. We have undertaken a thorough reexamination of Our holding will be spelled out with some specificity in the pages which follow, but,
the Escobedo decision and the principles it announced, and we reaffirm it. That case briefly stated, it is this: the prosecution may not use statements, whether
was but an explication of basic rights that are enshrined in our Constitution -- that exculpatory or inculpatory, stemming from custodial interrogation of the defendant
"No person . . . shall be compelled in any criminal case to be a witness against unless it demonstrates the use of procedural safeguards effective to secure the
himself," and that "the accused shall . . . have the Assistance of Counsel" -- rights privilege against self-incrimination. By custodial interrogation, we mean questioning
which were put in jeopardy in that case through official overbearing. These precious initiated by law enforcement officers after a person has been taken into custody or
rights were fixed in our Constitution only after centuries of persecution and struggle. otherwise deprived of his freedom of action in any significant way. [Footnote 4] As
And, in the words of Chief Justice Marshall, they were secured "for ages to come, and for the procedural safeguards to be employed, unless other fully effective means are
. . . designed to approach immortality as nearly as human institutions can approach devised to inform accused persons of their right of silence and to assure a
it,"Cohens v. Virginia, 6 Wheat. 264, 387 (1821). continuous opportunity to exercise it, the following measures are required. Prior to
any questioning, the person must be warned that he has a right to remain silent,
that any statement he does make may be used as evidence against him, and that he
Over 70 years ago, our predecessors on this Court eloquently stated:
has a right to the presence of an attorney, either retained or appointed. The
defendant may waive effectuation of these rights, provided the waiver is made
"The maxim nemo tenetur seipsum accusare had its origin in a protest against the voluntarily, knowingly and intelligently. If, however, he indicates in any manner and
inquisitorial and manifestly unjust methods of interrogating accused persons, which at any stage of the process that he wishes to consult with an attorney before
[have] long obtained in the continental system, and, until the expulsion of the speaking, there can be no questioning. Likewise, if the individual is alone and
Stuarts from the British throne in 1688 and the erection of additional barriers for the indicates in any manner that he does not wish to be interrogated, the police may not
protection of the people against the exercise of arbitrary power, [were] not question him. The mere fact that he may have answered some questions or
uncommon even in England. While the admissions or confessions of the prisoner, volunteered some statements on his own does not deprive him of the right to refrain
when voluntarily and freely made, have always ranked high in the scale of from answering any further inquiries until he has consulted with an attorney and
incriminating evidence, if an accused person be asked to explain his apparent thereafter consents to be questioned.
connection with a crime under investigation, the ease with which the questions put
to him may assume an inquisitorial character, the temptation to press the witness
I
unduly, to browbeat him if he be timid or reluctant, to push him into a corner, and to
entrap him into fatal contradictions, which is so painfully evident in many of the
earlier state trials, notably in those of Sir Nicholas Throckmorton and Udal, the The constitutional issue we decide in each of these cases is the admissibility of
Puritan minister, made the system so odious as to give rise to a demand for its total statements obtained from a defendant questioned while in custody or otherwise
abolition. The change in the English criminal procedure in that particular seems to deprived of his freedom of action in any significant way. In each, the defendant was
be founded upon no statute and no judicial opinion, but upon a general and silent questioned by police officers, detectives, or a prosecuting attorney in a room in
acquiescence of the courts in a popular demand. But, however adopted, it has which he was cut off from the outside world. In none of these cases was the
become firmly embedded in English as well as in American jurisprudence. So deeply defendant given a full and effective warning of his rights at the outset of the
did the iniquities of the ancient system impress themselves upon the minds of the interrogation process. In all the cases, the questioning elicited oral admissions, and
American colonists that the States, with one accord, made a denial of the right to in three of them, signed statements as well which were admitted at their trials. They
question an accused person a part of their fundamental law, so that a maxim, which all thus share salient features -- incommunicado interrogation of individuals in a
in England was a mere rule of evidence, became clothed in this country with the police-dominated atmosphere, resulting in self-incriminating statements without full
impregnability of a constitutional enactment." warnings of constitutional rights.

Brown v. Walker, 161 U. S. 591, 596-597 (1896). In stating the obligation of the An understanding of the nature and setting of this in-custody interrogation is
judiciary to apply these constitutional rights, this Court declared in Weems v. United essential to our decisions today. The difficulty in depicting what transpires at such
States, 217 U. S. 349, 373 (1910): interrogations stems from the fact that, in this country, they have largely taken
place incommunicado. From extensive factual studies undertaken in the early
1930's, including the famous Wickersham Report to Congress by a Presidential
". . . our contemplation cannot be only of what has been, but of what may be. Under
Commission, it is clear that police violence and the "third degree" flourished at that
any other rule, a constitution would indeed be as easy of application as it would be
time. [Footnote 5]
deficient in efficacy and power. Its general principles would have little value, and be
converted by precedent into impotent and lifeless formulas. Rights declared in words
might be lost in reality. And this has been recognized. The meaning and vitality of In a series of cases decided by this Court long after these studies, the police
the Constitution have developed against narrow and restrictive construction." resorted to physical brutality -- beating, hanging, whipping -- and to sustained and
protracted questioning incommunicado in order to extort confessions. [Footnote 6]
The Commission on Civil Rights in 1961 found much evidence to indicate that "some
This was the spirit in which we delineated, in meaningful language, the manner in
policemen still resort to physical force to obtain confessions," 1961 Comm'n on Civil
which the constitutional rights of the individual could be enforced against
Rights Rep. Justice, pt. 5, 17. The use of physical brutality and violence is not,
overzealous police practices. It was necessary in Escobedo, as here, to insure that
unfortunately, relegated to the past or to any part of the country. Only recently in
what was proclaimed in the Constitution had not become but a "form of
Kings County, New York, the police brutally beat, kicked and placed lighted cigarette
words," Silverthorne Lumber Co. v. United States, 251 U. S. 385, 392 (1920), in the
butts on the back of a potential witness under interrogation for the purpose of
hands of government officials. And it is in this spirit, consistent with our role as
judges, that we adhere to the principles of Escobedo today.

68
securing a statement incriminating a third party. People v. Portelli, 15 N.Y.2d 235, The efficacy of this tactic has been explained as follows:
205 N.E.2d 857, 257 N.Y.S.2d 931 (1965). [Footnote 7]
"If at all practicable, the interrogation should take place in the investigator's office or
The examples given above are undoubtedly the exception now, but they are at least in a room of his own choice. The subject should be deprived of every
sufficiently widespread to be the object of concern. Unless a proper limitation upon psychological advantage. In his own home, he may be confident, indignant, or
custodial interrogation is achieved -- such as these decisions will advance -- there recalcitrant. He is more keenly aware of his rights and more reluctant to tell of his
can be no assurance that practices of this nature will be eradicated in the indiscretions or criminal behavior within the walls of his home. Moreover his family
foreseeable future. The conclusion of the Wickersham Commission Report, made and other friends are nearby, their presence lending moral support. In his own office,
over 30 years ago, is still pertinent: the investigator possesses all the advantages. The atmosphere suggests the
invincibility of the forces of the law. [Footnote 11]"
"To the contention that the third degree is necessary to get the facts, the reporters
aptly reply in the language of the present Lord Chancellor of England (Lord Sankey):" To highlight the isolation and unfamiliar surroundings, the manuals instruct the
police to display an air of confidence in the suspect's guilt and, from outward
appearance, to maintain only an interest in confirming certain details. The guilt of
"It is not admissible to do a great right by doing a little wrong. . . . It is not sufficient
the subject is to be posited as a fact. The interrogator should direct his comments
to do justice by obtaining a proper result by irregular or improper means."
toward the reasons why the subject committed the act, rather than court failure by
asking the subject whether he did it. Like other men, perhaps the subject has had a
"Not only does the use of the third degree involve a flagrant violation of law by the bad family life, had an unhappy childhood, had too much to drink, had an unrequited
officers of the law, but it involves also the dangers of false confessions, and it tends desire for women. The officers are instructed to minimize the moral seriousness of
to make police and prosecutors less zealous in the search for objective evidence. As the offense, [Footnote 12] to cast blame on the victim or on society. [Footnote 13]
the New York prosecutor quoted in the report said, 'It is a short-cut, and makes the These tactics are designed to put the subject in a psychological state where his story
police lazy and unenterprising.' Or, as another official quoted remarked: 'If you use is but an elaboration of what the police purport to know already -- that he is guilty.
your fists, you are not so likely to use your wits.' We agree with the conclusion Explanations to the contrary are dismissed and discouraged.
expressed in the report, that"
The texts thus stress that the major qualities an interrogator should possess are
"The third degree brutalizes the police, hardens the prisoner against society, and patience and perseverance.
lowers the esteem in which the administration of Justice is held by the public."
One writer describes the efficacy of these characteristics in this manner:
"IV National Commission on Law Observance and Enforcement, Report on
Lawlessness in Law Enforcement 5 (1931)."
"In the preceding paragraphs, emphasis has been placed on kindness and
stratagems. The investigator will, however, encounter many situations where the
Again we stress that the modern practice of in-custody interrogation is sheer weight of his personality will be the deciding factor. Where emotional appeals
psychologically, rather than physically, oriented. As we have stated before, and tricks are employed to no avail, he must rely on an oppressive atmosphere of
dogged persistence. He must interrogate steadily and without relent, leaving the
subject no prospect of surcease. He must dominate his subject and overwhelm him
"Since Chambers v. Florida, 309 U. S. 227, this Court has recognized that coercion
with his inexorable will to obtain the truth. He should interrogate for a spell of
can be mental as well as physical, and that the blood of the accused is not the only
several hours, pausing only for the subject's necessities in acknowledgment of the
hallmark of an unconstitutional inquisition."
need to avoid a charge of duress that can be technically substantiated. In a serious
case, the interrogation may continue for days, with the required intervals for food
Blackburn v. Alabama, 361 U. S. 199, 206 (1960). Interrogation still takes place in and sleep, but with no respite from the atmosphere of domination. It is possible in
privacy. Privacy results in secrecy, and this, in turn, results in a gap in our knowledge this way to induce the subject to talk without resorting to duress or coercion. The
as to what, in fact, goes on in the interrogation rooms. A valuable source of method should be used only when the guilt of the subject appears highly probable.
information about present police practices, however, may be found in various police [Footnote 14]"
manuals and texts which document procedures employed with success in the past,
and which recommend various other effective tactics. [Footnote 8] These texts are
The manuals suggest that the suspect be offered legal excuses for his actions in
used by law enforcement agencies themselves as guides. [Footnote 9] It should be
order to obtain an initial admission of guilt. Where there is a suspected revenge
noted that these texts professedly present the most enlightened and effective
killing, for example, the interrogator may say:
means presently used to obtain statements through custodial interrogation. By
considering these texts and other data, it is possible to describe procedures
observed and noted around the country. "Joe, you probably didn't go out looking for this fellow with the purpose of shooting
him. My guess is, however, that you expected something from him, and that's why
you carried a gun -- for your own protection. You knew him for what he was, no good.
The officers are told by the manuals that the "principal psychological factor
Then when you met him, he probably started using foul, abusive language and he
contributing to a successful interrogation isprivacy -- being alone with the person
gave some indication that he was about to pull a gun on you, and that's when you
under interrogation. [Footnote 10]"
had to act to save your own life. That's about it, isn't it, Joe? [Footnote 15]"

69
Having then obtained the admission of shooting, the interrogator is advised to refer "Joe, you have a right to remain silent. That's your privilege, and I'm the last person
to circumstantial evidence which negates the self-defense explanation. This should in the world who'll try to take it away from you. If that's the way you want to leave
enable him to secure the entire story. One text notes that, this, O. K. But let me ask you this. Suppose you were in my shoes, and I were in
yours, and you called me in to ask me about this, and I told you, 'I don't want to
answer any of your questions.' You'd think I had something to hide, and you'd
"Even if he fails to do so, the inconsistency between the subject's original denial of
probably be right in thinking that. That's exactly what I'll have to think about you,
the shooting and his present admission of at least doing the shooting will serve to
and so will everybody else. So let's sit here and talk this whole thing over. [Footnote
deprive him of a self-defense 'out' at the time of trial. [Footnote 16]"
21]"

When the techniques described above prove unavailing, the texts recommend they
Few will persist in their initial refusal to talk, it is said, if this monologue is employed
be alternated with a show of some hostility. One ploy often used has been termed
correctly.
the "friendly-unfriendly," or the "Mutt and Jeff" act:

In the event that the subject wishes to speak to a relative or an attorney, the
". . . In this technique, two agents are employed. Mutt, the relentless investigator,
following advice is tendered:
who knows the subject is guilty and is not going to waste any time. He's sent a
dozen men away for this crime, and he's going to send the subject away for the full
term. Jeff, on the other hand, is obviously a kindhearted man. He has a family "[T]he interrogator should respond by suggesting that the subject first tell the truth
himself. He has a brother who was involved in a little scrape like this. He disapproves to the interrogator himself, rather than get anyone else involved in the matter. If the
of Mutt and his tactics, and will arrange to get him off the case if the subject will request is for an attorney, the interrogator may suggest that the subject save
cooperate. He can't hold Mutt off for very long. The subject would be wise to make a himself or his family the expense of any such professional service, particularly if he
quick decision. The technique is applied by having both investigators present while is innocent of the offense under investigation. The interrogator may also add, 'Joe,
Mutt acts out his role. Jeff may stand by quietly and demur at some of Mutt's tactics. I'm only looking for the truth, and if you're telling the truth, that's it. You can handle
When Jeff makes his plea for cooperation, Mutt is not present in the room. [Footnote this by yourself.' [Footnote 22] "
17] "
From these representative samples of interrogation techniques, the setting
The interrogators sometimes are instructed to induce a confession out of trickery. prescribed by the manuals and observed in practice becomes clear. In essence, it is
The technique here is quite effective in crimes which require identification or which this: to be alone with the subject is essential to prevent distraction and to deprive
run in series. In the identification situation, the interrogator may take a break in his him of any outside support. The aura of confidence in his guilt undermines his will to
questioning to place the subject among a group of men in a line-up. resist. He merely confirms the preconceived story the police seek to have him
describe. Patience and persistence, at times relentless questioning, are employed. To
obtain a confession, the interrogator must "patiently maneuver himself or his quarry
"The witness or complainant (previously coached, if necessary) studies the line-up
into a position from which the desired objective may be attained." [Footnote 23]
and confidently points out the subject as the guilty party. [Footnote 18]"
When normal procedures fail to produce the needed result, the police may resort to
deceptive stratagems such as giving false legal advice. It is important to keep the
Then the questioning resumes "as though there were now no doubt about the guilt subject off balance, for example, by trading on his insecurity about himself or his
of the subject." A variation on this technique is called the "reverse line-up": surroundings. The police then persuade, trick, or cajole him out of exercising his
constitutional rights.
"The accused is placed in a line-up, but this time he is identified by several fictitious
witnesses or victims who associated him with different offenses. It is expected that Even without employing brutality, the "third degree" or the specific stratagems
the subject will become desperate and confess to the offense under investigation in described above, the very fact of custodial interrogation exacts a heavy toll on
order to escape from the false accusations. [Footnote 19]" individual liberty, and trades on the weakness of individuals. [Footnote 24]

The manuals also contain instructions for police on how to handle the individual who This fact may be illustrated simply by referring to three confession cases decided by
refuses to discuss the matter entirely, or who asks for an attorney or relatives. The this Court in the Term immediately preceding our Escobedodecision. In Townsend v.
examiner is to concede him the right to remain silent. Sain, 372 U. S. 293 (1963), the defendant was a 19-year-old heroin addict, described
as a "near mental defective," id. at 307-310. The defendant in Lynumn v. Illinois, 372
U. S. 528 (1963), was a woman who confessed to the arresting officer after being
"This usually has a very undermining effect. First of all, he is disappointed in his
importuned to "cooperate" in order to prevent her children from being taken by relief
expectation of an unfavorable reaction on the part of the interrogator. Secondly, a
authorities. This Court, as in those cases, reversed the conviction of a defendant
concession of this right to remain silent impresses the subject with the apparent
in Haynes v. Washington, 373 U. S. 503 (1963), whose persistent request during his
fairness of his interrogator. [Footnote 20]"
interrogation was to phone his wife or attorney. [Footnote 25] In other settings, these
individuals might have exercised their constitutional rights. In the incommunicado
After this psychological conditioning, however, the officer is told to point out the police-dominated atmosphere, they succumbed.
incriminating significance of the suspect's refusal to talk:

70
In the cases before us today, given this background, we concern ourselves primarily any subject. The Trial of John Lilburn and John Wharton, 3 How.St.Tr. 1315 (1637). He
with this interrogation atmosphere and the evils it can bring. In No. 759, Miranda v. resisted the oath and declaimed the proceedings, stating:
Arizona, the police arrested the defendant and took him to a special interrogation
room, where they secured a confession. In No. 760,Vignera v. New York, the
"Another fundamental right I then contended for was that no man's conscience
defendant made oral admissions to the police after interrogation in the afternoon,
ought to be racked by oaths imposed to answer to questions concerning himself in
and then signed an inculpatory statement upon being questioned by an assistant
matters criminal, or pretended to be so."
district attorney later the same evening. In No. 761, Westover v. United States, the
defendant was handed over to the Federal Bureau of Investigation by local
authorities after they had detained and interrogated him for a lengthy period, both Haller & Davies, The Leveller Tracts 1647-1653, p. 454 (1944)
at night and the following morning. After some two hours of questioning, the federal
officers had obtained signed statements from the defendant. Lastly, in No.
On account of the Lilburn Trial, Parliament abolished the inquisitorial Court of Star
584, California v. Stewart, the local police held the defendant five days in the station
Chamber and went further in giving him generous reparation. The lofty principles to
and interrogated him on nine separate occasions before they secured his inculpatory
which Lilburn had appealed during his trial gained popular acceptance in England.
statement.
[Footnote 28] These sentiments worked their way over to the Colonies, and were
implanted after great struggle into the Bill of Rights. [Footnote 29] Those who
In these cases, we might not find the defendants' statements to have been framed our Constitution and the Bill of Rights were ever aware of subtle
involuntary in traditional terms. Our concern for adequate safeguards to protect encroachments on individual liberty. They knew that
precious Fifth Amendment rights is, of course, not lessened in the slightest. In each
of the cases, the defendant was thrust into an unfamiliar atmosphere and run
"illegitimate and unconstitutional practices get their first footing . . . by silent
through menacing police interrogation procedures. The potentiality for compulsion is
approaches and slight deviations from legal modes of procedure."
forcefully apparent, for example, in Miranda, where the indigent Mexican defendant
was a seriously disturbed individual with pronounced sexual fantasies, and
in Stewart, in which the defendant was an indigent Los Angeles Negro who had Boyd v. United States, 116 U. S. 616, 635 (1886). The privilege was elevated to
dropped out of school in the sixth grade. To be sure, the records do not evince overt constitutional status, and has always been "as broad as the mischief against which it
physical coercion or patent psychological ploys. The fact remains that in none of seeks to guard." Counselman v. Hitchcock, 142 U. S. 547, 562 (1892). We cannot
these cases did the officers undertake to afford appropriate safeguards at the outset depart from this noble heritage.
of the interrogation to insure that the statements were truly the product of free
choice.
Thus, we may view the historical development of the privilege as one which groped
for the proper scope of governmental power over the citizen. As a "noble principle
It is obvious that such an interrogation environment is created for no purpose other often transcends its origins," the privilege has come rightfully to be recognized in
than to subjugate the individual to the will of his examiner. This atmosphere carries part as an individual's substantive right, a "right to a private enclave where he may
its own badge of intimidation. To be sure, this is not physical intimidation, but it is lead a private life. That right is the hallmark of our democracy." United States v.
equally destructive of human dignity. [Footnote 26] The current practice of Grunewald, 233 F.2d 556, 579, 581-582 (Frank, J., dissenting), rev'd, 353 U.S. 391
incommunicado interrogation is at odds with one of our (1957). We have recently noted that the privilege against self-incrimination -- the
essential mainstay of our adversary system -- is founded on a complex of
values, Murphy v. Waterfront Comm'n, 378 U. S. 52, 55-57, n. 5 (1964); Tehan v.
Nation's most cherished principles -- that the individual may not be compelled to
Shott, 382 U. S. 406, 414-415, n. 12 (1966). All these policies point to one overriding
incriminate himself. Unless adequate protective devices are employed to dispel the
thought: the constitutional foundation underlying the privilege is the respect a
compulsion inherent in custodial surroundings, no statement obtained from the
government -- state or federal -- must accord to the dignity and integrity of its
defendant can truly be the product of his free choice.
citizens. To maintain a "fair state-individual balance," to require the government "to
shoulder the entire load," 8 Wigmore, Evidence 317 (McNaughton rev.1961), to
From the foregoing, we can readily perceive an intimate connection between the respect the inviolability of the human personality, our accusatory system of criminal
privilege against self-incrimination and police custodial questioning. It is fitting to justice demands that the government seeking to punish an individual produce the
turn to history and precedent underlying the Self-Incrimination Clause to determine evidence against him by its own independent labors, rather than by the cruel, simple
its applicability in this situation. expedient of compelling it from his own mouth. Chambers v. Florida, 309 U. S. 227,
235-238 (1940). In sum, the privilege is fulfilled only when the person is guaranteed
the right "to remain silent unless he chooses to speak in the unfettered exercise of
II
his own will." Malloy v. Hogan, 378 U. S. 1, 8 (1964).

We sometimes forget how long it has taken to establish the privilege against self-
The question in these cases is whether the privilege is fully applicable during a
incrimination, the sources from which it came, and the fervor with which it was
period of custodial interrogation.
defended. Its roots go back into ancient times. [Footnote 27] Perhaps the critical
historical event shedding light on its origins and evolution was the trial of one John
Lilburn, a vocal anti-Stuart Leveller, who was made to take the Star Chamber Oath in In this Court, the privilege has consistently been accorded a liberal
1637. The oath would have bound him to answer to all questions posed to him on construction. Albertson v. SACB, 382 U. S. 70, 81 (1965); Hoffman v. United
States,341 U. S. 479, 486 (1951); Arndstein v. McCarthy, 254 U. S. 71, 72-73

71
(1920);Counselman v. Hitchock, 142 U. S. 547, 562 (1892). We are satisfied that all Government concedes this point as well established in No. 761, Westover v. United
the principles embodied in the privilege apply to informal compulsion exerted by law States, stating:
enforcement officers during in-custody questioning. An individual swept from
familiar surroundings into police custody, surrounded by antagonistic forces, and
"We have no doubt . . . that it is possible for a suspect's Fifth Amendment right to be
subjected to the techniques of persuasion described above cannot be otherwise than
violated during in-custody questioning by a law enforcement officer. [Footnote 31]"
under compulsion to speak. As a practical matter, the compulsion to speak in the
isolated setting of the police station may well be greater than in courts or other
official investigations, where there are often impartial observers to guard against Because of the adoption by Congress of Rule 5(a) of the Federal Rules of Criminal
intimidation or trickery. [Footnote 30] Procedure, and this Court's effectuation of that Rule in McNabb v. United States, 318
U. S. 332 (1943), and Mallory v. United States, 354 U. S. 449(1957), we have had
little occasion in the past quarter century to reach the constitutional issues in
This question, in fact, could have been taken as settled in federal courts almost 70
dealing with federal interrogations. These supervisory rules, requiring production of
years ago, when, in Bram v. United States, 168 U. S. 532, 542 (1897), this Court
an arrested person before a commissioner "without unnecessary delay" and
held:
excluding evidence obtained in default of that statutory obligation, were nonetheless
responsive to the same considerations of Fifth Amendment policy that unavoidably
"In criminal trials, in the courts of the United States, wherever a question arises face us now as to the States. InMcNabb, 318 U.S. at 343-344, and in Mallory, 354
whether a confession is incompetent because not voluntary, the issue is controlled U.S. at 455-456, we recognized both the dangers of interrogation and the
by that portion of the Fifth Amendment . . . commanding that no person 'shall be appropriateness of prophylaxis stemming from the very fact of interrogation itself.
compelled in any criminal case to be a witness against himself.'" [Footnote 32]

In Bram, the Court reviewed the British and American history and case law and set Our decision in Malloy v. Hogan, 378 U. S. 1 (1964), necessitates an examination of
down the Fifth Amendment standard for compulsion which we implement today: the scope of the privilege in state cases as well. In Malloy, we squarely held the
privilege applicable to the States, and held that the substantive standards
underlying the privilege applied with full force to state court proceedings. There, as
"Much of the confusion which has resulted from the effort to deduce from the
in Murphy v. Waterfront Comm'n, 378 U. S. 52 (1964), and Griffin v. California, 380
adjudged cases what would be a sufficient quantum of proof to show that a
U. S. 609 (1965), we applied the existing Fifth Amendment standards to the case
confession was or was not voluntary, has arisen from a misconception of the subject
before us. Aside from the holding itself, the reasoning inMalloy made clear what had
to which the proof must address itself. The rule is not that, in order to render a
already become apparent -- that the substantive and procedural safeguards
statement admissible, the proof must be adequate to establish that the particular
surrounding admissibility of confessions in state cases had become exceedingly
communications contained in a statement were voluntarily made, but it must be
exacting, reflecting all the policies embedded in the privilege, 378 U.S. at 7-8.
sufficient to establish that the making of the statement was voluntary; that is to say,
[Footnote 33] The voluntariness doctrine in the state cases, as Malloy indicates,
that from the causes, which the law treats as legally sufficient to engender in the
encompasses all interrogation practices which are likely to exert such pressure upon
mind of the accused hope or fear in respect to the crime charged, the accused was
an individual as to disable him from making a free and rational choice. [Footnote 34]
not involuntarily impelled to make a statement, when, but for the improper
The implications of this proposition were elaborated in our decision in Escobedo v.
influences, he would have remained silent. . . ."
Illinois, 378 U. S. 478, decided one week after Malloy applied the privilege to the
States.
168 U.S. at 549. And see id. at 542.
Our holding there stressed the fact that the police had not advised the defendant of
The Court has adhered to this reasoning. In 1924, Mr. Justice Brandeis wrote for a his constitutional privilege to remain silent at the outset of the interrogation, and we
unanimous Court in reversing a conviction resting on a compelled confession,Wan v. drew attention to that fact at several points in the decision, 378 U.S. at 483, 485,
United States, 266 U. S. 1. He stated: 491. This was no isolated factor, but an essential ingredient in our decision. The
entire thrust of police interrogation there, as in all the cases today, was to put the
defendant in such an emotional state as to impair his capacity for rational judgment.
"In the federal courts, the requisite of voluntariness is not satisfied by establishing
The abdication of the constitutional privilege -- the choice on his part to speak to the
merely that the confession was not induced by a promise or a threat. A confession is
police -- was not made knowingly or competently because of the failure to apprise
voluntary in law if, and only if, it was, in fact, voluntarily made. A confession may
him of his rights; the compelling atmosphere of the in-custody interrogation, and not
have been given voluntarily, although it was made to police officers, while in
an independent decision on his part, caused the defendant to speak.
custody, and in answer to an examination conducted by them. But a confession
obtained by compulsion must be excluded whatever may have been the character of
the compulsion, and whether the compulsion was applied in a judicial proceeding or A different phase of the Escobedo decision was significant in its attention to the
otherwise. Bram v. United States, 168 U. S. 532." absence of counsel during the questioning. There, as in the cases today, we sought
a protective device to dispel the compelling atmosphere of the interrogation.
In Escobedo, however, the police did not relieve the defendant of the anxieties
266 U.S. at 14-15. In addition to the expansive historical development of the
which they had created in the interrogation rooms. Rather, they denied his request
privilege and the sound policies which have nurtured its evolution, judicial precedent
for the assistance of counsel, 378 U.S. at 481, 488, 491. [Footnote 35] This
thus clearly establishes its application to incommunicado interrogation. In fact, the
heightened his dilemma, and made his later statements the product of this
compulsion. Cf. Haynes v. Washington, 373 U. S. 503, 373 U. S. 514 (1963). The

72
denial of the defendant's request for his attorney thus undermined his ability to At the outset, if a person in custody is to be subjected to interrogation, he must first
exercise the privilege -- to remain silent if he chose or to speak without any be informed in clear and unequivocal terms that he has the right to remain silent.
intimidation, blatant or subtle. The presence of counsel, in all the cases before us For those unaware of the privilege, the warning is needed simply to make them
today, would he the adequate protective device necessary to make the process of aware of it -- the threshold requirement for an intelligent decision as to its exercise.
police interrogation conform to the dictates of the privilege. His presence would More important, such a warning is an absolute prerequisite in overcoming the
insure that statements made in the government-established atmosphere are not the inherent pressures of the interrogation atmosphere. It is not just the subnormal or
product of compulsion. woefully ignorant who succumb to an interrogator's imprecations, whether implied or
expressly stated, that the interrogation will continue until a confession is obtained or
that silence in the face of accusation is itself damning, and will bode ill when
It was in this manner that Escobedo explicated another facet of the pretrial privilege,
presented to a jury. [Footnote 37] Further, the warning will show the individual that
noted in many of the Court's prior decisions: the protection of rights at trial.
his interrogators are prepared to recognize his privilege should he choose to exercise
[Footnote 36] That counsel is present when statements are taken from an individual
it.
during interrogation obviously enhances the integrity of the factfinding processes in
court. The presence of an attorney, and the warnings delivered to the individual,
enable the defendant under otherwise compelling circumstances to tell his story The Fifth Amendment privilege is so fundamental to our system of constitutional
without fear, effectively, and in a way that eliminates the evils in the interrogation rule, and the expedient of giving an adequate warning as to the availability of the
process. Without the protections flowing from adequate warnings and the rights of privilege so simple, we will not pause to inquire in individual cases whether the
counsel, defendant was aware of his rights without a warning being given. Assessments of
the knowledge the defendant possessed, based on information as to his age,
education, intelligence, or prior contact with authorities, can never be more than
"all the careful safeguards erected around the giving of testimony, whether by an
speculation; [Footnote 38] a warning is a clear-cut fact. More important, whatever
accused or any other witness, would become empty formalities in a procedure where
the background of the person interrogated, a warning at the time of the
the most compelling possible evidence of guilt, a confession, would have already
interrogation is indispensable to overcome its pressures and to insure that the
been obtained at the unsupervised pleasure of the police."
individual knows he is free to exercise the privilege at that point in time.

Mapp v. Ohio, 367 U. S. 643, 685 (1961) (HARLAN, J., dissenting). Cf. Pointer v.
The warning of the right to remain silent must be accompanied by the explanation
Texas, 380 U. S. 400 (1965).
that anything said can and will be used against the individual in court. This warning
is needed in order to make him aware not only of the privilege, but also of the
III consequences of forgoing it. It is only through an awareness of these consequences
that there can be any assurance of real understanding and intelligent exercise of the
privilege. Moreover, this warning may serve to make the individual more acutely
Today, then, there can be no doubt that the Fifth Amendment privilege is available
aware that he is faced with a phase of the adversary system -- that he is not in the
outside of criminal court proceedings, and serves to protect persons in all settings in
presence of persons acting solely in his interest.
which their freedom of action is curtailed in any significant way from being
compelled to incriminate themselves. We have concluded that, without proper
safeguards, the process of in-custody interrogation of persons suspected or accused The circumstances surrounding in-custody interrogation can operate very quickly to
of crime contains inherently compelling pressures which work to undermine the overbear the will of one merely made aware of his privilege by his interrogators.
individual's will to resist and to compel him to speak where he would not otherwise Therefore, the right to have counsel present at the interrogation is indispensable to
do so freely. In order to combat these pressures and to permit a full opportunity to the protection of the Fifth Amendment privilege under the system we delineate
exercise the privilege against self-incrimination, the accused must be adequately today. Our aim is to assure that the individual's right to choose between silence and
and effectively apprised of his rights, and the exercise of those rights must be fully speech remains unfettered throughout the interrogation process. A once-stated
honored. warning, delivered by those who will conduct the interrogation, cannot itself suffice
to that end among those who most require knowledge of their rights. A mere
warning given by the interrogators is not alone sufficient to accomplish that end.
It is impossible for us to foresee the potential alternatives for protecting the privilege
Prosecutors themselves claim that the admonishment of the right to remain silent,
which might be devised by Congress or the States in the exercise of their creative
without more, "will benefit only the recidivist and the professional." Brief for the
rulemaking capacities. Therefore, we cannot say that the Constitution necessarily
National District Attorneys Association as amicus curiae, p. 14. Even preliminary
requires adherence to any particular solution for the inherent compulsions of the
advice given to the accused by his own attorney can be swiftly overcome by the
interrogation process as it is presently conducted. Our decision in no way creates a
secret interrogation process. Cf. Escobedo v. Illinois, 378 U. S. 478, 485, n. 5. Thus,
constitutional straitjacket which will handicap sound efforts at reform, nor is it
the need for counsel to protect the Fifth Amendment privilege comprehends not
intended to have this effect. We encourage Congress and the States to continue
merely a right to consult with counsel prior to questioning, but also to have counsel
their laudable search for increasingly effective ways of protecting the rights of the
present during any questioning if the defendant so desires.
individual while promoting efficient enforcement of our criminal laws. However,
unless we are shown other procedures which are at least as effective in apprising
accused persons of their right of silence and in assuring a continuous opportunity to The presence of counsel at the interrogation may serve several significant subsidiary
exercise it, the following safeguards must be observed. functions, as well. If the accused decides to talk to his interrogators, the assistance
of counsel can mitigate the dangers of untrustworthiness. With a lawyer present, the
likelihood that the police will practice coercion is reduced, and, if coercion is

73
nevertheless exercised, the lawyer can testify to it in court. The presence of a lawyer administration of justice. [Footnote 41] Denial of counsel to the indigent at the time
can also help to guarantee that the accused gives a fully accurate statement to the of interrogation while allowing an attorney to those who can afford one would be no
police, and that the statement is rightly reported by the prosecution at trial. See more supportable by reason or logic than the similar situation at trial and on appeal
Crooker v. California, 357 U. S. 433, 443-448 (1958) (DOUGLAS, J., dissenting). struck down in Gideon v. Wainwright, 372 U. S. 335 (1963), and Douglas v.
California, 372 U. S. 353 (1963).
An individual need not make a pre-interrogation request for a lawyer. While such
request affirmatively secures his right to have one, his failure to ask for a lawyer In order fully to apprise a person interrogated of the extent of his rights under this
does not constitute a waiver. No effective waiver of the right to counsel during system, then, it is necessary to warn him not only that he has the right to consult
interrogation can be recognized unless specifically made after the warnings we here with an attorney, but also that, if he is indigent, a lawyer will be appointed to
delineate have been given. The accused who does not know his rights and therefore represent him. Without this additional warning, the admonition of the right to consult
does not make a request may be the person who most needs counsel. As the with counsel would often be understood as meaning only that he can consult with a
California Supreme Court has aptly put it: lawyer if he has one or has the funds to obtain one. The warning of a right to counsel
would be hollow if not couched in terms that would convey to the indigent -- the
person most often subjected to interrogation -- the knowledge that he too has a right
"Finally, we must recognize that the imposition of the requirement for the request
to have counsel present. [Footnote 42] As with the warnings of the right to remain
would discriminate against the defendant who does not know his rights. The
silent and of the general right to counsel, only by effective and express explanation
defendant who does not ask for counsel is the very defendant who most needs
to the indigent of this right can there be assurance that he was truly in a position to
counsel. We cannot penalize a defendant who, not understanding his constitutional
exercise it. [Footnote 43]
rights, does not make the formal request, and, by such failure, demonstrates his
helplessness. To require the request would be to favor the defendant whose
sophistication or status had fortuitously prompted him to make it." Once warnings have been given, the subsequent procedure is clear. If the individual
indicates in any manner, at any time prior to or during questioning, that he wishes to
remain silent, the interrogation must cease. [Footnote 44] At this point, he has
People v. Dorado, 62 Cal.2d 338, 351, 398 P.2d 361, 369-370, 42 Cal.Rptr. 169, 177-
shown that he intends to exercise his Fifth Amendment privilege; any statement
178 (1965) (Tobriner, J.). In Carnley v. Cochran, 369 U. S. 506, 513 (1962), we stated:
taken after the person invokes his privilege cannot be other than the product of
compulsion, subtle or otherwise. Without the right to cut off questioning, the setting
"[I]t is settled that, where the assistance of counsel is a constitutional requisite, the of in-custody interrogation operates on the individual to overcome free choice in
right to be furnished counsel does not depend on a request." producing a statement after the privilege has been once invoked. If the individual
states that he wants an attorney, the interrogation must cease until an attorney is
present. At that time, the individual must have an opportunity to confer with the
This proposition applies with equal force in the context of providing counsel to
attorney and to have him present during any subsequent questioning. If the
protect an accused's Fifth Amendment privilege in the face of interrogation.
individual cannot obtain an attorney and he indicates that he wants one before
[Footnote 39] Although the role of counsel at trial differs from the role during
speaking to police, they must respect his decision to remain silent.
interrogation, the differences are not relevant to the question whether a request is a
prerequisite.
This does not mean, as some have suggested, that each police station must have a
"station house lawyer" present at all times to advise prisoners. It does mean,
Accordingly, we hold that an individual held for interrogation must be clearly
however, that, if police propose to interrogate a person, they must make known to
informed that he has the right to consult with a lawyer and to have the lawyer with
him that he is entitled to a lawyer and that, if he cannot afford one, a lawyer will be
him during interrogation under the system for protecting the privilege we delineate
provided for him prior to any interrogation. If authorities conclude that they will not
today. As with the warnings of the right to remain silent and that anything stated can
provide counsel during a reasonable period of time in which investigation in the field
be used in evidence against him, this warning is an absolute prerequisite to
is carried out, they may refrain from doing so without violating the person's Fifth
interrogation. No amount of circumstantial evidence that the person may have been
Amendment privilege so long as they do not question him during that time.
aware of this right will suffice to stand in its stead. Only through such a warning is
there ascertainable assurance that the accused was aware of this right.
If the interrogation continues without the presence of an attorney and a statement is
taken, a heavy burden rests on the government to demonstrate that the defendant
If an individual indicates that he wishes the assistance of counsel before any
knowingly and intelligently waived his privilege against self-incrimination and his
interrogation occurs, the authorities cannot rationally ignore or deny his request on
right to retained or appointed counsel. Escobedo v. Illinois,378 U. S. 478, 490, n. 14.
the basis that the individual does not have or cannot afford a retained attorney. The
This Court has always set high standards of proof for the waiver of constitutional
financial ability of the individual has no relationship to the scope of the rights
rights, Johnson v. Zerbst, 304 U. S. 458 (1938), and we reassert these standards as
involved here. The privilege against self-incrimination secured by the Constitution
applied to in-custody interrogation. Since the State is responsible for establishing the
applies to all individuals. The need for counsel in order to protect the privilege exists
isolated circumstances under which the interrogation takes place, and has the only
for the indigent as well as the affluent. In fact, were we to limit these constitutional
means of making available corroborated evidence of warnings given during
rights to those who can retain an attorney, our decisions today would be of little
incommunicado interrogation, the burden is rightly on its shoulders.
significance. The cases before us, as well as the vast majority of confession cases
with which we have dealt in the past, involve those unable to retain counsel.
[Footnote 40] While authorities are not required to relieve the accused of his An express statement that the individual is willing to make a statement and does not
poverty, they have the obligation not to take advantage of indigence in the want an attorney, followed closely by a statement, could constitute a waiver. But a

74
valid waiver will not be presumed simply from the silence of the accused after Our decision is not intended to hamper the traditional function of police officers in
warnings are given, or simply from the fact that a confession was, in fact, eventually investigating crime. See Escobedo v. Illinois, 378 U. S. 478, 492. When an individual
obtained. A statement we made in Carnley v. Cochran, 369 U. S. 506, 516 (1962), is is in custody on probable cause, the police may, of course, seek out evidence in the
applicable here: field to be used at trial against him. Such investigation may include inquiry of
persons not under restraint. General on-the-scene questioning as to facts
surrounding a crime or other general questioning of citizens in the factfinding
"Presuming waiver from a silent record is impermissible. The record must show, or
process is not affected by our holding. It is an act of responsible citizenship for
there must be an allegation and evidence which show, that an accused was offered
individuals to give whatever information they may have to aid in law enforcement. In
counsel but intelligently and understandingly rejected the offer. Anything less is not
such situations, the compelling atmosphere inherent in the process of in-custody
waiver."
interrogation is not necessarily present. [Footnote 46]

See also Glasser v. United States, 315 U. S. 60 (1942). Moreover, where in-custody
In dealing with statements obtained through interrogation, we do not purport to find
interrogation is involved, there is no room for the contention that the privilege is
all confessions inadmissible. Confessions remain a proper element in law
waived if the individual answers some questions or gives some information on his
enforcement. Any statement given freely and voluntarily without any compelling
own prior to invoking his right to remain silent when interrogated. [Footnote 45]
influences is, of course, admissible in evidence. The fundamental import of the
privilege while an individual is in custody is not whether he is allowed to talk to the
Whatever the testimony of the authorities as to waiver of rights by an accused, the police without the benefit of warnings and counsel, but whether he can be
fact of lengthy interrogation or incommunicado incarceration before a statement is interrogated. There is no requirement that police stop a person who enters a police
made is strong evidence that the accused did not validly waive his rights. In these station and states that he wishes to confess to a crime, [Footnote 47] or a person
circumstances, the fact that the individual eventually made a statement is who calls the police to offer a confession or any other statement he desires to make.
consistent with the conclusion that the compelling influence of the interrogation Volunteered statements of any kind are not barred by the Fifth Amendment, and
finally forced him to do so. It is inconsistent with any notion of a voluntary their admissibility is not affected by our holding today.
relinquishment of the privilege. Moreover, any evidence that the accused was
threatened, tricked, or cajoled into a waiver will, of course, show that the defendant
To summarize, we hold that, when an individual is taken into custody or otherwise
did not voluntarily waive his privilege. The requirement of warnings and waiver of
deprived of his freedom by the authorities in any significant way and is subjected to
rights is a fundamental with respect to the Fifth Amendment privilege, and not
questioning, the privilege against self-incrimination is jeopardized. Procedural
simply a preliminary ritual to existing methods of interrogation.
safeguards must be employed to protect the privilege, and unless other fully
effective means are adopted to notify the person of his right of silence and to assure
The warnings required and the waiver necessary in accordance with our opinion that the exercise of the right will be scrupulously honored, the following measures
today are, in the absence of a fully effective equivalent, prerequisites to the are required. He must be warned prior to any questioning that he has the right to
admissibility of any statement made by a defendant. No distinction can be drawn remain silent, that anything he says can be used against him in a court of law, that
between statements which are direct confessions and statements which amount to he has the right to the presence of an attorney, and that, if he cannot afford an
"admissions" of part or all of an offense. The privilege against self-incrimination attorney one will be appointed for him prior to any questioning if he so desires.
protects the individual from being compelled to incriminate himself in any manner; it Opportunity to exercise these rights must be afforded to him throughout the
does not distinguish degrees of incrimination. Similarly, for precisely the same interrogation. After such warnings have been given, and such opportunity afforded
reason, no distinction may be drawn between inculpatory statements and him, the individual may knowingly and intelligently waive these rights and agree to
statements alleged to be merely "exculpatory." If a statement made were, in fact, answer questions or make a statement. But unless and until such warnings and
truly exculpatory, it would, of course, never be used by the prosecution. In fact, waiver are demonstrated by the prosecution at trial, no evidence obtained as a
statements merely intended to be exculpatory by the defendant are often used to result of interrogation can be used against him. [Footnote 48]
impeach his testimony at trial or to demonstrate untruths in the statement given
under interrogation, and thus to prove guilt by implication. These statements are
IV
incriminating in any meaningful sense of the word, and may not be used without the
full warnings and effective waiver required for any other statement.
In Escobedo itself, the defendant fully intended his accusation of another as the A recurrent argument made in these cases is that society's need for interrogation
slayer to be exculpatory as to himself. outweighs the privilege. This argument is not unfamiliar to this Court. See, e.g.,
Chambers v. Florida, 309 U. S. 227, 240-241 (1940). The whole thrust of our
foregoing discussion demonstrates that the Constitution has prescribed the rights of
The principles announced today deal with the protection which must be given to the
the individual when confronted with the power of government when it provided in
privilege against self-incrimination when the individual is first subjected to police
the Fifth Amendment that an individual cannot be compelled to be a witness against
interrogation while in custody at the station or otherwise deprived of his freedom of
himself. That right cannot be abridged. As Mr. Justice Brandeis once observed:
action in any significant way. It is at this point that our adversary system of criminal
proceedings commences, distinguishing itself at the outset from the inquisitorial
system recognized in some countries. Under the system of warnings we delineate "Decency, security and liberty alike demand that government officials shall be
today, or under any other system which may be devised and found effective, the subjected to the same rules of conduct that are commands to the citizen. In a
safeguards to be erected about the privilege must come into play at this point. government of laws, existence of the government will be imperilled if it fail to
observe the law scrupulously. Our Government is the potent, the omnipresent
teacher. For good or for ill, it teaches the whole people by its example. Crime is

75
contagious. If the Government becomes a lawbreaker, it breeds contempt for law; it held four persons, who were in the defendant's house at the time of the arrest, in jail
invites every man to become a law unto himself; it invites anarchy. To declare that, for five days until defendant confessed. At that time, they were finally released.
in the administration of the criminal law, the end justifies the means . . . would bring Police stated that there was "no evidence to connect them with any crime." Available
terrible retribution. Against that pernicious doctrine this Court should resolutely set statistics on the extent of this practice where it is condoned indicate that these four
its face." are far from alone in being subjected to arrest, prolonged detention, and
interrogation without the requisite probable cause. [Footnote 53]
Olmstead v. United States, 277 U. S. 438, 485 (1928) (dissenting opinion). [Footnote
49] In this connection, one of our country's distinguished jurists has pointed out: Over the years, the Federal Bureau of Investigation has compiled an exemplary
"The quality of a nation's civilization can be largely measured by the methods it uses record of effective law enforcement while advising any suspect or arrested person,
in the enforcement of its criminal law." [Footnote 50] at the outset of an interview, that he is not required to make a statement, that any
statement may be used against him in court, that the individual may obtain the
services of an attorney of his own choice, and, more recently, that he has a right to
If the individual desires to exercise his privilege, he has the right to do so. This is not
free counsel if he is unable to pay. [Footnote 54] A letter received from the Solicitor
for the authorities to decide. An attorney may advise his client not to talk to police
General in response to a question from the Bench makes it clear that the present
until he has had an opportunity to investigate the case, or he may wish to be
pattern of warnings and respect for the rights of the individual followed as a practice
present with his client during any police questioning. In doing so an attorney is
by the FBI is consistent with the procedure which we delineate today. It states:
merely exercising the good professional judgment he has been taught. This is not
cause for considering the attorney a menace to law enforcement. He is merely
carrying out what he is sworn to do under his oath -- to protect to the extent of his "At the oral argument of the above cause, Mr. Justice Fortas asked whether I could
ability the rights of his client. provide certain information as to the practices followed by the Federal Bureau of
Investigation. I have directed these questions to the attention of the Director of the
Federal Bureau of Investigation, and am submitting herewith a statement of the
In fulfilling this responsibility, the attorney plays a vital role in the administration of
questions and of the answers which we have received."
criminal justice under our Constitution.

" (1) When an individual is interviewed by agents of the Bureau, what warning is
In announcing these principles, we are not unmindful of the burdens which law
given to him?"
enforcement officials must bear, often under trying circumstances. We also fully
recognize the obligation of all citizens to aid in enforcing the criminal laws. This
Court, while protecting individual rights, has always given ample latitude to law " The standard warning long given by Special Agents of the FBI to both suspects and
enforcement agencies in the legitimate exercise of their duties. The limits we have persons under arrest is that the person has a right to say nothing and a right to
placed on the interrogation process should not constitute an undue interference with counsel, and that any statement he does make may be used against him in court.
a proper system of law enforcement. As we have noted, our decision does not in any Examples of this warning are to be found in the Westover case at 342 F.2d 684
way preclude police from carrying out their traditional investigatory functions. (1965), and Jackson v. U.S., 337 F.2d 136 (1964), cert. den., 380 U.S. 935."
Although confessions may play an important role in some convictions, the cases
before us present graphic examples of the overstatement of the "need" for
" After passage of the Criminal Justice Act of 1964, which provides free counsel for
confessions. In each case, authorities conducted interrogations ranging up to five
Federal defendants unable to pay, we added to our instructions to Special Agents
days in duration despite the presence, through standard investigating practices, of
the requirement that any person who is under arrest for an offense under FBI
considerable evidence against each defendant. [Footnote 51] Further examples are
jurisdiction, or whose arrest is contemplated following the interview, must also be
chronicled in our prior cases. See, e.g., Haynes v. Washington, 373 U. S. 503, 518-
advised of his right to free counsel if he is unable to pay, and the fact that such
519 (1963); Rogers v. Richmond, 365 U. S. 534, 541 (1961); Malinski v. New
counsel will be assigned by the Judge. At the same time, we broadened the right to
York, 324 U. S. 401,402 (1945). [Footnote 52]
counsel warning to read counsel of his own choice, or anyone else with whom he
might wish to speak."
It is also urged that an unfettered right to detention for interrogation should be
allowed because it will often redound to the benefit of the person questioned. When
" (2) When is the warning given?"
police inquiry determines that there is no reason to believe that the person has
committed any crime, it is said, he will be released without need for further formal
procedures. The person who has committed no offense, however, will be better able " The FBI warning is given to a suspect at the very outset of the interview, as shown
to clear himself after warnings with counsel present than without. It can be assumed in the Westover case, cited above. The warning may be given to a person arrested
that, in such circumstances, a lawyer would advise his client to talk freely to police as soon as practicable after the arrest, as shown in the Jackson case, also cited
in order to clear himself. above, and in U.S. v. Konigsberg, 336 F.2d 844 (1964), cert. den., 379 U.S. 933, but,
in any event, it must precede the interview with the person for a confession or
admission of his own guilt."
Custodial interrogation, by contrast, does not necessarily afford the innocent an
opportunity to clear themselves. A serious consequence of the present practice of
the interrogation alleged to be beneficial for the innocent is that many arrests "for " (3) What is the Bureau's practice in the event that (a) the individual requests
investigation" subject large numbers of innocent persons to detention and counsel and (b) counsel appears?"
interrogation. In one of the cases before us, No. 584, California v. Stewart, police

76
" When the person who has been warned of his right to counsel decides that he consistent with our legal system that we give at least as much protection to these
wishes to consult with counsel before making a statement, the interview is rights as is given in the jurisdictions described. We deal in our country with rights
terminated at that point, Shultz v. U.S., 351 F.2d 287 (1965). It may be continued, grounded in a specific requirement of the Fifth Amendment of the Constitution,
however, as to all matters other than the person's own guilt or innocence. If he is whereas other jurisdictions arrived at their conclusions on the basis of principles of
indecisive in his request for counsel, there may be some question on whether he did justice not so specifically defined. [Footnote 64]
or did not waive counsel. Situations of this kind must necessarily be left to the
judgment of the interviewing Agent. For example, inHiram v. U.S., 354 F.2d 4 (1965),
It is also urged upon us that we withhold decision on this issue until state legislative
the Agent's conclusion that the person arrested had waived his right to counsel was
bodies and advisory groups have had an opportunity to deal with these problems by
upheld by the courts."
rulemaking. [Footnote 65] We have already pointed out that the Constitution does
not require any specific code of procedures for protecting the privilege against self-
" A person being interviewed and desiring to consult counsel by telephone must be incrimination during custodial interrogation. Congress and the States are free to
permitted to do so, as shown in Caldwell v. U.S., 351 F.2d 459 (1965). When counsel develop their own safeguards for the privilege, so long as they are fully as effective
appears in person, he is permitted to confer with his client in private. " as those described above in informing accused persons of their right of silence and
in affording a continuous opportunity to exercise it. In any event, however, the
issues presented are of constitutional dimensions, and must be determined by the
" (4) What is the Bureau's practice if the individual requests counsel, but cannot
courts. The admissibility of a statement in the face of a claim that it was obtained in
afford to retain an attorney?"
violation of the defendant's constitutional rights is an issue the resolution of which
has long since been undertaken by this Court. See Hopt v. Utah, 110 U. S.
" If any person being interviewed after warning of counsel decides that he wishes to 574 (1884). Judicial solutions to problems of constitutional dimension have evolved
consult with counsel before proceeding, further the interview is terminated, as decade by decade. As courts have been presented with the need to enforce
shown above. FBI Agents do not pass judgment on the ability of the person to pay constitutional rights, they have found means of doing so. That was our responsibility
for counsel. They do, however, advise those who have been arrested for an offense when Escobedo was before us, and it is our responsibility today. Where rights
under FBI jurisdiction, or whose arrest is contemplated following the interview, of a secured by the Constitution are involved, there can be no rulemaking or legislation
right to free counsel if they are unable to pay, and the availability of such counsel which would abrogate them.
from the Judge. [Footnote 55]"
V
The practice of the FBI can readily be emulated by state and local enforcement
agencies. The argument that the FBI deals with different crimes than are dealt with
Because of the nature of the problem and because of its recurrent significance in
by state authorities does not mitigate the significance of the FBI experience.
numerous cases, we have to this point discussed the relationship of the Fifth
[Footnote 56]
Amendment privilege to police interrogation without specific concentration on the
facts of the cases before us. We turn now to these facts to consider the application
The experience in some other countries also suggests that the danger to law to these cases of the constitutional principles discussed above. In each instance, we
enforcement in curbs on interrogation is overplayed. The English procedure, since have concluded that statements were obtained from the defendant under
1912 under the Judges' Rules, is significant. As recently strengthened, the Rules circumstances that did not meet constitutional standards for protection of the
require that a cautionary warning be given an accused by a police officer as soon as privilege.
he has evidence that affords reasonable grounds for suspicion; they also require that
any statement made be given by the accused without questioning by police.
No. 759. Miranda v. Arizona
[Footnote 57] The right of the individual to consult with an attorney during this
period is expressly recognized. [Footnote 58]
On March 13, 1963, petitioner, Ernesto Miranda, was arrested at his home and taken
in custody to a Phoenix police station. He was there identified by the complaining
The safeguards present under Scottish law may be even greater than in England.
witness. The police then took him to "Interrogation Room No. 2" of the detective
Scottish judicial decisions bar use in evidence of most confessions obtained through
bureau. There he was questioned by two police officers. The officers admitted at trial
police interrogation. [Footnote 59] In India, confessions made to police not in the
that Miranda was not advised that he had a right to have an attorney present.
presence of a magistrate have been excluded by rule of evidence since 1872, at a
[Footnote 66] Two hours later, the officers emerged from the interrogation room with
time when it operated under British law. [Footnote 60] Identical provisions appear in
a written confession signed by Miranda. At the top of the statement was a typed
the Evidence Ordinance of Ceylon, enacted in 1895. [Footnote 61] Similarly, in our
paragraph stating that the confession was made voluntarily, without threats or
country, the Uniform Code of Military Justice has long provided that no suspect may
promises of immunity and "with full knowledge of my legal rights, understanding any
be interrogated without first being warned of his right not to make a statement, and
statement I make may be used against me." [Footnote 67]
that any statement he makes may be used against him. [Footnote 62] Denial of the
right to consult counsel during interrogation has also been proscribed by military
tribunals. [Footnote 63] There appears to have been no marked detrimental effect At his trial before a jury, the written confession was admitted into evidence over the
on criminal law enforcement in these jurisdictions as a result of these rules. objection of defense counsel, and the officers testified to the prior oral confession
Conditions of law enforcement in our country are sufficiently similar to permit made by Miranda during the interrogation. Miranda was found guilty of kidnapping
reference to this experience as assurance that lawlessness will not result from and rape. He was sentenced to 20 to 30 years' imprisonment on each count, the
warning an individual of his rights or allowing him to exercise them. Moreover, it is sentences to run concurrently. On appeal, the Supreme Court of Arizona held that

77
Miranda's constitutional rights were not violated in obtaining the confession, and We reverse. The foregoing indicates that Vignera was not warned of any of his rights
affirmed the conviction. 98 Ariz. 18, 401 P.2d 721. In reaching its decision, the court before the questioning by the detective and by the assistant district attorney. No
emphasized heavily the fact that Miranda did not specifically request counsel. other steps were taken to protect these rights. Thus, he was not effectively apprised
of his Fifth Amendment privilege or of his right to have counsel present, and his
statements are inadmissible.
We reverse. From the testimony of the officers and by the admission of respondent,
it is clear that Miranda was not in any way apprised of his right to consult with an
attorney and to have one present during the interrogation, nor was his right not to No. 761. Westover v. United States
be compelled to incriminate himself effectively protected in any other manner.
Without these warnings, the statements were inadmissible. The mere fact that he
At approximately 9:45 p.m. on March 20, 1963, petitioner, Carl Calvin Westover, was
signed a statement which contained a typed-in clause stating that he had "full
arrested by local police in Kansas City as a suspect in two Kansas City robberies. A
knowledge" of his "legal rights" does not approach the knowing and intelligent
report was also received from the FBI that he was wanted on a felony charge in
waiver required to relinquish constitutional rights. Cf.373 U. S. Washington, 373 U.S.
California. The local authorities took him to a police station and placed him in a line-
up on the local charges, and, at about 11:45 p.m., he was booked. Kansas City police
503, 512-513 (1963); Haley v. Ohio, 332 U. S. 596, 601 (1948) (opinion of MR interrogated Westover
JUSTICE DOUGLAS).
on the night of his arrest. He denied any knowledge of criminal activities. The next
No. 760. Vignera v. New York day, local officers interrogated him again throughout the morning. Shortly before
noon, they informed the FBI that they were through interrogating Westover and that
the FBI could proceed to interrogate him. There is nothing in the record to indicate
Petitioner, Michael Vignera, was picked up by New York police on October 14, 1960,
that Westover was ever given any warning as to his rights by local police. At noon,
in connection with the robbery three days earlier of a Brooklyn dress shop. They took
three special agents of the FBI continued the interrogation in a private interview
him to the 17th Detective Squad headquarters in Manhattan. Sometime thereafter,
room of the Kansas City Police Department, this time with respect to the robbery of a
he was taken to the 66th Detective Squad. There a detective questioned Vignera
savings and loan association and bank in Sacramento, California. After two or two
with respect to the robbery. Vignera orally admitted the robbery to the detective.
and one-half hours, Westover signed separate confessions to each of these two
The detective was asked on cross-examination at trial by defense counsel whether
robberies which had been prepared by one of the agents during the interrogation. At
Vignera was warned of his right to counsel before being interrogated. The
trial, one of the agents testified, and a paragraph on each of the statements states,
prosecution objected to the question, and the trial judge sustained the objection.
that the agents advised Westover that he did not have to make a statement, that
Thus, the defense was precluded from making any showing that warnings had not
any statement he made could be used against him, and that he had the right to see
been given. While at the 66th Detective Squad, Vignera was identified by the store
an attorney.
owner and a saleslady as the man who robbed the dress shop. At about 3 p.m., he
was formally arrested. The police then transported him to still another station, the
70th Precinct in Brooklyn, "for detention." At 11 p.m., Vignera was questioned by an Westover was tried by a jury in federal court and convicted of the California
assistant district attorney in the presence of a hearing reporter, who transcribed the robberies. His statements were introduced at trial. He was sentenced to 15 years'
questions and Vignera's answers. This verbatim account of these proceedings imprisonment on each count, the sentences to run consecutively. On appeal, the
contains no statement of any warnings given by the assistant district attorney. At conviction was affirmed by the Court of Appeals for the Ninth Circuit. 342 F.2d 684.
Vignera's trial on a charge of first degree robbery, the detective testified as to the
oral confession. The transcription of the statement taken was also introduced in
We reverse. On the facts of this case, we cannot find that Westover knowingly and
evidence. At the conclusion of the testimony, the trial judge charged the jury in part
intelligently waived his right to remain silent and his right to consult with counsel
as follows:
prior to the time he made the statement. [Footnote 69] At the time the FBI agents
began questioning Westover, he had been in custody for over 14 hours, and had
"The law doesn't say that the confession is void or invalidated because the police been interrogated at length during that period. The FBI interrogation began
officer didn't advise the defendant as to his rights. Did you hear what immediately upon the conclusion of the interrogation by Kansas City police, and was
conducted in local police headquarters. Although the two law enforcement
authorities are legally distinct, and the crimes for which they interrogated Westover
I said? I am telling you what the law of the State of New York is."
were different, the impact on him was that of a continuous period of questioning.
There is no evidence of any warning given prior to the FBI interrogation, nor is there
Vignera was found guilty of first degree robbery. He was subsequently adjudged a any evidence of an articulated waiver of rights after the FBI commenced its
third-felony offender and sentenced to 30 to 60 years' imprisonment. [Footnote 68] interrogation. The record simply shows that the defendant did, in fact, confess a
The conviction was affirmed without opinion by the Appellate Division, Second short time after being turned over to the FBI following interrogation by local police.
Department, 21 App.Div.2d 752, 252 N.Y.S.2d 19, and by the Court of Appeals, also Despite the fact that the FBI agents gave warnings at the outset of their interview,
without opinion, 15 N.Y.2d 970, 207 N.E.2d 527, 259 N.Y.S.2d 857, remittitur from Westover's point of view, the warnings came at the end of the interrogation
amended, 16 N.Y.2d 614, 209 N.E.2d 110, 261 N.Y. .2d 65. In argument to the Court process. In these circumstances, an intelligent waiver of constitutional rights cannot
of Appeals, the State contended that Vignera had no constitutional right to be be assumed.
advised of his right to counsel or his privilege against self-incrimination.

78
We do not suggest that law enforcement authorities are precluded from questioning to no other construction than that he was compelled by persistent interrogation to
any individual who has been held for a period of time by other authorities and forgo his Fifth Amendment privilege.
interrogated by them without appropriate warnings. A different case would be
presented if an accused were taken into custody by the second authority, removed
Therefore, in accordance with the foregoing, the judgments of the Supreme Court Of
both in time and place from his original surroundings, and then adequately advised
Arizona in No. 759, of the New York Court of Appeals in No. 760, and of the Court of
of his rights and given an opportunity to exercise them. But here, the FBI
Appeals for the Ninth Circuit in No. 761, are reversed. The judgment of the Supreme
interrogation was conducted immediately following the state interrogation in the
Court of California in No. 584 is affirmed.
same police station -- in the same compelling surroundings. Thus, in obtaining a
confession from Westover the federal authorities were the beneficiaries of the
pressure applied by the local in-custody interrogation. In these circumstances, the It is so ordered.
giving of warnings alone was not sufficient to protect the privilege.
* Together with No. 760, Vignera v. New York, on certiorari to the Court of Appeals of
No. 584. California v. Stewart New York and No. 761, Westover v. United States, on certiorari to the United States
Court of Appeals for the Ninth Circuit, both argued February 28-March 1, 1966, and
No. 584, California v. Stewart, on certiorari to the Supreme Court of California,
In the course of investigating a series of purse-snatch robberies in which one of the
argued February 28-March 2, 1966.
victims had died of injuries inflicted by her assailant, respondent, Roy Allen Stewart,
was pointed out to Los Angeles police as the endorser of dividend checks taken in
one of the robberies. At about 7:15 p.m., January 31, 1963, police officers went to
Stewart's house and arrested him. One of the officers asked Stewart if they could
search the house, to which he replied, "Go ahead." The search turned up various
items taken from the five robbery victims. At the time of Stewart's arrest, police also
arrested Stewart's wife and three other persons who were visiting him. These four
were jailed along with Stewart, and were interrogated. Stewart was taken to the
University Station of the Los Angeles Police Department, where he was placed in a
cell. During the next five days, police interrogated Stewart on nine different
occasions. Except during the first interrogation session, when he was confronted
with an accusing witness, Stewart was isolated with his interrogators.

During the ninth interrogation session, Stewart admitted that he had robbed the
deceased and stated that he had not meant to hurt her. Police then brought Stewart
before a magistrate for the first time. Since there was no evidence to connect them
with any crime, the police then released the other four persons arrested with him.

Nothing in the record specifically indicates whether Stewart was or was not advised
of his right to remain silent or his right to counsel. In a number of instances,
however, the interrogating officers were asked to recount everything that was said
during the interrogations. None indicated that Stewart was ever advised of his rights.

Stewart was charged with kidnapping to commit robbery, rape, and murder. At his
trial, transcripts of the first interrogation and the confession at the last interrogation
were introduced in evidence. The jury found Stewart guilty of robbery and first
degree murder, and fixed the penalty as death. On appeal, the Supreme Court of
California reversed. 62 Cal.2d 571, 400 P.2d 97, 43 Cal.Rptr. 201. It held that, under
this Court's decision in Escobedo, Stewart should have been advised of his right to
remain silent and of his right to counsel, and that it would not presume in the face of
a silent record that the police advised Stewart of his rights. [Footnote 70]

We affirm. [Footnote 71] In dealing with custodial interrogation, we will not presume
that a defendant has been effectively apprised of his rights and that his privilege
against self-incrimination has been adequately safeguarded on a record that does
not show that any warnings have been given or that any effective alternative has
been employed. Nor can a knowing and intelligent waiver of these rights be
assumed on a silent record. Furthermore, Stewart's steadfast denial of the alleged
offenses through eight of the nine interrogations over a period of five days is subject

79
GALMAN V. PAMARAN, 138 SCRA 274 private respondents were charged as accessories, along with several principals, and
one accomplice.
(1985) Upon arraignment, all the accused, including the herein private ate Respondents
pleaded NOT GUILTY.
G.R. Nos. 71208-09 August 30, 1985
In the course of the joint trial of the two (2) aforementioned cases, the Prosecution
SATURNINA GALMAN AND REYNALDO GALMAN, petitioners,
represented by the Office of the petition TANODBAYAN, marked and thereafter
vs.
offered as part of its evidence, the individual testimonies of private respondents
THE HONORABLE PRESIDING JUSTICE MANUEL PAMARAN AND ASSOCIATE
before the Agrava Board. 6 Private respondents, through their respective counsel
JUSTICES AUGUSTO AMORES AND BIENVENIDO VERA CRUZ OF THE
objected to the admission of said exhibits. Private respondent Gen. Ver filed a formal
SANDIGANBAYAN, THE HONORABLE BERNARDO FERNANDEZ, TANODBAYAN,
"Motion to Exclude Testimonies of Gen. Fabian C. Ver before the Fact Finding Board
GENERAL FABIAN C. VER, MAJOR GENERAL PROSPERO OLIVAS, SGT. PABLO
as Evidence against him in the above-entitled cases" 7 contending that its admission
MARTINEZ, SGT. TOMAS FERNANDEZ, SGT. LEONARDO MOJICA SGT. PEPITO
will be in derogation of his constitutional right against self-incrimination and violative
TORIO, SGT. PROSPERO BONA AND AlC ANICETO ACUPIDO, respondents.
of the immunity granted by P.D. 1886. He prayed that his aforesaid testimony be
G.R. Nos. 71212-13 August 30, 1985 rejected as evidence for the prosecution. Major Gen. Olivas and the rest of the other
private respondents likewise filed separate motions to exclude their respective
PEOPLE OF THE PHILIPPINES, represented by the TANODBAYAN individual testimonies invoking the same ground. 8Petitioner TANODBAYAN opposed
(OMBUDSMAN), petitioner, said motions contending that the immunity relied upon by the private respondents in
vs. support of their motions to exclude their respective testimonies, was not available to
THE SANDIGANBAYAN, GENERAL FABIAN C. VER, MAJOR GEN. PROSPERO them because of their failure to invoke their right against self-incrimination before
OLIVAS, SGT. PABLO MARTINEZ, SGT. TOMAS FERNANDEZ, SGT. LEONARDO the ad hoc Fact Finding Board. 9 Respondent SANDIGANBAYAN ordered the
MOJICA, SGT. PEPITO TORIO, SGT. PROSPERO BONA AND AIC ANICETO TANODBAYAN and the private respondents to submit their respective memorandum
ACUPIDO, respondents. on the issue after which said motions will be considered submitted for resolution. 10
CUEVAS, JR., J.: On May 30, 1985, petitioner having no further witnesses to present and having been
On August 21, 1983, a crime unparalleled in repercussions and ramifications was required to make its offer of evidence in writing, respondent SANDIGANBAYAN,
committed inside the premises of the Manila International Airport (MIA) in Pasay City. without the pending motions for exclusion being resolved, issued a Resolution
Former Senator Benigno S. Aquino, Jr., an opposition stalwart who was returning to directing that by agreement of the parties, the pending motions for exclusion and
the country after a long-sojourn abroad, was gunned down to death. The the opposition thereto, together with the memorandum in support thereof, as well as
assassination rippled shock-waves throughout the entire country which reverberated the legal issues and arguments, raised therein are to be considered jointly in the
beyond the territorial confines of this Republic. The after-shocks stunned the nation Court's Resolution on the prosecution's formal offer of exhibits and other
even more as this ramified to all aspects of Philippine political, economic and social documentary evidences. 11 On June 3, 1985, the prosecution made a written "Formal
life. Offer of Evidence" which includes, among others, the testimonies of private
respondents and other evidences produced by them before the Board, all of which
To determine the facts and circumstances surrounding the killing and to allow a free, have been previously marked in the course of the trial. 12
unlimited and exhaustive investigation of all aspects of the tragedy, 1 P.D. 1886 was
promulgated creating an ad hoc Fact Finding Board which later became more All the private respondents objected to the prosecution's formal offer of evidence on
popularly known as the Agrava Board. 2 Pursuant to the powers vested in it by P.D. the same ground relied upon by them in their respective motion for exclusion.
1886, the Board conducted public hearings wherein various witnesses appeared and On June 13, 1985, respondent SANDIGANBAYAN issued a Resolution, now assailed in
testified and/or produced documentary and other evidence either in obedience to a these two (2) petitions, admitting all the evidences offered by the prosecution
subpoena or in response to an invitation issued by the Board Among the witnesses except the testimonies and/or other evidence produced by the private respondents
who appeared, testified and produced evidence before the Board were the herein in view of the immunity granted by P.D. 1886. 13
private respondents General Fabian C. Ver, Major General Prospero Olivas, 3 Sgt.
Pablo Martinez, Sgt. Tomas Fernandez, Sgt. Leonardo Mojica, Sgt. Pepito Torio, Sgt. Petitioners' motion for the reconsideration of the said Resolution having been
Prospero Bona and AIC Aniceto Acupido. 4 DENIED, they now come before Us by way of certiorari 14 praying for the amendment
and/or setting aside of the challenged Resolution on the ground that it was issued
UPON termination of the investigation, two (2) reports were submitted to His without jurisdiction and/or with grave abuse of discretion amounting to lack of
Excellency, President Ferdinand E. Marcos. One, by its Chairman, the Hon. Justice jurisdiction. Private prosecutor below, as counsel for the mother of deceased
Corazon Juliano Agrava; and another one, jointly authored by the other members of Rolando Galman, also filed a separate petition for certiorari 15 on the same ground.
the Board namely: Hon. Luciano Salazar, Hon. Amado Dizon, Hon. Dante Santos Having arisen from the same factual beginnings and raising practically Identical
and Hon. Ernesto Herrera. 'the reports were thereafter referred and turned over to issues, the two (2) petitioners were consolidated and will therefore be jointly dealt
the TANODBAYAN for appropriate action. After conducting the necessary preliminary with and resolved in this Decision.
investigation, the TANODBAYAN 5 filed with the SANDIGANBAYAN two (2)
Informations for MURDER-one for the killing of Sen. Benigno S. Aquino which was The crux of the instant controversy is the admissibility in evidence of the testimonies
docketed as Criminal Case No. 10010 and another, criminal Case No. 10011, for the given by the eight (8) private respondents who did not invoke their rights against
killing of Rolando Galman, who was found dead on the airport tarmac not far from self-incrimination before the Agrava Board.
the prostrate body of Sen. Aquino on that same fateful day. In both criminal cases,

80
It is the submission of the prosecution, now represented by the petitioner Among this class of witnesses were the herein private respondents, suspects in the
TANODBAYAN, that said testimonies are admissible against the private respondents, said assassination, all of whom except Generals Ver and Olivas, were detained
respectively, because of the latter's failure to invoke before the Agrava Board the (under technical arrest) at the time they were summoned and gave their testimonies
immunity granted by P.D. 1886. Since private respondents did not invoke said before the Agrava Board. This notwithstanding, Presidential Decree No. 1886 denied
privilege, the immunity did not attach. Petitioners went further by contending that them the right to remain silent. They were compelled to testify or be witnesses
such failure to claim said constitutional privilege amounts to a waiver thereof. 16 The against themselves. Section 5 of P.D. 1886 leave them no choice. They have to take
private respondents, on the other hand, claim that notwithstanding failure to set up the witness stand, testify or produce evidence, under pain of contempt if they failed
the privilege against self- incrimination before the Agrava Board, said evidences or refused to do so. 21 The jeopardy of being placed behind prison bars even before
cannot be used against them as mandated by Section 5 of the said P.D. 1886. They conviction dangled before their very eyes. Similarly, they cannot invoke the right not
contend that without the immunity provided for by the second clause of Section 5, to be a witness against themselves, both of which are sacrosantly enshrined and
P.D. 1886, the legal compulsion imposed by the first clause of the same Section protected by our fundamental law. 21-a Both these constitutional rights (to remain
would suffer from constitutional infirmity for being violative of the witness' right silent and not to be compelled to be a witness against himself) were right away
against self- incrimination. 17 Thus, the protagonists are locked in horns on the effect totally foreclosed by P.D. 1886. And yet when they so testified and produced
and legal significance of failure to set up the privilege against self-incrimination. evidence as ordered, they were not immune from prosecution by reason of the
testimony given by them.
The question presented before Us is a novel one. Heretofore, this Court has not been
previously called upon to rule on issues involving immunity statutes. The relative Of course, it may be argued is not the right to remain silent available only to a
novelty of the question coupled with the extraordinary circumstance that had person undergoing custodial interrogation? We find no categorical statement in the
precipitated the same did nothing to ease the burden of laying down the criteria constitutional provision on the matter which reads:
upon which this Court will henceforth build future jurisprudence on a heretofore
unexplored area of judicial inquiry. In carrying out this monumental task, however, ... Any person under investigation for the commission of an offense
We shall be guided, as always, by the constitution and existing laws. shall have the right to remain and to counsel, and to be informed
of such right. ... 22 (Emphasis supplied)
The Agrava Board, 18 came into existence in response to a popular public clamor that
an impartial and independent body, instead of any ordinary police agency, be Since the effectivity of the 1973 Constitution, we now have a mass of
charged with the task of conducting the investigation. The then early distortions and jurisprudence 23 on this specific portion of the subject provision. In all these cases, it
exaggerations, both in foreign and local media, relative to the probable motive has been categorically declared that a person detained for the commission of an
behind the assassination and the person or persons responsible for or involved in the offense undergoing investigation has a right to be informed of his right to remain
assassination hastened its creation and heavily contributed to its early formation. 19 silent, to counsel, and to an admonition that any and all statements to be given by
him may be used against him. Significantly however, there has been no
Although referred to and designated as a mere Fact Finding Board, the Board is in pronouncement in any of these cases nor in any other that a person similarly
truth and in fact, and to all legal intents and purposes, an entity charged, not only undergoing investigation for the commission of an offense, if not detained, is not
with the function of determining the facts and circumstances surrounding the killing, entitled to the constitutional admonition mandated by said Section 20, Art. IV of the
but more importantly, the determination of the person or persons criminally Bill of Rights.
responsible therefor so that they may be brought before the bar of justice. For
indeed, what good will it be to the entire nation and the more than 50 million The fact that the framers of our Constitution did not choose to use the term
Filipinos to know the facts and circumstances of the killing if the culprit or culprits "custodial" by having it inserted between the words "under" and investigation", as in
will nevertheless not be dealt with criminally? This purpose is implicit from Section fact the sentence opens with the phrase "any person " goes to prove that they did
12 of the said Presidential Decree, the pertinent portion of which provides not adopt in toto the entire fabric of the Miranda doctrine. 24 Neither are we
impressed by petitioners' contention that the use of the word "confession" in the last
SECTION 12. The findings of the Board shall be made public. sentence of said Section 20, Article 4 connotes the Idea that it applies only to police
Should the findings warrant the prosecution of any person, the investigation, for although the word "confession" is used, the protection covers not
Board may initiate the filing of proper complaint with the only "confessions" but also "admissions" made in violation of this section. They are
appropriate got government agency. ... (Emphasis supplied) inadmissible against the source of the confession or admission and against third
person. 25
The investigation therefor is also geared, as any other similar investigation of its
sort, to the ascertainment and/or determination of the culprit or culprits, their It is true a person in custody undergoing investigation labors under a more
consequent prosecution and ultimately, their conviction. And as safeguard, the P.D. formidable ordeal and graver trying conditions than one who is at liberty while being
guarantees "any person called to testify before the Board the right to counsel at any investigated. But the common denominator in both which is sought to be avoided is
stage of the proceedings." 20 Considering the foregoing environmental settings, it the evil of extorting from the very mouth of the person undergoing interrogation for
cannot be denied that in the course of receiving evidence, persons summoned to the commission of an offense, the very evidence with which to prosecute and
testify will include not merely plain witnesses but also those suspected as authors thereafter convict him. This is the lamentable situation we have at hand.
and co-participants in the tragic killing. And when suspects are summoned and
called to testify and/or produce evidence, the situation is one where the person All the private respondents, except Generals Ver and Olivas, are members of the
testifying or producing evidence is undergoing investigation for the commission of military contingent that escorted Sen. Aquino while disembarking from the plane
an offense and not merely in order to shed light on the facts and surrounding that brought him home to Manila on that fateful day. Being at the scene of the crime
circumstances of the assassination, but more importantly, to determine the as such, they were among the first line of suspects in the subject assassination.
character and extent of his participation therein. General Ver on the other hand, being the highest military authority of his co-
petitioners labored under the same suspicion and so with General Olivas, the first

81
designated investigator of the tragedy, but whom others suspected, felt and office the act of responding to interrogation was not voluntary and
believed to have bungled the case. The papers, especially the foreign media, and was not an effective waiver of the privilege against self-
rumors from uglywagging tongues, all point to them as having, in one way or incrimination.
another participated or have something to do, in the alleged conspiracy that brought
about the assassination. Could there still be any doubt then that their being asked to To buttress their precarious stand and breathe life into a seemingly hopeless cause,
testify, was to determine whether they were really conspirators and if so, the extent petitioners and amicus curiae (Ex-Senator Ambrosio Padilla) assert that the "right
of their participation in the said conspiracy? It is too taxing upon one's credulity to not to be compelled to be a witness against himself" applies only in favor of an
believe that private respondents' being called to the witness stand was merely to accused in a criminal case. Hence, it may not be invoked by any of the herein
elicit from them facts and circumstances surrounding the tragedy, which was private respondents before the Agrava Board. The Cabal vs. Kapunan 28 doctrine
already so abundantly supplied by other ordinary witnesses who had testified earlier. militates very heavily against this theory. Said case is not a criminal case as its title
In fact, the records show that Generals Ver and Olivas were among the last very clearly indicates. It is not People vs. Cabal nor a prosecution for a criminal
witnesses called by the Agrava Board. The subject matter dealt with and the line of offense. And yet, when Cabal refused to take the stand, to be sworn and to testify
questioning as shown by the transcript of their testimonies before the Agrava Board, upon being called as a witness for complainant Col. Maristela in a forfeiture of
indubitably evinced purposes other than merely eliciting and determining the so- illegally acquired assets, this Court sustained Cabal's plea that for him to be
called surrounding facts and circumstances of the assassination. In the light of the compelled to testify will be in violation of his right against self- incrimination. We did
examination reflected by the record, it is not far-fetched to conclude that they were not therein state that since he is not an accused and the case is not a criminal case,
called to the stand to determine their probable involvement in the crime being Cabal cannot refuse to take the witness stand and testify, and that he can invoke his
investigated. Yet they have not been informed or at the very least even warned right against self-incrimination only when a question which tends to elicit an answer
while so testifying, even at that particular stage of their testimonies, of their right to that will incriminate him is profounded to him. Clearly then, it is not the character of
remain silent and that any statement given by them may be used against them. If the suit involved but the nature of the proceedings that controls. The privilege has
the investigation was conducted, say by the PC, NBI or by other police agency, all consistently been held to extend to all proceedings sanctioned by law and to all
the herein private respondents could not have been compelled to give any cases in which punishment is sought to be visited upon a witness, whether a party or
statement whether incriminatory or exculpatory. Not only that. They are also entitled not. 29 If in a mere forfeiture case where only property rights were involved, "the
to be admonished of their constitutional right to remain silent, to counsel, and be right not to be compelled to be a witness against himself" is secured in favor of the
informed that any and all statements given by them may be used against them. Did defendant, then with more reason it cannot be denied to a person facing
they lose their aforesaid constitutional rights simply because the investigation was investigation before a Fact Finding Board where his life and liberty, by reason of the
by the Agrava Board and not by any police investigator, officer or agency? True, they statements to be given by him, hang on the balance. Further enlightenment on the
continued testifying. May that be construed as a waiver of their rights to remain subject can be found in the historical background of this constitutional provision
silent and not to be compelled to be a witness against themselves? The answer is against self- incrimination. The privilege against self- incrimination is guaranteed in
yes, if they have the option to do so. But in the light of the first portion of Section 5 the Fifth Amendment to the Federal Constitution. In the Philippines, the same
of P.D. 1886 and the awesome contempt power of the Board to punish any refusal to principle obtains as a direct result of American influence. At first, the provision in our
testify or produce evidence, We are not persuaded that when they testified, they organic laws were similar to the Constitution of the United States and was as follows:
voluntarily waived their constitutional rights not to be compelled to be a witness That no person shall be ... compelled in a criminal case to be a
against themselves much less their right to remain silent. witness against himself. 30
Compulsion as it is understood here does not necessarily connote As now worded, Section 20 of Article IV reads:
the use of violence; it may be the product of unintentional
statements. Pressure which operates to overbear his will, disable No person shall be compelled to be a witness against himself.
him from making a free and rational choice, or impair his capacity
The deletion of the phrase "in a criminal case" connotes no other import except to
for rational judgment would in our opinion be sufficient. So is
make said provision also applicable to cases other than criminal. Decidedly then, the
moral coercion 'tending to force testimony from the unwilling lips
right "not to be compelled to testify against himself" applies to the herein private
of the defendant. 26
respondents notwithstanding that the proceedings before the Agrava Board is not, in
Similarly, in the case of Louis J. Lefkowitz v. Russel 27 Turley" citing Garrity vs. New its strictest sense, a criminal case
Jersey" where certain police officers summoned to an inquiry being conducted by the
No doubt, the private respondents were not merely denied the afore-discussed
Attorney General involving the fixing of traffic tickets were asked questions following
sacred constitutional rights, but also the right to "due process" which is fundamental
a warning that if they did not answer they would be removed from office and that
fairness. 31 Quoting the highly-respected eminent constitutionalist that once graced
anything they said might be used against them in any criminal proceeding, and the
this Court, the former Chief Justice Enrique M. Fernando, due process
questions were answered, the answers given cannot over their objection be later
used in their prosecutions for conspiracy. The United States Supreme Court went ... is responsiveness to the supremacy of reason, obedience to the
further in holding that: dictates of justice. Negatively put, arbitrariness is ruled out and
unfairness avoided. To satisfy the due process requirement, official
the protection of the individuals under the Fourteenth Amendment
action, to paraphrase Cardozo, must not outrun the bounds of
against coerced statements prohibits use in subsequent
reason and result in sheer oppression. Due process is thus hostile
proceedings of statements obtained under threat or removal from
to any official action marred by lack of reasonableness. Correctly,
office, and that it extends to all, whether they are policemen or
it has been Identified as freedom from arbitrariness. It is the
other members of the body politic. 385 US at 500, 17 L Ed. 562.
embodiment of the sporting Idea of fair play(Frankfurter, Mr.
The Court also held that in the context of threats of removal from
Justice Holmes and the Supreme Court, 1983, pp. 32-33). It exacts

82
fealty "to those strivings for justice and judges the act of may be used against them. This, they were denied, under the pretense that they are
officialdom of whatever branch "in the light of reason drawn from not entitled to it and that the Board has no obligation to so inform them.
considerations of fairness that reflect (democratic) traditions of
legal and political thought."(Frankfurter, Hannah v. Larche 1960, It is for this reason that we cannot subscribe to the view adopted and urged upon Us
363 US 20, at 487). It is not a narrow or '"echnical conception with by the petitioners that the right against self-incrimination must be invoked before
fixed content unrelated to time, place and the Board in order to prevent use of any given statement against the testifying
circumstances."(Cafeteria Workers v. McElroy 1961, 367 US witness in a subsequent criminal prosecution. A literal interpretation fashioned upon
1230) Decisions based on such a clause requiring a 'close and Us is repugnant to Article IV, Section 20 of the Constitution, which is the first test of
perceptive inquiry into fundamental principles of our society. admissibility. It reads:
(Bartkus vs. Illinois, 1959, 359 US 121). Questions of due process No person shall be compelled to be a witness against himself. Any
are not to be treated narrowly or pedantically in slavery to form or person under investigation for the commission of an offense shall
phrases. (Pearson v. McGraw, 1939, 308 US 313). have the right to remain silent and to counsel, and to be informed
Our review of the pleadings and their annexes, together with the oral arguments, of such right. No force, violence, threat, intimidation, or any other
manifestations and admissions of both counsel, failed to reveal adherence to and means which vitiates the free will shall be used against him. Any
compliance with due process. The manner in which the testimonies were taken from confession obtained in violation of this section shall be
private respondents fall short of the constitutional standards both under the DUE inadmissible in evidence. (Emphasis supplied)
PROCESS CLAUSE and under the EXCLUSIONARY RULE in Section 20, Article IV. In the The aforequoted provision renders inadmissible any confession obtained in violation
face of such grave constitutional infirmities, the individual testimonies of private thereof. As herein earlier discussed, this exclusionary rule applies not only to
respondents cannot be admitted against them in ally criminal proceeding. This is confessions but also to admissions, 33 whether made by a witness in any proceeding
true regardless of absence of claim of constitutional privilege or of the presence of a or by an accused in a criminal proceeding or any person under investigation for the
grant of immunity by law. Nevertheless, We shall rule on the effect of such absence commission of an offense. Any interpretation of a statute which will give it a
of claim to the availability to private respondents of the immunity provided for in meaning in conflict with the Constitution must be avoided. So much so that if two or
Section 5, P.D. 1886 which issue was squarely raised and extensively discussed in more constructions or interpretations could possibly be resorted to, then that one
the pleadings and oral arguments of the parties. which will avoid unconstitutionality must be adopted even though it may be
Immunity statutes may be generally classified into two: one, which grants "use necessary for this purpose to disregard the more usual and apparent import of the
immunity"; and the other, which grants what is known as "transactional immunity." language used. 34 To save the statute from a declaration of unconstitutionality it
The distinction between the two is as follows: "Use immunity" prohibits use of must be given a reasonable construction that will bring it within the fundamental
witness' compelled testimony and its fruits in any manner in connection with the law. 35 Apparent conflict between two clauses should be harmonized. 36
criminal prosecution of the witness. On the other hand, "transactional immunity" But a literal application of a requirement of a claim of the privilege against self-
grants immunity to the witness from prosecution for an offense to which his incrimination as a condition sine qua non to the grant of immunity presupposes that
compelled testimony relates." 32 Examining Presidential Decree 1886, more from a layman's point of view, he has the option to refuse to answer questions and
specifically Section 5 thereof, which reads: therefore, to make such claim. P.D. 1886, however, forecloses such option of refusal
SEC. 5. No person shall be excused from attending and testifying by imposing sanctions upon its exercise, thus:
or from producing books, records, correspondence, documents, or SEC. 4. The Board may hold any person in direct or indirect
other evidence in obedience to a subpoena issued by the Board on contempt, and impose appropriate penalties therefor. A person
the ground that his testimony or the evidence required of him may guilty of .... including ... refusal to be sworn or to answer as a
tend to incriminate him or subject him to penalty or forfeiture; but witness or to subscribe to an affidavit or deposition when lawfully
his testimony or any evidence produced by him shall not be used required to do so may be summarily adjudged in direct contempt
against him in connection with any transaction, matter or thing by the Board. ...
concerning which he is compelled, after having invoked his
privilege against self-incrimination, to testify or produce evidence, Such threat of punishment for making a claim of the privilege leaves the witness no
except that such individual so testifying shall not be exempt from choice but to answer and thereby forfeit the immunity purportedly granted by Sec.
prosecution and punishment for perjury committed in so testifying, 5. The absurdity of such application is apparent Sec. 5 requires a claim which it,
nor shall he be exempt from demotion or removal from office. however, forecloses under threat of contempt proceedings against anyone who
(Emphasis supplied) makes such claim. But the strong testimonial compulsion imposed by Section 5 of
P.D. 1886 viewed in the light of the sanctions provided in Section 4,infringes upon
it is beyond dispute that said law belongs to the first type of immunity statutes. It the witness' right against self-incrimination. As a rule, such infringement of the
grants merely immunity from use of any statement given before the Board, but not constitutional right renders inoperative the testimonial compulsion, meaning, the
immunity from prosecution by reason or on the basis thereof. Merely testifying witness cannot be compelled to answer UNLESS a co-extensive protection in the
and/or producing evidence do not render the witness immuned from prosecution form of IMMUNITY is offered. 37 Hence, under the oppressive compulsion of P.D. 1886,
notwithstanding his invocation of the right against self- incrimination. He is merely immunity must in fact be offered to the witness before he can be required to answer,
saved from the use against him of such statement and nothing more. Stated so as to safeguard his sacred constitutional right. But in this case, the compulsion
otherwise ... he still runs the risk of being prosecuted even if he sets up his right has already produced its desired results the private respondents had all testified
against self- incrimination. The dictates of fair play, which is the hallmark of due without offer of immunity. Their constitutional rights are therefore, in jeopardy. The
process, demands that private respondents should have been informed of their only way to cure the law of its unconstitutional effects is to construe it in the manner
rights to remain silent and warned that any and all statements to be given by them

83
as if IMMUNITY had in fact been offered. We hold, therefore, that in view of the
potent sanctions imposed on the refusal to testify or to answer questions under Sec.
4 of P.D. 1886, the testimonies compelled thereby are deemed immunized under
Section 5 of the same law. The applicability of the immunity granted by P.D. 1886
cannot be made to depend on a claim of the privilege against self-incrimination
which the same law practically strips away from the witness.
With the stand we take on the issue before Us, and considering the temper of the
times, we run the risk of being consigned to unpopularity. Conscious as we are of,
but undaunted by, the frightening consequences that hover before Us, we have
strictly adhered to the Constitution in upholding the rule of law finding solace in the
view very aptly articulated by that well-known civil libertarian and admired defender
of human rights of this Court, Mr. Justice Claudio Teehankee, in the case of People
vs. Manalang 38 and we quote:
I am completely conscious of the need for a balancing of the
interests of society with the rights and freedoms of the individuals.
I have advocated the balancing-of-interests rule in an situations
which call for an appraisal of the interplay of conflicting interests
of consequential dimensions. But I reject any proposition that
would blindly uphold the interests of society at the sacrifice of the
dignity of any human being. (Emphasis supplied)
Lest we be misunderstood, let it be known that we are not by this disposition passing
upon the guilt or innocence of the herein private respondents an issue which is
before the Sandiganbayan. We are merely resolving a question of law and the
pronouncement herein made applies to all similarly situated, irrespective of one's
rank and status in society.
IN VIEW OF THE FOREGOING CONSIDERATIONS and finding the instant petitions
without merit, same are DISMISSED. No pronouncement as to costs.
SO ORDERED.
Aquino, J., concurs (as certified by Makasiar, C.J.).
Abad Santos, J., is on leave.

84
PEOPLE V. DELA CRUZ, 92 PHIL. 900 (1953) the stake, breaking on the wheel, disemboweling, and the like (15 Am. Jur., supra,
Note 35 L.R.A. p. 561). Fine and imprisonment would not thus be within the
G.R. No. L-5790 April 17, 1953 prohibition.

THE PEOPLE OF THE PHILIPPINES, plaintiff-appellee, However, there are respectable authorities holding that the inhibition applies as well
vs. to punishments that although not cruel and unusual in nature, may be so severe as
PABLO DE LA CRUZ, defendant-appellant. to fall within the fundamental restriction. (15 Am. Jur., p. 178.) These authorities
explain, nevertheless, that to justify a court's declaration of conflict with the
BENGZON, J.: Constitution, the prison term must be so disproportionate to the offense committed
as to shock the moral sense of all reasonable men as to what is right and proper
Having retailed a can of milk at ten centavos more than the ceiling price, Pablo de la under the circumstances (lb.). And seldom has a sentence been declared to be cruel
Cruz was sentenced, after trial, in the court of first instance of Manila, to and unusual solely on account of its duration (15 Am. Jur., p. 179).
imprisonment for five years, and to pay a fine of five thousand pesos plus costs. He
was also barred from engaging in wholesale and retail business for five years. Because it expressly enjoins the imposition of "excessive fines" the Constitution
might have contemplated the latter school of thought assessing punishments not
In this appeal he argues that the trial judge erred: (a) in not holding that the charge only by their character but also by their duration or extent. And yet, having applied
was fabricated; (b) in imposing a punishment wholly disproportionate to the offense "excessive" to fines, and "cruel and unusual" to punishment did it not intend to
and therefore unconstitutional and (c) in not invalidating Republic Act No. 509 in so distinguish "excessive" from "cruel" or "unusual"? And then, it has been heretofore
far as it prescribed excessive penalties. the practice that when a court finds the penalty to be "clearly excessive" it enforces
The evidence shows that in the morning of October 14, 1950, Eduardo Bernardo, Jr. the law but makes a recommendation to the Chief Executive for clemency (Art. 5
went to the defendant's store in Sampaloc, Manila, and purchased from him a six- Revised Penal Code). Did the Constitutional Convention intend to stop that practice?
ounce tin of "Carnation" milk for thirty centavos. As the purchase had been made for Or is that article unconstitutional?
Ruperto Austria, who was not in good terms with Pablo de la Cruz the matter So far as the writer of this opinion has been able to ascertain, these questions have
reached the City Fiscal's office and resulted in this criminal prosecution, because not been definitely passed upon by this court, 1 although in U.S. vs. Borromeo, 23
Executive Order No. 331 (issued by authority of Republic Act No. 509) fixed 20 Phil., 279 it was said that the prohibition of the Philippine Bill on punishments refer
centavos as the maximum price for that kind of commodity. not only to the mode but to the extent thereto.
The record is now before us, and from a reading thereof, we find it difficult to accept For the purposes of this decision, we may assume, without actually holding, that too
appellants contention that the charge had no foundation in fact. The People's case long a prison term might clash with the Philippine Constitution. But that brings up
has been established beyond reasonable doubt. again two opposing theories. On one side we are told the prohibition applies to
And his argument based on the principles of entrapment, may not be upheld, legislation only, and not to the courts' decision imposing penalties within the limits
because he was selling to the public, i.e., to anybody who would come to his store to of the statute (15 Am. Jur., "Criminal Law" sec. 526). On the other, authorities are
buy his commodities, and no special circumstances are shown to support the claim not lacking to the effect that the fundamental prohibition likewise restricts the
that he was led or induced to commit the offense. judge's power and authority (State vs. Ross 55 Or. 450, 104 Pac. 596; State vs.
Whitaker, 48 La. Am. 527, 19 So. 457). (See also U.S. vs. Borromeo, 23 Phil., 279.)
However, appellant's extensive discussion of his two propositions about the penalty,
deserves serious consideration. In other words, and referring to the penalty provided in Republic Act No. 509, under
the first theory the section would violate the Constitution, if the penalty is excessive
Republic Act No. 509 provides in part as follows: under any and all circumstances, the minimum being entirely out of proportion to
the kind of offenses prescribed. If it is not, the imposition by the judge of a stiff
SEC. 12. Imprisonment for a period of not less two months nor more than penalty but within the limits of the section will not be deemed
twelve years or a fine of not less than two thousand pesos nor more than unconstitutional.2 The second theory would contrast the penalty imposed by the
ten thousand pesos, or both, shall be imposed upon any person who sells court with the gravity of the particular crime or misdemeanor, and if notable
any article, goods, or commodity in excess of the maximum selling price disparity results, it would apply the constitutional brake, even if the statute would,
fixed by the president; . . . . under other circumstances, be not extreme or oppressive.
In addition to the penalties prescribed above, the persons, corporations, Now therefore, if we adopt the first doctrine the present issue would be: Is
partnerships, or associations found guilty of any violation of this Act or of imprisonment for two months or fine of two thousand pesos too excessive for a
any rule or regulations issued by the president pursuant to this Act shall be merchant who sells goods at prices beyond the ceilings established in the Executive
barred from the wholesome and retail business for a period of five years for Order? Obviously a negative answer must be returned, because in overstepping the
a first offense, and shall be permanently barred for the second or price barriers he might derive, in some instances, profits amounting to thousands of
succeeding offenses. pesos. Therefore under that doctrine, the penalty imposed in this case would not be
The constitution directs that "Excessive fines shall not be imposed, nor cruel and susceptible of valid attack, it being within the statutory limits.
unusual punishment inflicted." The prohibition of cruel and unusual punishments is Under the second theory the inquiry should be: Is five years and five thousand
generally aimed at the form or character of the punishment rather than its severity pesos, cruel and unusual for a violation that merely netted a ten-centavo profit to
in respect of duration or amount, and apply to punishment which never existed in the accused? Many of us do not regard such punishment unusual and cruel,
America of which public sentiment has regarded as cruel or obsolete (15 Am. Jur., p. remembering the national policy against profiteering in the matter of foodstuffs
172), for instance those inflicted at the whipping post, or in the pillory, burning at affecting the people's health, the need of stopping speculation in such essentials

85
and of safeguarding public welfare in times of food scarcity or similar stress. In our
opinion the damage caused to the State is not measured exclusively by the gains
obtained by the accused, inasmuch as one violation would mean others, and the
consequential breakdown of the beneficial system of price controls.
Some of us however are deeply moved by the plight of this modest store-owner with
a family to support, who will serve in Muntinglupa a stretch of five years, for having
attempted to earn a few extra centavos.
Fortunately there is an area of compromise, skirting the constitutional issue, yet
executing substantial justice: We may decrease the penalty, exercising that
discretion vested in the courts by the same statutory enactment.
Wherefore, reducing the imprisonment to six months and the fine to two thousand
pesos, we hereby affirm the appealed decision in all other respects.
Paras, C.J., Feria, Pablo, Tuason, Montemayor, Reyes, Jugo and Bautista Angelo,
JJ., concur.

86
PEOPLE V. BORJA, 91 SCRA 340 (1979) the members: he announced that their backpay was forthcoming at the rate of
P36,000.00 for a ranking officer, and a lesser amount for those of lower rank.
G.R. No. L-22947 July 12, 1979 When the meeting ended, Borja called an exclusive conference among selected
PEOPLE OF THE PHILIPPINES, plaintiff-appellee, officers and members, including the other accused Rufino Pavia, Inocencio Demen,
vs. Pedro Fustigo, Felipe Benavides, Dominador de los Santos, Alejo Balimbing, and Tito
PEDRO BORJA, PEDRO FUSTIGO, INOCENCIO DEMEN , RUFINO PAVIA, FELIPE Oljina (The last two, now deceased, are referred to in the title of this case as John
BENAVIDES, DOMINADOR DE LOS SANTOS, JOHN DOE, and RICHARD Doe and Richard Doe). At the secret meeting, Balimbing proposed to Borja that they
DOE, defendants-appellants. conduct a raid the following morning at the Hacienda San Miguel, located at San
Miguel Island, across the bay from Tabaco, Albay. That same evening, Demen
ABAD SANTOS, J.: cleaned a .45 caliber pistol. The conference over, all eight men slept in the bahay-
pulungan.
This is an appeal from the consolidated decision of September 8, 1960, by the Court
of First Instance of Albay, in Criminal Case No. 2578 for murder, and Criminal Case When they woke up the next day, December 19, 1958, the eight men held an early-
No. 2590 for frustrated murder, both bearing the Identical titles, PP. vs. Pedro morning conference. Balimbing aired to Borja his grievances against Santiago
Borja, Pedro Fustigo, Inocencio Demen, Rufino Pavia, Felipe Benavides, Dominador GAncayco, Jr. the manager of the hacienda. Balimbing charged that it was Gancayco
de los Santos, John Doe and Richard Doe. who killed Balimbing's cousin at Rawis, Tabaco, Albay. Balimbing further complained
that it was Gancayco who ordered that the camote plantations of the hacienda
The decision convicted the accused, as follows: squatters who were related to Balimbing should be bulldozed. It appears that
In Criminal Case No. 2590, the Court, fully convinced that Pedro Borja, Pedro Fustigo, Balimbing was only rehashing what he had already related to Borja several months
Inocencio Demen, Rufino Pavia, Felipe Benavides and Dominador de los Santos, are before, at the AFAG regional headquarters at Pili, Camarines Sur.
guilty beyond the peradventure of reasonable doubt of the crime of frustrated That same morning, the eight men left by bus for Tabaco, via Legaspi City. They were
murder, as principals, hereby sentences each of them to undergo imprisonment armed as follows: Borja had two pistols tucked in a shoulder holster; Pavia had a .45
ranging from six (6) years, one (1) month, and eleven (11) days of prision mayor, as caliber pistol; Balimbing had a hunting knife with a scabbard; and Fustigo had a
the minimum, to fourteen (14) years, ten (10) months, and twenty-one (21) days pistol. They were attired as follows: Borja wore a khaki suit, black jacket, buri hat,
ofreclusion temporal, as the maximum; to suffer inherent accessory penalties; to and sunglasses; Demen wore a khaki suit; Fustigo wore a blue shirt and khaki pants;
indemnify the offended party, Salustiano Isorena, in the sum of P5,000.00, as moral de los Santos wore a red T-shirt, brown jacket, and maong pants; Benavides wore
and exemplary damages, severally and jointly, but not to undergo subsidiary denim pants and printed polo shirt; Pavia wore white pants; and Oljina wore a brown
imprisonment in case of insolvency, by reason of the nature of the penalty imposed; polo shirt and khaki pants.
and to pay the costs of this proceeding on equal basis.
In Tabaco, the group went to Barrio San Jose, where they ate breakfast at the house
In Criminal Case No. 2578, the Court after having been convinced beyond the realm of de los Santos' brother. Upon Borja's instruction, Balimbing hired a motorboat
of reasonable doubt of the guilt of Pedro Borja, Pedro Fustigo, Inocencio Demen, operated by Mariano Burac, who observed at the trial that the eight men acted
Rufino Pavia, Felipe Benavides and Dominador de los Santos, of the crime of murder, suspiciously and conferred sotto voce among themselves. They crossed the bay, and
as principals, deeply hurting as it is, hereby sentences each of them to the the group disembarked at the hacienda. At a seashore conference, they agreed to
maximum penalty of death; to suffer inherent accessory penalties; to indemnify the pose as members of the Philippine Constabulary, ostensibly on a mission to inspect
offended parties, Mercedes Chuidian Vda. de Gancayco and her children in the sum the firearms of the hacienda. At this time, Borja started to wear a pair of white
of P6,000.00 for the death of Santiago Gancayco, Jr., as a matter of law and practice, gloves. They proceeded on their way, and Balimbing tried to open the bamboo gate
and another amount in the sum of P30,000.00 as moral and exemplary damages, of the fence surrounding the manager's house. He was accosted by Emilio Lanon, a
both severally and jointly, but not to undergo subsidiary imprisonment in case of security guard and barrio lieutenant of the hacienda, who was later one of the
insolvency, by reason of the nature of the penalty imposed; and to pay the costs of principal eyewitnesses for the prosecution. Balimbing and Lanon knew each other.
this proceeding on equal basis. Balimbing introduced his companions as PC soldiers, Identifying Borja as a major and
In the decision, the trial court pursuant to the Revised Penal Code, Article 5 Pavia as a sergeant.
recommended to the President, through the Secretary of Justice, with respect to the On Lanon's advice, Balimbing proceeded to the office and emerged with Salustiano
accused Dominador de los Santos, "that executive clemency be extended to him, or Isorena, the hacienda overseer. Isorena told Lanon to inform Gancayco about the
that at least his death penalty be minimized or commuted to life imprisonment." The presence of the visitors. Then, on Pavia's advice, he and Isorena went to the house,
court so recommended because "the testimony of this accused had contributed in a where Isorena informed Mrs. Gancayco about the visitors. On Mrs. Gancayco's
large measure to the Court in its pursuit of truth and justice in these cases." It suggestion, the group went to the office to await for Gancayco, who arrived later.
should be stated here that by resolution of January 24, 1966, the Court noted the
contents of de los Santos' motion withdrawing his appeal in this case. Gancayco shook hands with Borja and Pavia. He offered Borja a pack of cigarettes,
from which took one stick. Informed of the group's alleged mission, Gancayco
It appears that on December 18, 1958, the Anderson Fil-American Guerrillas (AFAG) instructed Isorena to present the license of the firearms. But Isorena failed to find
held a general meeting at thebahay-pulungan of the religious sect known as the license in the office, so Gancayco went to the house to look for it there. While in
Watawat Ng Lahi at Barrio Buragwis, Legaspi City. The locale of the meeting was so the house, he instructed his wife to prepare a meal for the guests. Gancayco
chosen because many AFAG members are also Watawat members. One of the returned to the office, gave the license to Isorena, and then left. Isorena presented
accused, Pedro Borja, presided over the meeting, which was attended by more than the license to Borja, who remarked that the license listed only five firearms, as
a hundred members. Borja, who has the rank of a full colonel, is the AFAG head for against reports received at the headquarters in Albay that there were ten firearms in
the entire Bicol region, which is said to have 36,000 members. He had good news for

87
the hacienda. Borja then ordered Isorena to produce the firearms for inspection; interjected that it was unbelievable for Gancayco, as manager of the hacienda, not
Isorena, in turn, instructed Lanon to get the firearms. Lanon went to the house and to have such a pistol. But Gancayco insisted that there was none.
obtained a carbine, the magazine of which he removed; as well as shotgun.
While this exchange was taking place, Gancayco, who was unarmed, stood with his
On his way back to the office, he met de los Santos and Oljina, who took the firearms back towards the foot of the stairs. He was surrounded by Balimbing who held a
and the magazine from him. Isorena again instructed Lanon to get the other hunting knife; Pavia who held a .45 caliber pistol concealed beneath his jacket;
firearms. In compliance, Lanon sent Jaime Rawit to get the grease-gun from Lanon's Demen, who held the grease-gun with his arms down and the muzzle of the gun
house, while Lanon himself went to Gancayco's house to get another grease-gun pointed slightly upward; and Oljina who held a shotgun. The four men were about
which had no magazine. Rawit and Lanon met at Gancayco's house and from there, two meters away from Gancayco. Up in the balcony, de los Santos stood guard with
while Lanon was carrying the two grease-guns, he met Demen and Benavides, who a carbine in ready position, standing slightly back of Isorena. Gancayco and the four
took them away. men were about four meters away from Isorena and de los Santos.
The group had now succeeded in obtaining possession of four of the five firearms, Gancayco had insisted that he had no .45 caliber pistol. At this fateful moment,
and they were not able to get the one remaining firearm only because it was with an Pavia suddenly jerked his right hand upward and fired point-blank with his .45 caliber
hacienda employee guarding cattle some kilometers away. An atmosphere of pistol at Gancayco. Hit, Gancayco stooped to holed the pit of his stomach and
menace descended on the hacienda as the men loaded the firearms and took a cried, "Aray ko po." Instinctively, he stepped backward and was turning around when
hostile stance. An apprehensive Isorena asked the equally perturbed Gancayco for Demen, fired at him with a grease-gun. He ran away in a crouching position towards
instructions, but was only cautioned to remain calm and deal courteously with the the citrus plantation, while Demen, continued firing at him. Then Demen, aimed
group, as Gancayco had already sent someone to Tabaco to request police rapid fire at Isorena, who fell flat on the balcony floor. Demen's line of fire
assistance. accidentaly caught de los Santos in the forehead.
During this time, Balimbing was investigating Lanon inside the bodega. Lanon was At the outburst of gunfire, Borja rushed out of the bodega. He saw Gancayco running
surrounded by Balimbing with a hunting knife, Pavia with a .45 caliber pistol, and towards the citrus plantation and he shouted: "Habulin! Habulin!" The men carrying
Benavides with a grease-gun. In the course of his interrogation, Balimbing told Lanon their firearms gave chase; they were led by Demen, and included de los Santos.
that they were going to kill Gancayco, Isorena, Lanon and one Pablo Balimbing, chief They ran along the road towards a cluster of houses in the northern part of the
herder of the hacienda, because these employees were boothlickers to the hacienda, but they stopped in front of the house of Estrella Cortezano. Balimbing
Gancaycos. Then Balimbing pushed Lanon out of the bodega. asked Cortezano whether she saw where Gancayco went, but he got a negative
answer. As the men ran in pursuit, Gancayco traversed the citrus plantation and
Gancayco and Isorena were talking at the balcony of the house. They were reached the cluster of, houses. Although he was bleeding profusely, he managed to
approached by Demen, who had a grease-gun, and Oljina who had a shotgun. De los cross the road and to open the gate by removing one of the bamboo railings. He
Santos with a carbine posted himself under the citrus tree near the water tank in then turned towards the trail that wound through the abaca plantation at the left
front of the house. Gancayco asked to see a search warrant from Demen, and Oljina side of the road.
who referred him to Borja. So Gancayco and Isorena descended and inquired for the
warrant from Borja, who was standing in front of the house. Instead of replying, Borja While the pursuers tracked their prey, Lanon left the bodega by the back door and
talked to Pavia, who was near the guardhouse. Borja then called Balimbing and, went to Gancayco's house. Mrs. Gancayco told him to close all the windows and take
referring to Gancayco and Isorena, ordered him to "fix them up" since it was already the children downstairs, and he complied. Then he left the house and looked for
getting on towards noon. Borja herded the other men of the hacienda inside the Gancayco whom he found in the abaca plantation, climbing a small hill towards
bodega, where they were guarded by Benavides who was armed with a grease-gun, another cluster of houses at the hilltop. Lanon found Gancayco bleeding profusely in
and Fustigo, with a pistol. Borja faced the hacienda personnel, holding in readiness the breast.
his two pistols. Lanon stood in front of him, facing out of the bodega.
Gancayco, his wife, and children, Isorena, and some other personnel of the
In the meantime, Balimbing, Pavia, Demen, de los Santos, and Oljina escorted hacienda, were able to rendezvous at the seashore. They got into a waiting
Gancayco and Isorena towards the office. Isorena went up the stairs and turned motorboat and sped away towards Tabaco, in the hope of taking the wounded men
around when he reached the top; behind him, de los Santos held a carbine in a ready to the hospital there. But this was not to be; for en route, Gancayco died in the arms
position. Benavides asked Gancayco for the magazine of the grease-gun he was of his wife. His body was brought to Manila, where it was examined and autopsied by
holding. When Gancayco answered that it had no magazine, Benavides accused him the National Bureau of Investigation. Although seriously wounded, Isorena survived.
of lying and sideswiped him with the grease-gun. Then Benavides left for the He received first-aid treatment at the Tabaco Hospital and was air lifted the next day
bodega. to Manila where he was confined, first at the North General Hospital, and then at the
National Orthopedic Hospital. Dr. Casiano Flaviano, a resident Physician at the
Now the men surrounded Gancayco in front of the office. Facing him was Balimbing, National Orthopedic Hospital who treated Isorena testified that the latter would have
who was swinging his hunting knife. Also facing him, to Balimbing's right, was Pavia died from his injuries had he not received immediate medical attention.
whose jacket, draped over his left arm, concealed his right hand which held a .45
caliber pistol. To Pavia's right was Demen, who was aiming the grease-gun at While events unfolded on Tabaco Bay, the group of eight men reached the seashore
Gancayco. Slightly behind Demen, Oljina aimed the shotgun at Gancayco. and chanced upon a motorboat anchored there. Balimbing wrapped the four firearms
in some anahaw leaves. When the men discovered that the motorboat did not have
Balimbing accused Gancayco of killing Balimbing's cousin, and demanded that enough gasoline, they looked for another and dragooned Bienvenido Taller into
Gancayco produce the .45 caliber pistol which he claimed was used to perpetrate transporting them. Taller observed that the eight men were excited and
the death. Gancayco explained that it was not he but Solon Demetrio who apprehensive. Balimbing told him, with some braggadocio that they had just killed
accidentally shot Balimbing's cousin, and that Gancayco had no such pistol. Pavia Gancayco and Isorena. The men alighted at the lighthouse at Malinao and

88
immediately afterwards, Taller reported to the police authorities of the town. requires only one qualifying circumstance, the other should be considered as an
However, he was advised to report to the police authorities of Tabaco, who had aggravating circumstance.
jurisdiction over the case.
The trial court found five aggravating circumstances against Fustigo, Demen, Pavia,
The eight men reached the poblacion of Malinao. On Borjas orders, they bought a Benavides, and de los Santos, to wit:
jute sack where they placed the firearms. Borja instructed Pavia and Demen, to
make their separate way towards Barrio Buragwis. Before he left with Demen, Pavia (1) the crimes of murder and frustrated murder were committed by a band, or with
left his .45 pistol with Balimbing. The remainder of the group boarded a passenger the aid of armed men;
bus going to Tiwi. As the bus stopped near the market at Tiwi another car overtook it (2) means were employed to weaken the defense, wherein is included taking
and policemen from Tabaco, headed by Chief of Police Ceferino Firaza, alighted and advantage of superior strength;
surrounded the bus. Firaza in a loud voice called for the surrender of all those in the
bus responsible for the killing at the Hacienda San Miguel. He was suddenly fired (3) craft, fraud and/or disguise were employed;
upon by Borja and then by Balimbing. The fusillade hit him on the right cheek, and
(4) there was promise of backpay in the commission of the crimes; and
he ordered his men to return fire. The encounter resulted in the death of Balimbing
and Oljina the capture of Benavides and de los Santos; and the escape of Borja and (5) there was treachery or evident premeditation, depending upon whatever is used
Fustigo, Lt. Melanio Rey of the Tabaco police confiscated the firearms, magazines, to qualify the crimes to murder and frustrated murder.
and ammunition taken from the hacienda; the .45 caliber pistol and its shell; the
hunting knife and its scabbard from the dead body of Balimbing; and other With respect to Pedro Borja in both cases the trial court considered against him four
paraphernalia from the dead bodies of Balimbing and Oljina. The encounter in Tiwi aggravating circumstances, consisting of the five above-mentioned, but excluding
took place in the afternoon of December 19, 1958. Subsequently, the rest of the the promise of backpay. It found another aggravating circumstance in the case of
band fell one by one into the hands of the law. Pavia, Demen, Fustigo, were arrested, frustrated murder, i.e. dwelling. Hence, in the case of frustrated murder, it found six
and Borja, the last one to fall, was captured on February 27, 1959 after a nationwide aggravating circumstances against Fustigo, Demen, Pavia, Benavides, and de los
manhunt. Santos; and five aggravating circumstances against Borja.

The trial judge characterized this narration of the shooting of Gancayco and Isorena All the five accused in the first trial admitted practically all the evidence for the
as both water tight and airtight. He found that the tenor of the evidence presented prosecution in their testimonies in their own behalf, and additionally in their
consisting in the main of the testimony Isorena, an eyewitness; and of the respective affidavits narrating their individual participation in the commission of the
accused de los Santos, who testified for the state during Borja's separate trial was two crimes. Moreover, they freely and voluntarily re-enacted the crime at the
confirmed by the testimonies of the medico-legal expert and the ballistician. He hacienda, in the presence of the trial judge. The re-enactment proceedings were
noted that the five accused who were separately tried from Borja admitted all the photographed and tape recorded, and bore out the version testified to by Isorena
facts leading to the shooting, but interposed the common defense of fear of Borja, and Lanon eyewitnesses for the prosecution.
who, they claimed, had threatened disobedient AFAG members with death. The trial
In the Brief for all accused-appellants except Pedro Borja, John Doe, and Richard Doe
judge was not persuaded and he discounted this common defense, characterizing it
(John Doe and Richard Doe refer to Alejo Balimbing and Tito Oljina who were killed
as "an after-thought to save their respective skins in the face of the overwhelming
during the encounter at Tiwi), the five accused Fustigo, Demen, Pavia, Benavides,
evidence of the prosecution pointing to their voluntary participation in the
and de los Santos do not deny their culpability for the offenses charged. In their
commission of the crimes of murder and frustrated murder." Instead, the trial judge
Brief, they prayed for the reduction of the penalty from death and its accessory
found that the five men participated in the killing "because they were inspired by the
penalties, to reclusion temporal and its accessory penalties. They admitted with
juicy thought or promise of an enormous amount of backpay for each."
candor: "The finding of facts in the decision of the trial court having been found to
Borja, who was still at large at the time the five accused were being tried, had a be a faithful narration of the incident as related during the trial of the case and given
separate trial. The trial court found that he "has a version entirely distinct and in the two ocular inspections of the premises where the shooting happened, it would
separate from that of the five accused, which version in turn is astronomically far seem a useless endeavor to reiterate said findings of facts, ..." (Brief for the
from the evidence presented by the prosecution." Borja washed his hands of any Accused-Appellants, except Pedro Borja, pages 6-7).
complicity in the killing which he sought to lay at Balimbing's door. Conveniently for
The five accused controverted the findings of the trial court that there were five
Borja, Balimbing is dead and cannot tell his tale. But like his colleagues, Borja failed
aggravating circumstances in the case for murder. Instead, they contended that the
to convince the trial judge. Noting that Borja was the commanding officer of eight
trial court should have appreciated only three aggravating circumstances. They
AFAG regiments in the entire Bicol region, while Balimbing was just a sergeant, the
reasoned that any of the alleged aggravating circumstances should necessarily be
trial court refused to believe that Balimbing openly defied Borja by instigating the
absorbed to qualify the crime of murder, thereby leaving only four aggravating
sanguinary episode at the hacienda. Instead, the trial court declared: "The contrary
circumstances. Moreover, they argued that the aggravating circumstance of promise
was the real and painful truth. Pedro Borja was the leader of the group that raided
of backpay was not alleged in the information, and consequently should not be taken
Hacienda San Miguel, and he was the very one who ordered the liquidation of
as an aggravating circumstance.
Santiago Gancayco Jr. and Salustiano Isorena."
We find that the trial court correctly considered that either treachery or evident
The trial court found that conspiracy was "conclusively established" and that "the
premeditation qualifies the crime to murder, and hence the other alternative
guilt of all the accused has been established by proof above the shadow of doubt." It
circumstance should be considered as aggravating. We reject the contention of the
found that the killing of Gancayco constitutes murder, while the shooting of Isorena
five accused; for while it is true that the aggravating circumstance of promise or
constitutes frustrated murder. The trial court held that either evident premeditation
reward was not alleged in the information, nevertheless, it was proven during the
or treachery qualifies the crimes to murder and frustrated murder; since the law
trial, and therefore can be considered as a generic aggravating circumstance,

89
though not a qualifying circumstance. (People of the Philippines vs. Navarro, et al., L- accused, and in not acquitting him upon the ground that his guilt was not proven
20860, November 28, 1964, 12 SCRA 530). On the other hand, as Borja beyond reasonable doubt. This is a blanket allegation which can be dealt with
contends, infra, the other aggravating circumstances are absorbed by alevosia or summarily, because the testimonies of common witnesses in both trials clearly
treachery. Hence, if it is treachery which is considered as the qualifying establish facts incriminatory to Borja. More specifically, Borja contended that the
circumstance, there remain two generic aggravating circumstances which attended trial court erred in finding that a conspiracy existed between Borja and his co-
the commission of the two crimes with respect to the five accused: evident accused. We find this contention to be an excursion outside the perimeters of
premeditation; and promise or reward (which does not apply with respect to Borja.) credibility. Borja was the AFAG commander for the Bicol region. After the second
secret conference, he donned spurious armed forces get-up, complete with uniform
The five accused contended that there were three mitigating circumstances in their and two guns. When he was introduced to the hacienda personnel as a PC major in
favor: lack of instruction; fear of Pedro Borja; and lack of motive. We find no merit in command of the group, it does not appear that he raised any protest. Instead, the
this contention. The argument of lack of instruction is based on the allegation that evidence shows that he issued the order to Balimbing to "fix them up" referring to
the five accused did not finish primary education. But the defense adduced no proof Gancayco and Isorena and he also issued the order "Habulin! Habulin!" when
to establish the existence of this circumstance, leaving in full force the holding that Gancayco ran for his life. Borja consistently acted the leader as he led the dash for
extenuating circumstances must be proven positively and cannot be based on mere illicit freedom. He led the group in fleeing the island; instructed that the firearms
deduction or inference. (PP. vs. Sakam, et al., 41566, December 7, 1934, 61 Phil. 27). should be secreted away in a jute sack; and ordered his men to separate into two
If by "lack of instruction" the defense refers to illiteracy, it is not sufficient to groups when his group was surrounded by Tabaco policemen near Tiwi, where Borja
constitute a mitigating circumstance, for there must also be lack of intelligence. (PP. led the gunfight.
vs. Gorospe, L-10644-45, February 19, 1959, 105 Phil. 184; PP. vs. Ripas, L-6246,
May 26, 1954, 95 Phil. 63; PP. vs. Semanada, L-11361, May 26, 1958, 103 Phil. 790; He eluded his trackers for more than two months until his capture. To flee the fold of
PP. vs. Tengyao, L-14675, November 29, 1961, 113 Phil. 465). the law is to admit that one has transgressed that law. (PP. vs. Wilson, et al., 30012-
15, March 7, 1929, 52 Phil. 907). Borja's uncontested actions would be gratuitous
The second mitigating circumstance advanced by the five accused is "awe and fear and illogical, unless located within the frame of conspiracy, which is their only
of Pedro Borja." But the element of fear is not one of those enumerated as a reasonable context. The evidence shows that Borja acted in concert with the other
mitigating circumstance under the Revised Penal Code, Article 13. If the defense accused in pursuance of the same objective. Hence, conspiracy attaches and it is no
refers to the element of "uncontrollable fear or duress" which is an exempting longer necessary to obtain proof as to the previous agreement or decision to commit
circumstance under the Revised Penal Code, Article 12, the argument is still invalid, the crime. (PP vs Cadag, L-13830, May 31, 1961, 2 SCRA 388; PP. vs. Peralta, L-
for it has been held that the element of duress should be based on real, imminent or 19069, October 29, 1968, 25 SCRA 759; PP. vs. Alcantara, L-26867, June 30, 1970,
reasonable fear for one's life or limb and should not be speculative, fanciful, or 33 SCRA 812).
remote fear. (PP. vs. Quilloy No. L-2313, January 10, 1951, 88 Phil. 53). We find no
evidence to support the claim that Borja threatened any or all of the other accused. Borja further contended that assuming his criminal liability, the trial court erred in
appreciating the qualifying circumstances of treachery and evident premeditation;
The third alleged mitigating circumstance is lack of motive. We are hard put to and the generic aggravating circumstances of band or aid of armed men; use of
discuss this contention because the Revised Penal Code, Article 13 does not include means to weaken the defense; craft, fraud, and/or disguise. This contention for the
"lack of motive" as one of the mitigating circumstances. Finally, the defense argues first part flies in the face of the evidence. Treachery was present because Gancayco
that "the undiplomatic attitude of Gancayco and Isorena, bordering on provocation" was killed while he was unarmed, and surrounded by enemies with firearms,
should be considered as another mitigating circumstance. This contention is not including two grease-guns. Thus, Gancayco was deprived of any means of defense
borne out by the evidence; on the contrary, it appears that instead of being while his enemies were exposed to no risk arising from the defense which the
"undiplomatic", Gancayco was pacific, and he counselled Isorena that "the best thing offended party might have made. True, Borja was inside the bodega when the
for us to do is to talk to them peacefully and follow them, whatever they want." shooting took place; but his physical absence does not exonerate him, for it was he
(T.s.n. p. 1466). This leaves as the only remaining assertion of the five accused the who ordered the execution. Moreover, where there is conspiracy, treachery is
claim that "the alleged premeditated conspiracy to kill Gancayco was not clearly considered against all the offenders. (PP. vs. Carandang, et al., 32039, February 26,
revealed in the records of the case." We shall discuss this assertion in dealing with 1930, 54 Phil. 503). The aggravating circumstance of evident premeditation was
the defense of Borja. sufficiently proved, for prior to the shooting on December 19, 1958, the accused met
In the Brief for the defendant-appellant Pedro Borja, the defense contended that in two secret conferences and discussed the raid on the hacienda in order to avenge
since a separate trial was held for Borja, the trial court erred in rendering a single themselves by exacting redress from Gancayco for allegedly killing Balimbing's
decision on which the findings of facts respecting Borja, based on evidence adduced cousin and bulldozing the camote plantations of Balimbing's relatives.
during his separate trial, are not distinctively set forth, thereby prejudicially However, Borja's contention for the second part is well taken. The defense argues
impairing Borja's substantial rights. We find no such impairment of the rights of the that the circumstance of band and aid of armed men, cannot be taken separately
accused. The evidence shows that the prosecution established the same facts in the from the circumstance of use of means to weaken the defense, and advantage of
two separate trials. Isorena and Lanon testified as eyewitnesses to the incident of superior strength. It was correctly pointed out that all these circumstances are
December 19, 1958; de los Santos testified as an eyewitness not only to the same absorbed in treachery and may not be considered independently. (U.S. vs. Estopia,
incident, but also to events before and after the shooting, showing conspiracy et al., No. 9411, September 29, 1914, 28 Phil. 97; U.S. vs. Oro, No. 5781, August 14,
among the accused. Other persons testified in both trials to supply corroborating 1911, 19 Phil. 548; U.S. vs. Vitug, et al., No. 5430, September 8, 1910, 17 Phil. 1; PP
evidence. vs. Sespene No. L-9346, October 30, 1957, 102 Phil. 199; PP. vs. Lumantas, L-28355,
Borja also contended that the trial court erred in not according credence to Borja's July 17, 1969, 28 SCRA 764; PP. vs. Agustin, L-18368, March 31, 1966, 16 SCRA 467;
defense and in finding him guilty on the basis in part of what is alleged to be PP. vs. Layson, L-25177, October 31, 1969, 30 SCRA 32). It was also correctly pointed
incompetent evidence adduced not at his separate trial but at that of his co- out that treachery absorbs the circumstance of craft, fraud and disguise. (PP. vs.

90
Malig, et al., L-2083, May 30, 1949, 83 Phil. 803). Nonetheless, this leaves the
aggravating circumstance of evident premeditation, which applies to all the accused;
and the aggravating circumstance of promise of backpay, which applies to all the
accused, except Borja.
Lastly, Borja contended that, assuming he is criminally liable, the trial court erred in
not according him the benefit of the mitigating circumstance of voluntary surrender.
We do not view this as error. It appears that Borja did not surrender but was
captured on February 27, 1959. His own witness, Captain Eliseo Farol of the Armed
Forces of the Philippines, testified that he received a report that Borja was holed up
at Barrio Sumakap, Cavinte, Laguna. Accordingly, he sent a ranger team which
located the house and called on Borja to come down. As the house was surrounded
by soldiers, Borja offered no resistance. Capt. Farol also testified that while Borja was
at large, he killed a PC soldier in an armed encounter at Catanauan, Quezon, as a
result of which he was indicted for murder in the court of first instance in that
province. Capt. Farol also declared that a prize of P2,000.00 was put on Borja's head
for being a dangerous fugitive. These are not indicia of the personality seeking
voluntary surrender.
Mr. Santiago Gancayco, Jr., manager of a 1,700-hectare hacienda and scion of a
prominent family, is dead. His demise when he was only in his early thirties was
rendered more tragic in that he breathed his last in the bosom of his grieving family,
consisting of his wife and six small children, in the course of his flight from
ostensible visitors who had suddenly been transformed into cold-blooded killers. Dr.
Rizalino Reyes, Chief of the Medico-Legal Division of the National Bureau of
Investigation who performed an autopsy on the body of Santiago Gancayco, Jr.
testified that his death was due to hemorrhage, severe, secondary to multiple
gunshot wounds of the body and that shock, traumatic was contributory. It was
abundantly established in the trial court that his killing was attended by treachery,
which qualifies the crime committed by the perpetrators into murder.
Five of the accused, namely: Pedro Fustigo, Inocencio Demen, Rufino Pavia, Felipe
Benavides and Dominador de los Santos have been in custody since December 19,
1958, or shortly thereafter; while Pedro Borja was apprehended on February 27,
1959. It can thus be seen that all of them have been under detention for over twenty
years.
WHEREFORE, the judgment of the lower court in Criminal Case No. 2590 is hereby
affirmed in toto; that in Criminal Case No. 2578 is modified in respect of the principal
penalty from death to reclusion perpetua for lack of necessary votes, and in respect
of the civil indemnity from P6,000 to P12,000.00. SO ORDERED.

91
PEOPLE V. MUOZ Muoz ordered Alejandro and his wife to lie down and then, even as Pedro pleaded
for his father's life, shot Alejandro twice in the head, killing him instantly. Millora,
G.R. No. L-38969-70 February 9, 1989 Tayaba and Mislang, along with their companions, merely stood by as the brutal act
was committed. Juana watched her husband's death in terror and the 12-year old
PEOPLE OF THE PHILIPPINES, plaintiff-appellee, boy made a desperate run for his life as one of the accused fired at him and
vs. missed. 6
FELICIANO MUOZ, alias "Tony", et al., accused, MARVIN MILLORA, TOMAS
TAYABA, alias "Tamy Tayaba" and JOSE MISLANG, defendants-appellants. The second victim having been murdered as the first, the accused then vented their
violence on Aquilino, whom Muoz again brutally kicked as the others looked on.
CRUZ, J.: Aquilino was entirely defenseless. Finally, Muoz ended the boy's agony and shot
him to death, hitting him in the head and body. Muoz and Minora then picked up all
Of the four persons convicted in this case, one has not appealed and thus impliedly the empty shells and fled with the rest of their companions, leaving the terrified
accepted his sentence. The others have questioned their conviction and insist that Juana with the two grisly corpses. 7
they are innocent. The prosecution did not think so, and neither does the Solicitor
General now. The brief for the appellee would affirm the finding of guilt and in fact The above events were narrated at the trial by Melecia Bulatao, 8 Mauro's daughter
even increase the penalty. and Aquilino's sister; Jose Bulatao, 9 Mauro's son and Aquilino's brother; Juana
Bulatao, 10 Alejandro's wife; and Pedro Bulatao, 11 their son. Their testimony was
The prosecution presented a bizarre case of arbitrary condemnation and instant corroborated by Dr. Juanita de Vera, 12 who performed the autopsy on the three
punishment meted out by what appear to be the members of a private army. Eleven victims.
persons, most of them bodyguards of the town mayor, went out in a jeep at the
behest of one of them who had complained of having been victimized by cattle Melecia and Jose testified on the killing of their father by Marvin Minora as the other
rustlers. Having found their supposed quarry, they proceeded to execute each one of accused stood by and the mauling of their brother Aquilino before he was dragged
them in cold blood without further ado and without mercy. One was shot in the away by the group. The trial court especially noted the straightforward account
mouth and died instantly as his son and daughter looked on in horror. The second given by Jose, who positively identified Minora as the killer and described the
was forced to lie down on the ground and then shot twice, also in the head, before participation of the others, including the savage kicking of his brother by
his terrified wife and son. The third, who was only sixteen years old, was kicked in Muoz. 13 Melecia earlier pointed to Mislang as the one who had shot her father but
the head until he bled before he too had his brains blown out. To all appearances, changed her mind later on cross-examination and named Millora as the actual killer.
the unfortunate victims were only innocent farmers and not the dangerous criminals She explained her turn-about by confessing that she had earlier agreed to exonerate
they were pronounced to be. Minora in exchange for the sum of P3,000.00 promised by his father although she
actually did not receive the money. 14 For her part, Juana related how she was
Bizarre but true, as the trial court agreed. threatened with death unless she accompanied the accused to where her husband
Of the eleven persons who were charged with murder in three separate was. She narrated in detail how Alejandro was killed before her very eyes and how
informations, the four who stood trial were found guilty. 1 The other seven have yet Aquilino was later kicked and then also shot to death, also by Muoz, while the other
to be identified and tried. The sentence of Feliciano Muoz, who did not appeal, has accused stood by. 15 Her testimony was corroborated by Pedro, her son, whom the
long become final and executory and is now being served. 2 We deal here only with accused had also thought of killing because he was "talkative" and indeed was shot
the appeals of the other convicts, namely, Marvin Millora, Tomas Tayaba, and Jose at when he successfully escaped after his father's murder. 16
Mislang, who all ask for a reversal. The defense makes much of the fact that it was only months after the killings that it
The killings occurred in the morning of June 30, 1972, in Balite Sur, San Carlos City, occurred to these witnesses to denounce the accused and suggests that this delay
Pangasinan. 3 should impugn their credibility. As correctly pointed out by the trial judge, however,
these witnesses were naturally deterred from doing so for fear that they would meet
As established by the prosecution, Feliciano Muoz, Marvin Millora, Tomas Tayaba, the same fate that befell their relatives. These were humble barrio folk whose
Jose Mislang, and the other seven unidentified men went to the house of Mauro timidity did not allow them to report their grievances beyond the barrio officials they
Bulatao and asked for the address of his son Arsenic. All four of them went inside knew, more so since the higher authorities appeared to be indifferent and gave no
while the rest surrounded the house. All eleven men were armed. Mauro, who was attention, much less encouragement, to their complaints.
then bathing his horse, was called by the accused. As he approached and while
under his house, he was met by Millora who simply shot him at arm's length with a It is true that there were several inconsistencies in the testimony of these witnesses
"long firearm," hitting him in the mouth and killing him as he fell. At that precise as painstakingly pointed out by the appellants, 17 but these are minor flaws that do
time, Muoz, Tayaba and Mislang were standing by Millora, evidently giving him not detract from the essential truthfulness of their accounts of the ruthless
armed support. None of them made any move to restrain or dissuade him. 4 killings. 18

After killing Mauro, the four accused dragged out of the house his sixteen year old The brutality of the murders and the veracity of the testimony of the said witnesses
son, Aquilino, and knocked him down. Muoz kicked him several times in the head as are emphasized by the medical reports 19 of the injuries sustained by the victims, as
he lay on the ground while the others looked on in silent approval or at least without follows:
objection. They then took the bleeding man with them to look for their third target, Mauro Bulatao:
Alejandro Bulatao. 5
1. Thru and thru gunshot wound with point of entrance at the upper lip left
In Alejandro's house, the group forced his wife, Juana to go with them and direct side around 1 cm. in diameter and with the exit at the middle of the back of
them to her husband. They found him tending to their cows with his son Pedro. the head around 1-1/2 cm. in diameter.

92
2. Gunshot wound at the lower lip left side of the mouth. Tayaba and Mislang offered a common defense, also of alibi. Both claimed that
Mislang having complained of cattle rustlers, a group of policemen, including
Alejandro Bulatao: Tayaba, stayed in the former's house the whole night of June 29, 1972, leaving only
1. Lacerated gunshot wound at the left eye with the whole eye practically at 8 o'clock the following morning of June 30, 1972, after Mislang had served them
lacerated. breakfast. 30Significantly, however, barrio Bacnar where Mislang's house was
located, is only two kilometers from Balite Sur. 31 Moreover, the trial court doubted
2. Lacerated gunshot wound of the right eye and the forehead practically the testimony given by Sgt. Lomibao, who corroborated them and spoke of having
opened with the brain tissue outside. heard the gunfire narrated by Millora's witnesses. The decision noted that Lomibao
was mysteriously absent when the police chief and Dr. de Vera went to the scene of
Aquiline Bulatao: the crime at 9 o'clock that morning to investigate the killings. In fact, it expressed
1. Thru and thru gunshot wound with point of entrance at the upper right the suspicion that Lomibao and Patrolman Liwanag, who also testified for the
jaw bone around 1- 1/2 cm. in diameter and with the exit at the middle of accused, might have been among the seven unidentified persons who were with
the back of the head around 2 cm. in diameter. Muoz and the three appellants herein when the Bulataos were murdered. 32

2. Gunshot wound at the upper left shoulder out the middle of the left All told, we affirm the findings of the trial judge, who had the opportunity to observe
clavicle around 1- 1/2 inches in diameter. the witnesses at the trial and assess their credibility. As we said in a previous case:

The three appellants invoked individual defenses which the trial court correctly We see no reason to reverse the factual findings of the trial judge, who had
rejected as false and unbelievable. All claimed the Bulataos were killed as a result of the opportunity to observe the demeanor of the witnesses and to assess
an exchange of gunfire with a rather hazy group and each claimed he was not their credibility. The written record will not show that nuance of tone or
involved in the shoot-out. voice, the meaningful contrast between the hesitant pause and the prompt
reply, and the expression or color or tilt of face that will affirm the truth or
Testifying for Millora on the alleged encounter between the Bulataos and their expose the fabrication. All these subtle factors could be considered by the
adversaries, Victoriano Bacani said that the latter included Tayaba, Mislang and five trial judge in weighing the conflicting declarations before him, and we do
others who fled from the scene in a jeep. 20 Graciano Muoz, corroborating Bacani, not find that he has erred. 33
said he himself saw seven men in a jeep coming from the sound of the gunfire after
he had paid Mauro P400.00 to redeem his stolen carabao. 21 Another witness for We agree that the three appellants, together with Muoz and their seven other
Millora, Orlando de los Santos, testified to having seen the encounter between the companions, participated in the killings of the three Bulataos in the manner
Bulataos and the other group and declared that the former were armed with described by the witnesses for the prosecution. The defenses of the herein
carbines and Garand rifles. 22 appellants should be, as they properly were, rejected as undeserving of belief in the
light of the more convincing and telling evidence submitted by the government.
The trial court rejected Bacani's testimony because he appeared hesitant and
suspicious on the stand and did not give the impression that he was telling the However, we do not accept the different degrees of participation assigned by the
truth. 23 Moreover, it took him all of one year to report the alleged shooting court a quo to each of the appellants in each of the three offenses imputed to them.
encounter, which he also did not mention that same afternoon when he visited In Criminal Case No. 0176, Millora was found guilty as principal and Muoz and the
Mauro's family to condole with them. 24 It is also not believable that the group would other two herein appellants only as accomplices, and in Criminal Case Nos. 0177 and
flee because they had no more bullets when their supposed three adversaries were 0178, Muoz was found guilty as principal and the herein appellants only as
already dead in the field. The alleged redemption made by Muoz was described by accomplices. 34 In support of this finding, the trial court said that there was no
the trial court as preposterous, especially since no shred of evidence had been evidence of conspiracy to justify holding each of the accused equally liable for the
presented to show that Mauro was a cattle rustler, let alone his 16 year old son. 25 As three murders.
for De los Santos, no firearms were discovered beside the dead bodies of the
We hold that there was. Indeed, it is clear that from the very start, when the eleven
Bulataos, including Mauro, who was found not in the supposed battleground but men went out to look for the suspected cattle rustlers, there was already an
under his house, as testified to by Dr. De Vera. 26
agreement among them to ferret out and punish the Bulataos whom they had
Millora's own defense was that he was in Dagupan City at the time of the killings, condemned beforehand. They knew whom they were looking for. They knew where
having gone there in the evening of June 29, 1972. He claimed he had stayed there to look for them. They sought each of them with drawn and ready weapons. When
overnight with a female companion after drinking beer with Atty. Antonio Resngit they reached Mauro Bulatao's house, four of them went inside while the rest
returning to San Carlos City only between 8 and 9 o'clock the following morning or deployed themselves in strategic positions. When Millora shot Mauro, the appellants
June 30, 1972.27 The lawyer corroborated him, 28 but he cannot be more credible and the others stood by with guns at the ready. Nobody moved to dissuade or stop
than Mauro's own children, Jose and Melecia, who positively identified Millora as the him. Together they dragged Aquilino from the house and the rest watched while
person who actually shot their father in the face and killed him instantly. Such a Muoz kicked him in the head while helpless on the ground. Together, they took him
traumatic experience could not have been forgotten by these witnesses who saw with them and then forced Juana Bulatao to lead them to her husband. The rest
their father murdered without warning or mercy nor could their memory of the stood by with their weapons as Muoz shot Alejandro in the head. No one interceded
heartless killer have been easily wiped out from their minds. to stop him from also killing Aquilino. There is no question that the group moved in
concert, pursuing a common design previously agreed upon, that made each of
It is stressed that Juana Bulatao and her son Pedro also categorically declared that them part of a conspiracy. 35 As such, each of them is liable in equal degree with the
Millora was with the group that she took to the field where her husband and Aquilino others for each of the three killings. Each member of the conspiracy to commit the
were killed by Muoz. 29 crime of murder is guilty as a co-principal, regardless of who actually pulled the

93
trigger that killed the three victims. It is settled that in a conspiracy the act of one is A reading of Section 19(l) of Article III will readily show that there is really nothing
the act of all. 36 therein which expressly declares the abolition of the death penalty. The provision
merely says that the death penalty shall not be imposed unless for compelling
Each of the three killings constituted the crime of murder, qualified by alevosia. reasons involving heinous crimes the Congress hereafter provides for it and, if
There was treachery because every one of the three victims was completely helpless already imposed, shall be reduced to reclusion perpetua. The language, while rather
and defenseless when shot and killed by the accused with no risk to themselves. awkward, is still plain enough. And it is a settled rule of legal hermeneutics that if
Mauro was completely taken by surprise when he was shot in the face. Alejandro the language under consideration is plain, it is neither necessary nor permissible to
was lying down when he was shot in the head. Aquilino was seated when he was resort to extrinsic aids, like the records of the constitutional convention, for its
shot in the head and shoulders. None of the three victims had a chance to resist. interpretation. 41
The penalty for murder under Article 248 of the Revised Penal Code was reclusion At that, the Court finds that such resort, even if made, would not be of much
temporal in its maximum period to death, but this was modified by Article III, Section assistance either in the case at bar. Accepting arguendo that it was the intention of
19(l) of the 1987 Constitution providing as follows: the framers to abolish the death penalty, we are still not convinced from the debates
Excessive fines shall not be imposed, nor cruel, degrading or inhuman in the Constitutional Commission that there was also a requirement to adjust the two
punishment inflicted Neither shall death penalty be imposed, unless, for remaining periods by dividing them into three shorter periods. This is not a
compelling reasons involving heinous crimes, the Congress hereafter necessary consequence of the provision as worded. The following exchange cited by
provides for it. Any death penalty already imposed shall be reduced those in favor of Masangkay is at best thought-provoking but not decisive of the
to reclusion perpetua. question:

Conformably, the Court has since February 2, 1987 not imposed the death penalty FR. BERNAS: The effect is the abolition of the death penalty from those
whenever it was called for under the said article but instead reduced the same to statutes-only the death penalty. The statute is not abolished, but the
reclusion perpetua as mandated by the above provision. The maximum period of the penalty is abolished.
penalty was thus in effect lowered to the medium, the same period applied, as MR. MAAMBONG: That is what I am worried about, because the statutes,
before, where the offense was not attended by any modifying circumstance, with the especially in the General Criminal Law, which is the Revised Penal Code, do
minimum period, i. e., reclusion temporalmaximum, being still applicable in all other not necessarily punish directly with death. Sometimes it has a range of
cases. The three-grade scheme of the original penalty, including death, was thus reclusion temporal to death or reclusion perpetua to death. And what would
maintained except that the maximum period was not imposed because of the be the effect on the judges, for example, if the range is reclusion temporal
constitutional prohibition. to death and he can no longer impose the death penalty? He will have
In People v. Gavarra 37 Justice Pedro L. Yap declared for the Court that "in view of the difficulty in computing the degrees.
abolition of the death penalty under Section 19, Article III of the 1987 Constitution, Could the committee enlighten us on how the judge will look at the specific
the penalty that may be imposed for murder is reclusion temporal in its maximum situation.
period to reclusion perpetua" thereby eliminating death as the original maximum
period. Later, without categorically saying so, the Court, through Justice Ameurfina FR. BERNAS: I grant that the judges will have difficulty, but I suppose that
Melencio-Herrera in People v. Masangkay 38 and through Justice Andres R. Narvasa the judges will be equal to their tasks. The only thing is, if there is a range,
in People v. Atencio 39 divided the modified penalty into three new periods, the limits the range cannot go as far as death (Record, CONCOM, July 18, 1986, Vol. I,
of which were specified by Justice Edgardo L. Paras in People v. Intino, 40 as follows: 749).
the lower half of reclusion temporal maximum as the minimum; the upper half
of reclusion temporal maximum as the medium; and reclusion perpetua as the FR. BERNAS: Certainly, the penalties lower than death remain.
maximum. MR. REGALADO: That would be reclusion perpetua. But the range of the
The Court has reconsidered the above cases and, after extended discussion, come to penalty for murder consists of three periods. The maximum period
the conclusion that the doctrine announced therein does not reflect the intention of of reclusion temporal under the present status is the minimum period for
the framers as embodied in Article III, Section 19(l) of the Constitution. This the penalty for murder. The medium period isreclusion perpetua. The
conclusion is not unanimous, to be sure. Indeed, there is much to be said of the maximum period is death. If we now remove the death penalty, we will,
opposite view, which was in fact shared by many of those now voting for its reversal. therefore, have a range of penalty of 17 years, 4 months and 1 day to 20
The majority of the Court, however, is of the belief that the original interpretation years ofreclusion temporal up to reclusion perpetua. You cannot
should be restored as the more acceptable reading of the constitutional provision in divide reclusion perpetua into two. While it has a duration of 30 years, it is
question. an indivisible penalty. Where do we get the medium period now until such
time that Congress gets around to accommodate this amendment?
The advocates of the Masangkay ruling argue that the Constitution abolished the
death penalty and thereby limited the penalty for murder to the remaining periods, FR. BERNAS: As I said, this is a matter which lawyers can argue with judges
to wit, the minimum and the medium. These should now be divided into three new about. All we are saying is, the judges cannot impose the death penalty
periods in keeping with the three-grade scheme intended by the legislature. Those (Record, CONCOM July 18, 1986, Vol. I, p. 750).
who disagree feel that Article III, Section 19(l) merely prohibits the imposition of the So there we have it "this is a matter which lawyers can argue with judges about."
death penalty and has not, by reducing it toreclusion perpetua, also correspondingly Assuming that Commissioner Bernas's answer reflected the consensus of the body,
reduced the remaining penalties. These should be maintained intact. we are still not persuaded that it was the intention of the framers to lower not only
the maximum period but also the other periods of the original penalty. That is not

94
necessarily inferable from his statement that "the judges will be equal to their task," Coming back to the case at bar, we find that there being no generic aggravating or
especially so since he also said and we think with more definiteness-that "all we are mitigating circumstance attending the commission of the offenses, the applicable
saying is that the judges cannot impose the death penalty" (Emphasis supplied). We sentence is the medium period of the penalty prescribed by Article 248 of the
understand this to mean that they were not saying more. Revised Penal Code which, conformably to the new doctrine here adopted and
announced, is stillreclusion perpetua. This is the penalty we impose on all the
The question as we see it is not whether the framers intended to abolish the death accused-appellants for each of the three murders they have committed in
penalty or merely to prevent its imposition. Whatever the intention was, what we conspiracy with the others. The award of civil indemnity for the heirs of each of the
should determine is whether or not they also meant to require a corresponding victims is affirmed but the amount thereof is hereby increased to P30,000.00 in line
modification in the other periods as a result of the prohibition against the death with the present policy.
penalty.
It remains to observe that the crimes inflicted upon the humble farmers would have
It is definite that such a requirement, if there really was one, is not at all expressed remained unpunished were it not for the vigilance of certain responsible officials,
in Article III, Section 19(l) of the Constitution or indicated therein by at least clear especially the police and the prosecuting officer, who took up the cudgels for the
and unmistakable implication. It would have been so easy, assuming such intention, victims' families. The courage and conscientiousness they displayed are still the
to state it categorically and plainly, leaving no doubt as to its meaning. One most potent weapons against those who, in their arrogance, believe that they can
searches in vain for such a statement, express or even implied. The writer of this flout the law and frustrate justice because they have the protection of powerful
opinion makes the personal observation that this might be still another instance patrons.
where the framers meant one thing and said another-or strangely, considering their
loquacity elsewhere did not say enough. WHEREFORE, the appealed decision is MODIFIED and all the accused-appellants are
hereby declared guilty as principals in Criminal Case Nos. 0176, 0177 and 0178.
The original ruling as applied in the Gavarra, Masangkay, Atencio and Intino cases Each of them is sentenced to suffer three (3) penalties ofreclusion perpetua, and to
represented the unanimous thinking of the Court as it was then constituted. All but pay solidarily to the heirs of their victims civil indemnity in the sum of P30,000.00
two members 42 at that time still sit on the Court today. If we have seen fit to take a for each of the deceased, or a total indemnity of P90,000.00, with costs.
second look at the doctrine on which we were all agreed before, it is not because of
a change in the composition of this body. It is virtually the same Court that is SO ORDERED.
changing its mind after reflecting on the question again in the light of new
perspectives. And well it might, and can, for the tenets it lays down are not Fernan, C.J., Gutierrez, Jr., Feliciano, Gancayco, Padilla, Bidin, Grio-Aquino and
immutable. The decisions of this Court are not petrified rules grown rigid once Medialdea, JJ., concur.
pronounced but vital, growing things subject to change as all life is. While we are
told that the trodden path is best, this should not prevent us from opening a fresh
trial or exploring the other side or testing a new idea in a spirit of continuing inquiry.
Accordingly, with the hope that "as judges, (we) will be equal to (our) tasks,"
whatever that means, we hereby reverse the current doctrine providing for three
new periods for the penalty for murder as reduced by the Constitution. Instead, we
return to our original interpretation and hold that Article III, Section 19(l) does not
change the periods of the penalty prescribed by Article 248 of the Revised Penal
Code except only insofar as it prohibits the imposition of the death penalty and
reduces it to reclusion perpetua. The range of the medium and minimum penalties
remains unchanged.
The Court relies that this interpretation may lead to certain inequities that would not
have arisen under Article 248 of the Revised Penal Code before its modification.
Thus, a person originally subject to the death penalty and another who committed
the murder without the attendance of any modifying circumstance will now be both
punishable with the same medium period although the former is concededly more
guilty than the latter. True enough. But that is the will not of this Court but of the
Constitution. That is a question of wisdom, not construction. Of some relevance
perhaps is the parable in the Bible of the workman who was paid the stipulated daily
wage of one penny although he had worked longer than others hired later in the day
also paid the same amount. When he complained because he felt unjustly treated by
the householder, the latter replied: "Friend, I do you no wrong. Did you not agree
with me for a penny?'
The problem in any event is addressed not to this Court but to the Congress.
Penalties are prescribed by statute and are essentially and exclusively legislative. As
judges, we can only interpret and apply them and have no authority to modify them
or revise their range as determined exclusively by the legislature. We should not
encroach on this prerogative of the lawmaking body.

95
PEOPLE V. OBSANIA, 23 SCRA 1249 (1968) always present in order that they may be considered in contemplation of
law.
G.R. No. L-24447 June 29, 1968 Nothing in the foregoing statement can be reasonably interpreted as requiring
THE PEOPLE OF THE PHILIPPINES, plaintiff-appellant, an explicit allegation of "lewd design" in a complaint for rape. We hold in no
vs. uncertain terms that in a complaint for rape it is not necessary to allege "lewd
WILLY OBSANIA, defendant-appellee. design" or "unchaste motive", for to require such averment is to demand a patent
superfluity. Lascivious intent inheres in rape and the unchaste design is manifest in
Office of the Solicitor General for plaintiff-appellant. the very act itself the carnal knowledge of a woman through force or intimidation,
Maximo V. Cuesta, Jr. for defendant-appellee. or when the woman is deprived of reason or otherwise unconscious, or when the
woman is under twelve years of age. 2
CASTRO, J.:
It is clear that the complaint here satisfies the requirements of legal sufficiency of an
Before us for review, on appeal by the People of the Philippines, is an order, dated indictment for rape as it unmistakably alleges that the accused had carnal
January 8, 1965, of the Court of First Instance of Pangasinan dismissing, upon motion knowledge of the complainant by means of violence and intimidation. We therefore
of the defense, an indictment for rape against Willy Obsania. hold that the trial judge erred in dismissing the case on the proffered grounds that
On November 22, 1964, barely a day after the occurence of the alleged crime, the complaint was defective for failure to allege "lewd design" and, as a
Erlinda Dollente, the 14-year old victim, and her parents, Ciriaco Dollente and consequence of such infirmity, that the court a quo did not acquire jurisdiction over
Carmelita Lureta, filed in the municipal court of Balungao, Pangasinan a complaint the case. The error of the trial judge was in confusing the concept of jurisdiction with
for rape with robbery, 1 alleging that of insufficiency in substance of an indictment.

That on or about the 21st day of November 1964, at around 2:00 to 3:00 in We come now to the more important issue of double jeopardy. The accused
the afternoon, particularly in sitio Cawakalan, barrio of Capulaan, maintains that "assuming, arguendo, that the argument is right that the court a
municipality of Balungao, Province of Pangasinan, Philippines and within the quo has jurisdiction, the appeal of the Government constitutes double jeopardy."
jurisdiction of the Honorable Court, the said accused Willy Obsania, armed An appeal by the prosecution in a criminal case is not available if the defendant
with a dagger, by means of violence and intimidation, willfully, unlawfully would thereby be placed in double jeopardy. 3 Correlatively, section 9, Rule 117 of the
and feloniously did then and there have carnal knowledge of the Revised Rules of Court provides:
complainant Erlinda Dollente, against her will and on the roadside in the
ricefields at the above-mentioned place while she was alone on her way to When a defendant shall have been convicted or acquitted, or the case
barrio San Raymundo. against him dismissed or otherwise terminated without the express consent
of the defendant, by a court of competent jurisdiction, upon a valid
After the case was remanded to the Court of First Instance of Pangasinan for further complaint or information or other formal charge sufficient in form and
proceedings, the assistant provincial fiscal filed an information for rape against the substance to sustain a conviction, and after the defendant had pleaded to
accused, embodying the allegations of the above complaint, with an additional the charge, the conviction or acquittal of the defendant or the dismissal of
averment that the offense was committed "with lewd designs". the case shall be a bar to another prosecution for the offense charged, or
The accused pleaded not guilty upon arraignment, and forthwith his counsel moved for any attempt to commit the same or frustration thereof, or for any
for the dismissal of the case, contending that the complaint was fatally defective for offense which necessarily includes or is necessarily included in the offense
failure to allege "lewd designs" and that the subsequent information filed by the charged in the former complaint or information.
fiscal which averred "lewd designs" did not cure the jurisdictional infirmity. The In order that the protection against double jeopardy may inure in favor of an
court a quogranted the motion and ordered dismissal of the action, ruling that "the accused, the following requisites must have obtained in the original prosecution: (a)
failure of the complaint filed by the offended party to allege that the acts committed a valid complaint or information; (b) a competent court; (c) the defendant had
by the accused were with 'lewd designs' does not give this Court jurisdiction to try pleaded to the charge; and (d) the defendant was acquitted, or convicted, or the
the case." From this order, the fiscal brought the instant appeal. case against him was dismissed or otherwise terminated without his express
Two issues are tendered for resolution, namely: first, are "lewd designs" an consent.
indispensable element which should be alleged in the complaint?, and, second, does The complaint filed with the municipal court in the case at bar was valid; the court a
the present appeal place the accused in double jeopardy? quo was a competent tribunal with jurisdiction to hear the case; the record shows
Both must be answered in the negative. that the accused pleaded not guilty upon arraignment. Hence, the only remaining
and decisive question is whether the dismissal of the case was without the express
The accused, in his motion to dismiss, as well as the trial judge, in his order of consent of the accused.
dismissal, rely basically on the ruling in People vs. Gilo (L-18202, April 30, 1964). In
the case which involved a prosecution for acts of lasciviousness this Court, in The accused admits that the controverted dismissal was ordered by the trial judge
passing, opined that "lewd design" is upon his motion to dismiss. However, he vehemently contends that under the
prevailing jurisprudence, citing People vs. Bangalao, et al. (94 Phil. 354, February 17,
... an indispensable element of all crimes against chastity, such as 1954), People vs. Labatete (L-12917, April 27, 1960), People vs. Villarin (L-19795,
abduction, seduction and rape, including acts of lasciviousness ... an July 31, 1964), and People vs. Cloribel (L-20314, August 31, 1964), an erroneous
element that characterizes all crimes against chastity, apart from the dismissal of a criminal action, even upon the instigation of the accused in a motion
felonious or criminal intent of the offender, and such element must be to quash or dismiss, does not bar him from pleading the defense of double jeopardy

96
in a subsequent appeal by the Government or in a new prosecution for the same person in authority, based on the same facts alleged in the former complaint for
offense. The accused suggests that the above-enumerated cases have abandoned slight physical injuries. Again, upon motion of the accused, the trial court dismissed
the previous ruling of this Court to the effect that when a case is dismissed, other the new indictment on the ground of double jeopardy. From this order, the
than on the merits, upon motion of the accused personally or through counsel, such prosecution appealed. In upholding the appeal of the Government, this Court
dismissal is to be regarded as with the express consent of the accused and observed that although the information for assault necessarily embraced the crime
consequently he is deemed to have waived 4 his right to plead double jeopardy of slight physical injuries for which the accused was indicted in the justice of the
and/or he is estopped 5 from claiming such defense on appeal by the Government or peace court,
in another indictment for the same offense.
... it appears that the appellee was neither convicted nor acquitted of the
This particular aspect of double jeopardy dismissal or termination of the original previous charge against him for slight physical injuries, for that case was
case without the express consent of the defendant has evoked varied and dismissed upon his own request before trial could be finished. Having
apparently conflicting rulings from this Court. We must untangle this jurisprudential himself asked for such dismissal, before a judgment of conviction or
maze and fashion out in bold relief a ruling not susceptible of equivocation. Hence, a acquittal could have been rendered, the appellee is not entitled to invoke
searching extended review of the pertinent cases is imperative. the defense of double jeopardy...
The doctrine of waiver of double jeopardy was enunciated and formally labelled as In Gandicela, this Court had occasion to reiterate the Salico ruling:
such for the first time in 1949 inPeople vs. Salico, supra, with three justices
dissenting. 6 In that case, the provincial fiscal appealed from the order of the trial But where a defendant expressly consents to, by moving for, the dismissal
court dismissing, upon motion of the defendant made immediately after the of the case against him, as in the present case, even if the court or judge
prosecution had rested its case, an indictment for homicide, on the ground that the states in the order that the dismissal is definite or does not say that the
prosecution had failed to prove that the crime was committed within the territorial dismissal is without prejudice on the part of the fiscal to file another
jurisdiction of the trial court, or, more specifically, that the municipality of Victorias information, the dismissal will not be a bar to a subsequent prosecution of
in which the crime was allegedly committed was compromised within the province of the defendant for the same offense. (People vs. Ylagan, 58 Phil. 851; People
Negros Occidental. Rejecting the claim of the accused that the appeal placed him in vs. Salico, 84 Phil. 722.).
double jeopardy, this Court held that the dismissal was erroneous because the And in denying the motion for reconsideration filed by the accused in that case, this
evidence on record showed that the crime was committed in the town of Victorias Court held:
and the trial judge should have taken judicial notice that the said municipality was
included within the province of Negros Occidental and therefore the offense charged According to Section 9 of Rule 13, if a criminal case is dismissed otherwise
was committed within the jurisdiction of the court of first instance of the said than upon the merits at any stage before judgment, without the express
province. In ruling that the appeal by the Government did not put the accused in consent of the defendant, by a court of competent jurisdiction, upon a valid
peril of a second jeopardy, this Court stressed that with "the dismissal of the case by complaint or information, and after the defendant has pleaded to the
the court below upon motion of the defendant, the latter has not been in jeopardy," charge, the dismissal of the case shall be definite or a bar to another
and "assuming, arguendo, that the defendant had been already in jeopardy in the prosecution for the same offense; but if it is dismissed upon the petition or
court below and would be placed in double jeopardy by the appeal, the defendant with the express consent of the defendant, the dismissal will be without
has waived his constitutional right not to be put in danger of being convicted twice prejudice or not a bar to another prosecution for the same offense,
for the same offense." Mr. Justice Felicisimo Feria, speaking for the majority, because, in the last case, the defendant's action in having the case
reasoned that dismissed constitutes a waiver of his constitutional right not to be
prosecuted again for the same offense.
... when the case is dismissed with the express consent of the defendant,
the dismissal will not be a bar to another prosecution for the same offense; In Pinuela, as in Salico, the prosecution had presented its evidence against the
because, his action in having the case dismissed constitutes a waiver of his defendant, and the trial court, upon motion of the accused, dismissed the criminal
constitutional right or privilege, for the reason that he thereby prevents the action for lack of evidence showing that the crime charged was committed within its
court from proceeding to the trial on the merits and rendering a judgment territorial jurisdiction. On appeal by the Government, this Court found that the
of conviction against him. evidence showed otherwise and, like in Salico, the majority rejected the plea of
double jeopardy interposed by the accused on the ground that his virtual instigation
The Salico doctrine was adhered to and affirmed in People vs. Marapao (85 Phil. 832, of the erroneous dismissal amounted to a waiver of his right against a second
March 30, 1950), Gandicela vs. Lutero (88 Phil. 299, March 5, 1951), People vs. jeopardy.
Pinuela, et al. (91 Phil. 53, March 28, 1952), Co Te Hue vs. Encarnacion (94 Phil. 258,
January 26, 1954), and People vs. Desalisa (L-15516, December 17, 1966). In Co Te Hue, it was the theory of the petitioner that the charge of estafa filed
against him having been dismissed, albeit provisionally, without his express consent,
In Marapao, the defendant was indicted for slight physical injuries in the municipal its revival constituted double jeopardy which bars a subsequent prosecution for the
court of Sibonga, Cebu. After the prosecution had rested its case, a continuance was same offense. This claim was traversed by the Solicitor General who contended that
had, and when trial was resumed, the court, upon motion of the defense, ordered considering what had transpired in the conference between the parties, the
the case dismissed for failure of the prosecution to appear. However, the court provisional dismissal was no bar to the subsequent prosecution for the reason that
reconsidered this order upon representation of the fiscal who appeared moments the dismissal was made with the defendant's express consent. This Court sustained
later, and ordered the defense to present its evidence. The accused moved to get the view of the Solicitor General, thus:
aside the latter order on the ground that it placed him in double jeopardy. Acceding
to this motion, the court dismissed the case. Subsequently, the accused was We are inclined to uphold the view of the Solicitor General. From the
charged in the Court of First Instance of Cebu with the offense of assault upon a transcript of the notes taken at the hearing in connection with the motion

97
for dismissal, it appears that a conference was held between petitioner and should the dismissal be found incorrect, the case would have to be
the offended party in the office of the fiscal concerning the case and that as remanded to the court of origin for further proceedings, to determine the
a result of that conference the offended party filed the motion to dismiss. It guilt or innocence of the defendant. (Emphasis supplied)
also appears that as no action has been taken on said motion, counsel for
petitioner invited the attention of the court to the matter who acted The doctrine of estoppel in relation to the plea of double jeopardy was first
thereon only after certain explanation was given by said counsel. And when enunciated in Acierto which held that when the trial court dismisses a case on a
the order came the court made it plain that the dismissal was merely disclaimer of jurisdiction, upon the instigation of the accused, the latter is estopped
provisional in character. It can be plainly seen that the dismissal was on appeal from asserting the jurisdiction of the lower court in support of his plea of
effected not only with the express consent of the petitioner but even upon second jeopardy. The doctrine of estoppel is in quintessence the same as the
the urging of his counsel. This attitude of petitioner, or his counsel, takes doctrine of waiver: the thrust of both is that a dismissal, other than on the merits,
this case out of the operation of the rule. sought by the accused in a motion to dismiss, is deemed to be with his express
consent and bars him from subsequently interposing the defense of double jeopardy
In essence, this Court held that where a criminal case is dismissed provisionally not on appeal or in a new prosecution for the same offense.
only with the express consent of the accused but even upon the urging of his
counsel, there can be no double jeopardy under section 9, Rule 113, if the In Acierto, the defendant was charged before a United States court-martial with
indictment against him is revived by the fiscal. This decision subscribes substantially having defrauded the Government of the United States, through falsification of
to the doctrine on waiver established in Salico. documents, within a military base of the United States in the Philippines. The
challenge by the accused against the jurisdiction of the military tribunal was
The validity and currency of the Salico doctrine were intimated in the recent case brushed aside, and he was convicted. On review, the verdict was reversed by the
of People vs. Fajardo (L-18257, June 29, 1966), and six months later were reaffirmed Commanding General who sustained Acierto's position on the ground of lack of
in People vs. Desalisa, supra. jurisdiction. Subsequently, he was convicted of estafa and falsification based on the
same facts by the Court of first Instance of Rizal. On appeal to this Court, he claimed
In Fajardo, this Court, through Mr. Justice Querube Makalintal, observed: former jeopardy in the court-martial proceedings, asserting that the military court
The record does not reveal that appellees expressly agreed to the dismissal actually had jurisdiction. In a unanimous 7 decision, this Court, through Mr. Justice
of the information as ordered by the trial Judge or that they performed any Pedro Tuason, ruled:
act which could be considered as express consent within the meaning of This is the exact reverse of the position defendant took at the military trial.
the rule. While they did file a motion asking that the case be quashed or As stated, he there attacked the court-martial's jurisdiction with the same
that a reinvestigation thereof be ordered, the court granted neither vigor that he now says the court-martial did have jurisdiction; and thanks to
alternative. What it did was to order the prosecution to amend the his objections, so we incline to believe, the Commanding General, upon
complaint. This order was in effect a denial of the motion to quash, and it consultation with, and the recommendation of, the Judge Advocate General
was only after the prosecution failed to amend that the court dismissed the in Washington, disapproved the court-martial proceedings.
case on that ground. Consequently, even under the theory enunciated in
some decisions of this Court (People vs. Salico, etc.) that if a valid and xxx xxx xxx
sufficient information is erroneously dismissed upon motion of the
defendant he is deemed to have waived the plea of double jeopardy in Irrespective of the correctness of the views of the Military authorities, the
connection with an appeal from the order of dismissal, appellees here are defendant was estopped from demurring to the Philippine court's
not precluded from making such plea. jurisdiction and pleading double jeopardy on the strength of his trial by the
court-martial, A party will not be allowed to make a mockery of justice by
To paraphrase, had the dismissal been anchored on the motion to dismiss, the taking inconsistent positions which if allowed would result in brazen
defendants would not have been entitled to protection against double jeopardy. deception. It is trifling with the courts, contrary to the elementary principles
of right dealing and good faith, for an accused to tell one court that it lacks
Then in Desalisa, this Court, in a unanimous decision penned by Mr. Justice authority to try him and, after he has succeeded in his effort, to tell the
Jesus Barrera, held that court to which he has been turned over that the first has committed error
... The ruling in the case of Salico, that the act of the defendant in moving in yielding to his plea. (Emphasis supplied)
for the dismissal of the case constitutes a waiver of the right to avail of the The Acierto ruling was reiterated in People vs. Amada Reyes, et al. (96 Phil. 827,
defense of double jeopardy, insofar as it applies to dismissals which do not April 30, 1955); People vs. Reyes, et al. (98 Phil. 646, March 23, 1956); People vs.
amount to acquittal or dismissal of the case on the merits, cannot be Casiano (L-15309, February 16, 1961), and People vs. Archilla (L-15632, February 28,
considered to have been abandoned by the subsequent decisions on the 1961).
matter. (Emphasis supplied)
The defendants in People vs. Amada Reyes, et al., were charged as accessories to
xxx xxx xxx the crime of theft committed by their brother, Anselmo, the principal accused. The
... an appeal of the prosecution from the order of dismissal (of the criminal latter pleaded guilty to simple theft and was sentenced accordingly. The former
complaint) by the trial court will not constitute double jeopardy if (1) the pleaded not guilty and subsequently filed a motion to quash on the ground that
dismissal is made upon motion, or with the express consent, of the being brothers and sisters of the principal accused, they were exempt from criminal
defendant, and (2) the dismissal is not an acquittal or based upon responsibility for the acts charged against them in the information. Thereupon, the
consideration of the evidence or of the merits of the case; and (3) the prosecution moved to amend the information so as to allege that the defendants
question to be passed upon by the appellate court is purely legal; so that profited from the effects of the crime. In view of this development, counsel for the
defendants moved to withdraw their motion to quash, and objected to the proposed

98
amendment which sought to change materially the information after plea without where the provincial fiscal filed an information for "illegal possession and use of false
the consent of the accused. Without acting on the petition to withdraw the motion to treasury or bank notes." Upon arraignment the defendant pleaded not guilty.
quash, the trial court denied the motion of the prosecution on the ground that the Subsequently, the defense filed a motion to dismiss on the thesis that there had
proposed amendment would substantially affect the fundamental rights of the been no preliminary investigation of the charge of illegal possession and use of false
accused who were exempt from liability under the information because of their treasury or bank notes, and that the absence of such preliminary investigation
relation to the principal culprit. Then the prosecution moved for the dismissal of the affected the jurisdiction of the trial court. The motion was granted on the ground
case against the alleged accessories with reservation to file a new information. The that the waiver made by the defendant in the justice of the peace court did not
court ordered the dismissal without ruling on the reservation. Subsequently, a new deprive her of the right to a preliminary investigation of an entirely different crime.
information was filed virtually reproducing the previous one except that now there On appeal to this Court, it was held that the dismissal was erroneous because the
was an added allegation of intent to gain. The lower court quashed the new allegations of the information filed in the Court of First Instance were included in
information upon motion of the accused on the ground of double jeopardy. On appeal those of the complaint filed in the justice of the peace court where the defendant
by the prosecution, this Court, thru Mr. Justice J. B. L. Reyes, held that the plea of had already waived her right to a preliminary investigation. On the question of
double jeopardy was erroneously sustained because whether the appeal placed the defendant in double jeopardy, this Court, thru Mr.
Chief Justice (then Associate Justice) Concepcion, observed that the situation of
In the first place, the accused-appellees herein filed a motion to quash on Casiano was identical to that of the accused in Acierto
the ground that they incurred no criminal liability under the facts alleged in
the information in the preceding case, No. Q-972, and the trial court instead ... were she to plead double jeopardy in this case, for such plea would
of allowing the withdrawal of the motion to quash, virtually sustained the require the assertion of jurisdiction of the court of first instance to try her
same when it denied the fiscal's motion to amend, thereby forcing the and that the same erred in yielding to her plea therein for lack of authority
latter to dismiss the case; hence, it can not be held that the former case therefor. In the language of our decision in the Acierto case, it is immaterial
was terminated without the express consent of the accused. Secondly, the whether or not the court a quohad said authority. It, likewise, makes no
defendants themselves showed that the information in the previous case difference whether or not the issue raised by defendant in the lower court
was insufficient to charge them with any criminal offense, in view of their affected its jurisdiction. The fact is that she contested its jurisdiction and
relationship with the principal accused; and it is well established doctrine that, although such pretense was erroneous, she led the court to believe
that for jeopardy to attach, there must be an information sufficient in form that it was correct and to act in accordance with such belief. The
and substance to sustain a conviction. Lastly, the herein accused having elementary principles of fair dealing and good faith demand, accordingly,
successfully contended that the information in the former case was that she be estopped now from taking the opposite stand in order to pave
insufficient to sustain a conviction, they cannot turn around now and claim the way for a plea of double jeopardy, unless the rule of estoppel laid down
that such information was after all, sufficient and did place them in danger in the Acierto case is revoked. As a matter of fact, said rule applies with
of jeopardy of being convicted thereunder. If, as they formerly contended, greater force to the case at bar than to the Acierto case, because the same
no conviction could be had in the previous case, they are in estoppel to involved two (2) separate proceedings before courts deriving their authority
contend now that the information in the second case places them in from different sovereignties, whereas the appeal in the case at bar is
jeopardy for the second time. Their case comes within the spirit of the rule a continuation of the proceedings in the lower court, which like this
laid down in People vs. Acierto. Supreme Court, is a creature of the same sovereignty. In short the
inconsistency and impropriety would be more patent and glaring in this
Again, in People vs. Reyes, et al., supra, this Court, speaking thru Mr. Chief Justice case than in that of Acierto, if appellant herein pleaded double jeopardy in
Paras, reiterated the Aciertoruling thus: this instance.
Where the complaint or information is in truth valid and sufficient, but the This Court then forthnightly stated that "the rule of estoppel applied in the Acierto
case is dismissed upon the petition of the accused on the ground that the case should be maintained, because:
complaint or information is invalid and insufficient, such dismissal will not
bar another prosecution for the same offense and the defendant is 1. It is basically and fundamentally sound and just.
estopped from alleging in the second information that the former dismissal
was wrong because the complaint or information was valid. 2. It is in conformity with the principles of legal ethics, which demand good
faith of the higher order in the practice of law.
In this particular case, upon motion of the defendants, the trial court dismissed the
information because it did not allege the use of violence, notwithstanding the fact 3. It is well settled that parties to a judicial proceeding may not, on appeal,
that the offense charged was coercion under article 287 of the Revised Penal Code. adopt a theory inconsistent with that which they sustained in the lower
On appeal, however, this Court ruled that the dismissal was erroneous because court.
"although the offense named in the information is coercion, it does not necessarily xxx xxx xxx
follow that the applicable provision is the first paragraph, since the second
paragraph also speaks of 'coercions'. Inasmuch as the recitals in the information do 4. The operation of the principle of estoppel on the question of jurisdiction
not include violence, the inevitable conclusion is that the coercion contemplated is seemingly depends whether the lower court actually had jurisdiction or not.
that described and penalized in the second paragraph." If it had no jurisdiction, but the case was tried and decided upon the theory
that it had jurisdiction, the parties are not barred on appeal, from assailing
We come now to the case of People vs. Casiano. In this case the accused was such jurisdiction, for the same "must exist as a matter of law, and may not
charged with estafa in a complaint filed with the justice of the peace court of be conferred by consent of the parties or by estoppel" (5 C.J.S. 861-863).
Rosales, Pangasinan. The accused waived her right to preliminary investigation and However, if the lower court had jurisdiction, and the case was heard and
the record was accordingly forwarded to the Court of First Instance of Pangasinan decided upon a given theory, such, for instance, as that the court had no

99
jurisdiction, the party who induced it to adopt such theory will not be dismissal was with the express consent of the accused because it was granted upon
permitted, on appeal, to assume an inconsistent position that the lower his instigation thru a motion to dismiss was not passed upon in Bangalao.
court hadjurisdiction. Here, the principle of estoppel applies. The rule that
jurisdiction is conferred by law, and does not depend upon the will of the A case of striking factual resemblance with Salico is People vs. Ferrer (100 Phil. 124,
parties, has no bearing thereon. October 23, 1956). In this case, after the prosecution had rested, the accused filed a
motion to dismiss on the ground that the territorial jurisdiction of the trial court had
Twelve days after Casiano, this Court, in People vs. Archilla, supra, invoked anew the not been published. Acting on this motion, the lower court dismissed the case. The
doctrine of estoppel. In this case Alfreda Roberts, together with Jose Archilla, was prosecution appealed. This Court found that the evidence on record, contrary to the
charged with bigamy. After pleading not guilty, Roberts, through his counsel, filed a finding of the trial court, amply proved the jurisdiction of the lower tribunal.
motion praying that the complaint be quashed with regard to her on the ground that However, without the defendant interposing the plea of double jeopardy, this Court
the facts alleged therein did not constitute the offense charged for failure to aver held that "the Government however meritorious its case cannot appeal the order of
that "insofar as Alfreda Roberts is concerned, her marriage to Jose Luis Archilla was dismissal without violating the right of the defendant not to be placed in double
her second marriage ..." On appeal, the prosecution contended that the trial court jeopardy." Again, like in Bangalao, this Court did not consider the nature of dismissal
erred in granting the motion to quash, because the complaint was sufficient and at whether it was with or without the express consent of the defendant.
least charged the accused as an accomplice. The defendant maintained that even if
that were true, the quashing of the information amounted to her acquittal which The accused in the case at bar avers that the Salico doctrine
prevented the prosecution from taking the said appeal as it would place her in was formally and expressly abandoned in People vs. Labatete, supra. In the latter
double jeopardy. Mr. Justice Felix Bautista Angelo, writing for the majority, ruled that case, the trial court, upon motion of the defendant, dismissed the original
the trial court erred, and proceeded to emphasize that the accused information for estafa on the ground that it did not allege facts constituting the
offense charged. The information recited that the accused had contracted a loan
... cannot now be allowed to invoke the plea of double jeopardy after from the complainant, giving as security the improvements and products of his
inducing the trial court to commit an error which otherwise it would not property (a piece of land), without averring that the said property, which was
have committed. In other words, appellee can not adopt a posture of double allegedly mortgaged by the accused to the Rehabilitation Finance Corporation,
dealing without running afoul with the doctrine of estoppel. It is well-settled formed part of the security. Consequently, the fiscal filed an amended complaint
that the parties to a justiciable proceeding may not, on appeal, adopt a alleging that the accused also gave as security the land in question, which he later
theory inconsistent with that which they sustained in the lower court mortgaged to the damage and prejudice of the complaining creditor. This amended
(Williams v. McMicking, 17 Phil. 408; Molina v. Somes, etc.). Consequently, information was also dismissed upon motion of the defendant on the ground of
appellee is now estopped from invoking the plea of double jeopardy upon double jeopardy. This Court, in sustaining the appealed order of dismissal, held:
the theory that she would still be convicted under an information which she
branded to be insufficient in the lower court. If the amended information were to be admitted, the accused will be
deprived of his defense of double jeopardy because by the amended
The accused in this case now before us nevertheless insists that the Salico doctrine information he is sought to be made responsible for the same act of
and "necessarily analogous doctrines" were abandoned by this Court borrowing on a mortgage for which he had already begun to be tried and
in Bangalao, Labatete, Villarin and Cloribel. acquitted by the dismissal of the original information.
In Bangalao, the complaint filed by the victim's mother alleged that the rape was xxx xxx xxx
committed "by means of force and intimidation" while the information filed by the
fiscal alleged that the offended party was a "minor and demented girl" and that the ... the trial court found that the accused could not be found guilty of any
defendants "successively had sexual intercourse with her by means of force and offense under the information. The judgment entered was not one of
against the will of Rosita Palban." After the accused had pleaded not guilty, the dismissal but of acquittal, and whether the judgment is correct or incorrect,
defense counsel moved for the dismissal of the case on the ground that the trial the same constitutes a bar to the presentation of the amended information
court lacked jurisdiction to try the offense of rape charged by the fiscal since it was sought to be introduced by the fiscal. (Emphasis supplied)
distinct from the one alleged in the complaint which did not aver that the victim was In not applying the Salico doctrine, this Court, through Mr. Justice Alejo Labrador,
a demented girl". The lower court sustained the motion and dismissed the case for expounded:
lack of jurisdiction. On appeal by the prosecution, this Court held that the trial judge
erred in dismissing the case for lack of jurisdiction, but ruled, however, that the ... The judgment of the trial court (in People vs. Salico) was in fact
appeal could not prosper because it placed the accused in double jeopardy. an acquittal because of the failure on the part of the fiscal to prove that the
crime was committed within the jurisdiction of the court. The judgment was
As the court below had jurisdiction to try the case upon the filing of the in fact a final judgment of acquittal. The mere fact that the accused asked
complaint by the mother of the offended party, the defendants-appellees for his acquittal after trial on the merits (after the prosecution had rested its
would be placed in double jeopardy if the appeal is allowed. case) is no reason for saying that the case was "dismissed" with his express
After mature analysis, we cannot agree that this Court in Bangalao impliedly consent and he may again be subjected to another prosecution.
abandoned the Salico doctrine on waiver. Bangalao was decided solely on the From the above named statement, it is clear that what in Salico was repudiated
question of jurisdiction. This Court, however, after holding that the lower tribunal in Labatete was the premise that the dismissal therein was not on the merits
had jurisdiction, decided outright to repress the appeal by the Government on the and not the conclusion that a dismissal, other than on the merits, sought by the
ground of double jeopardy without considering whether the appealed order of accused, is deemed to be with his express consent and therefore constitutes a
dismissal was issued with or without the express consent of the accused (this aspect waiver of his right to plead double jeopardy in the event of an appeal by the
of double jeopardy not being in issue). Hence, the ruling in Salico that the prosecution or a second indictment for the same offense. This Court, in Labatete,

100
merely pointed out that the controverted dismissal in Salico was in fact an As a final citation in support of his theory, the accused in the case at bar
acquittal." Reasoninga contrario, had the dismissal not amounted to acquittal, then invokes People vs. Clolibel, supra, where this Court, in sustaining the plea of double
the doctrine of waiver would have applied and prevailed. As a matter of fact we jeopardy interposed by the defendants, stated inter alia:
believe with the majority in Salico that the dismissal therein was not on the merits
and therefore did not amount to an acquittal: In asserting that Criminal Case No. 45717 may still be reinstated, the
petitioner adopts the ruling once followed by the Court to the effect that a
If the prosecution fails to prove that the offense was committed within the dismissal upon the defendant's own motion is a dismissal consented to by
territorial jurisdiction of the court and the case is dismissed, the dismissal is him and, consequently, will not be a bar to another prosecution for the
not an acquittal, inasmuch as if it were so the defendant could not be again same offense, because, his action in having the case dismissed constitutes
prosecuted for the same offense before a court of competent jurisdiction; a waiver of his constitutional right or privilege, for the reason that he
and it is elemental that in such case the defendant may again be thereby prevents the court from proceeding to the trial on the merits and
prosecuted for the same offense before a court of competent jurisdiction. rendering a judgment of conviction against him. (People v. Salico, 84 Phil.
772) But, this authority has long been abandoned and the ruling therein
Granting, however, that the Salico doctrine was abandoned in Labatete, it was expressly repudiated.
resurrected in Desalisa. Moreover,Labatete never mentioned the doctrine of
estoppel enunciated in Acierto which had been repeatedly reaffirmed. Thus, in the case of People v. Robles, G.R. No. L-12761, June 29, 1959, citing
People v. Bangalao, L-5610, February 17, 1954; People v. Diaz, L-6518,
To bolster his contention that the Salico doctrine has been dropped from the corpus March 30, 1954; People v. Abano, L-7862, May 17, 1955; and People v.
of our jurisprudence, the accused cites People vs. Villarin, supra. Here the accused Ferrer, L-9072, October 23, 1956, We said:
appealed to the Court of First instance his conviction in the inferior court for acts of
lasciviousness with consent. After conducting the preliminary investigation, the fiscal ... In reaching the above conclusion, this Court has not overlooked
charged the accused with corruption of minors. Villarin pleaded not guilty, and the ruling in People vs. Salico, 47 O.G. 4765, to the effect that a
before the case could be heard, his counsel filed a motion to dismiss on the ground dismissal upon defendant's motion will not be a bar to another
that the information did not allege facts constituting the crime charged. Acting on prosecution for the same offense as said dismissal was not without
this motion, the trial court dismissed the case. On appeal by the prosecution, this the express consent of the defendant, which ruling the prosecution
Court thru Mr. Justice Felix Angelo Bautista, held that the dismissal was erroneous, now invokes in support of its appeal; but said ruling is not now
but that this error controlling, having been modified or abandoned in subsequent
cases wherein this Court sustained the theory of double jeopardy
... cannot now be remedied by setting aside the order dismissal of the despite the fact that dismissal was secured upon motion of the
court a quo and by remanding the case to it for further proceedings as now accused. (Emphasis supplied)
suggested by the prosecution considering that the case was dismissed
without the express consent of the accused even if it was upon the motion Also, the rule that a dismissal upon defendant's motion will not be a bar to
of his counsel, for to do so would place the accused in double jeopardy. The another prosecution for the same offense as said dismissal is not without
only exception to the rule on the matter is when the dismissal is with the the express consent of the defendant, has no application to a case where
consent of the accused, and here this consent has not been obtained. the dismissal, as here, is predicated on the right of a defendant to a speedy
(Emphasis supplied) trial. (People vs. Tacneng, et al., G.R. No. L-12082, April 30, 1959).
(emphasis supplied)
Villarin gives the impression, as gleaned from the above statement, that this Court
therein sustained the plea of double jeopardy on the ground that dismissal was The above statements must be taken in the proper context and perspective. As
without the express consent of the defendant as it was ordered "upon the motion of previously explained, Bangalao, Ferrer, and even Labatete, did not actually abandon
his counsel" and not upon motion of the defendant himself. This conclusion is rather the doctrine of waiver in Salico (and not one of the said cases even implied the
unfortunate and must be rectified, for the settled rule is that the acts of counsel in a slightest departure from the doctrine of estoppel established in Acierto). In Diaz,
criminal prosecution bind his client. Thus, inPeople vs. Romero (89 Phil. 672, July 31, Abao, Tacnengand Robles which are cited above, like in Cloribel, the dismissals
1951), this Court held categorically that therein, all sought by the defendants, were considered acquittals because they were
all predicated on the right of a defendant to a speedy trial and on the failure of the
The fact that the counsel for the defendant, and not the defendant himself Government to prosecute. Therefore, even if such dismissals were induced by the
personally moved for the dismissal of the case against him, had the same accused, the doctrines of waiver and estoppel were obviously inapplicable for these
effect as if the defendant had personally moved for such dismissal, doctrines presuppose a dismissal not amounting to an acquittal.
inasmuch as the act of the counsel in the prosecution of the defendant's
cases was the act of the defendant himself , for the only case in which the This Court, through Mr. Justice Marceliano Montemayor, held in People vs. Diaz (94
defendant cannot be represented by his counsel is in pleading guilty Phil. 714, March 30, 1954):
according to Section 3, Rule 114, of the Rules of Court. (Emphasis supplied)
Here the prosecution was not even present on the day of trial so as to be in
On this consideration alone, we cannot agree with the accused in the case at bar a position to proceed with the presentation of evidence to prove the guilt of
that this Court in Villarin intended to abandon the Salico ruling. Had the motion to the accused. The case was set for hearing twice and the prosecution
dismiss filed by Villarin's counsel been considered as one made by the defendant without asking for postponement or giving any explanation, just failed to
himself, as should have been done, the Villarin case should have been resolved appear. So the dismissal of the case, though at the instance of defendant
consistent with the doctrine of waiver in Salico and/or that of estoppel in Acierto. Diaz may, according to what we said in the Gandicela case, be regarded as
an acquittal. (emphasis supplied)

101
A similar result was reached by this Court thru Mr. Justice Sabino Padilla, in People In Cloribel, the case dragged for three years and eleven months, that is, from
vs. Abano (97 Phil. 28, May 27, 1955), in this wise: September 27, 1958 when the information was filed to August 15, 1962 when it was
called for trial, after numerous postponements, mostly at the instance of the
After a perusal of the documents attached to the petition for a writ of prosecution. On the latter date, the prosecution failed to appear for trial, and upon
certiorari, we fail to find an abuse of discretion committed by the motion of defendants, the case was dismissed. This Court held "that the dismissal
respondent judge. He took pains to inquire about the nature of the ailment here complained of was not truly a 'dismissal' but an acquittal. For it was entered
from which the complaining witness claimed she was suffering. He upon the defendants' insistence on their constitutional right to speedy trial and by
continued the trial three times, to wit: on 27 May, 1 and 12 June. The reason of the prosecution's failure to appear on the date of trial." (Emphasis
defendant was entitled to a speedy trial. When on 15 June, the last day set supplied.)
for the resumption of the trial, the prosecution failed to secure the
continuance thereof and could not produce further evidence because of the Considering the factual setting in the case at bar, it is clear that there is no
absence of the complaining witness, the respondent judge was justified in parallelism between Cloribel and the cases cited therein, on the one hand, and the
dismissing the case upon motion of the defense ... The defendant was instant case, on the other. Here the controverted dismissal was predicated on the
placed in jeopardy for the offense charged in the information and the erroneous contention of the accused that the complaint was defective and such
annulment or setting aside of the order of dismissal would place him twice infirmity affected the jurisdiction of the court a quo, and not on the right of the
in jeopardy of punishment for the same offense. (emphasis supplied) accused to a speedy trial and the failure of the Government to prosecute. The
appealed order of dismissal in this case now under consideration did not terminate
Then in People vs. Tacneng (L-12082, April 30, 1959), Mr. Justice Pastor Endencia, the action on the merits, whereas in Cloribel and in the other related cases the
speaking for a unanimous Court, stressed that dismissal amounted to an acquittal because the failure to prosecute presupposed
... when criminal case No. 1793 was called for hearing for the third time and that the Government did not have a case against the accused, who, in the first
the fiscal was not ready to enter into trial due to the absence of his place, is presumed innocent.
witnesses, the herein appellees had the right to object to any further The application of the sister doctrines of waiver and estoppel requires two sine qua
postponement and to ask for the dismissal of the case by reason of their non conditions: first, the dismissal must be sought or induced by the defendant
constitutional right to a speedy trial; and if pursuant to that objection and personally or through his counsel; and second, such dismissal must not be on the
petition for dismissal the case was dismissed, such dismissal ammounted to merits and must not necessarily amount to an acquittal. Indubitably, the case at bar
an acquittal of the herein appellees which can be invoked, as they did, in a falls squarely within the periphery of the said doctrines which have been preserved
second prosecution for the same offense. (emphasis supplied) unimpaired in the corpus of our jurisprudence.
And this Court proceeded to distinguish the case from People vs. Salico, thus: ACCORDINGLY, the order appealed from is set aside. This case is hereby remanded
We are fully aware that pursuant to our ruling in the case of Peo. v. Salico, to the court of origin for further proceedings in accordance with law. No costs.
45 O.G. No. 4, 1765-1776, and later reiterated in Peo vs. Romero, L-4517- Concepcion, C.J., Reyes, J.B.L., Dizon, Makalintal, Zaldivar, Sanchez, Angeles and
20, July 31, 1951, a dismissal upon defendant's motion will not be a bar to Fernando, JJ., concur.
another prosecution for the same offense as said dismissal was not without
the express consent of the defendant. This ruling, however, has no
application to the instant case, since the dismissal in those cases was not
predicated, as in the case at bar, on the right of a defendant to a speedy
trial, but on different grounds. In the Salico case, the dismissal was based
on the ground that the evidence for the prosecution did not show that the
crime was committed within the territorial jurisdiction of the court which, on
appeal, we found that it was, so the case was remanded for further
proceedings; and in the Romero case the dismissal was due to the non-
production of other important witnesses by the prosecution on a date fixed
by the court and under the understanding that no further postponement at
the instance of the government would be entertained. In both cases, the
right of a defendant to a speedy trial was never put in issue. (emphasis
supplied)
The gravamen of the foregoing decisions was reiterated in People vs. Robles (L-
12761, June 29, 1959) where the trial court, upon motion of the defendant,
dismissed the case on the ground that the failure of the prosecution to present its
evidence despite several postponements granted at its instance, denied the accused
a speedy trial. In rejecting the appeal of the Government, this Court held:
In the circumstances, we find no alternative than to hold that the dismissal
of Criminal Case No. 11065 is not provisional in character but one which is
tantamount to acquittal that would bar further prosecution of the accused
for the same offense.

102
MELO V. PEOPLE, 85 PHIL. 766 (1950) It must be noticed that the protection of the Constitution inhibition is against a
second jeopardy for the same offense, the only exception being, as stated in the
G.R. No. L-3580 March 22, 1950 same Constitution, that "if an act is punished by a law and an ordinance, conviction
or acquittal under either shall constitute a bar to another prosecution for the same
CONRADO CARMELO, petitioner-appellant, act." The phrase same offense, under the general rule, has always been construed
vs. to mean not only the second offense charged is exactly the same as the one alleged
THE PEOPLE OF THE PHILIPPINES and THE COURT OF FIRST INSTANCE OF in the first information, but also that the two offenses are identical. There is identity
RIZAL, respondent-appellees. between the two offenses when the evidence to support a conviction for one offense
would be sufficient to warrant a conviction for the other. This so called "same-
Jose A. Fojas for petitioner. evidence test" which was found to be vague and deficient, was restated by the Rules
First Assistant Solicitor General Roberto A. Gianzon and Solicitor Martiniano P. Vivo of Court in a clearer and more accurate form. Under said Rules there is identity
for respondents. between two offenses not only when the second offense is exactly the same as the
MORAN, C.J.: first, but also when the second offense is an attempt to commit the first or a
frustration thereof, or when it necessary includes or is necessarily included in the
Petitioner Conrado Melo was charged in the Court of First Instance of Rizal, on offense charged in the first information. (Rule 113, sec. 9; U.S. vs. Lim Suco, 11 Phil.,
December 27, 1949, with frustrated homicide, for having allegedly inflicted upon 484; U. S. vs. Ledesma, 29 Phil., vs. Martinez, 55 Phil., 6.) In this connection, an
Benjamin Obillo, with a kitchen knife and with intent to kill, several serious wounds offense may be said to necessarily include another when some of the essential
on different parts of the body, requiring medical attendance for a period of more ingredients of the former as alleged in the information constitute the latter. And vice-
than 30 days, and incapacitating him from performing his habitual labor for the versa, an offense may be said to be necessarily included in another when all the
same period of time. On December 29, 1949, at eight o'clock in the morning, the ingredients of the former constitute a part of the elements constituting the latter
accused pleaded not guilty to the offense charged, and at 10:15 in the evening of (Rule 116, sec. 5.) In other words, on who has been charged with an offense cannot
the same day Benjamin Obillo died from his wounds. Evidence of death was be again charged with the same or identical offense though the latter be lesser or
available to the prosecution only on January 3, 1950, and on the following day, greater than the former. "As the Government cannot be with the highest, and then
January 4, 1950, an amended information was filed charging the accused with go down step to step, bringing the man into jeopardy for every dereliction included
consummated homicide. The accused filed a motion to quash the amended therein, neither can it begin with the lowest and ascend to the highest with precisely
information alleging double jeopardy, motion that was denied by the respondent the same result." (People vs. Cox, 107 Mich., 435, quoted with approval in U. S. vs.
court; hence, the instant petition for prohibition to enjoin the respondent court from Lim Suco, 11 Phil., 484; see also U. S. vs. Ledesma, 29 Phil., 431 and People vs.
further entertaining the amended information. Martinez, 55 Phil., 6, 10.)
Brushing aside technicalities of procedure and going into the substance of the issues This rule of identity does not apply, however when the second offense was not in
raised, it may readily be stated that amended information was rightly allowed to existence at the time of the first prosecution, for the simple reason that in such case
stand. Rule 106, section 13, 2d paragraph, is as follows: there is no possibility for the accused, during the first prosecution, to be convicted
for an offense that was then inexistent. Thus, where the accused was charged with
If it appears at may time before the judgment that a mistake has been physical injuries and after conviction the injured person dies, the charge for
made in charging the proper offense, the court may dismiss the original homicide against the same accused does not put him twice in jeopardy. This is the
complaint or information and order the filing of a new one charging the ruling laid down by the Supreme Court of the United States in the Philippines case of
proper offense, provided the defendant would not be placed thereby in Diaz vs. U. S., 223 U. S. 442, followed by this Court in People vs. Espino, G. R. No.
double jeopardy, and may also require the witnesses to give bail for their 46123, 69 Phil., 471, and these two cases are similar to the instant case. Stating it in
appearance at the trial. another form, the rule is that "where after the first prosecution a new fact
Under this provision, it was proper for the court to dismiss the first information and supervenes for which the defendant is responsible, which changes the character of
order the filing of a new one for the treason that the proper offense was not charged the offense and, together with the fact existing at the time, constitutes a new and
in the former and the latter did not place the accused in a second jeopardy for the distinct offense" (15 Am. Jur., 66), the accused cannot be said to be in second
same or identical offense. jeopardy if indicated for the new offense.

"No person shall be twice put in jeopardy of punishment for the same offense," This is the meaning of "double jeopardy" as intended by our constitution for was the
according to article III, section 1 (20) of our constitution. The rule of double jeopardy one prevailing in jurisdiction at the time the Constitution was promulgated, and no
had a settled meaning in this jurisdiction at the time our Constitution was other meaning could have been intended by our Rules of Court.
promulgated. It meant that when a person is charged with an offense and the case is Accordingly, an offense may be said to necessarily include or to be necessarily
terminated either by acquittal or conviction or in any other manner without the included in another offense, for the purpose of determining the existence of double
consent of the accused, the latter cannot again be charged with the same or jeopardy, when both offenses were in existence during the pendency of the first
identical offense. This principle is founded upon the law of reason, justice and prosecution, for otherwise, if the second offense was then inexistence, no jeopardy
conscience. It is embodied in the maxim of the civil law non bis in idem, in the could attach therefor during the first prosecution, and consequently a subsequent
common law of England, and undoubtedly in every system of jurisprudence, and charge for the same cannot constitute second jeopardy. By the very nature of things
instead of having specific origin it simply always existed. It found expression in the there can be no double jeopardy under such circumstance, and our Rules of Court
Spanish Law and in the Constitution of the United States and is now embodied in our cannot be construed to recognize the existence of a condition where such condition
own Constitution as one of the fundamental rights of the citizen. in reality does not exist. General terms of a statute or regulation should be so limited
in their application as not to lead to injustice, oppression, or an absurd consequence.

103
It will always, therefore, be presumed that exceptions have been intended to their
language which would avoid results of this character. (In re Allen, 2 Phil., 641.)
When the Rules of Court were drafted, there was absolutely no intention of
abandoning the ruling laid down in the Diaz case, and the proof of this is that
although the said Rules were approved on December 1939, yet on January 30, 1940,
this Court decided the Espino case reiterating therein the Diaz doctrine. Had that
doctrine been abandoned deliberately by the Rules of Court as being unwise, unjust
or obnoxious, logically it would have likewise been repudiated in the Espino case by
reason if consistency and as a matter of justice to the accused, who should in
consequence have been acquitted instead of being sentenced to a heavy penalty
upon the basis of a doctrine that had already been found to be wrong. There was
absolutely no reason to preclude this Court from repealing the doctrine in the Espino
case, for as a mere doctrine it could be repealed at any time in the decision of any
case where it is invoked, is a clear proof that the mind of the Court, even after the
approval of the Rules, was not against but in favor of said doctrine.
For these reasons we expressly repeal the ruling laid down in People vs. Tarok, 73
Phil., 260, as followed in People vs. Villasis, 46 Off. Gaz. (Supp. to No. 1), p. 268.
Such ruling is not only contrary to the real meaning of "double jeopardy" as intended
by the Constitution and by the Rules of Court but is also obnoxious to the
administration of justice. If, in obedience to the mandate of the law, the prosecuting
officer files an information within six hours after the accused is arrested, and the
accused claiming his constitutional right to a speedy trial is immediately arraigned,
and later on new fact supervenes which, together with the facts existing at the time,
constitutes a more serious offense, under the Tarok ruling, no way is open by which
the accused may be penalized in proportion to the enormity of his guilt.
Furthermore, such a ruling may open the way to suspicions or charges of conclusion
between the prosecuting officers and the accused, to the grave detriment of public
interest and confidence in the administration of justice, which cannot happen under
the Diaz ruling.
Before closing, it is well to observe that when a person who has already suffered his
penalty for an offense, is charged with a new and greater offense under the Diaz
doctrine herein reiterated, said penalty may be credited to him in case of conviction
for the second offense.
For all the foregoing, the petition is denied, and the respondent court may proceed
to the trial of the criminal case under the amended information. Without costs.
Ozaeta, Pablo, Padilla, Tuason, Montemayor and Reyes, JJ., concur.

104
PEOPLE V. YORAC, 42 SCRA 230 (1971) As succinctly set forth in the brief of the People of the Philippines: "The sole issue in
this case is whether the defendant, who had already been convicted of slight
G.R. No. L-29270 November 23, 1971 physical injuries before the City Court of Bacolod for injuries inflicted upon Lam
Hock, and had served sentence therefore, may be prosecuted anew for frustrated
THE PEOPLE OF THE PHILIPPINES, plaintiff-appellant, murder for the same act committed against the same person." 7 The position taken
vs. by the appellant is in the affirmative but, as indicated at the outset, the controlling
RODRIGO YORAC, defendant-appellee. force of People v. Buling would preclude us from reversing the resolution of Judge
Alampay.
FERNANDO, J.:
1. The Constitution, to repeat, is quite explicit: "No person shall be twice put in
The constitutional right not to be put twice in jeopardy for the same offense 1 was jeopardy of punishment for the same offense. As Justice Laurel made clear in an
the basis for a motion to quash filed by the accused, now appellee, Rodrigo Yorac. address as delegate before the Constitutional Convention, such a provision finds its
He was prosecuted for frustrated murder arising allegedly from having assaulted, origin" from the days when sanguinary punishments were frequently resorted to by
attacked, and hit with a piece of wood the offended party, for which he had been despots." 9 A defendant in a criminal case should therefore, according to him, be
previously tried and sentenced for slight physical injuries, his plea being one of guilt. adjudged either guilty or not guilty and thereafter left alone in peace, in the latter
The later information for frustrated murder was based on a second medical case the State being precluded from taking an appeal. 10 It is in that sense that the
certificate after the lapse of one week from the former previously given by the same right against being twice put in jeopardy is considered as possessing many features
physician who, apparently, was much more thorough the second time, to the effect in common with the rule of finality in civil cases. For the accused is given assurance
that the victim did suffer a greater injury than was at first ascertained. The lower that the matter is closed, enabling him to plan his, future accordingly, protecting him
court, presided by the Honorable Judge Nestor B. Alampay, considering that there from continued distress, not to mention saving both him and the state from the
was no, supervening fact that would negate the defense of double jeopardy, expenses incident to redundant litigation. There is likewise the observation that this
sustained the motion to quash in an order of June 21, 1968. The People appealed. As constitutional guarantee helps to equalize the adversary capabilities of two grossly
the order complained of is, fully supported by the latest authoritative ruling of this mismatched litigants, a poor and impecunious defendant hardly in a position to keep
Tribunal, People v. Buling, 2 we have to affirm. on shouldering the costs of a suit.
In the brief for the People of the Philippines, it was shown that the accused Yorac Then, as a member of the Supreme Court, Justice Laurel had the first opportunity to
was charged with slight physical injuries before the City Court of Bacolod, the give meaning to what, under the Constitution, should be considered "the same
offended party being a certain Lam Hock who, according to the medical certificate offense." In the case of People v. Tarok, decided in 1941, 11 the then comparatively
issued in April 10, 1968 by a Dr. Rogelio Zulueta, a resident physician of the new Rules of Court in its Section 9 of Rule 113 speaks of a bar to another
Occidental Negros Provincial Hospital, was confined "since April 8, 1968 up to the prosecution for the offense charged after a defendant shall have been convicted or
present time for head injury." 3 Then came a plea of guilty by the accused on April acquitted or the case against him dismissed or otherwise terminated without his
16, 1968 resulting in his being penalized to suffer ten days of arresto menor. He express consent, "or for any attempt to commit the same or frustration thereof or
started serving his sentence forthwith. On April 18, 1968, the provincial fiscal filed for, any offense which necessarily includes or is necessarily included in the offense
an information, this time in the Court of First Instance of Negros Occidental, charging charged in the former complaint or information." 12
the same defendant with frustrated murder arising from the same act against the
aforesaid victim Lam Hock upon another medical certificate dated April 17, 1968 In the Tarok case, the conviction for parricide of the accused was sought to be set
issued by the same Dr. Zulueta. In the medical certificate of April 17, 1968, it was aside, as previously he had been indicted for the crime of serious physical injuries,
made to appear that the confinement of the offended party in the hospital was the to which he had pleaded guilty. He was sentenced and was actually incarcerated by
result of: "1. Contusion with lacerated wound 4 inches parieto-occipital region scalp virtue of such penalty imposed. The offended party was his wife whom he hacked
mid portion. 2. Cerebral concussion, moderately severe, secondary." 4Moreover, it with bolo, his ire being aroused by certain, remarks made her. While he was thus
further contained a statement that the X-ray finding did not yield any "radiographic serving sentence, the victim died resulting in the new prosecution for parricide of
evidence of fracture." The healing period barring complications, was declared to be which he was convicted. On appeal to this Court, it was decided over the dissents of
from eighteen to twenty-one days. 5 the then Justice Moran and Justice Diaz that the offense of serious physical injury of
which he was found guilty being included in parricide his previous conviction was a
Afterwards, a motion to quash was filed by the accused on June 10, 1968 on the bar to such subsequent prosecution for the more serious crime. The lower court
ground that, having been previously convicted of slight physical injuries by the City judgement of conviction was thus reversed. According to Justice Laurel who spoke
Court of Bacolod and having already served the penalty imposed on him for the very for the Court: "To our mind, the principle embodied in the New Rules of Court is a
same offense, the prosecution for frustrated murder arising out of the same act clear expression of selection of rule amidst conflicting theories. We take the position
committed against the same offended party, the crime of slight physical injuries that when we amended section 26 of General Orders No. 58 by providing that the
necessarily being included in that of frustrated murder, he would be placed in conviction or acquittal of the defendant or the dismissal of the case shall be a bar to
second jeopardy if indicted for the new offense. 6 In its well-reasoned resolution of another prosecution for any offense not only necessarily therein included but which
June 21, 1968 granting the motion to quash and ordering the dismissal of a criminal necessarily includes the offense charged in the former complaint or information, we
case for frustrated murder against the accused, Judge Alampay relied on People v. meant what we have, in plain language, stated. We certainly did not mean to
Buling which, in his opinion, was squarely applicable as "nothing in the later medical engage in the simple, play of words." 13
certificate [indicated] that a new or supervening fact had developed or arisen since
the time of the filing of the original action" against the accused. A motion for 2. Such a ruling was however re-examined and set aside in Melo v. People, 14 where
reconsideration being unavailing, an appeal was elevated to us. it was held that an accused who pleaded guilty to the offense of frustrated homicide,
the offended party thereafter dying in the evening of the same day, could not rely on
a plea of double jeopardy if, as a result thereof, the information was amended to

105
charge him with homicide. 15 As was clarified in the opinion of this Court through the to quash and to dismiss the information against appellee Yorac. No error could
then Chief Justice Moran, one of the dissenters in the Tarok case: "This rule of therefore be rightfully imputed to it.
identity does not apply, however, when the second offense was not in existence at
the time of the first prosecution, for the simple reason that in such case there is no WHEREFORE, the resolution of June 21, 1968 of Judge Nestor B. Alampay granting
possibility for the accused, during the first prosecution, to be convicted for an the motion to quash, ordering the dismissal of the case and the immediate release
offense that was then inexistent. Thus, where the accused was charged with of the appellee Rodrigo Yorac, is affirmed. Without costs.
physical injuries and after conviction the injured person dies, the charge for Concepcion, C.J., Reyes, J.B.L., Makalintal, Zaldivar, Castro, Teehankee, Villamor and
homicide against the same accused does not put him twice in jeopardy." 16 Stated Makasiar, JJ., concur.
differently, if after the first prosecution "a new fact supervenes on which defendant
may be held liable, resulting in altering the character of the crime and giving rise to Barredo, J., took no part.
a new and distinct offense, "the accused cannot be said to be in second jeopardy if
indicted for the new offense." 17 It is noteworthy, however, that in the Melo ruling,
there was a reiteration of what was so emphatically asserted by Justice Laurel in the
Tarok case in these words: "As the Government cannot begin with the highest, and
then down step by step, bringing the man into jeopardy for every dereliction
included therein, neither can it begin the lowest and ascend to the highest with
precisely the same result." 18
3. There is then the indispensable requirement of the existence of "a new fact
[which] supervenes for which the defendant is responsible" changing the character
of the crime imputed to him and together with the facts existing previously
constituting a new and distinct offense. The conclusion reached in People v.
Buling, 19 the latest case in point relied upon by Judge Alampay in the resolution no
appeal, was thus, predictable. As set forth in the opinion of Justice Labrador in the
case, there was a medical certification that the wounds for which the accused
Buenaventura as first prosecuted for less serious physical injuries would require
medical attendance from a period of from ten days to fifteen days. He pleaded guilty
and on December 8, 1956, sentenced by the Justice of the Peace of Cabalian Leyte,
to one month and one day of arresto mayor. He started serving his sentence on the
same day. On January 18, 1957, however, another physician examined the offended
party and with the use of an X-ray apparatus, certified that he did suffer a fracture
requiring a treatment of from one and one-half months to two and one half months,
barring complications. As a result, on February 20, 1957, an information was filed
against the same accused, this time before the Court of First Instance of Leyte,
charging him with serious physical injuries. He stood trial and was found guilty of
such an offense and sentenced to imprisonment of four months of arresto mayor as
minimum to one year of prision correccional as maximum. On appeal to this Court,
his invocation of the defense of double jeopardy struck a responsive chord, and he
was acquitted.
4. The opinion of Justice Labrador explained with clarity why the constitutional right
against being put twice in jeopardy was a bar to the second prosecution. Thus: "If
the X-ray examination discloses the existence of a fracture on January 17, 1957, that
fracture must have existed when the first examination was made on December 10,
1956. There is therefore, no view or supervening fact that could be said to have
developed or arisen since the filing of the original action, which would justify the
application of the ruling enunciated by us in the cases if Melo vs. People andPeople
vs. Manolong ... . We attribute the new finding of fracture, which evidently
lengthened the period of healing of the wound, to the very superficial and
inconclusive examination made on December 10, 1956. Had an X-ray examination
been taken at the time, the fracture would have certainly been disclosed. The wound
causing the delay in healing was already in existence at the time of the first
examination, but said delay was, caused by the very superficial examination then
made. As we have stated, we find therefore that no supervening fact had occurred
which justifies the application of the rule in the case of Melo vs. People and People
vs. Manolong for which reason we are constrained to apply the general rule of
double jeopardy." 20 It is quite apparent, in the light of the foregoing, why the lower
court, submitting to the compulsion of the Buling decision, had to sustain the motion

106
PEOPLE VS. DELA TORRE dela Torre in the amount of P50,000.00 as and by way of civil
damages.
[G.R. Nos. 137953-58. April 11, 2002]
5) In Crim. Case No. RTC 2183-I of the crime of Rape, defined and
PEOPLE OF THE PHILIPPINES, appellant, vs. WILFREDO DELA penalized under Article 335 of the Revised Penal Code, is sentenced to
TORRE, appellee. suffer the penalty of reclusion perpetua, and to indemnify Mary Rose
dela Torre in the amount of P50,000.00 as and by way of civil
damages.
DECISION
6) In Crim. Case No. RTC 2184-I of the crime of Rape, defined and
PANGANIBAN, J.: penalized under Article 335 of the Revised Penal Code, is sentenced to
suffer the penalty of reclusion perpetua, and to indemnify Mary Rose
The prosecution cannot appeal a decision in a criminal case whether to reverse dela Torre in the amount of P50,000.00 as and by way of civil
an acquittal or to increase the penalty imposed in a conviction. damages.[4]

The Case The two Amended Informations for acts of lasciviousness, dated July 1, 1997,
were similarly worded as follows:

The prosecution appeals the March 31, 1998 Decision [1] and June 3, 1998
Order[2] issued by the Regional Trial Court (RTC) of Iba, Zambales (Branch 69) [3] in That on or about the 30th day of September, 1996 at Brgy. Guisguis, municipality of
Criminal Cases Nos. 2179-I, 2180-I, 2181-I, 2182-I, 2183-I and 2184-I. The assailed Sta. Cruz, Province of Zambales, Philippines, and within the jurisdiction of this
Decision convicted Wilfredo Dela Torre of two counts of acts of lasciviousness and Honorable Court, the said accused, being the father of one Mary Rose de la Torre,
four counts of rape, while the challenged Order denied the Motion for actuated by lust and by means of coercion, threats, intimidation and other
Reconsideration filed by plaintiff (now appellant). consideration, did then and there wilfully, unlawfully and feloniously commit acts of
lasciviousness on the person of Mary Rose de la Torre, a minor of 11 years old, to the
damage and prejudice of the said Mary Rose de la Torre.[5]
The dispositive portion of the Decision is reproduced hereunder:
The other Information[6] charged appellee with the same crime against the
WHEREFORE, premises considered, accused Wilfredo dela Torre is found GUILTY same victim on a different date, October 10, 1996.
beyond reasonable doubt as follows:
On the other hand, the four Informations charging him with rape, dated July 1,
1) In Crim. Case No. RTC 2179-I of the crime of Acts of Lasciviousness, 1997, similarly read as follows:
defined and penalized under Article 336 of the Revised Penal Code, is
sentenced to suffer an imprisonment of six (6) months and one (1)
day to two (2) years of prision correccional, and to indemnify Mary That on or about the 18th day of October, 1996 at Brgy. Guisguis, municipality of
Rose dela Torre in the amount ofP10,000.00 as and by way of civil Sta. Cruz, Province of Zambales, Philippine[s], and within the jurisdiction of this
damages. Honorable Court, the said accused, being the father of one Mary Rose de la Torre,
with lewd design by means of coercion, threats, intimidation and other
consideration, did then and there wilfully, unlawfully and feloniously, have carnal
2) In Crim. Case No. RTC 2180-I of the crime of Acts of Lasciviousness, knowledge with one Mary Rose de la Torre, a minor of 11 years old, without her
defined and penalized under Article 336 of the Revised Penal Code, is consent and against her will, to the damage and prejudice of the latter. [7]
sentenced to suffer an imprisonment of six (6) months and one (1)
day to two (2) years of prision correccional, and to indemnify Mary
Rose dela Torre in the amount ofP10,000.00 as and by way of civil The three other Amended Informations recited the same allegations on
damages. different dates: November 1,[8] November 12[9] and December 23,[10] 1996.When
arraigned on August 13, 1997, appellee pleaded [11] not guilty[12] to all six (6)
Informations. After trial in due course, the RTC rendered the challenged Decision.
3) In Crim. Case No. RTC 2181-I of the crime of Rape, defined and
penalized under Article 335 of the Revised Penal Code, is sentenced to
suffer the penalty of reclusion perpetua, and to indemnify Mary Rose Appellee did not appeal, but the prosecution filed a Notice of Appeal [13] dated
dela Torre in the amount of P50,000.00 as and by way of civil June 9, 1998.
damages.
The Facts
4) In Crim. Case No. RTC 2182-I of the crime of Rape, defined and
penalized under Article 335 of the Revised Penal Code, is sentenced to Appellants Version
suffer the penalty of reclusion perpetua, and to indemnify Mary Rose

107
In its Brief,[14] the Office of the Solicitor General (OSG) presents the Felina Sobrevilla, teacher of Mary Rose, noticed sudden changes in her behavior and
prosecutions version of the facts as follows: when confronted, the latter admitted that she was sexually abused by her
father. Her head teacher informed her Aunt Elpidia Balindo about the sexual
abuses. They referred the case to the DSWD who took her under its custody.
Appellee Wilfredo dela Torre had a common-law relationship with Melinda dela
Torre. The latter gave birth to three children, Mary Rose, Mark Anthony, and Mark
Domil. When Mary Rose was about seven (7) years old, her mother left the conjugal Mary Rose testified that her father committed sexual abuses on her on the following
abode with Mark Domil, leaving her and sibling Mark Anthony in the care of appellee, dates: September 30, 1996, October 10, 1996, October 18, 1996, November 01,
who resided with his progeny in a one-room hut in Sitio Pao, Guis-guis, Sta. Cruz, 1996, November 12, 1996 and December 23, 1996.
Zambales.
Appellee, on the other hand denies vehemently the charges being imputed on him
Mary Rose and her brother Mark Anthony studied at the Guinabon Elementary by her daughter and said that the only reason he can think of why the daughter filed
School. She was the brightest in her class, even though because of their poverty, the charges is because he did not allow her to stay with her teacher, Mrs. Sobrevilla.
[18]
she had to walk from their hut to the school everyday. (Citations omitted)

In January of 1997, Felita Sobrevilla, noticed a sudden change in the behavior and Ruling of the Trial Court
performance of Mary Rose, who was twelve-year[s] old at th[at] time. The latter
appeared sleepy, hungry and snobbish. She also urinated on her panty. When
The RTC ruled that it was duly established that accused Wilfredo committed
confronted by Generosa Mayo, the head teacher, Mary Rose admitted to her that she
acts of lasciviousness against Mary Rose on 30 September 1996 and 10 October
was abused repeatedly by appellee. Mayo informed Elpidia Balindo, the aunt of Mary
1996, and had carnal knowledge [of] Mary Rose on 18 October 1996, 01 November
Rose, about the abuses. They then decided to refer the matter to the Department of
1996, 12 November 1996 and 23 December 1996. [19] Further, the trial court added
Social Welfare and Development (DSWD), who took Mary Rose under its custody.
that the moral ascendancy of appellee over the victim was equivalent to
intimidation. It did not give any probative value to his uncorroborated and
It turned out that on September 30, 1996, Mary Rose was about to sleep when unsubstantiated defenses of denial and alibi.
appellee told her, anak puwede ba nating subukan? She did not understand what
that meant and continued to sleep. Appellee then placed himself on top of Mary
However, the court a quo refused to impose the supreme penalty of death on
Rose. After removing her shorts as well as his shorts, he poked his penis into her
appellee. It maintained that there were circumstances that mitigated the gravity of
organ. He also kissed and embraced Mary Rose, who just wept. The same incident
the offenses, as follows:
was repeated in the evening of October 10, 1996.

1. As testified to (supra) there was absence of any actual, physical


In the evening of October 18, 1996, appellee was able to insert his penis into the
violence or intimidation in the commission of the acts complained of.
vagina of Mary Rose. After the act, her whole body ached. She started to fear
appellee. He also had sexual intercourse with his minor daughter on three more
occasions, that is, on November 1 and 12 and December 23, 1996. xxxxxxxxx

A medical examination conducted by Dr. Milagrina Mayor, Rural Health Physician of 2. The abandonement by Melinda (common-law wife of accused Wilfredo
Sta. Cruz, Zambales, on Mary Rose revealed that her vagina admitted one finger and mother of Mary Rose) when Mary Rose was seven (7) years old
with ease. She was no longer a virgin. Her hymen was broken with healed leaving behind Wilfredo, Mary Rose and her brother, Mark Anthony.
lacerations at the 3:00, 6:00 and 9:00 nine oclock positions. The girl also suffered
from urinary tract infection.[15] (Citations omitted)
3. The extreme poverty on the life of Wilfredo, Mary Rose and Mark
Anthony.
Appellees Version
4. After the mother of Mary Rose left the conjugal home, for more than
On the other hand, appellees statement of facts, [16] as contained in his Brief, five (5) years, Wilfredo, Mary Rose and Mark Anthony were living
[17]
is reproduced as follows: together as a family and Mary Rose was never molested by her
father.
Appellee WILFREDO DELA TORRE had three (3) children with Melinda Torre, namely:
Mary Rose, Mark Anthony and Mark Ronnil. Melinda left her family when Mary Rose 5. There is reason to deprive Wilfredo of the love of her daughter Mary
was about seven (7) years old bringing with her Mark Ronnil. The victim lived with Rose but there is no reason to deprive Mark Anthony of the love of
her father and brother Mark Anthony in Sta. Cruz, Zambales. his father considering that both Mary Rose and Mark Anthony have
no one to call as a mother.[20]

Hence, this appeal.[21]

108
The Issue The Kepner doctrine was clarified in a 1987 case.[29] Speaking through Justice
Isagani A. Cruz, the Court explained that an appeal of the prosecution from a
judgment of acquittal (or for the purpose of increasing the penalty imposed upon the
In this appeal, the solicitor general assigns this single error for our
convict) would place him in double jeopardy. [30]
consideration:

Double jeopardy provides three related protections: (1) against a second


The Court a quo erred in penalizing appellee with reclusion perpetua in each of the
prosecution for the same offense after acquittal, (2) against a second prosecution for
four indictments for rape, instead of imposing the supreme penalty of death as
the same offense after conviction, and (3) against multiple punishments for the
mandated by R.A. No. 7659.[22]
same offense.[31]

The Courts Ruling


Although Kepner technically involved only a single proceeding, the Court
regarded the practice as equivalent to two separate trials, and the evil that the
The appeal has no merit. Court saw in the procedure was plainly that of multiple prosecution. [32]

Lone Issue: The ban on double jeopardy is deeply rooted in jurisprudence. The doctrine has
several avowed purposes. Primarily, it prevents the State from using its criminal
processes as an instrument of harassment to wear out the accused by a multitude of
Propriety of Appeal by the Prosecution
cases with accumulated trials. [33] It also serves the additional purpose of precluding
the State, following an acquittal, from successively retrying the defendant in the
The prosecution asks this Court to modify the RTC Decision by imposing the hope of securing a conviction. [34] And finally, it prevents the State, following
supreme penalty of death on the accused. It argues that it has proven that the conviction, from retrying the defendant again in the hope of securing a greater
victim is the daughter of the accused, and that she was below eighteen (18) years penalty.[35]
old when the rapes took place. As a consequence, the trial court should have
imposed the penalty of death pursuant to Section 11 of RA 7659.[23]
Being violative of the right against double jeopardy, the instant appeal filed by
the prosecution cannot prosper. The rule is clear -- the prosecution cannot appeal on
Under Section 1, Rule 122 of the 2000 Rules of Criminal Procedure, any party the ground that the accused should have been given a more severe penalty. [36]
may appeal from a judgment or final order, unless the accused will be placed in
double jeopardy. This provision is substantially the same as that provided by the
Besides, even assuming that the penalties imposed by the RTC were erroneous,
1985 Rules.
these cannot be corrected by this Court on an appeal by the prosecution.Said the
Court:
The question now is whether an increase in the penalty imposed by the lower
court will violate the right of the accused against double jeopardy.
Whatever error may have been committed by the lower court was merely an error of
judgment and not of jurisdiction. It did not affect the intrinsic validity of the
In several cases, this Court has already definitively ruled on this decision. This is the kind of error that can no longer be rectified on appeal by the
issue. Recently, in People v. Leones,[24] it unmistakably declared that [w]hile it is true prosecution no matter how obvious the error may be. [37]
that this Court is the Court of last resort, there are allegations of error committed by
a lower court which we ought not to look into to uphold the right of the
The only way to nullify an acquittal or to increase the penalty is through a
accused. Such is the case in an appeal by the prosecution seeking to increase the
proper petition for certiorari to show grave abuse of discretion. This was explained
penalty imposed upon the accused for this runs afoul of the right of the accused
in People v. CA and Maquiling[38] as follows:
against double jeopardy.[25] It added:

While certiorari may be used to correct an abusive acquittal, the petitioner in such
This Court has not just once ruled that where the accused after conviction by the
extraordinary proceeding must clearly demonstrate that the lower court blatantly
trial court did not appeal his conviction, an appeal by the government seeking to
abused its authority to a point so grave as to deprive it of its very power to dispense
increase the penalty imposed by the trial court places the accused in double
justice. On the other hand, if the petition, regardless of its nomenclature, merely
jeopardy and should therefore be dismissed.[26]
calls for an ordinary review of the findings of the court a quo, the constitutional right
against double jeopardy would be violated. Such recourse is tantamount to
This doctrine was applied as early as 1904 in Kepner v. United converting the petition for certiorari into an appeal, contrary to the express
States[27] (hereinafter Kepner), as follows: injunction of the Constitution, the Rules of Court and prevailing jurisprudence on
double jeopardy.[39]
The Court of First Instance, having jurisdiction to try the question of the guilt or
innocence of the accused, found Kepner not guilty; to try him again upon the merits, WHEREFORE, the appeal is hereby DENIED. No pronouncement as to costs.
even in an appellate court, is to put him a second time in jeopardy for the same
offense.[28]

109
SO ORDERED.

Melo, (Chairman), Vitug, Sandoval-Gutierrez, and Carpio, JJ., concur.

110
AMARILLO ET AL. VS. SANDIGANBAYAN Petitioner Carolina D. Querijero submitted her counter-affidavit, denying the material
allegations in the complaint, and attached thereto a photocopy of the letter dated
G.R. Nos. 145007-08 January 28, 2003 January 4, 1996 by Mayor Fidel Salamera, addressed to Mr. Joselito Ong, president
and general manager of IDC, requesting said company to allow petitioner Querijero
FIDEL AMARILLO, SANCHITO C. CABANA, ANDRES M. MARMOL, ALEXANDER to utilize its crane and boom in the repair of Dyos and Pugo bridges; and the letter of
G. SURIL, ROGELIO R. GUERRERO, JESUS L. CAPIENDO, ARTURO P. RUZOL, a certain Wilbur Dee, general manager of IDC, granting the above request of Mayor
LORETO G. SURIL and CAROLINA D. QUERIJERO,petitioners, Salamera.
vs.
THE HONORABLE SANDIGANBAYAN (THIRD DIVISION) and THE PEOPLE OF The graft investigator found reasonable ground to include Mayor Salamera as one of
THE PHILIPPINES,respondents. the respondents, hence, he was directed to submit his counter-affidavit and
supporting evidence.
YNARES-SANTIAGO, J.:
In his counter-affidavit, Mayor Salamera stated, among others, that on November 3,
This is a petition 1 for certiorari, with prayer for a writ of preliminary injunction, 1995, heavy rains hit Casiguran, Aurora which resulted in widespread destruction of
seeking to annul the orders dated August 16 2 and September 27, 2000 of the properties, both public and private, including Dyos and Pugo bridges; that the
Sandiganbayan in Criminal Cases Nos. 25813 and 25518, which denied petitioners' bridges were repaired by Caroline Construction, pursuant to the contract awarded by
second motion for leave of court to file a motion for reinvestigation. the DPWH Aurora Engineering District; and that Danay's statements were not
accurate and were politically motivated.
The facts as culled from the parties' pleadings are as follows:
On June 23, 1999, graft investigator Gruta issued a resolution finding sufficient
On March 21, 1997, the Fact-Finding and Intelligence Bureau, through its director, ground to hold petitioners for trial on two counts of estafa through falsification of
filed a complaint-affidavit with the Evaluation and Preliminary Investigation Bureau official documents, thus:
(EPIB) of the Office of the Ombudsman against petitioners, namely, Fidel C. Amarillo,
Sanchito C. Cabana, Andres M. Marmol, Alexander G. Suril, Rogelio R. Guerrero, Jesus After a circumspect evaluation of the record, it has become apparent that there was
L. Capiendo, Arturo P. Ruzol, and Loreto G. Suril, all officials of the Department of conspiracy among the respondents to make it appear that Pugo and Dyos Bridges
Public Works and Highways, Aurora Engineering District, together with Carolina were repaired in January 1996 by Caroline Construction in the total amount of
Querijero, a private contractor, for estafa through falsification of public documents. P381,824.04 when in truth and in fact no repair/construction was actually
The complaint alleged that petitioners simulated a contract with Querijero for the undertaken, to the prejudice of the government. Accordingly, we find sufficient
repair of the Pugo and Dyos bridges, which were allegedly damaged by flashfloods ground to bring the respondents to trial for two counts of estafa through falsification
on November 3, 1995. of official documents in violation of Article 315 par. 2(a) in relation to Article 171 of
the Revised Penal Code.
Finding sufficient basis for a preliminary investigation, Graft Investigation Officer
(GIO) Melinda S. Diaz directed petitioners to submit their counter-affidavits. Wherefore, it is most respectfully recommended that the appropriate
However, instead of doing so, they submitted a letter dated May 20, 1997, stating information be filed before the Sandiganbayan against Fidel T. Salamera,
that they were adopting as their counter-affidavit Amarillo's affidavit in the Fidel C. Amarillo, Sanchito C. Cabana, Andres M. Marmol, Alexander G. Suril,
administrative complaint which involved the same subject matter. The case was Rogelio R. Guerrero, Jesus L. Capiendo, Arturo P. Ruzol, Loreto G. Suril and
then assigned to GIO Bienvenida A. Gruta. Carolina D. Qurijero.
5
In her investigation, Gruta considered, among others, (a) the sworn statements SO RESOLVED.
executed by Zosimo C. Danay, Jr., and Rolando L. Eresmas, the Barangay
Chairperson and Kagawad, respectively, of Barangay Tinib, Casiguran, Aurora; (b) The recommendation was approved by the Ombudsman.
the certification dated March 12, 1999 of Geraldine B. Santos, a social welfare officer Consequently, an Information was filed with the Sandiganbayan charging petitioners
of the same barangay; and (c) the memorandum dated December 28, 1995 of with estafa through falsification of official documents, docketed as Criminal Case No.
Helideliza R. Vicente, the Municipal Agriculturist of Casiguran, Aurora. 25518.
In his affidavit, 3 Danay stated that Pugo and Dyos bridges are located in Barangay On September 17, 1999, petitioners filed a motion for reconsideration with prior
Tinib, Casiguran, Aurora; that the same were repaired by the Industrial Development leave of court, with the Office of the Special Prosecutor on the ground that they were
Corporation (IDC) sometime in October or November 1995 for free; and that no not given the chance to answer the affidavits executed by Danay and Eresmas.
repair was made on the said bridges in January 1996.
On December 2, 1999, the Office of the Special Prosecutor recommended the denial
Eresmas, in his affidavit, 4 corroborated Danay's statements that the Dios and of the motion for reconsideration on the ground that no new evidence was
Nalinan (also known as Pugo) bridges in Barangay Tinib, Casiguran, Aurora were discovered which materially affects the resolution, and no errors of law or
repaired by IDC; and that since then, it had not been damaged. He added that the irregularities were committed prejudicial to the interest of the movant. 6 It also
flashflood occurred not on November 3, 1995, but on December 24, 1995. recommended the filing of an additional information with the Sandiganbayan,
The certification by social welfare officer Geraldine B. Santos disclosed that the considering that only one charge was filed despite the approved EPIB finding that
flashflood that affected Barangay Tinib, Casiguran, Aurora occurred on December there were two counts of estafa committed by petitioners. Likewise, it was observed
24, 1995. This was strengthened by the memorandum submitted by Helideliza R. by the Office of the Special Prosecutor that the information in Criminal Case No.
Vicente containing the final report on the damage on crops and livestock resulting 25518 contained some defects such as (a) the non-inclusion of the phrase
from the flashflood that occurred on the said date. "committing the offense in relation to office", and (b) failure to state the

111
participation of accused Loreto G. Suril, et al. 7 Hence, the Special Prosecutor could have raised the same in its motion for reconsideration, but they failed to do so.
recommended the amendment of the information in Criminal Case No. 25518. Neither were petitioners denied the chance to refute the allegations of barangay
officials Danay and Eresmas. The truth is, petitioners, in their motion for
Accordingly, on February 17, 2000, another information for estafa through reconsideration, tried to discredit the statements of the said barangay officials by
falsification of official documents was filed and docketed as Criminal Case No. saying that the latter could not possibly have an accurate recollection of events
25813. The information in Criminal Case No. 25518 was amended to conform to the which happened two years ago. However, the Special Prosecutor did not give weight
recommendation of the Office of the Special Prosecutor. to petitioners' allegations.
On July 4, 2000, petitioners filed a second motion for leave of court to file a motion The petition centers on the issue of whether or not the Sandiganbayan committed
for reinvestigation on the ground of newly discovered evidence, which consisted of grave abuse of discretion in denying the second motion for leave of court to file a
an affidavit executed by Mineo F. Taduyo, administrative officer of IDC, attesting that motion for reinvestigation.
indeed Mayor Salamera requested IDC in writing to allow Querijero to utilize its
heavy equipments for the repair of Pugo and Dyos bridges and that Wilbur Dee, the The petition lacks merit.
general manager of the aforesaid company, granted the request in his letter dated
January 10, 1996. Due process of law means giving opportunity to be heard before judgment is
rendered. It is a law which hears before it condemns, which proceeds upon inquiry
On August 16, 2000, the Sandiganbayan denied the motion on the ground that a and renders judgment only after trial. There is no violation of due process even if no
second motion for reinvestigation is prohibited under Republic Act No. 6770. hearing was conducted, where the party was given a chance to explain his side of
Petitioners moved for reconsideration, but the same was denied in open court on the controversy. What is frowned upon is the denial of the opportunity to be heard. 9
September 27, 2000. Hence, this petition.
In the case at bar, petitioners cannot claim that they were denied due process of
Petitioners contend that the Sandiganbayan committed grave abuse of discretion law. The Evaluation and Preliminary Investigation Bureau, upon a finding of sufficient
when it denied their second motion for leave of court to file a motion for basis to conduct a preliminary investigation, directed petitioners to submit their
reinvestigation and their motion to reconsider said denial, thus, depriving them of respective counter-affidavits, but they did not. Instead, they manifested that they
their right to due process. According to petitioners, they were not given the were adopting, as their counter-affidavits, the affidavit filed by Amarillo in the
opportunity to refute the statements of Danay and Eresmas that no repair was made administrative charge filed against him and petitioners. Clearly, petitioners were
by Caroline Construction on the bridges and the finding of conspiracy by the Special given the opportunity to explain their side. Thus, petitioners cannot shift the blame
Prosecutor. Petitioners further argue that their motion for reinvestigation is based on to respondent if in its determination of probable cause, it found well-founded the
meritorious grounds, i.e., newly discovered evidence. Petitioners aver that legalistic recommendation of Graft Investigation Officer Gruta, which was based on the
formalities and narrow technicalities that would derail an otherwise valid claim are affidavits of Danay and Eresmas.
frowned upon; and that procedural formalities must yield where the interest of
justice would make unfair the strict application of the rules. Further, petitioners filed a motion for reconsideration where they assailed the
accuracy of the statements given by Danay and Eresmas. Where the parties were
Public respondent Sandiganbayan, through the Office of the Special Prosecutor, given the opportunity to seek a reconsideration of the action or ruling complained of,
maintains that there was nothing capricious and whimsical in denying petitioners' they cannot claim denial of due process of law. 10
second motion for leave of court to file motion for reinvestigation, for it merely
applied the Office of the Ombudsman's Rules of Procedure. 8 Public respondent Anent the second ground alleged by petitioner, we cannot consider Taduyo's
claims that the alleged repair of the bridges has been investigated twice, first by affidavit as a newly discovered evidence. Under the Rules of Court, 11 the requisites
EPIB in its preliminary investigation, and second by the Office of the Special for newly discovered evidence are: (a) the evidence was discovered after trial (in
Prosecutor. To give due course to the second motion for leave is tantamount to this case, after investigation); (b) such evidence could not have been discovered and
allowing a determination, for the third time, of the existence of probable cause. produced at the trial with reasonable diligence; and (c) that it is material, not merely
There must be an end to a reevaluation of evidence. cumulative, corroborative or impeaching, and is of such weight that, if admitted, will
probably change the judgment. 12
Public respondent likewise asseverates that the affidavit of Taduyo cannot qualify as
newly discovered evidence. First, it was available during the preliminary Taduyo's affidavit falls short of the second and third requirements. It could have
investigation of the complaint. Second, it contains nothing that would alter the easily been produced during the investigation of the case. There was no showing of
findings of the Office of the Special Prosecutor that the two bridges were not Taduyo's non-availability at the time of the investigation or the absence of the
damaged and that no repair was done by Caroline Construction. Taduyo merely correspondence between Mayor Salamera and Wilbur Dee from IDC records. Also,
attested in said affidavit that IDC received the letter of Mayor Salamera requesting assuming that said affidavit could not have been reasonably produced during the
said company to allow the use of its heavy equipment in the repair of the two investigation, still it can not qualify as newly discovered evidence, because it was
bridges and that said request was granted by Wilbur Dee, the company's general not material to the issue. As alleged by public respondent, the affidavit merely
manager. Taduyo did not attest that he saw the actual repair of the two bridges by stated that IDC received the letter-request of Mayor Salamera and that Wilbur Dee
Caroline Construction in 1996. Public respondent maintains that a motion for granted the request, but there was no allegation by Taduyo that he actually saw or
reinvestigation on the ground of newly discovered evidence is allowed only where had personal knowledge of the repair of the Pugo and Dyos bridges by petitioners
said evidence was not available during the probe, which evidence, if proven or through Caroline Construction. In fact, in the same affidavit, Taduyo also attested
admitted, may probably change the prosecutor's findings as to the existence of a that IDC undertook in September-October 1995, the construction of six bridges, two
prima facie case. of which were the Pugo and Dyos bridges, and that it was only on May 8, 1996 that
IDC prepared and presented the necessary deed of donation for the said bridges to
Public respondent also refutes petitioners' claim that they were denied opportunity the municipal government of Casiguran, Aurora.
to rebut the findings of conspiracy by the Office of the Special Prosecutor. Petitioners

112
Based on the foregoing, no grave abuse of discretion was committed by public
respondent Sandiganbayan in denying the subject motions. Grave abuse of
discretion implies such capricious and whimsical exercise of judgment as equivalent
to lack of jurisdiction, or, in other words, where the power is exercised in an arbitrary
or despotic manner by reason of passion or personal hostility, and it must be so
patent and gross as to amount to an evasion of positive duty or to a virtual refusal to
perform the duty enjoined or to act at all in contemplation of law. 13 The records
show that public respondent Sandiganbayan did none of these; it acted in
accordance with the law.
WHEREFORE, the petition is DISMISSED. The assailed Orders of the Sandiganbayan
are AFFIRMED. This decision is immediately executory.
Costs against the petitioners.
SO ORDERED.

113
COQUILLA VS. COMMISSION ON ELECTIONS Respondents frequent or regular trips to the Philippines and stay in Oras,
Eastern Samar after his retirement from the U.S. Navy in 1985 cannot be
G.R. No. 151914 July 31, 2002 considered as a waiver of his status as a permanent resident or immigrant .
. . of the U.S.A. prior to November 10, 2000 as would qualify him to acquire
TEODULO M. COQUILLA, petitioner, the status of residency for purposes of compliance with the one-year
vs. residency requirement of Section 39(a) of the Local Government Code of
THE HON. COMMISSION ON ELECTIONS and MR. NEIL M. 1991 in relation to Sections 65 and 68 of the Omnibus Election Code. The
ALVAREZ, respondents. one (1) year residency requirement contemplates of the actual residence of
a Filipino citizen in the constituency where he seeks to be elected.
MENDOZA, J.:
All things considered, the number of years he claimed to have resided or
This is a petition for certiorari to set aside the resolution, 1 dated July 19, 2001, of the stayed in Oras, Eastern Samar since 1985 as an American citizen and
Second Division of the Commission on Elections (COMELEC), ordering the permanent resident of the U.S.A. before November 10, 2000 when he
cancellation of the certificate of candidacy of petitioner Teodulo M. Coquilla for the reacquired his Philippine citizenship by [repatriation] cannot be added to his
position of mayor of Oras, Eastern Samar in the May 14, 2001 elections and the actual residence thereat after November 10, 2000 until May 14, 2001 to
order, dated January 30, 2002, of the COMELEC en banc denying petitioners motion cure his deficiency in days, months, and year to allow or render him eligible
for reconsideration. to run for an elective office in the Philippines. Under such circumstances, by
The facts are as follows: whatever formula of computation used, respondent is short of the one-year
residence requirement before the May 14, 2001 elections. 9
Petitioner Coquilla was born on February 17, 1938 of Filipino parents in Oras, Eastern
Samar. He grew up and resided there until 1965, when he joined the United States Petitioner filed a motion for reconsideration, but his motion was denied by the
Navy. He was subsequently naturalized as a U.S. citizen. 2 From 1970 to 1973, COMELEC en banc on January 30, 2002. Hence this petition.
petitioner thrice visited the Philippines while on leave from the U.S. I.
Navy.3 Otherwise, even after his retirement from the U.S. Navy in 1985, he remained
in the United States. Two questions must first be resolved before considering the merits of this case: (a)
whether the 30-day period for appealing the resolution of the COMELEC was
On October 15, 1998, petitioner came to the Philippines and took out a residence suspended by the filing of a motion for reconsideration by petitioner and (b) whether
certificate, although he continued making several trips to the United States, the last the COMELEC retained jurisdiction to decide this case notwithstanding the
of which took place on July 6, 2000 and lasted until August 5, 2000. 4 Subsequently, proclamation of petitioner.
petitioner applied for repatriation under R.A. No. 8171 5 to the Special Committee on
Naturalization. His application was approved on November 7, 2000, and, on A. With respect to the first question, private respondent contends that the
November 10, 2000, he took his oath as a citizen of the Philippines. Petitioner was petition in this case should be dismissed because it was filed late; that the
issued Certificate of Repatriation No. 000737 on November 10, 2000 and Bureau of COMELEC en banc had denied petitioners motion for reconsideration for being pro
Immigration Identification Certificate No. 115123 on November 13, 2000. forma; and that, pursuant to Rule 19, 4 of the COMELEC Rules of Procedure, the
said motion did not suspend the running of the 30-day period for filing this petition.
On November 21, 2000, petitioner applied for registration as a voter of Butnga, He points out that petitioner received a copy of the resolution, dated July 19, 2001,
Oras, Eastern Samar. His application was approved by the Election Registration of the COMELECs Second Division on July 28, 2001, so that he had only until August
Board on January 12, 2001. 6 On February 27, 2001, he filed his certificate of 27, 2001 within which to file this petition. Since the petition in this case was filed on
candidacy stating therein that he had been a resident of Oras, Eastern Samar for February 11, 2002, the same should be considered as having been filed late and
"two (2) years."7 should be dismissed.
On March 5, 2001, respondent Neil M. Alvarez, who was the incumbent mayor of Private respondents contention has no merit.
Oras and who was running for reelection, sought the cancellation of petitioners
certificate of candidacy on the ground that the latter had made a material Rule 19 of the COMELEC Rules of Procedure provides in pertinent parts:
misrepresentation in his certificate of candidacy by stating that he had been a
resident of Oras for two years when in truth he had resided therein for only about six Sec. 2. Period for Filing Motions for Reconsideration. A motion to
months since November 10, 2000, when he took his oath as a citizen of the reconsider a decision, resolution, order, or ruling of a Division shall be filed
Philippines. within five days from the promulgation thereof. Such motion, if not pro-
forma, suspends the execution for implementation of the decision,
The COMELEC was unable to render judgment on the case before the elections on resolution, order, or ruling.
May 14, 2001. Meanwhile, petitioner was voted for and received the highest number
of votes (6,131) against private respondents 5,752 votes, or a margin of 379 votes. Sec. 4. Effect of Motion for Reconsideration on Period to Appeal. A motion
On May 17, 2001, petitioner was proclaimed mayor of Oras by the Municipal Board to reconsider a decision, resolution, order, or ruling, when not pro-forma,
of Canvassers.8 He subsequently took his oath of office. suspends the running of the period to elevate the matter to the Supreme
Court.
On July 19, 2001, the Second Division of the COMELEC granted private respondents
petition and ordered the cancellation of petitioners certificate of candidacy on the The five-day period for filing a motion for reconsideration under Rule 19, 2 should
basis of the following findings: be counted from the receipt of the decision, resolution, order, or ruling of the
COMELEC Division.10 In this case, petitioner received a copy of the resolution of July
19, 2001 of the COMELECs Second Division on July 28, 2001. Five days later, on

114
August 2, 2001, he filed his motion for reconsideration. On February 6, 2002, he SECTION 6. Effect of Disqualification Case. Any candidate who has been
received a copy of the order, dated January 30, 2002, of the COMELEC en declared by final judgment to be disqualified shall not be voted for, and the
banc denying his motion for reconsideration. Five days later, on February 11, 2002, votes cast for him shall not be counted. If for any reason a candidate is not
he filed this petition for certiorari. There is no question, therefore, that petitioners declared by final judgment before an election to be disqualified and he is
motion for reconsideration of the resolution of the COMELEC Second Division, as well voted for and receives the winning number of votes in such election, the
as his petition for certiorari to set aside of the order of the COMELEC en banc, was Court or Commission shall continue with the trial and hearing of the action,
filed within the period provided for in Rule 19, 2 of the COMELEC Rules of Procedure inquiry, or protest and, upon motion of the complainant or any intervenor,
and in Art. IX(A), 7 of the Constitution. may during the pendency thereof order the suspension of the proclamation
of such candidate whenever the evidence of his guilt is strong. (Emphasis
It is contended, however, that petitioners motion for reconsideration before the added)
COMELEC en banc did not suspend the running of the period for filing this petition
because the motion was pro forma and, consequently, this petition should have SECTION 7. Petition to Deny Due Course To or Cancel a Certificate of
been filed on or before August 27, 2001. It was actually filed, however, only on Candidacy. The procedure hereinabove provided shall apply to petitions
February 11, 2002. Private respondent cites the finding of the COMELEC en to deny due course to or cancel a certificate of candidacy as provided in
banc that Section 78 of Batas Pambansa Blg. 881.
An incisive examination of the allegations in the Motion for Reconsideration The rule then is that candidates who are disqualified by final judgment before the
shows that the same [are] a mere rehash of his averments contained in election shall not be voted for and the votes cast for them shall not be counted. But
his Verified Answer and Memorandum. Neither did respondent raise new those against whom no final judgment of disqualification had been rendered may be
matters that would sufficiently warrant a reversal of the assailed resolution voted for and proclaimed, unless, on motion of the complainant, the COMELEC
of the Second Division. This makes the said Motion pro forma.11 suspends their proclamation because the grounds for their disqualification or
cancellation of their certificates of candidacy are strong. Meanwhile, the proceedings
We do not think this contention is correct. The motion for reconsideration was not for disqualification of candidates or for the cancellation or denial of certificates of
pro forma and its filing did suspend the period for filing the petition for certiorari in candidacy, which have been begun before the elections, should continue even after
this case. The mere reiteration in a motion for reconsideration of the issues raised by such elections and proclamation of the winners. In Abella v. COMELEC19 and Salcedo
the parties and passed upon by the court does not make a motion pro forma; II v. COMELEC,20 the candidates whose certificates of candidacy were the subject of
otherwise, the movants remedy would not be a reconsideration of the decision but a petitions for cancellation were voted for and, having received the highest number of
new trial or some other remedy.12 But, as we have held in another case:13 votes, were duly proclaimed winners. This Court, in the first case, affirmed and, in
Among the ends to which a motion for reconsideration is addressed, one is the second, reversed the decisions of the COMELEC rendered after the proclamation
precisely to convince the court that its ruling is erroneous and improper, of candidates, not on the ground that the latter had been divested of jurisdiction
contrary to the law or the evidence; and in doing so, the movant has to upon the candidates proclamation but on the merits.
dwell of necessity upon the issues passed upon by the court. If a motion for II.
reconsideration may not discuss these issues, the consequence would be
that after a decision is rendered, the losing party would be confined to filing On the merits, the question is whether petitioner had been a resident of Oras,
only motions for reopening and new trial. Eastern Samar at least one (1) year before the elections held on May 14, 2001 as he
represented in his certificate of candidacy. We find that he had not.
Indeed, in the cases where a motion for reconsideration was held to be pro
forma, the motion was so held because (1) it was a second motion for First, 39(a) of the Local Government Code (R.A No. 7160) provides:
reconsideration,14 or (2) it did not comply with the rule that the motion must specify
the findings and conclusions alleged to be contrary to law or not supported by the Qualifications. - (a) An elective local official must be a citizen of the
evidence,15 or (3) it failed to substantiate the alleged errors, 15 or (4) it merely alleged Philippines; a registered voter in the barangay, municipality, city, or
that the decision in question was contrary to law, 17 or (5) the adverse party was not province or, in the case of a member of the sangguniang panlalawigan,
given notice thereof.18 The 16-page motion for reconsideration filed by petitioner in sangguniang panlungsod, or sangguniang bayan, the district where he
the COMELEC en banc suffers from none of the foregoing defects, and it was error intends to be elected; a resident therein for at least one (1) year
for the COMELEC en banc to rule that petitioners motion for reconsideration was pro immediately preceding the day of the election; and able to read and write
forma because the allegations raised therein are a mere "rehash" of his earlier Filipino or any other local language or dialect. (Emphasis added)
pleadings or did not raise "new matters." Hence, the filing of the motion suspended The term "residence" is to be understood not in its common acceptation as referring
the running of the 30-day period to file the petition in this case, which, as earlier to "dwelling" or "habitation,"21but rather to "domicile" or legal residence, 22 that is,
shown, was done within the reglementary period provided by law. "the place where a party actually or constructively has his permanent home, where
B. As stated before, the COMELEC failed to resolve private respondents petition for he, no matter where he may be found at any given time, eventually intends to return
cancellation of petitioners certificate of candidacy before the elections on May 14, and remain (animus manendi)."23 A domicile of origin is acquired by every person at
2001. In the meantime, the votes were canvassed and petitioner was proclaimed birth. It is usually the place where the childs parents reside and continues until the
elected with a margin of 379 votes over private respondent. Did the COMELEC same is abandoned by acquisition of new domicile (domicile of choice). 24
thereby lose authority to act on the petition filed by private respondent? In the case at bar, petitioner lost his domicile of origin in Oras by becoming a U.S.
R.A. No. 6646 provides: citizen after enlisting in the U.S. Navy in 1965. From then on and until November 10,
2000, when he reacquired Philippine citizenship, petitioner was an alien without any

115
right to reside in the Philippines save as our immigration laws may have allowed him a Balikbayan Program), the term balikbayan includes a former Filipino citizen who
to stay as a visitor or as a resident alien. had been naturalized in a foreign country and comes or returns to the Philippines
and, if so, he is entitled, among others, to a "visa-free entry to the Philippines for a
Indeed, residence in the United States is a requirement for naturalization as a U.S. period of one (1) year" (3(c)). It would appear then that when petitioner entered the
citizen. Title 8, 1427(a) of the United States Code provides: country on the dates in question, he did so as a visa-free balikbayanvisitor whose
Requirements of naturalization. Residence stay as such was valid for one year only. Hence, petitioner can only be held to have
waived his status as an alien and as a non-resident only on November 10, 2000 upon
(a) No person, except as otherwise provided in this subchapter, shall be taking his oath as a citizen of the Philippines under R.A. No. 8171. 32 He lacked the
naturalized unless such applicant, (1) immediately preceding the date of requisite residency to qualify him for the mayorship of Oras, Eastern, Samar.
filing his application for naturalization has resided continuously, after being
lawfully admitted for permanent residence, within the United States for at Petitioner invokes the ruling in Frivaldo v. Commission on Elections33 in support of his
least five years and during the five years immediately preceding the date of contention that the residency requirement in 39(a) of the Local Government Code
filing his petition has been physically present therein for periods totaling at includes the residency of one who is not a citizen of the Philippines. Residency,
least half of that time, and who has resided within the State or within the however, was not an issue in that case and this Court did not make any ruling on the
district of the Service in the United States in which the applicant filed the issue now at bar. The question in Frivaldo was whether petitioner, who took his oath
application for at least three months, (2) has resided continuously within of repatriation on the same day that his term as governor of Sorsogon began on June
the United States from the date of the application up to the time of 30, 1995, complied with the citizenship requirement under 39(a). It was held that
admission to citizenship, and (3) during all the period referred to in this he had, because citizenship may be possessed even on the day the candidate
subsection has been and still is a person of good moral character, attached assumes office. But in the case of residency, as already noted, 39(a) of the Local
to the principles of the Constitution of the United States, and well disposed Government Code requires that the candidate must have been a resident of the
to the good order and happiness of the United States. (Emphasis added) municipality "for at least one (1) year immediately preceding the day of the
election."
In Caasi v. Court of Appeals,25 this Court ruled that immigration to the United States
by virtue of a "greencard," which entitles one to reside permanently in that country, Nor can petitioner invoke this Courts ruling in Bengzon III v. House of
constitutes abandonment of domicile in the Philippines. With more reason then does Representatives Electoral Tribunal.34 What the Court held in that case was that, upon
naturalization in a foreign country result in an abandonment of domicile in the repatriation, a former natural-born Filipino is deemed to have recovered his original
Philippines. status as a natural-born citizen.

Nor can petitioner contend that he was "compelled to adopt American citizenship" Third, petitioner nonetheless says that his registration as a voter of Butnga, Oras,
only by reason of his service in the U.S. armed forces. 26 It is noteworthy that Eastern Samar in January 2001 is conclusive of his residency as a candidate because
petitioner was repatriated not under R.A. No. 2630, which applies to the repatriation 117 of the Omnibus Election Code requires that a voter must have resided in the
of those who lost their Philippine citizenship by accepting commission in the Armed Philippines for at least one year and in the city or municipality wherein he proposes
Forces of the United States, but under R.A. No. 8171, which, as earlier mentioned, to vote for at least six months immediately preceding the election. As held in Nuval
provides for the repatriation of, among others, natural-born Filipinos who lost their v. Guray,35 however, registration as a voter does not bar the filing of a subsequent
citizenship on account of political or economic necessity. In any event, the fact is case questioning a candidates lack of residency.
that, by having been naturalized abroad, he lost his Philippine citizenship and with it Petitioners invocation of the liberal interpretation of election laws cannot avail him
his residence in the Philippines. Until his reacquisition of Philippine citizenship on any. As held in Aquino v. Commission on Elections:36
November 10, 2000, petitioner did not reacquire his legal residence in this country.
A democratic government is necessarily a government of laws. In a
Second, it is not true, as petitioner contends, that he reestablished residence in this republican government those laws are themselves ordained by the people.
country in 1998 when he came back to prepare for the mayoralty elections of Oras Through their representatives, they dictate the qualifications necessary for
by securing a Community Tax Certificate in that year and by "constantly declaring" service in government positions. And as petitioner clearly lacks one of the
to his townmates of his intention to seek repatriation and run for mayor in the May essential qualifications for running for membership in the House of
14, 2001 elections.27 The status of being an alien and a non-resident can be waived Representatives, not even the will of a majority or plurality of the voters of
either separately, when one acquires the status of a resident alien before acquiring the Second District of Makati City would substitute for a requirement
Philippine citizenship, or at the same time when one acquires Philippine citizenship. mandated by the fundamental law itself.
As an alien, an individual may obtain an immigrant visa under 13 28 of the Philippine
Immigration Act of 1948 and an Immigrant Certificate of Residence (ICR) 29 and thus Fourth, petitioner was not denied due process because the COMELEC failed to act on
waive his status as a non-resident. On the other hand, he may acquire Philippine his motion to be allowed to present evidence. Under 5(d), in relation to 7, of R.A.
citizenship by naturalization under C.A. No. 473, as amended, or, if he is a former No. 6646 (Electoral Reforms Law of 1987), proceedings for denial or cancellation of a
Philippine national, he may reacquire Philippine citizenship by repatriation or by an certificate of candidacy are summary in nature. The holding of a formal hearing is
act of Congress,30 in which case he waives not only his status as an alien but also his thus notde rigeur. In any event, petitioner cannot claim denial of the right to be
status as a non-resident alien. heard since he filed a Verified Answer, a Memorandum and a Manifestation, all dated
March 19, 2001, before the COMELEC in which he submitted documents relied by
In the case at bar, the only evidence of petitioners status when he entered the him in this petition, which, contrary to petitioners claim, are complete and intact in
country on October 15, 1998, December 20, 1998, October 16, 1999, and June 23, the records.
2000 is the statement "Philippine Immigration [] Balikbayan" in his 1998-2008 U.S.
passport. As for his entry on August 5, 2000, the stamp bore the added inscription III.
"good for one year stay."31 Under 2 of R.A. No. 6768 (An Act Instituting

116
The statement in petitioners certificate of candidacy that he had been a resident of
Oras, Eastern Samar for "two years" at the time he filed such certificate is not true.
The question is whether the COMELEC was justified in ordering the cancellation of
his certificate of candidacy for this reason. We hold that it was. Petitioner made a
false representation of a material fact in his certificate of candidacy, thus rendering
such certificate liable to cancellation. The Omnibus Election Code provides:
SEC. 74. Contents of certificate of candidacy. The certificate of candidacy
shall state that the person filing it is announcing his candidacy for the office
stated therein and that he is eligible for said office; if for Member of the
Batasang Pambansa, the province, including its component cities, highly
urbanized city or district or sector which he seeks to represent; the political
party to which he belongs; civil status; his date of birth; residence; his post
office address for all election purposes; his profession or occupation; that
he will support and defend the Constitution of the Philippines and will
maintain true faith and allegiance thereto; that he will obey the laws, legal
orders, and decrees promulgated by the duly constituted authorities; that
he is not a permanent resident or immigrant to a foreign country; that the
obligation imposed by his oath is assumed voluntarily, without mental
reservation or purpose of evasion; and that the facts stated in the
certificate of candidacy are true to the best of his knowledge.
SEC. 78. Petition to deny due course to or cancel a certificate of candidacy.
A verified petition seeking to deny due course or to cancel a certificate of
candidacy may be filed by any person exclusively on the ground that any
material representation contained therein as required under Section 74
hereof is false. The petition may be filed at any time not later than twenty-
five days from the time of the filing of the certificate of candidacy and shall
be decided, after due notice and hearing, not later than fifteen days before
the election.
Indeed, it has been held that a candidates statement in her certificate of candidacy
for the position of governor of Leyte that she was a resident of Kananga, Leyte when
this was not so37 or that the candidate was a "natural-born" Filipino when in fact he
had become an Australian citizen 38 constitutes a ground for the cancellation of a
certificate of candidacy. On the other hand, we held in Salcedo II v. COMELEC39 that a
candidate who used her husbands family name even though their marriage was
void was not guilty of misrepresentation concerning a material fact. In the case at
bar, what is involved is a false statement concerning a candidates qualification for
an office for which he filed the certificate of candidacy. This is a misrepresentation of
a material fact justifying the cancellation of petitioners certificate of candidacy. The
cancellation of petitioners certificate of candidacy in this case is thus fully justified.
WHEREFORE, the petition is DISMISSED and the resolution of the Second Division of
the Commission on Elections, dated July 19, 2001, and the order, dated January 30,
2002 of the Commission on Elections en banc are AFFIRMED.
SO ORDERED.
Davide, Jr., C.J., Bellosillo, Puno, Vitug, Kapunan, Panganiban, Quisumbing, Ynares-
Santiago, Sandoval-Gutierrez, Carpio, Austria-Martinez, and Corona, JJ., concur.

117
CRUZ ET AL. VS. CIVIL SERVICE Service Subprofessional Examinations which she took last July 26,
1987 and July 31, 1988 respectively. It would appear that the
COMMISSION purported picture of Cruz pasted in the Picture Seat Plan of the
said July 30, 1989 examination is the picture of a different person.
G.R. No. 144464 November 27, 2001 Further verification showed that this picture belongs to a certain
Zenaida Paitim, Municipal Treasurer of Norzagaray, Bulacan who
GILDA G. CRUZ and ZENAIDA C. PAITIM, petitioner, apparently took the said examination on behalf of Cruz and on the
vs. basis of the application bearing the name and personal
THE CIVIL SERVICE COMMISSION, respondent. circumstances of Cruz."
KAPUNAN, J.: WHEREFORE, Gilda Cruz and Zenaida Paitim are hereby directed to answer
Assailed in the instant petition is the decision of the Court of Appeals upholding in writing and under oath within five (5) days from receipt hereof. To
Resolution No. 981695 of the Civil Service Commission for allegedly being contrary support your Answer, you may submit supporting documents/sworn
to law and jurisprudence. statements.

The facts are as follows: In your Answer, you should state whether you elect to have a formal
investigation or waive your right to said investigations should your Answer
On September 9, 1994, the Chairperson of the Civil Service Commission (CSC), be found not satisfactory.
received a letter from a private individual, Carmelita B. Esteban, claiming that,
during the examinations for non-professional in the career civil service, given by the You are advised that you are entitled to the assistance of a counsel.
Civil Service Commission, on July 30, 1989 in Quezon City, Zenaida C. Paitim, the
Municipal Treasurer of Norzagaray, Bulacan, falsely pretending to be the examinee, By Authority of the Commission:
Gilda Cruz, a co-employee in the said office, took the examinations for the latter.
Carmelita Esteban requested the CSC to investigate the matter, appending to said (Sgd.) Bella A. Amilhasan
letter, pictures purporting to be those of Gilda Cruz and Zenaida Paitim. Director IV1

On September 20, 1994, Erlinda A. Rosas, Director IV of the Commission, issued a The petitioners filed their Answer to the charge entering a general denial of the
Memorandum to Eliseo Gatchalian, the Director of the Management Information material averments of the "Formal Charge." They also declared that they were
Office of the Commission, requesting the latter to furnish her with the picture seat electing a formal investigation on the matter. The petitioners subsequently filed a
plan of the room where Gilda G. Cruz was during the said examination, to ascertain Motion to Dismiss averring that if the investigation will continue, they will be
the veracity of the letter-complaint. Eliseo S. Gatchalian did furnish Erlinda Rosas deprived of their right to due process because the Civil Service Commission was the
with certified true copies of the picture seat plans of the rooms where Gilda G. Cruz complainant, the Prosecutor and the Judge, all at the same time.
was assigned not only in the 1989 but also in the 1987 and 1988 career service
(sub-professional) examinations. On November 8, 1994, Erlinda Rosas thereby wrote On July 17, 1995, Director Bella A. Amilhasan issued an order denying the
a Memorandum to Civil Service Commissioner Thelma P. Gaminde, dated November motion.2 The subsequent motion for reconsideration of said order was likewise
8, 1994, declaring that based on the record, she found a prima facie case against dismissed.
Zenaida Paitim and Gilda G. Cruz.
Dulce J. Cochon, Attorney III of the CSC was thereby directed to conduct the formal
On the basis of said memorandum, a fact finding investigation was conducted. On administrative investigation of petitioners' case.
March 31, 1995, a "Formal Charge" for "Dishonesty, Grave Misconduct, and Conduct
Prejudicial to the Best Interest of the Service" signed by Bella Amilhasan, Director IV On November 16, 1995, Dulce J. Cochon issued an "Investigation Report and
of the Civil Service Commission Regional Office No. 3 was filed against Gilda Cruz Recommendation" finding the Petitioners guilty of "Dishonesty" and ordering their
and Zenaida C. Paitim, with the Civil Service Commission, docketed as dismissal from the government service, the decretal portion of which reads as
Administrative Case No. D3-9S-052, which reads as follows: follows:

FORMAL CHARGE WHEREFORE, foregoing premises considered, this Office recommends the
dismissal from the service with all its accessory penalties of respondents
MESDAMES: Zenaida Paitim and Gilda Cruz, both employees of the Municipality of
Norzagaray, Bulacan for the offenses of Dishonesty, Grave Misconduct and
This Office has found after a fact finding investigation that a prima facie Conduct Prejudicial to the Best Interest of the Service. Furthermore, this
case exists against you for DISHONESTY, GRAVE MISCONDUCT and Office recommends the filing of criminal charges against them that shall
CONDUCT PREJUDICIAL TO THE BEST INTEREST OF THE SERVICE, committed serve as a deterrent to all possible plans of making a mockery to the
as follows: sanctity of Civil Service Law and Rules as well as the constitutional
"That Gilda Cruz applied to take the July 30, 1989 Career Service mandate that 'A public office is a public trust. (Idem. Supra.)3
Subprofessional examination. A verification of our records revealed The aforesaid "Investigation Report and Recommendation" was then forwarded, to
that the picture of Cruz pasted in the Picture Seat Plan of the said the Civil Service Commission for its consideration and resolution.
examination held at Room 21 of the Ramon Magsaysay Elementary
School, Quezon City, bears no resemblance to the pictures of Cruz
as appearing in the picture seat plans of the previous Career

118
On July 1, 1998, the Civil Service Commission issued Resolution No. 981695 finding SECTION 47. Disciplinary Jurisdiction. (1) The Commission shall decide
the petitioners guilty of the charges and ordered their dismissal from the upon appeal all administrative disciplinary cases involving the imposition of
government service. The decretal portion reads as follows: a penalty of suspension for more than thirty days, or a fine in an amount
exceeding thirty days' salary, demotion in rank or salary or
WHEREFORE, Zenaida Paitim and Gilda Cruz are hereby found guilty of transfer, removal or dismissal from office. A complaint may be filed
Dishonesty. Accordingly, they are imposed the penalty of dismissal from the directly with the Commission by a private citizen against a government
service with all its accessory penalties. The Civil Service (Subprofessional) official or employee in which case it may hear and decide the case or
Eligibility of Gilda Cruz is also cancelled. it may deputize any department or agency or official or group of officials to
Let a copy of this Resolution, as well as other relevant documents, be conduct the investigation. The results of the investigation shall be
furnished the Office of the Ombudsman for whatever action it may take submitted to the Commission with recommendation as to the penalty to be
under the premises."4 imposed or other action to be taken.7

Petitioners then went up to the Court of Appeals assailing the resolution of the CSC. (Emphasis supplied.)

On November 29, 1999, the Court of Appeals dismissed the petition before it. The Petitioners' invocation of the law is misplaced. The provision is applicable to
motion for reconsideration was, likewise, denied on August 9, 2000. instances where administrative cases are filed against erring employees in
connection with their duties and functions of the office. This is, however, not the
Hence, this petition. scenario contemplated in the case at bar. It must be noted that the acts complained
of arose from a cheating caused by the petitioners in the Civil Service
In the instant petition, petitioners raised the following assignment of errors: (Subprofessional) examination. The examinations were under the direct control and
I supervision of the Civil Service Commission. The culprits are government employees
over whom the Civil Service Commission undeniably has jurisdiction. Thus, after the
THE COURT OF APPEALS GRAVELY AND SERIOUSLY ERRED IN HOLDING THAT petitioners were duly investigated and ascertained whether they were indeed guilty
PETITIONERS' CONSTITUTIONAL RIGHT TO DUE PROCESS WAS NOT of dishonesty, the penalty meted was dismissal from the office.
VIOLATED IN ADMINISTRATIVE CASE NO. D3-95-052 WHERE RESPONDENT
COMMISSION ACTED AS THE INVESTIGATOR, THE COMPLAINANT, THE Section 28, Rule XIV of the Omnibus Civil Service Rules and Regulations explicitly
PROSECUTOR, AND THE JUDGE, ALL AT THE SAME TIME, AGAINST provides that the CSC can rightfully take cognizance over any irregularities or
PETITIONERS. IN SO DOING, RESPONDENT COMMISSION COMMITTED A anomalies connected to the examinations, as it reads:
MOCKERY OF ADMINISTRATIVE JUSTICE AND THE COURT OF APPEALS Sec. 28. The Commission shall have original disciplinary jurisdiction over all
SANCTIONED IT.
its officials and employees and over all cases involving civil service
II examination anomalies or irregularities."

THE COURT OF APPEALS GRAVELY AND SERIOUSLY ERRED IN RULING THAT Petitioners' contention that they were denied due process of law by the fact that the
RESPONDENT COMMISSION HAS ORIGINAL JURISDICTION TO HEAR AND CSC acted as investigator, complainant, prosecutor and judge, all at the same time
DECIDE A COMPLAINT OR CHARGE WHETHER FILED BY A PRIVATE CITIZEN against the petitioners is untenable. The CA correctly explained that the CSC is
OR BY THE CIVIL SERVICE COMMISSION ITSELF. THE LAW VESTS IN mandated to hear and decide administrative case instituted by it or instituted before
RESPONDENT COMMISSION ONLY APPELLATE, NOT ORIGINAL, JURISDICTION it directly or on appeal including actions of its officers and the agencies attached to
IN ALL ADMINISTRATIVE CASES AGAINST A PUBLIC OFFICIAL OR EMPLOYEE it pursuant to Book V, Title 1, Subtitle A, Chapter 3, Section 12, paragraph 11 of the
INVOLVING THE IMPOSITION OF A PENALTY OF REMOVAL OR DISMISSAL Administrative Code of 1987 which states:
FROM OFFICE WHERE THE COMPLAINT THEREFORE WAS NOT FILED BY A (11) Hear and decide administrative cases instituted by or brought before it
PRIVATE CITIZEN AS IN ADMINISTRATIVE CASE NO. D3-95-052 OF
directly or on appeal, including contested appointments, and review
RESPONDENT COMMISSION.5 decisions and actions of its offices and of the agencies attached to it.
We find no merit in the petition. Officials and employees who fail to comply with such decisions, orders, or
rulings shall be liable for contempt of the Commission. Its decisions, orders,
There is no question that petitioner Zenaida Paitim, masquerading herself as or rulings shall be final and executory. Such decisions, orders, or rulings
petitioner Gilda Cruz, took the civil service examinations in her behalf. Gilda Cruz may be brought to the Supreme Court on certiorari by the aggrieved party
passed the examinations. On the basis of a tip-off that the two public employees within thirty (30) days from receipt of a copy thereof;
were involved in an anomalous act, the CSC conducted an investigation and verified
that the two employees were indeed guilty of dishonesty. Thus, in accordance with The fact that the complaint was filed by the CSC itself does not mean that it could
the CSC law, the petitioners merited the penalty of dismissal. not be an impartial judge. As an administrative body, its decision was based on
substantial findings. Factual findings of administrative bodies, being considered
Petitioners maintain that the CSC did not have original jurisdiction to hear and experts in their field, are binding on the Supreme Court. 8 The records clearly disclose
decide the administrative case. Allegedly, in accordance with Section 47(1), Chapter that the petitioners were duly investigated by the CSC and found that:
7, Subtitle A, Title 1, Book V, Administrative Code of 1987, the CSC is vested with
appellate jurisdiction only in all administrative cases where the penalty imposed is After a careful examination of the records, the Commission finds
removal or dismissal from the office and where the complaint was filed by a private respondents guilty as charged.
citizen against the government employee. 6It reads:

119
The photograph pasted over the name Gilda Cruz in the Picture Seat Plan dismissed by private respondent Camara Steel, Inc., and as a consequence, ordered
(PSP) during the July 30, 1989 Career Service Examination is not that of his immediate reinstatement. Specifically, the dispositive portion of the Labor
Cruz but of Paitim. Also, the signature over the name of Gilda Cruz in the Arbiter's Decision promulgated 23 May 1999 states
said document is totally different from the signature of Gilda Cruz.
WHEREFORE, premises considered, respondent Camara Steel Industries,
It should be stressed that as a matter of procedure, the room examiners Inc. is hereby ordered to reinstate complainant Cecilio de los Santos to his
assigned to supervise the conduct of a Civil Service examination closely former position within ten (10) days from receipt of this Resolution without
examine the pictures submitted and affixed on the Picture Seat Plan (CSC loss of seniority rights and other benefits with full back wages from date of
Resolution No. 95-3694, Obedencio, Jaime A.). The examiners carefully dismissal up to actual date of reinstatement which is hereby computed as
compare the appearance of each of the examinees with the person in the of even date as follows:
picture submitted and affixed on the PSP. In cases where the examinee
does not look like the person in the picture submitted and attached on the From 8/23/93-12/15/93 = 3.73 mos.
PSP, the examiner will not allow the said person to take the examination
(CSC Resolution No. 95-5195, Taguinay, Ma. Theresa) P118 x 26 days x 3.73 mos. = P11,443.64
The facts, therefore, that Paitim's photograph was attached over the name
of Gilda Cruz in the PSP of the July 30, 1989 Career Service Examination, 12/16/93 - 3/29/94 = 3.43 mos.
shows that it was Paitim who took the examination.
135 x 26 days x 3.43 mos. = 12,039.30
In a similar case, the Commission ruled:
"It should be stressed that the registered examinee's act of asking or Total Backwages as of 3/29/94 P23,482.94
allowing another person to take the examination in her behalf constitutes
that the evidence on record clearly established that another person took Respondent Camara Steel Industries, Inc. is also ordered to pay
the Civil Service Examination for De Guzman, she should be held liable for complainant 10% for and as attorney's fees.
the said offense."
All other claims are hereby dismissed for lack of merit.
At the outset, it is axiomatic that in the offense of impersonation, two persons are
always involved. In the instant case, the offense cannot prosper without the active On 3 May 1991 petitioner De los Santos started working at Camara Steel Industries
participation of both Arada and de Leon. Thus, the logical conclusion is that de Leon Inc. (CAMARA STEEL), a company engaged in the manufacture of steel products such
took the examination for and in behalf of Arada. Consequently, they are both as LPG cylinders and drums. He was first assigned at the LPG assembly line, then
administratively liable. (Arada, Carolina C. and de Leon, Ponciana Anne M.) 9 later, as operator of a blasting machine. While performing his task as such operator,
he met an accident that forced him to go on leave for one and a half (1-1/2) months.
It can not be denied that the petitioners were formally charged after a finding that Upon his return, he was designated as a janitor assigned to clean the premises of
a prima facie case for dishonesty lies against them. They were properly informed of the company, and occasionally, to transfer scrap and garbage from one site to
the charges. They submitted an Answer and were given the opportunity to defend another.1
themselves. Petitioners can not, therefore, claim that there was a denial of due
process much less the lack of jurisdiction on the part of the CSC to take cognizance On 11 May 1993 petitioner was doing his usual chores as a janitor of CAMARA STEEL
of the case. We do not find reversible error with the decision of the Court of Appeals when he momentarily left his pushcart to answer the call of Narciso Honrado, scrap
in upholding the CSC Resolution. in-charge, who summoned him to the company clinic. There Honrado handed him a
box which he placed on top of a drum in his pushcart for transfer to the other lot of
WHEREFORE, the petition is DENIED. The assailed decision of the Court of Appeals is the company near gate 2. On his way out of gate 2, however, the security guard on
AFFIRMED. SO ORDERED. duty found in the box handed to him by Honrado two (2) pieces of electric cable
measuring 2.26 inches each and another piece of 1.76 meters with a total estimated
value of P50.00 to P100.00. Apprehensive that he might be charged with theft,
G.R. No. 121327 December 20, 2001 petitioner De los Santos explained that the electric cord was declared a scrap by
Honrado whose instructions he was only following to transfer the same to the
CECILIO P. DE LOS SANTOS and BUKLOD MANGGAGAWA NG CAMARA adjacent lot of the company as scrap.
(BUMACA), petitioners,
vs. Narciso Honrado admitted responsibility for the haul and his error in declaring the
NATIONAL LABOR RELATIONS COMMISSION (SECOND DIVISION), HON. electric cables as scrap. The general manager, apparently appeased by Honrado's
COMMISSIONERS VICTORIANO R. CALAYCAY, RAUL T. AQUINO, and ROGELIO apology, issued a memorandum acknowledging receipt of his letter of apology and
I. RAYALA, CAMARA STEEL INDUSTRIES INC., JOSELITO JACINTO, ALBERTO F. exculpated him of any wrongdoing.
DEL PILAR, DENNIS ALBANO, MERCEDITA G. PASTRANA, TOP-FLITE and
Taking an unexpected volte face, however, the company through its counsel filed on
RAUL RUIZ, respondents.
9 July 1993 a criminal complaint for frustrated qualified theft against Honrado and
BELLOSILLO, J.: herein petitioner De los Santos. The complaint however was subsequently dismissed
by the Provincial Prosecutor of Pasig for lack of evidence. 2
This is a petition for certiorari under Rule 65 assailing the Decision of public
respondent National Labor Relations Commission (NLRC) which remanded this case On 23 August 1993, upon request of Top-Flite, alleged manpower agency of De los
to the Labor Arbiter who ruled that petitioner Cecilio P. de los Santos was illegally Santos, CAMARA STEEL terminated his services.

120
Aggrieved by his illegal termination, De los Santos sought recourse with the Labor Camara's Personnel Manager, Dennis Albano, approving; (i) Annex "G-3" Signature
Arbiter who on 29 March 1994 rendered a decision ordering respondent CAMARA of Camara's Department Head where petitioner is working, Mr. Narisma, approving;
STEEL to reinstate Delos Santos to his former position within ten (10) days without (j) Annex "H" to "H-1" Petitioner's Daily Time Card (representative samples) with
loss of seniority rights and other benefits with full back wages from date of dismissal name and logo of Camara Steel Industries Inc.; and, (k) Annex "J" Affidavit of
up to actual reinstatement as herein before stated. Complainant.
CAMARA STEEL went to the NLRC for recourse. Top-Flite filed a Motion for All these pieces of evidence which, according to petitioner De los Santos, were not
Intervention praying that it be permitted to intervene in the appeal as co-respondent properly considered by NLRC, plainly and clearly show that the power of control and
and, accordingly, be allowed to submit its own memorandum and other pleadings. 3 supervision over him was exercised solely and exclusively by the managers and
supervisors of CAMARA STEEL. Even the power to dismiss was also lodged with
On 23 May 1995 the NLRC reversed the Labor Arbiter and ordered the return of the CAMARA STEEL when it admitted in page 3 of its Reply that upon request by Top-
entire records of the case to the arbitration branch of origin for further proceedings. Flite, the steel company terminated his employment after being allegedly caught
In its Decision, NLRC specified the reasons for the remand to the Labor Arbiter 4 committing theft.
First, as respondents have broadly implied, having alleged that he was an Petitioner De los Santos also advances the view that Top-Flite, far from being his
employee of Camara Steel, it was complainant's burden to prove this employer, was in fact a "labor-only" contractor as borne out by a contract whereby
allegation as a fact, not merely through his uncorroborated statements but Top-Flite undertook to supply CAMARA STEEL workers with "warm bodies" for its
through independent evidence. As noted by respondents, he has not factory needs and edifices. He insists that such contract was not a job contract but
submitted one piece of evidence to support his premise on this matter the supply of labor only. All things considered, he is of the firm belief that for all legal
except for his sworn statement. intents and purposes, he was an employee a regular one at that of CAMARA
Secondly, the Arbiter maintained that the contract of services submitted by STEEL.
respondents was insufficient to prove that complainant was an employee of In its comment, private respondent CAMARA STEEL avers that far from being its
Top-Flite, but he has obviously omitted consideration of Annexes F, G, H and employee, De los Santos was merely a project employee of Top-Flite who was
I which are time sheets of the complainant with Top-Flite and the assigned as janitor in private respondent company. This much was acknowledged by
corresponding time cards which he punches in for Camara Steel. Top-Flite in its Motion for Intervention filed before the NLRC.6 Such allegation,
The NLRC further noted that under the circumstances it became appropriate to according to private respondent CAMARA STEEL, supports all along its theory that De
conduct a formal hearing on the particular issue of whether an employer-employee los Santos' assignment to the latter as janitor was based on an independent contract
relationship existed between the parties, which issue was determinative of the executed between Top-Flite and CAMARA STEEL.7
nature of petitioner's dismissal by CAMARA STEEL. That being so, according to the Respondent CAMARA STEEL further argues that crystal clear in the Motion for
NLRC, it was necessary for the Labor Arbiter to issue the appropriate directive to Intervention of Top-Flite is its allegation that it was in fact petitioner's real employer
summon Top-Flite as a necessary party to the case, for the manpower agency to as his salaries and benefits during the contractual period were paid by Top-Flite; not
submit its own evidence on the actual status of petitioner. only that, De los Santos was dismissed by CAMARA STEEL upon the recommendation
As pointed out by petitioner, the errors in the disputed decision by the NLRC are: (a) of Top-Flite. These ineluctably show that Top-Flite was not only a job contractor but
NLRC violated due process of law when it did not consider the evidence on record; was in truth and in fact the employer of petitioner.
(b) CAMARA STEEL, and not Top-Flite, is the real employer of petitioner; (c) Contrary In his petition, De los Santos vigorously insists that he was the employee of
to the finding of NLRC, Top-Flite was made a party respondent in the illegal dismissal respondent CAMARA STEEL which in turn was not only denying the allegation but
case docketed as NLRC-NCR No. 00-08-05302-93 and the NLRC was therefore in was finger-pointing Top-Flite as petitioner's real employer. De los Santos again
error in remanding the case to the Labor Arbiter for further proceedings. objects to this assertion and claims that Top-Flite, far from being an employer, was
Petitioner De los Santos contends that NLRC was in grave error when it ruled that, merely a "labor-only" contractor.
with the exception of a bare assertion on his sworn statement, he "has not In the maze and flurry of claims and counterclaims, several contentious issues
submitted one piece of evidence to support his premise" 5 that he was in fact an continue to stick out like a sore thumb. Was De los Santos illegally dismissed? If so,
employee of CAMARA STEEL. by whom? Was his employer respondent CAMARA STEEL, in whose premises he was
To underscore NLRC's oversight, petitioner brings to our attention and specifies the allegedly caught stealing, or was it Top-Flite, the manpower services which allegedly
pieces of evidence which he presented before the Labor Arbiter on 19 November hired him?
1993 also appended as Annexes to petitioner's "Traverse to Camara's Position Inextricably intertwined in the resolution of these issues is the determination of
Paper and Reply:" (a) Annex "E" to "E-1" Approval signature of Camara's whether there existed an employer-employee relationship between CAMARA STEEL
Department head, Reynaldo Narisma, without which petitioner cannot render and respondent De Los Santos, and whether Top-Flite was an "independent
overtime; (b) Annex "F" Petitioner's daily time record for 8/3/92 to 8/9/92; (c) contractor" or a "labor-only" contractor. A finding that Top-Flite was a "labor-only"
Annex "F-1" Signature of private respondent Mercedita Pastrana, approving in her contractor reduces it to a mere agent of CAMARA STEEL which by statute would be
capacity as Assistant Manager of Camara Steel; (d) Annex "F-2" Signature of responsible to the employees of the "labor-only" contractor as if such employees had
private respondent Dennis Albano, Personnel Manager of Camara Steel Industries been directly employed by the employer.
Inc. also co-signing for approval; (e) Annex "F-3" Signature of Narisma, as
Department Head of Camara Steel Industries Inc. where petitioner is working; (f) Etched in an unending stream of cases are the four (4) standards in determining the
Annex "G" Daily Time Record of petitioner for 7/6/92 to 7/12/92; (g) Annex "G-1" existence of an employer-employee relationship, namely: (a) the manner of selection
Signature of Camara Steel Assistant Manager; (h) Annex "G-2" Signature of and engagement of the putative employee; (b) the mode of payment of wages; (c)

121
the presence or absence of power of dismissal; and, (d) the presence or absence of Of course, it must be stressed that loss of confidence as a just cause for the
control of the putative employee's conduct. Most determinative among these factors termination of employment is based on the premise that the employee holds a
is the so-called "control test." position of trust and confidence, as when he is entrusted with responsibility
involving delicate matters, and the task of a janitor does not fall squarely under this
As shown by the evidence on record, De los Santos was hired by CAMARA STEEL category.
after undergoing an interview with one Carlos Suizo, its timekeeper who worked
under the direct supervision of one Renato Pacion, a supervisor of CAMARA STEEL. Petitioner De los Santos argues that Top-Flite was merely a "labor-only" contractor.
These allegations are contained in the affidavit 8 executed by De los Santos and were To fortify his stance, De los Santos brings to our attention the contract of
never disputed by CAMARA STEEL. Also remaining uncontroverted are the pieces of service9 dated 8 February 1991 between CAMARA STEEL and Top-Flite which
documentary evidence adduced by De los Santos consisting of daily time records provides:
marked Annexes "F" and "G" which, although bearing the heading and logo of Top-
Flite, were signed by officers of respondent CAMARA STEEL, and Annexes "H" and "I" 1) The contractor (Top-Flite) shall provide workers (non-skilled) six (6) days
with the heading and logo of CAMARA STEEL. a week for the Client's (Camara) factory and edifices.

Incidentally, we do not agree with NLRC's submission that the daily time records However, both respondent CAMARA STEEL and Top-Flite10 are adamant in their belief
serve no other purpose than to establish merely the presence of De los Santos that the latter was not a "labor-only" contractor as they rely on another provision of
within the premises of CAMARA STEEL. Contrarily, these records, which were signed the contract which states
by the company's officers, prove that the company exercised the power of control 2) The Contractor warrants the honesty, reliability, industry and cooperative
and supervision over its employees, particularly De los Santos. There is dearth of disposition of the person it employs to perform the job subject to this
proof to show that Top-Flite was the real employer of De los Santos other than a contract, and shall employ such persons only as are in possession of health
naked and unsubstantiated denial by CAMARA STEEL that it has no power of control certificates and police clearances x x x
over De los Santos. Records would attest that even the power to dismiss was vested
with CAMARA STEEL which admitted in its Reply that "Top-Flite requested CAMARA The preceding provisions do not give a clear and categorical answer as regards the
STEEL to terminate his employment after he was caught by the security guard real character of Top-Flite's business. For whatever its worth, the invocation of the
committing theft." contract of service is a tacit admission by both parties that the employment of De
los Santos was by virtue of such contract. Be that as it may, Top-Flite, much less
A cursory reading of the above declaration will confirm the fact that the dismissal of CAMARA STEEL, cannot dictate, by the mere expedient of a unilateral declaration in
De los Santos could only be effected by CAMARA STEEL and not by Top-Flite as the a contract, the character of its business, i.e., whether as "labor-only" contractor, or
latter could only "request" for De los Santos' dismissal. If Top-Flite was truly the job contractor, it being crucial that its character be measured in terms of and
employer of De los Santos, it would not be asking permission from or "requesting" determined by the criteria set by statute. The case of Tiu v. NLRC11 succinctly
respondent CAMARA STEEL to dismiss De los Santos considering that it could very enunciates this statutory criteria
well dismiss him without CAMARA STEEL's assent.
Job contracting is permissible only if the following conditions are met: 1) the
All the foregoing considerations affirm by more than substantial evidence the contractor carries on an independent business and undertakes the contract
existence of an employer-employee relationship between De los Santos and CAMARA work on his own account under his own responsibility according to his own
STEEL. manner and method, free from the control and direction of his employer or
As to whether petitioner De los Santos was illegally terminated from his principal in all matters connected with the performance of the work except
employment, we are in full agreement with the Labor Arbiter's finding that he was as to the results thereof; and 2) the contractor has substantial capital or
illegally dismissed. As correctly observed by the Labor Arbiter, it was Narciso investment in the form of tools, equipment, machineries, work premises,
Honrado, scrap in-charge, who handed the box containing the electrical cables to De and other materials which are necessary in the conduct of the business.
los Santos. No shred of evidence can show that De los Santos was aware of its "Labor-only contracting" as defined in Sec. 4, par. (f), Rule VIII-A, Book III, of
contents, or if ever, that he conspired with Honrado in bilking the company of its the Omnibus Rules Implementing the Labor Code states that a "labor-only"
property. What is certain however is that while Honrado admitted, in a letter of contractor, prohibited under this Rule, is an arrangement where the contractor or
apology, his culpability for the unfortunate incident and was unconditionally forgiven subcontractor merely recruits, supplies or places workers to perform a job, work or
by the company, De los Santos was not only unceremoniously dismissed from service for a principal and the following elements are present: (a) The contractor or
service but was charged before the court for qualified theft (later dismissed by the subcontractor does not have substantial capital or investment to actually perform
public prosecutor for lack of evidence). For sure, De los Santos cannot be held more the job, work or service under its own account or responsibility; and, (b) The
guilty than Honrado who, being the scrap in-charge, had the power to classify the employees recruited, supplied or placed by such contractor or subcontractor are
cables concerned as scrap. performing activities which are directly related to the main business of the principal.
Neither can we gratify CAMARA STEEL's contention that petitioner was validly Applying the foregoing provisions, the Court finds Top-Flite to be a "labor-only"
dismissed for loss of trust and confidence. As provided for in the Labor Code: contractor, a mere supplier of labor to CAMARA STEEL, the real employer. Other than
Art. 282. Termination by employment An employer may terminate an its open declaration that it is an independent contractor, no substantial evidence
employment for any of the following causes: x x x (c) Fraud or willful breach was adduced by Top-Flite to back up its claim. Its revelation that it provided a
by the employee of the trust reposed in him by his employer or duly sweeper to petitioner would not suffice to convince this Court that it possesses
authorized representative x x x x adequate capitalization to undertake an independent business. 12 Neither will the
submission prosper that De los Santos did not perform a task directly related to the
principal business of respondent CAMARA STELL. As early as in Guarin v. NLRC13 we

122
ruled that "the jobs assigned to the petitioners as mechanics, janitors, gardeners,
firemen and grasscutters were directly related to the business of Novelty as a
garment manufacturer," reasoning that "for the work of gardeners in maintaining
clean and well-kept grounds around the factory, mechanics to keep the machines
functioning properly, and firemen to look out for fires, are directly related to the daily
operations of a garment factory."
In its comment respondent CAMARA STEEL emphatically argues that Top-Flite,
although impleaded as respondent in NLRC-NCR Cases Nos. 00-0704761-93 and 00-
0805061-93, subject of the present appeal, was never summoned for which reason it
was deprived of procedural due process; basically the same line of argument
adopted by the NLRC in its decision to remand the case to the arbitration branch of
origin. CAMARA STEEL obviously wants to impress upon us that Top-flite, being a
necessary party, should have been summoned and the failure to do so would justify
the remand of the case to the Labor Arbiter.
We are not persuaded. The records show that Top-Flite was not only impleaded in the
aforementioned case but was in fact afforded an opportunity to be heard when it
submitted a position paper. This much was admitted by Top-Flite in par. 5 of its
Motion for Intervention where it stated that "movant submitted its position paper in
the cases mentioned in the preceding paragraph but the Presiding Arbiter ignored
the clear and legal basis of the position of the movant." 14 In other words, the failure
of Top-Flite to receive summons was not a fatal procedural flaw because it was never
deprived of the opportunity to ventilate its side and challenge petitioner in its
position paper, not to mention the comment which it submitted through counsel
before this Court.15 It moved to intervene not because it had no notice of the
proceedings but because its position paper allegedly was not considered by the
Labor Arbiter. While jurisdiction over the person of the defendant can be acquired by
service of summons, it can also be acquired by voluntary appearance before the
court which includes submission of pleadings in compliance with the order of the
court or tribunal. A fortiori, administrative tribunals exercising quasi-judicial powers
are unfettered by the rigidity of certain procedural requirements subject to the
observance of fundamental and essential requirements of due process in justiciable
cases presented before them. In labor cases, a punctilious adherence to stringent
technical rules may be relaxed in the interest of the workingman. A remand of the
case, as the NLRC envisions, would compel petitioner, a lowly worker, to tread once
again the calvary of a protracted litigation and flagellate him into submission with
the lash of technicality.
WHEREFORE, the petition is GRANTED and the appealed Decision of the NLRC is
REVERSED and SET ASIDE and the Decision of the Labor Arbiter promulgated 23 May
1999 is REINSTATED and ADOPTED as the Decision in this case.
SO ORDERED.
Mendoza, Quisumbing and De Leon, Jr., JJ ., concur.
Buena, J ., on official business

123
EMIN VS. DE LEON ET AL During the hearing, the six teachers cited in the charge sheet, namely: Eufrocina
Sicam, Ma. Elisa Sarce, Lilia Millondaga, Merla Entiero, Lourdes Limbaga and Florida
G.R. No. 139794 February 27, 2002 P. Alforjas were presented as witnesses for the prosecution. Felixberta Ocho and
Araceli G. Delgado who were also holders of fake certificates of eligibility were
MARTIN S. EMIN, petitioner, likewise presented as witnesses.
vs.
CHAIRMAN CORAZON ALMA G. DE LEON, COMMISSIONERS THELMA P. Alforjas and Delgado identified petitioner and a certain Teddy Cruz as the persons
GAMINDE and RAMON P. ERENETA, JR., of the CIVIL SERVICE who facilitated their applications for R.A. 6850 eligibility. The other witnesses
COMMISSION, respondents. corroborated Alforjas and Delgados testimonies. They all identified petitioner as the
person who helped them obtain the fake certificates of eligibility.
DECISION
On June 29, 1994, Director Buenaflor submitted a report 5 to the Chairman of the Civil
QUISUMBING, J.: Service Commission. The CSC found that there was sufficient evidence to warrant
the conviction of petitioner. On May 14, 1996, the Civil Service Commission in its
This is a petition to review the decision dated October 30, 1998 of the Court of resolution decreed:
Appeals in CA-G.R. S.P. No. 46549, affirming Civil Service Commission Resolution
Nos. 96-3342 and 97-4049 finding petitioner Martin Emin, guilty of dishonesty, grave WHEREFORE, Martin S. Emin is hereby found guilty of Grave Misconduct.
misconduct and conduct prejudicial to the best interest of the service, and Accordingly, the penalty of dismissal from the service including all its accessory
dismissing him from the service as Non-Formal Education (NFE) Supervisor of the penalties is imposed upon him.6
Department of Education, Culture and Sports (DECS), Kidapawan, Cotabato.
Not satisfied with the abovecited resolution, the petitioner filed a motion for
The facts are as follows: reconsideration,7 but it was denied.
Sometime in the year 1991, appointment papers for a change of status from On January 16, 1998, petitioner elevated the case to the Court of Appeals, but it was
provisional to permanent under Republic Act No. 6850 of teachers were submitted to dismissed for failure to comply with Section 5, Rule 43 of the 1997 Rules of Civil
the Civil Service Field Office-Cotabato at Amas, Kidapawan, Cotabato. Attached to Procedure.8
these appointment papers were photocopies of certificates of eligibility of the
teachers. However, the CA granted petitioners motion for reconsideration 9 and time to amend
his petition.10 In his amended petition, he raised before the CA the twin issues of (1)
Director Gantungan U. Kamed noticed that the certificates of eligibility were of whether the CSC had original jurisdiction over the administrative cases against the
doubtful authenticity. He called the Head Civil Service Field Officer. While the public school teachers; and (2) whether petitioner was accorded due process. 11
certificates seemed to be authentic, the signature of Civil Service Commission
Director Elmer R. Bartolata and the initials of the processors of said certificates were Finding the petition unmeritorious, the appellate court ruled on the appeal, thus:
clearly forgeries. Director Kamed initially forwarded five (5) appointments to Civil WHEREFORE, premises considered, the petition (appeal) is DISMISSED, hereby
Service Regional Office No. XII for verification of their R.A. 6850 eligibilities and for affirming public respondents assailed appealed resolutions (Resolution No. 963342,
appropriate action through an indorsement letter dated September 26, 1991. The dated May 14,1996; and Resolution No. 974049, dated October 14, 1997).
appointment papers of the same nature subsequently submitted to the Field Office
were likewise forwarded to the CSRO No. XII. SO ORDERED.12

Upon verification of the records of CSRO No. XII, it was found that said applications Petitioner is now before us raising the following issues:
for civil service eligibility under R.A. 6850 were disapproved. However, the
I. WHETHER OR NOT THE COURT OF APPEALS ERRED IN FINDING THAT THE
certificates of eligibility they submitted were genuine as their control number
CIVIL SERVICE COMMISSION HAS ORIGINAL JURISDICTION OVER
belonged to the batch issued to CSRO No. XII by the CSC Central Office. But the
ADMINISTRATIVE CASES AGAINST PUBLIC SCHOOL TEACHERS.
records showed that these certificates were never issued to any one.
II. WHETHER OR NOT THE COURT OF APPEALS ERRED IN NOT FINDING THAT
Two separate investigations1 were conducted by Director Cesar P. Buenaflor of
THE PETITIONER WAS NOT ACCORDED HIS RIGHT TO DUE PROCESS.
Regional Office No. 12 of the Civil Service Commission in Cotabato City: (1) on how
the R.A. 6850 certificates were issued/released from the Office, and (2) on how the III. WHETHER OR NOT THE COURT OF APPEALS ERRED IN FINDING THAT
teachers got said certificates. The teachers concerned were asked to report to the THERE WAS SUFFICIENT GROUND TO DISMISS THE PETITIONER FROM
Office and bring the original copies of their certificates of eligibility. On several SERVICE.
dates, the teachers appeared and gave their sworn statements pointing to petitioner
as the person who gave them the R.A. 6850 certificates of eligibility they had IV. WHETHER OR NOT THE COURT OF APPEALS ERRED IN NOT ADMITTING
attached to their appointments for a fee. Upon finding a prima facie case, petitioner THE NEWLY DISCOVERED EVIDENCE.13
was formally charged with dishonesty, grave misconduct and conduct prejudicial to
Notwithstanding petitioners formulation, we find that the issues to be resolved are:
the best interest of the service.2
(1) whether or not the CSC has original jurisdiction over the present case; and (2)
In his sworn letter dated April 8, 1992 to the CSC Regional Director, petitioner denied whether or not petitioner was accorded due process.
the accusation.3 He filed a motion to dismiss, dated June 5, 1992,4 but the motion
Petitioner avers that as a teacher, original jurisdiction over the administrative case
was denied on July 8, 1992.
against him is lodged with a committee and not with the CSC, as provided for by

124
Republic Act 4670 otherwise known as the "Magna Carta for Public School Teacher," to other matters.19 Exclusio unios est inclusio alterius. Had Congress intended to
specifically, Section 9 thereof, which provides: exclude an NFE Division Supervisor from the coverage of R.A. 4670, it could have
easily done so by clear and concise language.
Sec. 9. Administrative Charges.- Administrative charges against a teacher shall be
heard initially by a committee composed of the corresponding School As petitioner is covered by R.A. 4670, it is the Investigating Committee that should
Superintendent of the Division or a duly authorized representative who should at have investigated his case conformably with Section 9 of R.A. 4670, now being
least have the rank of a division supervisor, where the teacher belongs, as chairman, implemented by Section 2, Chapter VII of DECS Order No. 33, S. 1999, otherwise
a representative of the local, or, in its absence, any existing provincial or national known as the DECS Rules of Procedure.20
teachers organization and a supervisor of the Division, the last two to be designated
by the Director of Public Schools within thirty days from the termination of the However, at this late hour, the proceedings conducted by the public respondent CSC
hearings: Provided, however, That where the school superintendent is the can no longer be nullified on procedural grounds. Under the principle of estoppel by
complainant or an interested party, all the members of the committee shall be laches, petitioner is now barred from impugning the CSCs jurisdiction over his case.
appointed by the Secretary of Education. But we must stress that nothing herein should be deemed as overriding the
For public respondent CSC, the Office of the Solicitor General maintains that original provision in the Magna Carta for Teachers on the jurisdiction of the Committee to
jurisdiction over the present case is with the CSC pursuant to the Constitution and investigate public school teachers as such, and the observance of due process in
P.D. 807 (Civil Service Law) which provide that the civil service embraces every administrative proceedings involving them, nor modifying prior decided cases of
branch, agency, subdivision, and instrumentality of the government, including teachers on the observance of the said Magna Carta such as Fabella vs. Court of
government-owned or controlled corporations whether performing governmental or Appeals.21
proprietary function. Here what is crucial, in our view, is that the Civil Service Commission had afforded
We find merit in petitioners contention that R.A. 4670 is good law and is applicable petitioner sufficient opportunity to be heard and defend himself against charges of
to this case. R.A. 4670 has not been expressly repealed by the general law P.D. 807, participation in faking civil service eligibilities of certain teachers for a fee. Not only
nor has R.A. 4670 been shown to be inconsistent with the presidential did he answer the charges before the CSC Regional Office but he participated in the
decree.14 Section 2 thereof specified those who are covered by the term "teacher" as hearings of the charges against him to the extent that we are left with no doubt that
follows: his participation in its proceedings was willful and voluntary.

SEC. 2. Title Definition. - This Act shall be known as the "Magna Carta for Public As held previously, participation by parties in the administrative proceedings without
School Teachers" and shall apply to all public school teachers except those in the raising any objection thereto bars them from raising any jurisdictional infirmity after
professorial staff of state colleges and universities. an adverse decision is rendered against them. 22 In the case at bar, petitioner raised
the issue of lack of jurisdiction for the first time in his amended petition for
As used in this Act, the term "teacher" shall mean all persons engaged in classroom review23 before the CA. He did not raise this matter in his Motion to Dismiss 24 filed
teaching, in any level of instruction, on full-time basis, including guidance before the CSC Regional Office. Notably, in his Counter-Affidavit, he himself invoked
counselors, school librarians, industrial arts or vocational instructors, and all other the jurisdiction of the Commission by stating that he was "open to further
persons performing supervisory and/or administrative functions in all schools, investigation by the CSC to bring light to the matter" 25 and by further praying for
colleges and universities operated by the Government or its political "any remedy or judgment which under the premises are just and equitable." 26 It is an
subdivisions; but shall not include school nurses, school physicians, school dentists, undesirable practice of a party participating in the proceedings, submitting his case
and other school employees. for decision, and then accepting the judgment only if favorable, but attacking it for
lack of jurisdiction, when adverse.27
Petitioner is the Non-Formal Education Supervisor of the DECS, in Kidapawan,
Cotabato, in-charge of the out-of-school programs. 15 The 1993 Bureau of Non-formal Equally unmeritorious is petitioners contention that he was denied due process. He
Education Manual16 outlines the functions of a NFE Division Supervisor which include, avers that he was not allowed cross-examination. It is well to remember that in
"(5) implementation of externally assisted NFE programs and projects; (6) administrative proceedings, technical rules of procedure and evidence are not
monitoring and evaluation of NFE programs and projects (8) supervision of the strictly applied and administrative due process cannot be fully equated with due
implementation of NFE programs/projects at the grassroots level." 17 Clearly, process in its strict judicial sense.28
petitioner falls under the category of "all other persons performing supervisory
and/or administrative functions in all schools, colleges and universities operated by Nothing on record shows he asked for cross-examination as most of the submissions
the government or its political subdivisions." were written. In our view, petitioner cannot argue that he has been deprived of due
process merely because no cross-examination took place. The rule is well
Under Section 2 of R.A. 4670, the exclusions in the coverage of the term "teachers" established that due process is satisfied when the parties are afforded fair and
are limited to: (1) public school teachers in the professorial staff of state colleges reasonable opportunity to explain their side of the controversy or given opportunity
and universities; and (2) school nurses, school physicians, school dentists, and other to move for a reconsideration of the action or ruling complained of. 29 In the present
school employees under the category of "medical and dental personnel". Under the case, the record clearly shows that petitioner not only filed his Counter-
principle ofejusdem generis, general words following an enumeration of persons or Affidavit30 during the preliminary investigation, and later his Motion to Dismiss. 31 He
things, by words of a particular and specific meaning, are not to be construed in also filed a Motion for Reconsideration32 of the October 19, 1993 Order of the
their widest extent, but are to be held as applying only to persons or things of the Commission. The essence of due process in administrative proceedings is an
same kind or class as those specifically mentioned.18 Too, the enumeration of opportunity to explain ones side or an opportunity to seek reconsideration of the
persons excluded from the coverage of the term "teachers" is restricted, limited and action or ruling complained of.33
exclusive to the two groups as abovementioned. Where the terms are expressly
limited to certain matters, it may not by interpretation or construction be extended

125
Neither is there merit in petitioners assertion that he was denied the right to due
process when the CSC Regional Office, according to him, acted as investigator,
prosecutor, judge and executioner. He laments that Director Buenaflor who formally
filed the charge nominally was also the hearing officer, and that prosecutor Atty.
Anabelle Rosell was also the one who submitted the recommendation to the CSC for
the dismissal of petitioner. Recall, however, that it was ultimately the Civil Service
Chairman who promulgated the decision. The report submitted by Atty. Rosell based
on the hearing where Director Buenaflor sat as hearing officer, was merely
recommendatory in character to the Civil Service Commission itself. Such procedure
is not unusual in an administrative proceeding.1wphi1
Petitioner claims that there was no valid case to dismiss him as Director Elmer
Bartolata was not presented to ascertain the alleged forged signature contained in
the questioned certificates of eligibility. The Court of Appeals and the Civil Service
Commission made a finding on this fact of forgery. It is not this Courts function now
to evaluate factual questions all over again. This is particularly true in this case,
where the Commission and the appellate court agree on the facts. 34
Lastly, petitioner contends that the affidavit of Teodorico Cruz 35 should have been
admitted as newly discovered evidence. Petitioner raised this issue for the first time
on appeal, when he filed his Motion for New Trial and to Admit Newly Discovered
Evidence before the CA. For a particular piece of evidence to be regarded as "newly
discovered" for purposes of a new trial, it is essential that the offering party had
exercised reasonable diligence in seeking to locate such evidence before or during
trial but had nonetheless failed to secure it. The OSG 36 observed that despite the
knowledge of the importance of Mr. Cruzs testimony on the matter, petitioner did
not ask for a subpoena duces tecum to obtain said "newly discovered evidence."
Neither did petitioner, on his own, secure said affidavit or testimony during the
proceedings to support his cause. We note too, that the said affidavit attempts to
exonerate the petitioner and Cruz and points to someone else ("Jing") as the culprit,
leaving the impression that the idea of the affidavit was a mere afterthought, a last
ditch effort to clear petitioners name. Thus, we are not persuaded by petitioners
claim of newly discovered evidence, for it appears to us as a dilatory contrivance for
petitioners benefit.
WHEREFORE, there being no reversible error committed by the Court of Appeals
and the respondent officials of the CSC, the instant petition is hereby DENIED. The
Decision dated October 30, 1998 of the Court of Appeals in CA-G.R. S.P. No. 46549 is
AFFIRMED. Costs against petitioner.
SO ORDERED.
Davide, Jr., C.J., Bellosillo, Melo, Puno, Vitug, Kapunan, Mendoza, Panganiban, Buena,
Ynares-Santiago, De Leon, Jr., Sandoval-Gutierrez, and Carpio, JJ., concur.

126
JAYLO ET AL. VS. SANDIGANBAYAN mortal wounds on his body which directly and immediately caused his
(victim) death."2
G.R. Nos. 111502-04* November 22, 2001 Before the trial of the criminal cases, or on February 16, 1993, petitioners filed a
REYNALDO H. JAYLO, WILLIAM VALENZONA, ANTONIO HABALO and motion3 praying that they be allowed to take oral depositions of the three (3) DEA
EDGARDO CASTRO, petitioners, agents before a consular official of the Philippine Embassy stationed in the United
vs. States of America (USA). The agents residential addresses 4 are as follows:
SANDIGANBAYAN (First Division), respondent. Andrew Fendrich - DEA Headquarters, Boston, Massachusets
SANDOVAL-GUTIERREZ, J.: Phil Needham - San Francisco, California
This petition for certiorari1 seeks the nullification of two Resolutions of the Jake Fernandez - DEA Headquarters, Boston, Massachusets
Sandiganbayan (First Division) promulgated on March 9, 1993 and August 10, 1993,
in Criminal Cases Nos. 17984 17986, denying petitioners "Motion To Take Oral The motion is basically premised on the following grounds:
Deposition Outside The Philippines."
(a) Accused is entitled under the Constitution to secure the attendance of
Petitioners Reynaldo Jaylo, William Valenzona, Antonio Habalo and Edgardo Castro witnesses and the production of evidence in his behalf;
were former members of the defunct Philippine Constabulary-Integrated National
(b) Section 4, Rule 119 of the 1988 Revised Rules on Criminal Procedure
Police (PC-INP), assigned at the Western Police District (WPD), Manila, but detailed
recognizes the right of the accused, upon motion with notice, to have
with the National Bureau of Investigation (NBI).
witnesses conditionally examined in his behalf;
On July 10, 1990, petitioners figured in a shoot-out incident at the Magallanes
(c) Due to the limitations of Rule 119, the Sandiganbayan, in the interest of
Commercial Complex, Makati City, which resulted in the death of Colonel Rolando de
substantive justice, should liberally allow the utilization of the pertinent
Guzman, Major Franco Calanog (both members of the Philippine Army) and Avelino
provisions of Rule 24 on Depositions and Discovery of the Revised Rules of
Manguerra (reportedly a civilian agent of the Criminal Investigation Services [CIS]).
Court to secure the testimony of said probable witnesses who are willing to
The NBI and CIS were then tasked to investigate the incident. Both agencies testify outside the Philippines; and
submitted conflicting findings.
(d) The taking of depositions before the Philippine consulate abroad does
The NBI reported that the shooting incident was a consequence of the NBI-WPD Joint not raise any extraterritorial problems, particularly on the jurisdiction of the
Heroin Drug Buy-Bust Operation headed by petitioner Reynaldo Jaylo, in close Sandiganbayan.
coordination with the agents of the Drug Enforcement Agency (DEA) of the United
The pertinent allegations in support of the motion state:
States of America (USA). The DEA agents were identified as Phil Needham, Andrew
Fendrich and Jack Fernandez, all American citizens and residents of the USA. The NBI "In the instant case, there are three probable witnesses whose testimonies
further reported that in the course of effecting the arrest of the suspects, the latter are of utmost materiality in maintaining the innocence of the accused of
fired upon petitioners who retaliated in self-defense. the crime charged. As such, their attendance and production of other
evidence in their possession are indispensable to the speedy, just and
Upon the other hand, the CIS concluded that petitioners shot the victims at close
impartial trial of the accused.
range, without any chance to defend themselves.
"These witnesses, Messrs. Phil Needham, Andrew Fendrich and Jake
Due to the conflicting versions of the two agencies, then President Corazon C.
Fernandez, are Drug Enforcement Administration (DEA) agents who planned
Aquino created a Presidential Fact-Finding Committee headed by Magdangal Elma
and coordinated with the National Bureau of Investigation (NBI) regarding
(the "Elma Committee") to investigate the incident.
the drug-buy bust operations involving the accused and the deceased.
After hearing the testimonies of 44 witnesses, the Elma Committee recommended
"Specifically, Mr. Fendrich was the head of the DEA in the Philippines who
the prosecution of four (4) participants in the drug buy-bust operation.
approached and informed then NBI Director Alfredo Lim of ten kilos of
Consequently, on September 12, 1992, the Office of the Special Prosecutor filed with heroin being offered for sale [t.s.n., 26 July 1990, Tuazon & Regodon (Elma
the Sandiganbayan three (3) Amended Informations for murder against the Committee). p. 11]. Likewise, he was the one who coordinated and planned
petitioners. The Informations contain the following similarly worded allegations: with one of the accused, Reynaldo Jaylo, to entrap the sellers of said
prohibited drugs, from 2 July 1990 until the fateful shooting incident of 10
"That on or about July 10, 1990, and for sometime prior thereto, in Makati, July 1990. As part of strategy, Fendrich invited another American agent of
Metro Manila, Philippines, and within the jurisdiction of this Honorable the DEA, Phil Needham, to act as a poseur-buyer in order to establish
Court, the above-named accused, all public officers, being then members of contact with, and eventually apprehend, the suspects [t.s.n., 26 July 1990,
the Western Police District (WPD) on special detail with the National Bureau Tuazon and Regodon (Elma Committee), p. 33. He was also primarily
of Investigation (NBI), conspiring, confederating and mutually helping one responsible for the videotaping of the negotiations between the poseur-
another, on the occasion of a police operation, thus committing the offense buyer, the go-between, and the sellers inside one of the rooms of Philippine
in relation to their duties, together with several John Does, did then and Plaza Hotel.
there willfully, unlawfully and with intent to kill, evident premeditation and
abuse of superior strength and treachery, suddenly and simultaneously and "Mr. Phil Needham, on the other hand, is a major player in the said drug
with the use of their firearms attack and shoot (i.e., Calanog, etc.) causing buy-bust operation. He acted as the poseur-buyer and negotiated with the

127
agent of the deceased, Ms. Estrella Arrastia, and met Rico and Rolly, who addition to the accused, other witnesses are available on the same facts.
turned out to be Major Calanog and Colonel de Guzman respectively (Elma There is also no showing that the alleged videotapes and memoranda in
Committee Report, pp. 97-98). Above all these considerations, he was the possession of the United States Drug Enforcement Administration (DEA)
personally present at the scene of the alleged crime. Certainly, he could chronicling the events prior to and during the shooting incident cannot be
shed light in ferreting out the truth in order to maintain the innocence of made available except through the proposed deponents (Fajardo vs.
the accused. Garcia, 98 SCRA 514 [1980] and Manila Railroad Co. vs. Mitchel, 49 Phil.
801 [1926]).
"Lastly, Mr. Jake Fernandez was the unidentified driver of a Toyota car with
diplomatic plate who picked up Needham at the scene of the drug buy-bust, "Hence, no necessity appears for the conditional examination of defense
as alluded to in the Elma Committee Report (p. 100). He was actually at the witnesses under Sections 4 and 5, Rule 119, 1985 Rules on Criminal
Magallanes Commercial Center to serve us a back-up security for Needham Procedure, assuming that as argued extensively by the accused, the
in case any untoward incident occurs. During the drug buy-bust operations, conditional examination provided in those sections, supplemented by
Mr. Fernandez was in his car parked near the Volkswagen and Saab cars of pertinent provisions of Rule 24, Revised Rules of Court, may be validly held
the deceased. He was in a vantage point where he saw the events that outside the Philippines."7 (Emphasis ours)
precipitated the shootings. His testimony, therefore, is of utmost
importance in strengthening the defense of the accused against the crime In his concurring opinion, 8 Presiding Justice Garchitorena stated additional grounds
charged. for the denial of the motion, to wit:

"Aside from these indispensable testimonial evidence, the DEA has in its "a) The unwillingness of the intended witnesses of the defense to come to
possession videotapes and memoranda which chronicled the events prior to the Philippines to testify since they allegedly are afraid for their lives, is not
and during the alleged shooting incident. a valid ground to authorize the deposition of their testimony where they
want it to be taken;
"The need to present the testimonies of Fendrich, Needham and Fernandez,
including the documents and videotapes in their possession, becomes more b) The power of the court in criminal cases is restricted by its territorial
imperative considering that the Elma Committee had discredited the jurisdiction;
testimonies of Apollo Jacinto and Ramon Zuiga, Jr., the only witnesses c) Venue in criminal cases, unlike in civil cases, is not consensual; and
presented who claimed to have seen the actual incident.
d) The taking of the testimony of the DEA agents in the US would unduly
"It is unfortunate, however, that the aforementioned evidence are beyond burden the adverse party, the Philippine Government, with extra-ordinary
the jurisdiction of this Honorable Court. Messrs. Fendrich, Needham and inconvenience and extra-ordinary cost. "
Fernandez are residents and citizens of the United States of America who
have previously expressed to accused Jaylo their refusal to testify in the Dissatisfied, petitioners filed a motion for reconsideration 9 and a
Philippines for security reasons. Documents, including the videotapes supplement10 thereto, contending that:
chronicling the events prior to and during the alleged shootings, are in their
"a) There are clear and imminent threats to the lives and safety of the DEA
possession and need to be identified by them in order to be presented as
agents if they come to the Philippines. The proposed deposition of their
evidence.
testimonies, although corroborative in nature, are absolutely necessary in
"Yet, the importance of said evidence to the defense of the accused is so the absence of credible and competent persons, besides the accused, who
encompassing that the failure to present the same would be tantamount to were actually present at the time of the alleged shooting incident;
a serious violation of the constitutional rights of the accused and may result
b) The Motion does not pray for the issuance of the necessary compulsory
to a serious miscarriage of justice. Indeed, the noble ends of this Court to
process in a foreign court but merely asks to validate certain acts and/or
search for truth and dispense justice might not be achieved in the absence
deeds taken abroad in order to be admissible as competent evidence in a
of said evidence.
court of law;
"But this dilemma can still be solved because of the willingness of said
c) Venue should not be an issue since the parties have not agreed to
witnesses to testify and produce the necessary documents and videotapes
change and/or alter the same;
outside the country. The present case, therefore, presents less a question
of availment by the accused of compulsory processes, as it raises the issue d) The government need not incur expenses as the Honorable Court may
of alternative, suppletory remedies if such witnesses are beyond such deputize and direct the concerned consulate to be the prosecutions
compulsory processes. x x x."5 representative in accordance with the pertinent provisions of the Rules of
Court;
Ruling that there are other witnesses available in the Philippines to testify on the
same facts sought to be proved by the proposed deponents, the Sandiganbayan e) Oral depositions in civil procedure under Rule 24 of the Revised Rules of
(First Division) denied the motion in a Resolution6 dated March 3, 1993 promulgated Court must be allowed in criminal cases due to the civil liabilities that are
on March 9, 1993. The pertinent portions of the Resolution read: impliedly instituted therewith;
"The Motion to Take Oral Deposition Outside the Philippines dated f) Memorandum Circular No. 13 of the Supreme Court, promulgated on 1
February 15, 1993, of all the accused is denied, there being no showing July 1987, encourages the availment of discovery procedures by the parties
that the proposed deponents are the only witnesses upon the facts sought without any distinction as to the kind of cases, be they civil or criminal
to be proved. On the contrary, it would appear from the record that in case; and

128
g) The denial of the motion to take oral deposition would be tantamount to events prior to and during the shooting incident could not be produced except
excluding such testimony as evidence notwithstanding the fact that its through the same deponents.
admissibility is guaranteed by the Revised Rules on Evidence.
We have carefully observed the finding of the Sandiganbayan specified in its
Petitioners motion for reconsideration and the supplement thereto were opposed by assailed Resolution of August 10, 1993 that the reasons for denying the motion were
the prosecution. "not disputed by the accused (now petitioners); on the contrary, they expressly
confirmed it when they conceded that for the most part, deponents testimony are
In a Resolution11 promulgated on August 10, 1993, the Sandiganbayan denied the corroborative in nature."14
motion for reconsideration.
We also noted that, in their motion for reconsideration of the first assailed Resolution
Hence, this petition, assailing the two (2) Resolutions of the Sandiganbayan as promulgated March 9, 1993, petitioners sole reason why they filed the motion for
having been issued "with grave abuse of discretion amounting to excess of oral deposition is that:
jurisdiction."12
"The shooting incident itself in Magallanes needs the collaborative
Petitioners contend inter alia that their "Motion To Take Oral Deposition Outside the testimony of these foreign deponents; otherwise it might boil down to a
Philippines" "finds support in Sections 4 and 5, Rule 119 of the 1988 Revised Rules of my-word-against-yours situation.
Criminal Procedure."13 Both Sections 4 and 5 are now incorporated in Sections 12
and 13, Rule 119 of the Revised Rules of Criminal Procedure, as amended, effective "That while it is conceded that for the most part, deponents testimonies
December 1, 2000, which provide: are corroborative in nature, even then, however, that does not deter, or
lessen any, its probative value." 15 (Emphasis ours)
"SEC. 12. Application for examination of witness for accused before trial. -
When the accused has been held to answer for an offense, he may, upon It is clear that, although the proposed deponents testimonies are admittedly
motion with notice to the other parties, have witnesses conditionally corroborative in nature, what prompted petitioners to file the motion for oral
examined in his behalf. The motion shall state: (a) the name and residence deposition was their seeming apprehension that the Sandiganbayan might not
of the witnesses; (b) the substance of his testimony; and (c) that the consider their very own testimonies credible. Petitioners posture is certainly
witness is sick or infirm as to afford reasonable ground for believing that he speculative and cannot be a valid ground for seeking an oral deposition. In fact, such
will not be able to attend the trial, or resides more than one hundred (100) apprehension contravenes the legal presumption that a trial judge can fairly weigh
kilometers from the place of trial and has no means to attend to the same, and appraise the evidence submitted by the respective parties. 16
or that other similar circumstances exist that would make him unavailable
or prevent him from attending the trial. The motion shall be supported by Moreover, petitioners bare allegation that the DEA American agents cannot come to
an affidavit of the accused and such other evidence as the court may the Philippines to testify for security reasons, is not a compelling justification to take
require. (4a)" (Emphasis ours) their deposition in the US. The taking of deposition in criminal cases may be allowed
only in exceptional situation in order to prevent a failure of justice.
"SEC. 13. Examination of defense witness; how made. - If the court is
satisfied that the examination of a witness for the accused is necessary, an In the case at bar, there is no showing that the DEA agents could not attend the trial.
order shall be made directing that the witness be examined at a specific It is quite unusual and preposterous that the said agents who, by the nature of their
date, time and place and that a copy of the order be served on the profession, are used to risking their lives to apprehend and prosecute drug
prosecutor at least three (3) days before the scheduled examination. The traffickers, suddenly refused to testify in a case wherein they have a vital role.
examination shall be taken before a judge, or, if not practicable, a member We thus hold that in issuing the assailed Resolutions, the Sandiganbayan did not act
of the Bar in good standing so designated by the judge in the order, or if with grave abuse of discretion. It is manifest that the court was not convinced that
the order be made by a court of superior jurisdiction, before an inferior the examination of those witnesses is necessary. In fact, it was aware that the
court to be designated therein. The examination shall proceed proposed deponents testimonies would be merely corroborative in nature.
notwithstanding the absence of the prosecutor provided he was duly
notified of the hearing. A written record of the testimony shall be taken. Petitioners cannot likewise feign denial of due process since they admitted that they
(5a)" (Emphasis ours) have other witnesses to testify on the same facts sought to be testified to by the
proposed deponents.
Under Section 13, the application for oral deposition in a criminal case shall be
granted only "(i)f the court is satisfiedthat the examination of a witness for the WHEREFORE, the petition is DISMISSED. Costs against petitioners.
accused is necessary". Thus, the determination of whether or not an oral deposition
SO ORDERED.
of a defense witness is necessary is addressed to the sound discretion of the court
where the application was made. Petitioners assert that such discretion was gravely Davide, Jr., C.J., Bellosillo, Melo, Puno, Vitug, Kapunan, Mendoza, Panganiban,
abused by the Sandiganbayan. Quisumbing, Buena, Ynares-Santiago, De Leon, Jr., and Carpio, JJ., concur.
Pardo, J., no part.
We do not agree.
The Sandiganbayan correctly denied petitioners motion. It properly ruled that no
necessity existed for the conditional examination of the three (3) proposed
witnesses for the defense because (a) other witnesses appearing on record are
available to testify on the same facts on which the proposed deponents would
testify; and that (b) petitioners failed to show that the video tapes recording the

129
MARCIALES VS. COURT OF APPEALS However, after the recess, the public prosecutor declined to present the NBI agent,
and instead manifested that he was not presenting any further evidence. The
G.R. No. 124171 March 18, 2002 defense then moved that the cases be deemed submitted for decision, and asked
leave of court to file a demurrer to evidence.
LETICIA R. MERCIALES, petitioner,
vs. On August 29, 1994, the Solicitor General filed [in G.R. No. 113273-78] a motion for
THE HONORABLE COURT OF APPEALS, THE PEOPLE OF THE PHILIPPINES, issuance of a writ of preliminary injunction or temporary restraining order with the
JOSELITO NUADA, PAT. EDWIN MORAL, ADONIS NIEVES, ERNESTO LOBETE, Supreme Court, to enjoin the respondent judge from proceeding with the resolution
DOMIL GRAGEDA, and RAMON "POL" FLORES,respondents. of the case. However, on September 19, 1994, the motion was denied by the
Supreme Court.
YNARES-SANTIAGO, J.:
In due time, the accused filed their demurrer to evidence x x x. 4
Petitioner seeks the reversal of the Decision of the Court of Appeals 1 in CA-G.R. SP
No. 37341, denying her petition to annul the Order of the Regional Trial Court of On October 21, 1994, the trial court issued the assailed Order, the dispositive
Legazpi City, Branch 8,2 in Criminal Case Nos. 6307-6312, which dismissed the portion of which reads:
charge of rape with homicide based on a demurrer to evidence filed by private For lack of sufficient evidence to prove the guilt of the accused beyond
respondents, accused therein. reasonable doubt, all the accused in all these cases are hereby ACQUITTED
The antecedent facts as succinctly synthesized by the respondent court are as and the cases filed against them are hereby DISMISSED. The accused in all
follows: these cases, being detention prisoners, are hereby ordered RELEASED from
detention, unless they are being detained for some other legal cause.
On August 12, 1993, Criminal Case Nos. 6307, 6308, 6309, 6310, 6311, and 6312,
for rape with homicide, in connection with the death of one Maritess Ricafort SO ORDERED.5
Merciales, were filed against the private respondents, Joselito Nuada, Pat. Edwin Petitioner Leticia Merciales, who is the mother of the victim in the said criminal
Moral, Adonis Nieves, Ernesto Lobete, Domil Grageda and Ramon "Pol" Flores, before cases, filed before the respondent Court of Appeals a petition to annul the foregoing
the Regional Trial Court, Fifth Judicial Region, Legaspi City. The said cases were Order of the trial court. However, the Court of Appeals dismissed the petition on
consolidated in Branch 8, presided over by the respondent judge. October 4, 1995.
During the trial, after presenting seven witnesses, the public prosecutor filed a A motion for reconsideration was denied on March 6, 1996. Hence, the instant
motion for the discharge of accused Joselito Nuada, in order that he may be utilized petition based on the ground that:
as a state witness. However, the prosecution contended that it was not required to
present evidence to warrant the discharge of accused Nuada, since the latter had THE COURT OF APPEALS COMMITTED A REVERSIBLE ERROR WHEN IT
already been admitted into the Witness Protection Program of the Department of REFUSED TO NULLIFY THE ORDER DATED OCTOBER 21, 1994 OF THE TRIAL
Justice. Consequently, the respondent judge denied the motion for discharge, for COURT FOR BEING NULL AND VOID ON THE GROUND THAT THE TRIAL
failure of the prosecution to present evidence as provided for by Section 9, Rule 119 JUDGE TOLERATED AND/OR COMMITTED INJUSTICE BY FAILING TO REQUIRE
of the 1985 Rules on Criminal Procedure. THE PROSECUTION TO PRESENT ALL THEIR EVIDENCE INSTEAD OF
SUPPRESSING THEM APPARENTLY TO FAVOR THE ACCUSED IN VIOLATION OF
On December 22, 1993, the prosecution filed a petition 3 for certiorari [G.R. No. THE CONSTITUTIONAL RIGHT OF THE PEOPLE TO DUE PROCESS, OUSTING
113273-78] before the Supreme Court, questioning the respondent judge's denial of THE TRIAL COURT OF ITS JURISDICTION.6
the motion to discharge the accused Nuada. Despite the fact that the petition did
not contain a prayer for a temporary restraining order, the trial judge did not set the The case was set for oral argument on December 11, 2001. Counsel for petitioner
case for further hearing so as to give the prosecution time to secure such temporary and the Solicitor General appeared. During the oral argument, the Solicitor General
restraining order from the Supreme Court. manifested that he was joining the cause of petitioner in order to prevent a
miscarriage of justice. The Court directed the parties to submit their respective
On July 13, 1994, herein private respondents filed a motion to set the case for memoranda in amplification of the points raised during the oral argument.
hearing, invoking their constitutional right to speedy trial. The respondent judge
granted the motion, and set the case for hearing on July 29, 1994. Petitioner maintains that the reopening of the criminal case will not violate the
accused's right to double jeopardy. More particularly, she ascribes prosecutorial and
On the said date, the prosecution filed a motion for reconsideration, instead of judicial misconduct in the undue haste which attended the prosecution's premature
presenting further evidence. The respondent Judge postponed the hearing and reset resting and the trial court's grant of the demurrer to evidence when the presentation
the same for August 9, 1994. of the evidence for the prosecution has not been completed.
On August 9, 1994, again the prosecution filed a motion for reconsideration, Private respondent Ramon Flores filed his Memorandum, arguing that petitioner,
invoking its pending petition for certiorari with the Supreme Court. The private being the private complainant in the criminal case below, has no legal standing to
respondents, thru counsel, objected to any further resetting as this would constitute appeal the acquittal of private respondents; that there was no extrinsic fraud, abuse
a violation of their right to a speedy trial. The respondent judge called for a recess so of discretion or jurisdictional defect to warrant either a petition for annulment of
as to let the prosecution decide whether or not to present an NBI agent, who was judgment or certiorari; and that the reopening of the criminal case will violate the
then present, to prove the due execution of the accused Nuada's extrajudicial accused's right against double jeopardy.
confession.
It is true that a private complainant cannot bring an action questioning a judgment
of acquittal, except insofar as the civil aspect of the criminal case is concerned. 7 In

130
the case at bar, we agree with petitioner that this issue was rendered moot when In the case at bar, the public prosecutor knew that he had not presented sufficient
the Solicitor General, in representation of the People, changed his position and evidence to convict the accused. Yet, despite repeated moves by the accused for the
joined the cause of petitioner, thus fulfilling the requirement that all criminal actions trial court to continue hearing the case, he deliberately failed to present an available
shall be prosecuted under the direction and control of the public prosecutor. 8 witness and thereby allowed the court to declare that the prosecution has rested its
case. In this sense, he was remiss in his duty to protect the interest of the offended
In any event, petitioner has an interest in the maintenance of the criminal parties. More specifically, the public prosecutor in this case was guilty of blatant
prosecution, being the mother of the deceased rape victim. The right of offended error and abuse of discretion, thereby causing prejudice to the offended party.
parties to appeal an order of the trial court which deprives them of due process has Indeed, the family of the deceased victim, Maritess Merciales, could do nothing
always been recognized, the only limitation being that they cannot appeal any during the proceedings, having entrusted the conduct of the case in the hands of the
adverse ruling if to do so would place the accused in double jeopardy. 9 said prosecutor. All they could do was helplessly watch as the public prosecutor, who
The criminal case below was for rape with homicide. Although the public prosecutor was under legal obligation to pursue the action on their behalf, renege on that
presented seven witnesses, none of these actually saw the commission of the crime. obligation and refuse to perform his sworn duty.
It was only Joselito Nuada, one of the accused, who came forward and expressed Indeed, Rule 119, Section 9 (now Section 17) of the Rules of Court expressly requires
willingness to turn state witness. His testimony was vital for the prosecution, as it the presentation of evidence in support of the prosecution's prayer for the discharge
would provide the only eyewitness account of the accused's complicity in the crime. of an accused to be a state witness, viz:
The trial court required the public prosecutor to present evidence to justify Nuada's
discharge as a state witness, but the latter insisted that there was no need for such When two or more persons are jointly charged with the commission of any
proof since Nuada had already been admitted into the Witness Protection Program of offense, upon motion of the prosecution before resting its case, the court
the Department of Justice. The public prosecutor's obstinate refusal to present the may direct one or more of the accused to be discharged with their consent
required evidence prompted the trial court to deny the motion to discharge Nuada. so that they may be witnesses for the state when after requiring the
prosecution to present evidence and the sworn statement of each proposed
The prosecution elevated the matter to the Supreme Court on a petition for state witness at a hearing in support of the discharge, xxx xxx xxx.
certiorari. Meanwhile, the accused moved to set the case for hearing, invoking their
constitutional right to speedy trial. The trial court granted the motion. The public By refusing to comply with the trial court's order to present evidence, the public
prosecutor moved for a continuance, and the trial court acceded. At the next prosecutor grossly violated the above-quoted rule. Moreover, the public prosecutor
scheduled hearing, however, the trial court denied a similar motion by the violated his bounden duty to protect the interest of the offended party, at least
prosecution in view of the objection of the accused. The trial court directed the insofar as the criminal aspect is concerned. After the trial court denied his motion to
public prosecutor to present Atty. Carlos S. Caabay, the NBI Agent who took Nuada's discharge Nuada as a state witness, he should have proceeded to complete the
extrajudicial confession. At the resumption of the hearing, the public prosecutor evidence of the prosecution by other means. Instead, he willfully and deliberately
declared that he was resting the prosecution's case, knowing fully well that the refused to present an available witness, i.e., the NBI Agent who was present in court
evidence he has presented was not sufficient to convict the accused. Consequently, on that date and time. The public prosecutor was duty-bound to exhaust all available
the ensuing demurrer to evidence filed by the accused was granted by the trial proofs to establish the guilt of the accused and bring them to justice for their offense
court. against the injured party.
It is clear from the foregoing that the public prosecutor was guilty of serious Likewise guilty for serious nonfeasance was the trial court. Notwithstanding its
nonfeasance. It is the duty of the public prosecutor to bring the criminal proceedings knowledge that the evidence for the prosecution was insufficient to convict,
for the punishment of the guilty.10 Concomitant with this is the duty to pursue the especially after the public prosecutor tenaciously insisted on utilizing Nuada as state
prosecution of a criminal action and to represent the public interest. A crime is an witness, the trial court passively watched as the public prosecutor bungled the case.
offense against the State, and hence is prosecuted in the name of the People of the The trial court was well aware of the nature of the testimonies of the seven
Philippines. For this reason, Section 5 of Rule 110 provides that "all criminal actions prosecution witnesses that have so far been presented. Given this circumstance, the
either commenced by complaint or by information shall be prosecuted under the trial court, motu proprio, should have called additional witnesses for the purpose of
direction and control of the fiscal x x x." As the representative of the State, the questioning them himself in order to satisfy his mind with reference to particular
public prosecutor has the right and the duty to take all steps to protect the rights of facts or issues involved in the case.13
the People in the trial of an accused. 11 If the public prosecutor commits a
nonfeasance in refusing to perform a specific duty imposed on him by law, he can be Based on the foregoing, it is evident that petitioner was deprived of her day in court.
compelled by an action for mandamus.12 Indeed, it is not only the State, but more so the offended party, that is entitled to
due process in criminal cases. Inasmuch as the acquittal of the accused by the
court a quo was done without regard to due process of law, the same is null and
void. It is as if there was no acquittal at all, and the same cannot constitute a claim
for double jeopardy.14
By contending that the challenged Decision is void for having been issued
with grave abuse of discretion amounting to lack or excess of jurisdiction,
the petition does not violate the right of the accused against double
jeopardy. It is elementary that double jeopardy attaches only when the
following elements concur: (1) the accused are charged under a complaint
or information sufficient in form and substance to sustain their conviction;
(2) the court has jurisdiction; (3) the accused have been arraigned and

131
have pleaded; and (4) they are convicted or acquitted, or the case is
dismissed without their consent.
Thus, even assuming that a writ of certiorari is granted, the accused would
not be placed in double jeopardy because, from the very beginning, the
lower tribunal had acted without jurisdiction. Precisely, any ruling issued
without jurisdiction is, in legal contemplation, necessarily null and void and
does not exist.15
Otherwise put, the dismissal of the case below was invalid for lack of a fundamental
prerequisite, that is, due process. 16 In rendering the judgment of dismissal, the trial
judge in this case acted without or in excess of jurisdiction, for a judgment which is
void for lack of due process is equivalent to excess or lack of jurisdiction. 17Indeed,
"jurisdiction" is the right to hear and determine, not to determine without hearing. 18
Lack of jurisdiction is one of the grounds for the annulment by the Court of Appeals
of judgments or final orders and resolutions of Regional Trial Courts. 19 Hence, the
remedy taken by petitioner before the Court of Appeals was correct.
WHEREFORE, in view of the foregoing, the petition is GRANTED. The Decision of
the Court of Appeals in CA-G.R. SP No. 37341 is REVERSED AND SET ASIDE. The
Order dismissing Criminal Case Nos. 6307-6312 is ANNULLED, and this case
is REMANDED to the Regional Trial Court of Legazpi City, Branch 8, for further
proceedings. The public prosecutor is ORDERED to complete the presentation of all
available witnesses for the prosecution.
SO ORDERED.

132
PADILLA ET AL. VS. COURT OF APPEALS represented by Padilla in his capacity as its President and General Manager.
And Phoenix Omega. which was not a party, to the July 28. 1988 lease
G.R. No. 123893 November 22, 2001 contract sought to be amended but which was a party to the amended
contract. was also represented by Padilla as Chairman of the Board of
LUISITO PADILLA and PHOENIX-OMEGA DEVELOPMENT AND MANAGEMENT Directors of Phoenix Omega.
CORPORATION, petitioners,
vs. PKA's building permit was later revoked due to certain violations of the
THE HONORABLE COURT OF APPEALS and SUSANA REALTY, National Building Code (BP 344).
INC., respondents. On August 24, 1989, PKA was allowed by the (Department) of Public Works
QUISUMBING, J.: and Highway(s) to resume construction on the leased premises subject to
PKA's correction of the defects in the construction to conform to BP 344.
This petition for review seeks the reversal of the Court of Appeals decision 1 in CA-
G.R. SP No. 36685, refusing to set aside (1) the order dated November 29, 1994 of As SRI's approval of PKA's amended plans in the construction was required,
the Regional Trial Court of Pasay City, Branch 113, which authorized the issuance of PKA transmitted the same to SRI which withheld approval thereof pending
an alias writ of execution in connection with Civil Case No. 7302 filed before said PKA's correction of the defects in the construction.
court; and (2) the order dated February 10, 1995, which denied petitioners' motion Repeated requests for approval of its amended plans not having been
for reconsideration of the order of November 29, 1994, regarding the annulment of heeded by SRI, PKA filed at the courta quo the action at bar for rescission of
the alias writ of execution and cancellation of the notice of levy and sale dated contract of lease against SRI, alleging that SRI's refusal to approve the
December 16, 1994, issued pursuant to the implementation of said alias writ. plans without any justifiable reason deprived it of the use of the commercial
The antecedent facts, as summarized by the Court of Appeals, are as follows: stalls, thereby incurring losses.

"On June 27, 1983, Susana Realty, Inc. (SRI), by a deed of absolute sale, SRI, upon the other hand, claimed that it was PKA which violated the terms
sold to the Light Rail Transit Authority (LRTA) several parcels of land located of their contract, alleging that PKA failed to complete within six months the
in Taft Avenue Extension, San Rafael District, Pasay City. Under paragraph 7 construction of the commercial stalls during which period it was not paying
of the deed of sale, SRI reserved to itself the right of first refusal to develop any rentals and that PKA undertook the construction without first having its
and/or improve the property sold should the LRTA decide to lease and/or plans approved."2(Underscoring in the original.)
assign to any person the right to develop and/or improve the property. On January 7,1991, the RTC rendered its decision, as follows:
On November 28, 1986, the LRTA and Phoenix Omega Development and "WHEREFORE, judgment is hereby rendered:
Management Corporation (Phoenix Omega) entered into a Commercial Stall
Concession Contract authorizing the latter to construct and develop 1. Declaring the rescission and termination of the Contract of Lease, as
commercial stalls on a 90 sq. m. portion of the property bought from SRI. amended, and the passing in ownership of all the improvements now
SRI opposed the agreement as having violated the deed of sale it entered existing on the premises, and ordering plaintiff to surrender possession of
with LRTA. A tripartite agreement was later concluded by the parties, the leased premises to the defendant.
however, whereby SRI agreed to honor the terms of the concession contract
2. Ordering plaintiff to pay to the defendant the following sums of money:
and to lease to Phoenix Omega its (SRI's) property (remaining property)
adjacent to the 90 sq. m. portion subject of the concession (a) P1,750,000.00 as of April 30, 1990, plus monthly rental of
contract.1wphi1.nt P200,000 per month starting in May, 1990, until plaintiff shall turn
over possession of the premises to the defendant, with interest at
A contract was thus entered into on July 28, 1988 between Phoenix Omega
1% per month until fully paid;
and SRI with LRTA whereby Phoenix Omega undertook to construct
commercial stalls on the 90-sq. m. property in accordance with plans and (b) Moral damages in the amount of P100,000.00;
specifications prepared by the latter, the construction to begin, however,
only upon SRI's approval of such plans and specifications. Also on July 28, (c) Exemplary damages in the amount of P100,000.00; and
1988, Phoenix Omega, by a deed of assignment, assigned its right and
(d) Attorney's fees in the amount of P150,000.00; and
interests over the remaining property unto its sister company, PKA
Development and Management Corporation (PKA). Signatories to the deed (e) The cost of suit."3
of assignment were Eduardo Gatchalian in his capacity as President of
Phoenix Omega, and Luisito B. Padilla (Padilla), one of the petitioners PKA appealed the RTC decision to the Court of Appeals. On October 2, 1992, the CA
herein, in his capacity as President and General Manager of PKA. The affirmed the RTC decision, decreeing as follows:
development of the remaining property having been assigned to PKA, it "WHEREFORE, with MODIFICATIONS that the award of P100,000.00 for
entered into a contract of lease with SRI likewise on July 28, 1988: moral damages and P100,000.00 for exemplary damages is DELETED from
In the meantime, SRI sold part of its remaining property to a third party. An the judgment appealed from, the rest thereof not inconsistent herewith is
amended contract of lease was thus forged in January 1989 among SRI, PKA AFFIRMED. No costs."4
and Phoenix Omega, whereby the parties agreed to substitute the already PKA' S motion for reconsideration was denied by the CA in a resolution dated March
sold portion ,of SRI's remaining property with 2 parcels of land also 15, 1993. PKA then filed before this Court a petition for review on certiorari, which
belonging to SRI. In this amended contract of lease, PKA was again

133
we denied in a resolution dated September 27, 1993. We likewise denied PKA's Alleging that the writ of execution cannot be enforced against them, herein
motion for reconsideration in a resolution dated January 17, 1994. petitioners filed with the RTC on December 15, 1994, an omnibus motion for the
reconsideration of the order of November 29, 1994, and for annulment of the alias
A writ of execution was issued in due course by the RTC, which reads as follows: writ of the same date and cancellation of the notice of levy and sale dated
"NOW THEREFORE, you are hereby commanded to cause the execution of December 16, 1994. Petitioners assailed these orders as confiscatory, since they
the aforesaid decision, ordering the plaintiff and all persons claiming under were never parties to the case filed by PKA against SRI, and they were unable to
it to surrender possession of the premises to the defendant, and that of the present evidence on their behalf. The motion was denied on February 10, 1995.
goods and chattels of the plaintiff you cause to be made the sum of! Subsequently, on March 8, 1995, petitioners filed with the Court of Appeals a
P1,750,000.00 plus monthly rental of P200,000.00 starting in May, 1990 petition for certiorari and prohibition under Rule 65 of the Rules of Court. This
until plaintiff shall turn over possession of the premises to defendant with petition was also denied; so was petitioners' motion for reconsideration of said
interest of 1% per month until fully paid, and the further sum of denial.
PI50,000.00 as attorney's fees, and the cost of suit, together with your
lawful fees for service of this execution all in Philippine currency, and that The Court of Appeals agreed with the RTC's finding that there is evidence on record
you tender the same to defendant Susana Realty, Inc. aside from your own to support the RTC's conclusion that PKA and Phoenix-Omega are one and the same,
fees on this execution and to likewise return this writ to this Court within or that the former is a mere conduit of the latter. It pointed out that petitioner Padilla
sixty (60) days from receipt hereof with your proceedings endorsed is both president and general manager of PKA and at the same time chairman of the
thereon. board of directors and controlling stockholder of Phoenix-Omega. PKA and Phoenix-
Omega also shared officers, laborers, and offices.
But if sufficient personal property of the plaintiff cannot be found whereof
to satisfy the amount of said judgment, you are hereby directed to levy the While aware that the dispositive portion of the RTC decision holds only PKA liable to
real property of the said plaintiff and to sell the same or so much thereof in SRI, the Court of Appeals pointed out that the intent of the RTC was clearly to hold
the manner provided for by law for the satisfaction of the said judgment." 5 PKA, Phoenix-Omega, and Padilla liable, as shown in the body of the RTC decision.
The rule that the dispositive portion of a decision is the subject of execution only
Possession of the subject properties was subsequently restored to SRI, but the applies where the disposition is clear and unequivocal, according to the CA, unlike in
monetary award was left unsatisfied. Thus, on November 14, 1994, SRI filed a this case where there is uncertainty and ambiguity. The body of the decision may be
motion for issuance of an alias writ against herein petitioners, based on the trial consulted to construe the judgment in this case.
court's observation that PKA and Phoenix-Omega are one and the same entity. This
was granted by the RTC in an order6 dated November 29, 1994, which reads: On the claim that Phoenix-Omega and Padilla were not parties to the case, the CA
ruled that
"WHEREFORE, as prayed for by the defendant-judgment creditor Susana
Realty, Inc., let an alias writ of execution issue against the properties, both "a person not so impleaded to an action is deemed to be a party to a suit
real and personal, of PKA Development and Management Corporation, of when he has the right to control the proceedings, to make defense, to
Phoenix-Omega Development Corporation, and of Luisito B. Padilla, for the adduce and cross examine witnesses, and to appeal from a decision ( 67
enforcement of the decision dated January 7, 1991, promulgated by this C.J.S. 887 cited in Albert v. University Publishing Co., 13 SCRA 84). That
Court, the same be implemented by deputy sheriff Edilberto A. Santiago." petitioner Padilla is in reality the one who had and duly exercised these
(Underscoring by petitioners.) rights is glaringly borne by the records."8
The RTC issued an alias writ on the same day pursuant to the above order: Hence, this petition for review, in which petitioners allege that the CA erred:
"NOW THEREFORE, you are hereby commanded to cause the execution of I. ...IN RENDERING THE DECISION AND RESOLUTION IN QUESTION... IN
the aforesaid decision and that of the goods and chattels of the plaintiff, DEFIANCE OF LAW AND JURISPRUDENCE BY SUSTAINING THE TRIAL
PKA Development and Management Corporation, Phoenix- Omega, caused COURT'S ORDER AND WRIT BOTH DATED NOVEMBER 29, 1994... FINDING
to be made the sum of P1,750,000.00 plus monthly rentals of P200,000.00 PETITIONERS JOINTLY AND SEVERALLY LIABLE WITH PKA, THEREBY
starting in May, 1990 with interest of 1% per month, until fully paid, and the AUTHORIZING THE EXECUTION OF THE DECISION... AGAINST THEIR
further sum of P150,000.00 as attorney's fees; P100,000.00 moral damages PROPERTIES, DESPITE THE ADMITTED FACT THAT --
and the cost of suit, together with your lawful fees for service of this
execution all in Philippine currency, and that you tender the same to the A. PETITIONERS WERE NEVER IMPLEADED AS PARTIES IN THE CASE
defendant SUSANA REALTY, INC., aside from your own fees on this BEFORE THE TRIAL COURT (CIVIL CASE NO. 7302), THEREBY
execution and to likewise return this writ to this Court within 60 days from CONFIRMING THE OPPRESSIVE AND CONFISCATORY NATURE OF
receipt hereof with your proceeding indorsed thereon. THE ORDER AND WRIT (ANNEXES N AND O);

But if sufficient personal properties of the plaintiff cannot be found whereof B. PETITIONERS COULD NOT AND DID NOT HAVE ANY
to satisfy the amount of said judgment, you are directed to levy the real OPPORTUNITY TO ADDUCE EVIDENCE TO REFUTE THE CAUSES OF
property of the plaintiff, PKA Development and Management Corporation, ACTIONS ALLEGED IN RESPONDENT SRI'S COMPLAINT BEFORE THE
Phoenix-Omega Development and Management Corporation and Luisito B. TRIAL COURT (CIVIL CASE NO. 7302) THUS VIOLATING THEIR RIGHT
Padilla and to sell the same or so much thereof in the manner provided for TO DUE PROCESS OF LAW.
by law for the satisfaction of the said judgment." 7 II. ...IN CONCLUSIONS REACHED IN THE DECISION AND RESOLUTION IN
QUESTION... BY AFFIRMING THE ORDER AND WRIT... AS ISSUED BY THE
TRIAL COURT IN CIVIL CASE NO. 7302 WHICH EXPANDED THE SCOPE OF

134
THE WRIT HOLDING PETITIONERS SOLIDARILY LIABLE WITH PKA "Generally accepted is the principle that no man shall be affected by any
NOTWITHSTANDING THAT THIS FINDING WAS NOT CONTAINED IN THE proceeding to which he is a stranger, and strangers to a case are not bound
DISPOSITIVE PORTION OF THE DECISION..., IN DEFIANCE OF LAW AND by judgment rendered by the court. xxx",12
JURISPRUDENCE ON THE MATTER.
In the present case, we note that the trial court never acquired jurisdiction over
III. ...IN APPLYING THE DOCTRINE OF PIERCING THE VEIL OF CORPORATE petitioners through any of the modes mentioned above. Neither of the petitioners
FICTION TO THE CASE AT BAR DESPITE THE FACT THAT THE GROUNDS FOR was even impleaded, as a party to the case. 13
ITS APPLICATION UNDER CASE LAW HAVE NOT BEEN SHOWN, THEREBY
ABROGATING PRONOUNCEMENTS OF THIS HONORABLE COURT IN Without the trial court having acquired jurisdiction over petitioners, the latter could
NUMEROUS DECISIONS ON THE SUBJECT . not be bound by the decision of the court. Execution can only be issued against a
party and not against one who was not accorded his day in court. 14 To levy upon
IV. ...IN AFFIRMING THE ORDER AND WRIT... OF THE TRIAL COURT their properties to satisfy a judgment in a case in which they were not even parties
NOTWITHSTANDING THE ABSENCE OF ANY MISTAKE, OMISSION OR is not only inappropriate; it most certainly is deprivation of property without due
AMBIGUITY IN THE JANUARY 9, 1991 DECISION IN THE MAIN CASE... AS process of law.15 This we cannot allow.
WOULD HAVE JUSTIFIED ITS MODIFICATION PURSUANT TO EXTANT
JURISPRUDENCE ON THE MATTER.9 The courts a quo ruled that petitioner Padilla, in particular, had his day in court. As
general manager of PKA, he actively participated in the case in the trial court. He
Petitioners stress that the RTC, the CA, and this Court, in the main case (Civil Case "ha(d) the right to control the proceedings, to make defense, to adduce and cross
No. 7302), did not find them solidarily liable with PKA, and rightly so since PKA and examine witnesses, and to appeal from a decision." 16 Therefore, Padilla and Phoenix-
Phoenix-Omega are two different entities. Phoenix-Omega's only participation in the Omega, of which Padilla is chairman of the board, could not now argue that they did
properties subject of the main case was as the construction company that would not have the opportunity to present their case in court, according to private
develop the properties on behalf of PKA. Phoenix-Omega was involved in the respondent.
amended lease agreement between SRI and PKA only to the extent that it had to
apply the terms of the tripartite agreement (among LRTA, SRI, and Phoenix-Omega) To begin with, it is clear that Padilla participated in the proceedings below as general
to the development of the LRTA-owned property situated in front of the lots leased to manager of PKA and not in any other capacity. The fact that at the same time he was
PKA by SRI.10 Petitioners argue that the amended lease contract was, in reality, only the chairman of the board of Phoenix-Omega cannot, by any stretch of reasoning,
between SRI and PKA. equate to participation by Phoenix- Omega in the same proceedings. We again
stress that Phoenix-Omega was not a party to the case and so could not have taken
Petitioners protest the piercing of the veil of corporate fiction between themselves part therein.
and PKA. They contend, citingFilmerco Commercial Co., Inc. v. IAC, No. L-70661, 149
SCRA 193 (1987), that the court must first acquire jurisdiction over the corporation Private respondent, however, insists that the trial court had pierced the veil of
attempting to misuse the corporate vehicle to shield the commission of a fraud. corporate fiction protecting petitioners, and this justifies execution against their
properties.
Petitioners contend that the finding by the trial court as regards the single
personality of PKA and Phoenix-Omega was made only to refute PKA's claim that it The general rule is that a corporation is clothed with a personality separate and
was not liable for constructions made by Phoenix- Omega outside the leased areas. distinct from the persons composing it. It may not be held liable for the obligations
of the persons composing it, and neither can its stockholders be held liable for its
On the other hand, private respondent argues that there is no error in the issuance obligations.17
of the alias writ of execution against the properties of petitioners since the trial
court, the CA, and this Court had all ruled that petitioners and PKA are in reality one This veil of corporate fiction may only be disregarded in cases where the corporate
and the same entity. This is the reason why, when the first writ of execution was vehicle is being used to defeat public convenience, justify wrong, protect fraud, or
returned unsatisfied, SRI moved for the issuance of an alias writ of execution not defend crime.18 PKA and Phoenix-Omega are admittedly sister companies, and may
only against the properties of PKA but those of petitioners as well. There is no be sharing personnel and resources, but we find in the present case no allegation,
violation of petitioners' right to due process since petitioner Padilla actively much less positive proof, that their separate corporate personalities are being used
participated in the proceedings before the R TC as the responsible officer of both PKA to defeat public convenience, justify wrong, protect fraud, or defend crime. "For the
and Phoenix-Omega. separate juridical personality of a corporation to be disregarded, the wrongdoing
must be clearly and convincingly established. It cannot be presumed." 19 We find no
Private respondent also contends that the CA ruled on the necessity of construing reason to justify piercing the corporate veil in this instance.
the dispositive portion of the judgment along with its text, which petitioners
allegedly accepted by not discussing the issue in their pleadings. We understand private respondent's frustration at not being able to have the
monetary award in their favor satisfied. But given the circumstances of this case,
To our mind, the main issue for our consideration is whether or not the trial court public respondent cannot order the seizure of petitioners' properties without
had jurisdiction over petitioners, to justify the issuance of an alias writ of execution violating their constitutionally enshrined right to due process, merely to compensate
against their properties. private respondent.1wphi1.nt
A court acquires jurisdiction over a person through either a valid service of summons WHEREFORE, the instant petition is GRANTED. The assailed decision and
or the person's voluntary appearance in court. 11 A court must necessarily have resolution of the Court of Appeals in CA-G.R. SP No. 36685 are SET ASIDE, and the
jurisdiction over a party for the latter to be bound by a court decision. order of the trial court dated November 29, 1994 and the alias writ of execution
issued on the same date in connection with Civil Case No. 7302, are declared NULL
and VOID.

135
Costs against private respondent.
SO ORDERED.
Bellosillo, Mendoza, Buena, De Leon, Jr., JJ., concur.

136
PEOPLE VS. GLABO PERPETUA, with the accessory penalties of civil interdiction for life and of
perpetual absolute disqualification; to pay complainant moral damages in
G.R. No. 129248 December 7, 2001 the amount of P50,000.00; to give support to complainant's child who was
born as a result of this offense; and to pay the costs.2
PEOPLE OF THE PHILIPPINES, plaintiff-appellee,
vs. Hence, this direct appeal.
JUSTINIANO GLABO alias "TOTO BUGOY", accused-appellant. After carefully reviewing the evidence on record, we find no reason to reverse the
YNARES-SANTIAGO, J.: judgment of the trial court. The prosecution established by sufficient evidence the
guilt of the accused-appellant by proof beyond reasonable doubt.
One afternoon in October, 1991, 21-year old victim Mila Lobrico, a mental retardate,
and her 11-year old sister, Judith, were summoned by accused-appellant, their Sexual crimes where the culprit denies culpability is actually a test of credibility. 3 The
maternal uncle, to his house. He told them to wash the clothes of his wife. After the issue of credibility has, time and again, been settled by this Court as a question best
two sisters finished their chore, accused-appellant ordered Judith to wash the dishes addressed to the province of the trial court because of its unique position of having
in the nearby creek, about 200 meters away from his house. When Judith was gone, observed that elusive and incommunicable evidence of the witnesses' deportment
accused-appellant dragged Mila from the yard, where she was hanging the washed on the stand while testifying which opportunity is denied to the appellate courts.
clothes, into the house. He pushed her to the floor and made her lie down. He Absent any substantial reason which would justify the reversal of the trial court's
undressed the victim, then he inserted his penis into her private organ and made assessments and conclusions, the reviewing court is generally bound by the former's
push and pull motions. Mila was overpowered by accused-appellant's brute strength. findings, particularly when no significant facts and circumstances were shown to
She shouted for help, but there were no neighbors nearby. have been overlooked or disregarded which when considered would have affected
the outcome of the case.
Suddenly, it started to rain hard, so Judith had to run back to the house for shelter.
She went directly under the house, which was elevated 3 feet above the ground. Moreover, the fact that the victim was about six months pregnant in March 1992
While underneath the house, she heard someone crying on the floor above. She confirms the commission of the rape sometime in October 1991. There was no
looked up through the bamboo floor and saw accused-appellant on top of her elder showing that Mila had sexual relations with other men during that time.
sister. Both were naked. Judith went to the kitchen, and she saw accused-appellant's It was sufficiently proved that Mila was a mental retardate. Both her parents testified
penis as he stood up and raised his briefs. that their child, Mila, is mentally retarded. In addition, the psychiatrist who examined
The two girls went home silently. They did not say a word about the incident. her testified as to her mental state:
However, the victim became pregnant as a result of the rape, and after six months Q Will you please tell the court whether that sickness of Mila Lubrico
her condition could no longer be concealed. Severino Lobrico, Mila's father, was in-born or only acquired?
confronted her, but she said nothing. It was her sister, Judith, who told their father
that accused-appellant raped Mila. Severino brought Mila to the police and filed a A Based on the medical history of Mila Lobrico she has a medical
complaint for rape before the Municipal Trial Court. After the preliminary sickness with typhoid which lasted for three (3) weeks and that otherwise
investigation, the following Information was filed against accused-appellant: fever might have affected or impaired her mental faculties.

That on or about the month of October, 1991, at Sitio Siniaran, Bgy. Q At the time you examined Mila Lubrico will you say in your own
Banbanan, in the Municipality of Taytay, Province of Palawan, Philippines opinion that she was suffering from mental disturbance.
and within the jurisdiction of this Honorable Court, the said accused with
A Yes, sir.4
lewd design and by means of force, threat and intimidation, did then and
there wilfully, unlawfully and feloniously have carnal knowledge with one Thus, Mila could not have consented to engage in sexual intercourse with accused-
Mila Lobrico against her will and consent to the damage and prejudice of appellant.5 Her condition falls under the definition of a person "deprived of reason."
said Mila Lobrico in such amount as may be awarded her by the court. These include those suffering from mental abnormality or deficiency; or some form
of mental retardation; the feeble minded but coherent; or even those suffering from
CONTRARY TO LAW.1
mental abnormality or deficiency of reason. 6
In his defense, accused-appellant alleged that during the entire month of October
Mentally deficient persons generally share certain social behavior characteristics
1991, he was plowing the field of one of his sisters in Sitio Yakal, new Guinto, Taytay,
that undermine their ability to give statements voluntarily, knowingly and
Palawan. The victim's mother, Gloria Glabo-Lobrico, testified for the defense. She
intelligently they "may be vulnerable to exploitation by others." 7 Where the rape
stated that she wanted the case to be settled to restore her good relationship with
victim is feeble-minded, even if there may have been no physical force employed on
accused-appellant, who is her brother.
her, there is authority to the effect that the force required by law is the sexual act
On September 30, 1996, the Regional Trial Court of Puerto Princesa City, Palawan, itself.8
Branch 51, rendered judgment convicting accused-appellant, thus:
Accused-appellant was positively identified as the perpetrator of the rape by both
WHEREFORE, premises considered, judgment is hereby rendered, finding the victim and her sister. There is no showing that the latter harbored ill motive
the accused JUSTINIANO GLABO, ALIAS TOTO BUGOY, guilty beyond against accused-appellant. Hence, his defense of alibi must fail. 9 Alibi and denial are
reasonable doubt as principal in the crime of rape, and there being no inherently weak and unreliable defenses.10
modifying circumstances appreciated and not being entitled to the benefits
The silence of the victim and her sister for about six months does not render their
of the Indeterminate Sentence Law, he is hereby sentenced to RECLUSION
testimony doubtful. Delay for six months in reporting a sexual attack does not

137
detract from the veracity of her charge. 11 The failure to disclose defilement or rape SO ORDERED.
does not warrant the conclusion that she was not sexually molested and that her
charges are baseless, untrue and fabricated. 12 Rape is not only a physical but also an Davide, Jr., C .J ., Puno, Kapunan, and Pardo, JJ ., concur.
emotional assault on women, which places tremendous stress on the human mind.
Hence victims of rape react, even cope, differently from one another some may
shout, others may faint; some may collapse into a trance-like state, and others may
lose their sanity.13Moreover, there is no standard form of behavioral response
whenever one is confronted with a startling or frightful experience. 14 Rape is
unquestionably a frightful experience. To discredit a witness merely for her
understandable procrastination is to forever seal the lips of any reluctant or fearful
witness.15
Accused-appellant contends that the victim's father coached his children to make up
the charges because he maltreats his wife and he wants a piece of land belonging to
his family. The contention has no merit. It is unnatural for a parent to use his
daughter as an instrument of malice, especially if it will subject her to
embarrassment and even stigma.16
The fact that the Information did not specify the precise date in October 1991 when
the rape was committed does not render the same defective. The precise time of
commission of rape is not an essential element. 17 Such allegation in the Information
does not violate due process and is sufficient to sustain guilt. 18 Besides, considering
the mental status of the victim and the tender age of her sister, they cannot be
expected to recall and keep track of the date, particularly when they had apparently
chosen not to recall that day.
With respect to the monetary awards, moral damages is automatically awarded to
rape victims without need of proof, for it is assumed that she has suffered moral
injuries entitling her to such award.19 It is granted separate and distinct from the civil
indemnity, which is likewise automatically awarded upon proof of the commission of
the crime and the accused-appellant's responsibility therefor. 20 Moral damages is
currently fixed at P50,000.00 and the civil indemnity also at P50,000.00. 21 The rape
of a niece by her own uncle is incestuous in character, hence exemplary damages is
also justified.22
Concerning the acknowledgment and support of the offspring of rape, Article 345 of
the Revised Penal Code provides for three kinds of civil liability that may be imposed
on the offender: a) indemnification, b) acknowledgment of the offspring, unless the
law should prevent him from so doing, and c) in every case to support the offspring.
With the passage of the Family Code, the classification of acknowledged natural
children and natural children by legal fiction was eliminated and they now fall under
the specie of illegitimate children. Since parental authority is vested by Article 176
of the Family Code upon the mother and considering that an offender sentenced
to reclusion perpetuaautomatically loses the power to exercise parental authority
over his children, no "further positive act is required of the parent as the law itself
provides for the child's status." Hence, accused-appellant should only be ordered to
indemnify and support the victim's child. However, the amount and terms of support
shall be determined by the trial court after due notice and hearing in accordance
with Article 201 of the Family Code.
WHEREFORE, in view of the foregoing, the decision dated September 30, 1996 of the
Regional Trial Court of Palawan and Puerto Princesa City, Branch 51, in Criminal Case
No. 10275, finding accused-appellant guilty beyond reasonable doubt of rape and
sentencing him to suffer the penalty of reclusion perpetua, is AFFIRMED with the
MODIFICATION that accused-appellant is ordered to pay the victim the sums of
P50,000.00 as civil indemnity and P25,000.00 as exemplary damages, in addition to
moral damages of P50,000.00. Accused-appellant is further ordered to provide
support to the victim's child born out of the rape, subject to the amount and terms
to be determined by the trial court in a proper proceeding.

138
PRODUCERS BANK VS. COURT OF APPEALS Asiatrust as they failed to exercise reasonable care and caution to determine the
true ownership of the bills before allowing the proceeds to be paid to Milagros B.
G.R. No. 126620 April 17, 2002 Nayve, Elizabeth C. Garcia and Alberto Limjoco, Sr. Petitioner sought to hold
Asiatrust solidarily liable with the other defendants for the payment of the value of
PRODUCERS BANK OF THE PHILIPPINES, petitioner, the treasury bills and for damages. Subsequently, the complaint was dismissed as
vs. against the CBP on motion of petitioner on the ground that the latter had lifted
HONORABLE COURT OF APPEALS, ASIA TRUST DEVELOPMENT BANK, petitioners conservatorship and allowed the return of the management and assets
RAINELDA A. ANDREWS, SAMSON FLORES, ALFONSO LEONG, JR., RHODORA to petitioners Board of Directors. The CBPs lifting of the conservatorship was
D. LANDRITO, JOSEPH CHUA, RAMON YU, EDUARDO G. ESCOBAR, MILAGROS conditioned upon petitioners dropping of all its cases pending against the CBP.
B. NAYVE, ELIZABETH C. GARCIA, ALBERTO LIMJOCO, SR., GLORIA E.
MENPIN and ESPERANZA FLORENDO, respondents. The defendants filed their respective Answers, after which the issues were joined
and trial on the merits ensued.
CARPIO, J.:
On August 30, 1993, the law firm of Quisumbing, Torres and Evangelista ("QTE" for
The Case brevity) entered its appearance for petitioner in substitution of Atty. Antonio M. Pery.
In this Petition for Review on Certiorari under Rule 45 of the 1997 Rules of Civil Petitioners handling counsel, Atty. Alvin Agustin T. Ignacio ("Atty. Ignacio" for
Procedure, petitioner Producers Bank of the Philippines ("petitioner" for brevity) brevity) of QTE arrived late during the hearing held on May 17, 1995. On motion of
assails the September 19, 1996 Resolution 1 of the Court of Appeals in CA-G.R. CV No. Asiatrusts counsel, the RTC issued an Order on the same day dismissing the case for
50016 which dismissed petitioners appeal for being filed out of time. The Court of lack of interest to prosecute.
Appeals decreed thus:
On June 9, 1995, Atty. Ignacio filed a motion to reconsider the Order dated May 17,
"WHEREFORE, finding the Motion to Dismiss Appeal to be meritorious, the 1995, explaining that his late arrival at the hearing was due to the unexpected
same is granted. The appeal is DISMISSED. heavy traffic at Roxas Boulevard in front of Baclaran Church. He also offered his
apologies to the RTC for his unintended tardiness. QTE received a copy of the Order
SO ORDERED."2 dated August 1, 1995 denying the motion for reconsideration on August 11, 1995. At
The Antecedent Facts that time, Atty. Ignacio was indisposed for allegedly suffering from "fatigue and
stress". It was only on August 25, 1995 that Atty. Ignacio found out that the Order
On March 29, 1988, petitioner through its former counsel, Atty. Antonio M. Pery, filed denying the motion for reconsideration was received by the law firm on August 11,
a Complaint to recover the sum of P11,420,000.00 from Asia Trust Development 1995. He filed a Notice of Appeal on August 25, 1995.
Bank ("Asiatrust" for brevity) and the Central Bank of the Philippines ("CBP" for
brevity) before the Regional Trial Court of Makati, Branch 147 ("RTC" for brevity). On November 13, 1995, Asiatrust, et al. filed a Motion to Dismiss Appeal with the
Thereafter, petitioner filed an amended complaint, impleading additional Court of Appeals. On March 8, 1996, QTE filed its Comment to the Motion to Dismiss
defendants.3 Appeal.

Petitioner sought to recover the proceeds of several treasury bills amounting In the Resolution dated September 19, 1996, the Court of Appeals granted the
to P11,420,000.00. According to petitioner, said proceeds were fraudulently credited motion to dismiss petitioners appeal.
to the demand deposit account of Asiatrust with the CBP and withdrawn by Milagros Ruling of the Court of Appeals
B. Nayve, Elizabeth C. Garcia and Alberto Limjoco, Sr.
In granting the motion to dismiss appeal, the Court of Appeals held in part:
It appears that petitioner owned several treasury bills. On the respective maturity
dates of these bills, petitioner caused these bills to be delivered to the CBP. The bills "xxx. We hold that the failure of plaintiff-appellant to file the Notice of
were initially received by Manuel B. Ala, petitioners rediscounting clerk together Appeal on time was inexcusable negligence. These are the reasons:
with a letter of transmittal and a receipt for the bills addressed to the CBP. Mr. Ala
turned over the bills together with the accompanying documents to Rogelio Carrera One. In paragraph 7.28 of the Comment (to the Motion to Dismiss), plaintiff-
for delivery to the CBP. Alberto Limjoco, Sr., Elizabeth C. Garcia and Milagros B. appellant states that
Nayve4 came into possession of these bills which they in turn delivered to Rainelda "On 11 August 1995 at 3:00 pm., plaintiff-appellant received a
A. Andrews and Rhodora B. Landrito5. Petitioner alleged that Andrews and Landrito copy of the Order dated 1 August 1995 denying its motion for
failed to ascertain the lawful ownership of the bills, and caused their transmittal and reconsideration of the dismissed order."
delivery to the CBP, through a letter signed by Eduardo G. Escobar and Alfonso
Leong, Jr..6 The proceeds of the bills were credited to the account of Asiatrust which Since, the last day for plaintiff-appellant to file the Notice of Appeal was
approved the managers check applications and facilitated payment to the bearers August 12, 1995, why did it not file the Notice of Appeal right away
of the bills. Petitioner discovered that the proceeds of the bills were not credited to considering that its preparation and mailing could not take two hours? If
its demand deposit account with the CBP. Upon such discovery, petitioner informed counsel for plaintiff-appellant did not take advantage of the two remaining
the CBP which furnished petitioner with a copy of the acknowledgment from office hours on August 11, 1995, why did it not file the Notice of Appeal at
Asiatrust of receipt of the bills and that the proceeds were credited to the account of anytime, the following day, August 12? In failing to do that, the law firm
Asiatrust. Petitioner claimed that Rainelda A. Andrews, Samson Flores, Alfonso counsel was guilty of gross and inexcusable negligence.
Leong, Jr., Rhodora D. Landrito, Joseph Chua, Ramon Yu and Eduardo G. Escobar
TWO. If the counsel for plaintiff-appellant did not know that the last day to
were negligent in the performance of their duties and responsibilities as officers of
file the Notice of Appeal was on August 12, 1995, why did it not ask the

139
handling lawyer about it? There was no impediment to do that because the Court of Appeals.7 Petitioner enumerates the similarities between theLegarda case
handling lawyer was not comatose. The counsel was inexcusably negligent and its own, as follows:
for failing to make that inquiry.
"First, like the petitioner in the Legarda case, petitioner herein was not
THREE. The handling lawyer knew that if the Motion for Reconsideration negligent in choosing a counsel to represent them in the case. The former
would be denied as in fact it was he would have only a day after receipt engaged the services of former law school dean, Dean Antonio Coronel,
of the order of denial to file a notice of appeal. Why did he not forewarn his while the latter engaged the service of the well known and reputable law
law firm about such fact so that even in his absence, the latter could file firm, Quisumbing, Torres and Evangelista which is associated with Baker
said notice? Assuming that the handling lawyer was really sick, his ailment and McKenzie of the United States, as counsels to their respective cases.
which was allegedly just "fatigue and stress" was not at all serious much
less incapacitating. In fact he was not even hospitalized for he was just In fact, the diligence of petitioner can be shown by the fact that it even
advised to rest for at least two weeks. With all the communication facilities replaced its first counsel, Atty. Antonio Pery in favor of Quisumbing, Torres
in Metro-Manila, there was no reason for said counsel even if sick not to and Evangelista, hoping that by hiring the services of that law firm the case
have gotten in touch with his law firm to check on the result of his Motion would be handled better and would have a better chance of winning.
for Reconsideration. It was, therefore, inexcusable negligence for him to Unfortunately, such hope was dampened by the gross negligence and
have failed doing that which an ordinarily prudent lawyer would have done. blunders committed by the law firm.

The inexcusable negligence of plaintiff-appellants counsel is made more Second, just like in the case of Legarda, the previous counsel of the
glaring by the fact that the Notice of Appeal was late not only by 2 or 4 petitioner committed two blunders. In the case of the former, counsel failed
days but all of 13 days.1wphi1.nt to file an answer in the trial court and failed to timely appeal the case to
the appellate courts, while in the latter case, counsel caused the dismissal
We are not unaware of the rule that technicality should not smother the of the case by arriving late at the trial date and also by failing to timely
right of a litigant to a day in court. But the Supreme Court instructs us perfect an appeal to the Court of Appeals.
that strict adherence to reglementary periods fixed in the Rules of Court is
necessary to ensure the efficient and orderly disposition of cases (Panes v. Third, in both cases the Court of Appeals has found that both counsels
Court of Appeals, 120 SCRA 509). We cannot also close Our eyes to the rule committed negligence. The only difference would be that in the case
that perfecting an appeal within the period permitted by law is not of Legarda, the Court of Appeals only held that there was only pure and
only mandatory but jurisdictional and the failure to perfect the appeal on simple negligence on the part of Dean Antonio Coronel, while in the case at
time renders the judgment of the court final and executory. (Bank of bar, the Court of Appeals found that there was gross and inexcusable
America, Gerochi, Jr. 230 SCRA 9; Philippine Commercial International Bank negligence on the part of Quisumbing, Torres and Evangelista Law
v. Court of Appeals, 229 SCRA 560). Well rooted is the principle that once a Firm.
decision becomes final the appellate court is without jurisdiction to Thus, the Court of Appeals committed an error in stating that: "The plaintiff
entertain the appeal (Sumbillo v. IAC, 165 SCRA 232; Hensy Zoilo Llido v. appellant has to bite the bullet for it cannot shake itself of the inexcusable
Marquez, 166 SCRA 61)." negligence of its counsel" (Alabanza. vs. Intermediate Appellate
Hence, the instant petition. Court, 204 SCRA 304), because of its own findings that there was a gross
and inexcusable negligence on the part of the previous counsel. The
The Issue applicable decision of the Supreme Court to the case at bar should be the
case of Legarda vs. Court of Appeals. (195 SCRA 418)."
Petitioner now comes before us with the following assignment of error:
The Courts Ruling
THE RESPONDENT COURT OF APPEALS ERRED IN FINDING THAT THE ACTS
OF PETITIONERS PREVIOUS COUNSEL, QUISUMBING, TORRES AND The petition is bereft of merit. We uphold the dismissal of the appeal by the Court of
EVANGELISTA, SHOULD BIND THE PETITIONER, DESPITE THE FINDINGS IN Appeals.
ITS RESOLUTION THAT THE LAW FIRM COUNSEL WAS GROSSLY AND
INEXCUSABLY NEGLIGENT. The general rule is that a client is bound by the acts, even mistakes, of his counsel in
the realm of procedural technique. The exception to this rule is when the negligence
The threshold issue in this petition for review on certiorari is whether the Court of of counsel is so gross, reckless and inexcusable that the client is deprived of his day
Appeals erred in dismissing an appeal that was filed 13 days late despite its own in court. In which case, the remedy then is to reopen the case and allow the party
findings that petitioners counsel was grossly negligent. who was denied his day in court to adduce his evidence. 8 However, a thorough
review of the instant case reveals that petitioner cannot seek refuge or obtain
Petitioner argues that a client should not be bound by counsels gross and reprieve under these principles.
inexcusable negligence. Petitioner admits that its handling counsel, Atty. Ignacio of
QTE, committed two blunders: first, he failed to arrive on time during one of the Legarda case is not applicable
hearings allegedly due to the traffic at Roxas Boulevard in front of Baclaran Church;
second, he failed to file the notice of appeal within the reglementary period due to Petitioners reliance on the Legarda case which was promulgated on March 18, 1991
"fatigue and stress". Petitioner further admits that Atty. Ignacio offered a "flimsy is clearly misplaced. In said case, the Court declared that petitioners counsel, Atty.
excuse" for his tardiness and an "out of this world excuse" for his failure to file the Antonio Coronel, a well-known practicing lawyer and dean of a law school,
notice of appeal on time. Petitioner, however, submits that such gross negligence committed not just ordinary or simple negligence, but reckless and gross negligence
and mistake of counsel should not bind the client in line with the case of Legarda vs. which deprived his client of her property without due process of law. According to
the Legarda decision-

140
"xxx, the negligence of the then counsel for petitioner when he failed to file coming to the aid of someone, who through her weakness, ignorance or
the proper motion to dismiss or to draw a compromise agreement if it was misjudgment may have been bested in a legal joust which complied with all
true that they agreed on a settlement of the case; or in simply filing an the rules of legal proceedings."
answer; and that after having been furnished a copy of the decision by the
court he failed to appeal therefrom or to file a petition for relief from the In sum, the court did not relieve the client from the consequences of her counsels
order declaring petitioner in default." 9 negligence and mistakes considering that she was given an opportunity to defend
her interests in due course. Certainly, it cannot be said that she was denied due
was so gross and inexcusable that it should not bind his client. The Court declared process. Consequently, the Legarda case does not support petitioners cause.
that the counsels acts or omissions "consigned (the client) to penury" because "her
lawyer appeared to have abandoned her case not once but repeatedly." The Court No Denial of Due Process
noted that counsels "lack of devotion to duty is so gross and palpable that this Contrary to petitioners stance, the Legarda case supports the view that petitioner
Court must come to the aid of his distraught client." Thus, the Court held as null and was not denied its day in court. The Constitution mandates that "(n)o person shall be
void the decisions of the trial and appellate courts against Atty. Coronels client and deprived of life, liberty, or property without due process of law x x x ." 11 The right to
ordered, among other things, the reconveyance of the property in her favor. due process of law has been interpreted to mean as follows:
However, the decision in said case was not yet final in 1991. The private respondent "The essence of due process is to be found in the reasonable opportunity to
therein filed a timely motion for reconsideration. In granting the motion for be heard and submit any evidence one may have in support of ones
reconsideration, the Court en banc held: defense. `To be heard' does not mean only verbal arguments in court; one
"Under the Gancayco ruling, the order of reconveyance was premised on may be heard also through pleadings. Where opportunity to be heard,
the alleged gross negligence of Legardas counsel which should not be either through oral arguments or pleadings, is accorded, there is no denial
allowed to bind her as she was deprived of her property `without due of due process."12 (Emphasis supplied)
process of law. Verily, so long as a party is given the opportunity to advocate her cause or defend
It is, however, basic that as long as a party was given the opportunity to her interest in due course, it cannot be said that there was denial of due process. In
defend her interests in due course, she cannot be said to have been denied petitioners case as in the Legarda case, the chronology of events shows that the
due process of law, for this opportunity to be heard is the very essence of case took its regular course in the trial court.
due process. The chronology of events shows that the case took its regular On December 8, 1992, petitioner presented its first witness, Mr. Manuel B. Ala, the
course in the trial and appellate courts but Legardas counsel failed to act Accounting Clerk of its Accounting Department. He was cross-examined by CBP,
as any ordinary counsel should have acted, his negligence every step of the Nayve and Garcia on February 10, 1993. On March 1, 1993, petitioner presented its
way amounting to "abandonment, " in the words of the Gancayco decision. second witness, Ms. Josie Fernandez, the Security Custodian of its Treasury Bills. On
Yet, it cannot be denied that the proceedings which led to the filing of this July 5, 1993, petitioner presented its third witness, Atty. Leopoldo Cotaco, head of its
case were not attended by any irregularity. The judgment by default was Department of Security and Internal Affairs. At this hearing, only counsels of Nayve
valid, so was the ensuing sale at public auction. If Cabrera was adjudged and Garcia were present. The counsels of CBP and Asiatrust were absent. On August
highest bidder in said auction sale, it was not through any machination on 25, 1993, the direct testimony of Atty. Leopoldo Cotaco was terminated. On August
his part. All of his actuations that led to the final registration of the title in 30, 1993, QTE entered its appearance in substitution of Atty. Antonio M. Pery. On
his name were aboveboard, untainted by any irregularity." September 8, 1993, QTE moved for the postponement of the cross-examination of
xxx Atty. Cotaco since it had only recently entered its appearance. On September 16,
1993, petitioner moved to dismiss the complaint against CBP on the ground that the
Neither Cathay nor Cabrera10 should be made to suffer for the gross latter had lifted the conservatorship and allowed the return of the management and
negligence of Legardas counsel. If she may be said to be "innocent" assets to petitioners Board of Directors. The motion was granted in an Order dated
because she was ignorant of the acts of negligence of her counsel, with September 28, 1993. The hearings on November 17 and 24, 1993 were postponed
more reason are respondents truly "innocent." As between two parties who upon petitioners motion since former counsel, Atty. Antonio M. Pery, refused to turn
may lose due to the negligence or incompetence of the counsel of one, the over the records and files of the case due to a dispute over legal fees. The hearings
party who was responsible for making it happen should suffer the were reset to February 21 and March 9 and 16, 1994. On February 18, 1994,
consequences. This reflects the basic common law maxim, so succinctly petitioner again moved that the hearing scheduled on February 21, 1994 be reset for
stated by Justice J.B.L. Reyes, that ". . . (B)etween two innocent parties, the the same reason. The hearings on March 9 and 16, 1994 were likewise postponed
one who made it possible for the wrong to be done should be the one to due to former counsels adamant refusal to turn over the files. The hearings were
bear the resulting loss." In this case, it was not respondents, but Legarda, reset to May 18 and 25, 1994. By May 18, 1994, petitioners former counsel still
who misjudged and hired the services of the lawyer who practically refused to turn over the files of the case, prompting petitioner to request for another
abandoned her case and who continued to retain him even after his proven postponement. The hearings were reset to July 6, 20 and August 15, 1994. The
apathy and negligence. hearing scheduled on July 6, 1994 was postponed on the ground that there was no
proof of service of the notice of hearing to counsels for the defendants. The hearing
The Gancayco decision makes much of the fact that Legarda is now scheduled on July 20, 1994 was postponed by Asiatrust on the ground that its
"consigned to penury" and, therefore, this Court "must come to the aid of counsel was not available for said hearing. The hearings were reset to August 31 and
the distraught client." It must be remembered that this Court renders September 19, 1994. The hearing scheduled on August 15, 1994 was cancelled. The
decisions, not on the basis of emotions but on its sound judgment, applying hearing on August 31, 1994 was reset to September 19, 1995 because there was no
the relevant, appropriate law. Much as it may pity Legarda, or any losing proof of service of the notice of hearing on counsels. The hearing on September 19,
litigant for that matter, it cannot play the role of a "knight in shining armor"

141
1994 was also reset to November 21, 1994 for lack of proof that the contending Counsel for petitioner committed simple negligence
counsels received the notice of hearing for said date. On November 16, 1994,
Asiatrust filed an urgent motion to postpone the hearing on November 21, 1994, due We also find that the negligence of the law firm engaged by the petitioner to litigate
to unavailability of its counsel. Consequently, the hearings were reset to January 30 its cause was not gross butsimple negligence. Petitioner capitalizes on the following
and February 15, 1995. However, the hearing on said dates were cancelled since the "blunders" of the law firm to establish gross negligence: (1) arriving late during the
presiding judge was indisposed, and reset to May 17, 1995. As mentioned earlier, hearing on May 17, 1995 and (2) filing the notice of appeal thirteen (13) days late.
petitioners handling lawyer, Atty. Ignacio of QTE arrived late during the hearing on Tardiness is plain and simple negligence. On the other hand, counsels failure to file
May 17, 1995 allegedly due to the unexpectedly heavy traffic on Roxas Boulevard in the notice of appeal within the reglementary period did not deprive petitioner of due
front of Baclaran Church. On motion of Asiatrusts counsel, the trial court issued the process of law.1wphi1.nt
Order dismissing the case for lack of interest to prosecute. We also do not miss the fact that petitioners were represented by a law firm which
Upon said dismissal, petitioners counsel filed a timely motion for reconsideration. meant that any of its members could lawfully act as their counsel during the
The same was denied by the trial court. However, it must trial."20 As such, "[w]hen a client employs the services of a law firm, he does not
be emphasized that petitioner was not left without any relief. Upon the denial employ the services of the lawyer who is assigned to personally handle the case.
thereof, the situation could have been easily remedied by filling a notice of appeal Rather, he employs the entire law firm. In the event that the counsel appearing for
within the reglementary period13 considering that a dismissal for failure to prosecute the client resigns, the firm is bound to provide a replacement." 21 Petitioner cannot
is an adjudication on the merits.14 As correctly pointed out by Asiatrust, all that is now complain of counsels errors. It has been held that "[l]itigants, represented by
required is a singled-paged, pro forma notice of appeal, the accomplishment of counsel, should not expect that all they need to do is sit back, relax and await the
which does not require a high degree of legal skill. Despite this, counsel failed to file outcome of their case. x x x 22." Especially in this case, where petitioner has a legal
its notice of appeal on time and the proffered excuse that he was suffering from department to monitor its pending cases and to liaise with its retained counsel. To
"stress and fatigue" while highly unacceptable, does not amount to gross, palpable, agree with petitioners stance would enable every party to render inutile any
pervasive and reckless negligence so as to deprive counsels client its day in court. adverse order or decision through the simple expedient of alleging gross negligence
As the proceedings in the trial court all the way up to the appellate court would on the part of its counsel. The Court will not countenance such a farce which
show, petitioner was not deprived of due process. contradicts long-settled doctrines of trial and procedure.23

Indeed, by failing to file its appeal within the reglementary period, it could not be Hence, there is no justifiable reason to exempt petitioner from the general rule that
successfully argued that petitioner was deprived of its day in court. clients should suffer the consequences of the negligence, mistake or lack of
competence of the counsel whom they themselves hired and had the full authority
Time and again it has been held that the right to appeal is not a natural right or a to fire at any time and replace with another even without any justifiable reason. 24
part of due process, it is merely a statutory privilege, and may be exercised only in
the manner and in accordance with the provisions of the law. 15 The party who seeks In sum, this is not a case where the negligence of counsel is one that is so gross,
to avail of the same must comply with the requirements of the rules. 16 Failing to do palpable, pervasive and reckless which is the type of negligence that deprives a
so, the right to appeal is lost.17 party of his or her day in court. For this reason, the Court need no longer concern
itself with the merits of petitioners causes of action nor consider the propriety of the
The Court has had several occasions to hold that "rules of procedure, especially dismissal of the case by the trial court for lack of interest to prosecute. The Court is
those prescribing the time within which certain acts must be done, have oft been bound by the trial courts judgment which had become final and executory due to
held as absolutely indispensable to the prevention of needless delays and to the the simple negligence of the petitioners counsel in allowing the reglementary period
orderly and speedy discharge of business. The reason for rules of this nature is to lapse without perfecting the appeal.
because the dispatch of business by courts would be impossible, and intolerable
delays would result, without rules governing practice. Such rules are a necessary WHEREFORE, there being no reversible error committed by the Court of Appeals,
incident to the proper, efficient and orderly discharge of judicial functions. Thus, we the petition for review on certiorari is DENIED and the assailed Resolution dated
have held that the failure to perfect an appeal within the prescribed reglementary September 19, 1996 dismissing the appeal is AFFIRMED.
period is not a mere technicality, but jurisdictional.18 SO ORDERED.
Neither could petitioner plead leniency in the application of the rules considering Vitug, Panganiban, and Sandoval-Gutierrez, JJ., concur.
that the period to appeal is prescribed not only by the Rules of Court but also by Melo, J., Abroad, on official leave.
statute, particularly Sec. 39 of Batas Pambansa Blg. 129 which provides
Sec. 39. Appeals. The period for appeal from final orders, resolutions,
awards, judgments, or decisions of any court in all cases shall be fifteen
(15) days counted from the notice of the final order, resolution, award,
judgment, or decision appealed from: Provided, however, That habeas
corpus, the period for appeal shall be forty-eight (48) hours from the notice
of the judgment appealed from x x x x
Clearly, the perfection of an appeal in the manner and within the period prescribed
by law is not only mandatory but jurisdictional, and failure to perfect an appeal has
the effect of rendering the judgment final and executory. Public policy and sound
practice demand that judgments of courts should become final and irrevocable at
some definite date fixed by law."19

142
QUINTOS VS. COMMISSION ON ELECTIONS
G.R. No. 149800 November 21, 2002
RICARDO V. QUINTOS, petitioner,
vs.
COMMISSION ON ELECTIONS and JOSE T. VILLAROSA, respondents.
DECISION
CARPIO, J.:
The Case
Before us is a petition for certiorari with prayer for the issuance of a temporary
restraining order and a writ of preliminary injunction under Rule 65 of the Rules of
Court. Petitioner seeks to annul the Orders ("Assailed Orders" for brevity) dated
August 27, 2001 and September 12, 2001 of the Commission on Elections Second
Division ("COMELEC" for brevity) in Election Protest Case No. 2001-34 entitled
"Ricardo V. Quintos v. Jose T. Villarosa." The Assailed Orders deferred the delivery to
the COMELEC of the protested and counter-protested ballot boxes ("Contested Ballot
Boxes" for brevity) from the Municipality of Paluan, Occidental Mindoro. The same
ballot boxes were also the subject of two pending municipal election protests 1 in the
Regional Trial Court of Mamburao, Branch 44, Occidental Mindoro.
The COMELEC, under the Assailed Orders, gave the Regional Trial Court of Mamburao
priority in the custody and revision of the ballots in the Contested Ballot Boxes. The
Assailed Orders deviated from the order of preference prescribed in Section 2 of
COMELEC Resolution No. 2812 promulgated on October 17, 1995. Under this
COMELEC Resolution, when contested ballot boxes are the subject of simultaneous
protests before the COMELEC and the Regional Trial Court, the COMELEC enjoys
preference in the custody of the ballot boxes. Petitioner contends that in issuing the
Assailed Orders, the COMELEC acted without or in excess of jurisdiction, or with
grave abuse of discretion, and there is no appeal or any speedy remedy in the
ordinary course of law.
Thus, this petition for certiorari.
The Antecedents
Petitioner Ricardo V. Quintos ("petitioner" for brevity) and private respondent Jose T.
Villarosa ("private respondent" for brevity) were candidates for Governor of
Occidental Mindoro in the May 14, 2001 elections. On May 26, 2001, the Provincial
Board of Canvassers proclaimed private respondent winner with 57,136 votes.
Petitioner received 56,043 votes and lost to private respondent by 1,093
votes.2 Petitioner filed with the COMELEC an election protest, docketed as Election
Protest Case No. 2001-34,3 against private respondent claiming that massive fraud
and illegal electoral practices were committed in the contested precincts (Precinct
Nos. 13A and 14A, Barangay Lumangbayan, Municipality of Paluan) during the
registration, the voting and the counting of the votes. Petitioner claimed that
because of the fraud and illegal electoral practices, he was deprived of votes cast in
his favor while private respondent was illegally benefited by votes which should not
be credited to him. After revision of the ballots in the protested precincts, petitioner
prayed that he be declared the winner in the elections and thus be proclaimed as
the duly elected Governor of the Province of Occidental Mindoro.
On June 15, 2001, private respondent filed his Answer with Counter-Protest and
Counterclaim.4 He counter-protested thirteen (13) ballot boxes from 13 precincts in
the Municipality of Paluan, namely, Precinct Nos. 23A, 25A, 24A, 3A, 47A1/48A,
29A/30A, 35A, 27A/28A, 7A/8A, 26A1, 9A, 36A and 47A.

143
In a hearing on August 7, 2001, private respondents counsel moved that the subject ballot boxes and other election documents can be turned over to
Regional Trial Court of Mamburao be allowed to take first custody of the Contested the Commission in due time. No revision of ballots shall be conducted
Ballot Boxes before their transmittal to the COMELEC. The Contested Ballot Boxes without prior notice being given to both parties in the above-entitled case.
were also the subject of two election protests, pending before the Regional Trial
Court of Mamburao, involving the positions of mayor and councilors in the SO ORDERED."9
Municipality of Mamburao.5 The COMELEC issued an Order dated August 7, 2001 On September 4, 2001, petitioner filed a Motion for Reconsideration 10 of the Order of
denying private respondents motion because of COMELEC Resolution No. 2812 the COMELEC dated August 27, 2001.
giving the COMELEC preference in the custody of the Contested Ballot Boxes over
the Regional Trial Court.6 On September 12, 2001, the COMELEC issued the second Assailed Order, 11 the
dispositive portion of which reads:
From the COMELECs Order, private respondent filed a Manifestation and Motion for
Partial Reconsideration.7Private respondent stated that petitioner identified only one "Wherefore, in view of the foregoing, the Commission (Second Division) hereby
(1) ballot box (Barangay Lumangbayan, Precinct No. 13A/14A [clustered]) in the DENIES the instant Motion for Reconsideration. It is however, emphasized that one
Municipality of Paluan, Occidental Mindoro as subject of his protest. Private (1) week prior to the completion of the revision of the protested ballots, the Regional
respondent, however, identified thirteen (13) ballot boxes from the same Trial Court, Branch 44, of Mamburao, Occidental Mindoro shall be notified to transmit
municipality in his counter-protest. Thus, the total number of Contested Ballot Boxes, the ballot box of Precinct 13A/14A to this Commission whether or not the said court
both in the protest and counter-protest, is fourteen (14). Private respondent has completed its revision of the ballots therein.
explained that the Contested Ballot Boxes were also the subject of two municipal
SO ORDERED."
election protests both filed with the Regional Trial Court of Mamburao, and both
handled by counsel of private respondent. In an Order dated June 25, 2001, 8 the Hence, petitioner filed this petition for certiorari seeking the annulment of the
Regional Trial Court directed the Municipal Treasurer of Paluan to bring before the Assailed Orders dated August 27, 2001 and September 12, 2001 based on the
court the Contested Ballot Boxes. following grounds:
Private respondent averred that if COMELEC Resolution No. 2812 were strictly "1. The Manifestation and Motion for Partial Reconsideration is not verified
implemented, the resolution of the election protests in the Municipality of Paluan pursuant to Sec. 3, Rule 19, Comelec Rules of Procedure and is considered
would suffer undue delay. The COMELEC would take first custody of the 14 Contested an unsigned pleading which should be denied (Soller vs. Comelec, et al.,
Ballot Boxes. The resolution of the election protests in the Municipality of Paluan G.R. No. 139853, September 5, 2000; Lalic vs. Casupanan, EAC No. 73-98,
would have to wait for the COMELEC to complete its revision of the ballots in the January 12, 2001, G.R. No. 146868, February 27, 2001).
Contested Ballot Boxes. Private respondent argued that if the COMELEC reconsidered
its Order dated August 7, 2001, the local election protests would be resolved 2. The Order of August 27, 2001 was issued without giving the petitioner
expeditiously without causing any undue delay in the resolution of petitioners and other parties concerned the opportunity to comment/oppose the
election protest and private respondents counter-protest. motion for partial reconsideration filed by the respondent in violation of due
process.
Private respondent prayed that the COMELEC allow the one ballot box (Precinct Nos.
13A/14A, Barangay Lumangbayan) from the Municipality of Paluan, subject of 3. The Order of August 27, 2001 is impractical and would clog one ballot
petitioners protest, to be delivered first to the Regional Trial Court of Mamburao. The with 26 different exhibit marks as there are four (4) parties in Election
Regional Trial Court could transmit this one ballot box to the COMELEC after the Protest Case No. 19, RTC, Branch 44, Mamburao, Occidental Mindoro
revision of the ballots in the election protests pending with the trial court. involving the Office of Mayor of Paluan, Occidental Mindoro as well as
Alternatively, private respondent prayed that the COMELEC first allow the delivery of twenty-two (22) parties in Election Protest Case No. 20, RTC, Branch 44,
the 13 ballot boxes subject of the counter-protest to the Regional Trial Court of Mamburao, Occidental Mindoro, involving the Office of Municipal Councilors
Mamburao. of Paluan, Occidental Mindoro.

On August 27, 2001, the COMELEC rendered the Assailed Order granting private 4. There is no compelling and urgent reason to disturb the order of
respondents Manifestation and Motion for Partial Reconsideration subject to the preferences in the custody and revision of ballots and other election
following guidelines: documents contained in the ballot boxes sequentially enumerated in
Section 2 of Comelec Resolution No. 2812 promulgated October 17, 1995
"Accordingly, the retrieval of the ballot box from Precinct No. 13A/14A of Barangay and still enforced up to the present time."12
Lumangbayan, and those from the precincts mentioned in the aforequoted
manifestation, all from the Municipality of Paluan, Occidental Mindoro, is hereby In his Comment and Manifestation, 13 private respondent manifested that the
ordered deferred subject to the following guidelines: Regional Trial Court of Mamburao had completed the revision of the ballots in the
ballot box of Precincts 13A and 14A of Barangay Lumangbayan, Paluan. Private
1. That the said ballot boxes should be delivered to the Commission respondent submitted the supporting Certification14 dated November 19, 2001
(Second Division) one (1) week prior to the termination of the revision of issued by the Clerk of Court of the trial court. Private respondent stated that the
the other protested ballot boxes, the schedule of which shall be set after Assailed Orders of the COMELEC, which petitioner sought to restrain, were already
the delivery of the said ballot boxes; and implemented. The COMELEC could direct at any time the Regional Trial Court to
transmit to the COMELEC in Manila the ballot box of Precincts 13A and 14A of
2. The Regional Trial Court of Mamburao, Occidental Mindoro, Branch 44, is
Barangay Lumangbayan. Private respondent thus asserted that this development
requested to conduct the revision and appreciation proceedings in Election
rendered the instant petition for certiorari, with prayer for a temporary restraining
Cases Nos. 19 and 20 (Ocampo vs. Pangilinan and Terana vs. Velandria,
order and preliminary injunction, moot and academic.
respectively) in the most expeditious manner possible in order that the

144
In its Comment,15 the Office of the Solicitor General, as counsel for the COMELEC, maintains that if the ballots and other election documents in the Contested Ballot
stated that the issue is already moot and academic because the trial court had Boxes are initially placed in the custody of the COMELEC, the possibility of tampering
completed its review of the ballot boxes included in petitioners protest. These ballot of these ballots and documents is very remote, if not impossible.
boxes are ready for transmittal to the COMELEC.
COMELEC Resolution No. 2812 addresses "the matter of impounding, transfer and
In his Reply, petitioner stated that while he concedes that indeed the Regional Trial control of ballot boxes, election documents and paraphernalia which are subject of
Court had completed its review of the ballots subject of his protest, still the issue simultaneous protests before the Electoral Tribunals, the Commission and Regional
should not be considered moot and academic. Petitioner asserts that it is of Trial Courts." Section 2 of the Resolution provides as follows:
paramount interest that the COMELEC strictly observe the order of preference in the
custody and revision of ballots as prescribed by Section 2 of COMELEC Resolution "Sec. 2. The following order of preference in the custody and revision of
No. 2812. ballots and other documents contained in the ballot boxes shall be:

The Issues 2.1 Presidential Electoral Tribunal (PET);

This petition raises the following issues: 2.2 Senate Electoral Tribunal (SET);

1. Whether the COMELEC, in issuing the Assailed Orders giving the Regional 2.3 House of Representatives Electoral Tribunal (HRET);
Trial Court of Mamburao priority in the custody and revision of the ballots in 2.4 Commission on Elections (Commission);
the Contested Ballot Boxes, acted without or in excess of jurisdiction or with
grave abuse of discretion because (a) there was no compelling reason to 2.5 Regional Trial Court (RTC)."
disturb the order of preference prescribed in Section 2 of COMELEC
In its Assailed Order dated September 12, 2001, the COMELEC explained
Resolution No. 2812; and (b) the Order dated August 27, 2001 is
the reason why it gave the Regional Trial Court of Mamburao, priority in the
impractical.
custody of the Contested Ballot Boxes, thus:
2. Whether the Unverified Manifestation and Motion for Partial
"After due deliberation, the Commission (Second Division) finds that no
Reconsideration should have been denied.
substantial right of protestant was violated by the issuance of the said
3. Whether the failure to give petitioner the opportunity to comment or assailed Order.
oppose private respondents Manifestation and Motion for Partial
It should bear emphasis that in the resolution of electoral protest cases, the
Reconsideration is a denial of due process.
paramount consideration taken into account by this Commission is the
Since the Clerk of Court of the Regional Trial Court of Mamburao certified on orderly, efficient and effective ascertainment of the will of the people vis a
November 19, 2001 that "the revision/counting of ballot boxes of Precincts 13A and vis the rights of parties to a speedy determination of their respective
14A of Barangay Lumangbayan, Paluan, Occidental Mindoro has already been interests. In granting the motion of protestee to allow the Regional Trial
completed,"16 the issue is moot and academic with respect to the ballot box subject Court of Mamburao to first take custody of the ballot box of Precinct No.
of petitioners protest. There is no showing, however, that the revision of ballots in 13A/14A subject of herein protest together with the counter-protested
the 13 ballot boxes of the 13 precincts in the Municipality of Paluan, which are the ballot-boxes, all from the Municipality of Paluan, Occidental Mindoro, the
subject of the counter-protest, was also completed. Hence, the issue is not moot and Commission (Second Division) sought to give the parties therein immediate
academic with regard to the 13 counter-protested ballot boxes, the initial custody of relief rather than have them unreasonably wait until the instant protest
which was also given to the Regional Trial Court of Mamburao in the Assailed Orders case involving the whole Province of Occidental Mindoro be resolved to
of the COMELEC. their detriment. This was likewise intended to avoid the protested ballots
being transported back and forth from Paluan to Comelec, Manila then back
The Courts Ruling again to Paluan or Mamburao. One thing more, in the questioned Order, this
We rule that the COMELEC did not act without or in excess of jurisdiction, or commit Division did not altogether vacate the protestants right over the subject
grave abuse of discretion, in issuing the Assailed Orders giving the Regional Trial ballot box involved in his protest considering that the privilege given to the
Court of Mamburao first access to the Contested Ballot Boxes. lower court to take first custody of the said ballot box is limited only to that
period when the other protested ballot boxes are being revised by the
First Issue: (a) whether there is no compelling reason to disturb Commission. In this connection, a request was made to the RTC for it to
expedite its revision of the said ballots and to deliver the same to the
the order of preference in COMELEC Resolution No. 2812. Commission "one (1) week prior to the termination of the revision of the
Petitioner contends that there is no compelling and urgent reason to disturb the other protested ballot boxes" upon prior notice by this Division." 17
order of preference in the custody of the Contested Ballot Boxes, which order is The privilege given to the Regional Trial Court to take first custody of the
prescribed sequentially in Section 2 of COMELEC Resolution No. 2812. Petitioner Contested Ballot Boxes is limited only to the period when the other
alleges that private respondents only reason in asking for observance of the order protested ballot boxes are being revised by the COMELEC. The COMELEC
of preference is to delay the election protest. Petitioner points out that when the clarified that this limitation applies not only to the ballot box subject of
COMELEC promulgated Resolution No. 2812, the Electoral Tribunals and the petitioners protest, but also to the 13 ballot boxes subject of private
COMELEC knew that delays would happen as a matter of course in the observance of respondents counter-protest. Thus, in its Assailed Order dated August 27,
the order of preference. Petitioner asserts that the rationale in the order of 2001, the COMELEC deferred the retrieval of "the ballot box from Precinct
preference prescribed in COMELEC Resolution No. 2812 is the importance of the No. 13A/14A of Barangay Lumangbayan, and those from the precincts
office involved insofar as the revision of the ballots is concerned. Petitioner

145
mentioned in the aforequoted manifestation 18 (referring to the 13 counter- are twenty-six parties in the two municipal election cases pending before the
protested ballot boxes), all from the Municipality of Paluan, Occidental Regional Trial Court of Mamburao.
Mindoro."
We are not persuaded. The COMELEC has shown the practicality of its Assailed Order
The COMELEC further issued the guideline that "said ballot boxes should be dated August 27, 2001. First, in giving the Regional Trial Court first access to the
delivered to the Commission (Second Division) one (1) week prior to the Contested Ballot Boxes while the COMELEC is still revising other protested ballot
termination of the revision of the other protested ballot boxes, the schedule boxes, delay is prevented in the resolution of the protest cases before the trial court.
of which shall be set after the delivery of the said ballot boxes x x x." The Second, the Contested Ballot Boxes will be transported only once from the Regional
COMELEC even emphasized that "one (1) week prior to the completion of Trial Court of Mamburao in Occidental Mindoro to the COMELEC in Manila. Petitioner
the revision of the protested ballots, the Regional Trial Court, Branch 44, of fears that the 26 parties in the two municipal election cases before the Regional Trial
Mamburao, Occidental Mindoro shall be notified to transmit the ballot box Court will clutter the ballots with 26 different exhibit marks. If the parties wish to
of Precinct 13A/14A to this Commission whether or not the said court has have the ballots marked as their exhibits, that is their right regardless of who takes
completed its revision of the ballots therein."19 In the same manner, the first custody of the ballots. These markings, however, are not expected to prevent a
COMELEC directed the delivery of the 13 counter-protested ballot boxes proper review of the ballots in the petitioners protest case before the COMELEC.
whether or not the Regional Trial Court has completed its revision of the
ballots in these ballot boxes. Second Issue: whether the unverified Manifestation and Motion

Admittedly, the COMELEC enjoys preference over the Regional Trial Court of for Partial Reconsideration should have been denied outright.
Mamburao in the custody and revision of the ballots in the Contested Ballot Petitioner contends that since private respondent failed to verify the Manifestation
Boxes. However, the COMELEC may for good reason waive this preference and Motion for Partial Reconsideration as required by the COMELEC Rules of
and allow the Regional Trial Court first access to the Contested Ballot Boxes. Procedure,22 the same is an unsigned pleading, and should have been denied
The COMELEC allowed the Regional Trial Court to have first access to the outright.
Contested Ballot Boxes "to give the parties therein immediate relief rather
than have them unreasonably wait until the instant protest case (Election We agree with the Solicitor General that the alleged lack of verification of private
Protest Case No. 2001-34) involving the whole Province of Occidental respondents Manifestation and Motion for Partial Reconsideration is merely a
Mindoro be resolved to their detriment." Another reason was "to avoid the technicality that should not defeat the will of the electorate. The COMELEC may
protested ballots being transported back and forth from Paluan to Comelec, liberally construe23 or even suspend24 its rules of procedure in the interest of justice,
Manila then back again to Paluan or Mamburao." including obtaining a speedy disposition of all matters pending before the COMELEC.

The COMELEC insured that this arrangement did not prejudice petitioners Third Issue: whether the failure to require comment
right to a speedy resolution of his election protest. The COMELEC limited
or opposition to the Manifestation and Motion for
the privilege given to the Regional Trial Court only to the period when the
COMELEC was still revising other protested ballot boxes. The COMELEC Partial Reconsideration is a denial of due process.
requested the Regional Trial Court to expedite its revision of the ballots and
to deliver the Contested Ballot Boxes to the COMELEC one week prior to the Petitioner contends that the COMELEC issued the Assailed Order of August 27, 2001
COMELECs termination of the revision of other protested ballot boxes. Such without giving him the opportunity to comment or oppose the motion for partial
an arrangement is deemed covered under Section 3 of COMELEC Resolution reconsideration. This, bewails petitioner, violates his right to due process.
No. 2812 which provides:
The Solicitor General correctly stated that there was no denial of due process since
"Sec. 3. The Tribunals, the Commission and the Courts shall coordinate and petitioner subsequently filed a motion for reconsideration which the COMELEC
make arrangement with each other so as not to delay or interrupt the considered and acted on, albeit unfavorably, in the Order dated September 12,
revision of ballots being conducted. The synchronization of revision of 2001. The essence of due process is simply an opportunity to be heard, or as applied
ballots shall be such that the expeditious disposition of the respective to administrative proceedings, an opportunity to explain ones side or an opportunity
protest cases shall be the primary concern." to seek a reconsideration of the assailed action or ruling. 25 Petitioner cannot
successfully invoke deprivation of due process since the COMELEC gave petitioner
The order of preference in Section 2 of COMELEC Resolution No. 2812 may therefore the chance to be heard in his motion for reconsideration.26
yield to the primary concern for the expeditious disposition of protest cases. Instead
of leaving the Contested Ballot Boxes unutilized during the time the COMELEC would The COMELEC, in issuing the Assailed Orders allowing the Regional Trial Court to take
be revising other protested ballot boxes, the COMELEC allowed the Regional Trial prior custody of the Contested Ballot Boxes, did not act without or in excess of
Court to have access to the Contested Ballot Boxes. This would prevent delay in the jurisdiction, or with grave abuse of discretion. In Sahali v. Commission on
disposition of the two municipal election cases pending before the trial court. Elections,27 this Court held:
Election contests involve the public interest, 20 and should be decided expeditiously
"Certiorari as a special civil action can be availed of only if there is concurrence of
and economically without unnecessary delays.21
the essential requisites, to wit: (a) a tribunal, board or officer exercising judicial
First Issue: (b) whether the Assailed Order functions has acted without or in excess of jurisdiction or with grave abuse of
discretion amounting to lack or in excess of jurisdiction, and (b) there is no appeal,
of August 27, 2001 is impractical. nor any plain, speedy and adequate remedy in the ordinary course of law for the
purpose of annulling or modifying the proceeding. There must be a capricious,
Petitioner also contends that the Assailed Order of August 27, 2001 is impractical.
arbitrary and whimsical exercise of power for it to prosper.
The Assailed Order will clutter one ballot with 26 different exhibit marks since there

146
To question the jurisdiction of the lower court or the agency exercising judicial or
quasi-judicial functions, the remedy is a special civil action for certiorari under Rule
65 of the Rules of Court. The petitioner in such cases must clearly show that the
public respondent acted without jurisdiction or with grave abuse of discretion
amounting to lack or excess of jurisdiction. Grave abuse of discretion defies exact
definition, but generally refers to "capricious or whimsical exercise of judgment as is
equivalent to lack of jurisdiction. The abuse of discretion must be patent and gross
as to amount to an evasion of positive duty or a virtual refusal to perform a duty
enjoined by law, or to act at all in contemplation of law, as where the power is
exercised in an arbitrary and despotic manner by reason of passion and hostility.
It has been held, however, that no grave abuse of discretion may be attributed to a
court simply because of its alleged misappreciation of facts and evidence. A writ of
certiorari may not be used to correct a lower tribunals evaluation of the evidence
and factual findings. In other words, it is not a remedy for mere errors of judgment,
which are correctible by an appeal or a petition for review under Rule 45 of the Rules
of Court.
In fine, certiorari will issue only to correct errors of jurisdiction, not errors of
procedure or mistakes in the findings or conclusions of the lower court. As long as a
court acts within its jurisdiction, any alleged errors committed in the exercise of its
discretion will amount to nothing more than errors of judgment which are reviewable
by timely appeal and not by special civil action for certiorari."
In the instant case, the COMELEC has original jurisdiction over the election protest of
petitioner who sought to be proclaimed as the duly elected Governor of the Province
of Occidental Mindoro. Under Article IX-C, Section 2, paragraph 2 of the Constitution,
the COMELEC exercises "exclusive original jurisdiction over all contests relating to
the elections x x x of all elective, provincial, and city officials x x x." Since the
COMELEC has jurisdiction over petitioners election protest, it had the authority to
issue the Assailed Orders.
Moreover, the COMELEC did not gravely abuse its discretion in issuing the Assailed
Orders since these orders are not tainted with a capricious, arbitrary or whimsical
exercise of power. In giving the Regional Trial Court first access to the Contested
Ballot Boxes, the COMELEC sought to prevent delay in the resolution of the two
protest cases pending before the trial court. However, first access by the Regional
Trial Court is only limited to the period of time when the COMELEC is still revising
other protested ballot boxes. The primary concern for such arrangement is the
expeditious disposition of protest cases, which is underscored in Section 3 of
COMELEC Resolution No. 2812.
WHEREFORE, the petition for certiorari is DISMISSED. No pronouncement as to
costs.
SO ORDERED.
Davide, Jr., C.J., Bellosillo, Puno, Vitug, Mendoza, Panganiban, Quisumbing, Ynares-
Santiago, Sandoval-Gutierrez, Austria-Martinez, Carpio-Morales, Callejo, Sr., and
Azcuna, JJ., concur.
Corona, J., on official leave.

147
REPUBLIC VS. EXTELCOM On May 17, 1999, Bayantel filed an Ex-Parte Motion to Revive Case, 7 citing the
availability of new frequency bands for CMTS operators, as provided for under
G.R. No. 147096 January 15, 2002 Memorandum Circular No. 3-3-99.

REPUBLIC OF THE PHILIPPINES, represented by NATIONAL On February 1, 2000, the NTC granted BayanTel's motion to revive the latter's
TELECOMMUNICATIONS COMMISSION,petitioner, application and set the case for hearings on February 9, 10, 15, 17 and 22,
vs. 2000.8 The NTC noted that the application was ordered archived without prejudice to
EXPRESS TELECOMMUNICATION CO., INC. and BAYAN its reinstatement if and when the requisite frequency shall become available.
TELECOMMUNICATIONS CO., INC., respondents. Respondent Express Telecommunication Co., Inc. (Extelcom) filed in NTC Case No.
x---------------------------------------------------------x 92-486 an Opposition (With Motion to Dismiss) praying for the dismissal of
Bayantel's application.9 Extelcom argued that Bayantel's motion sought the revival
G.R. No. 147210 January 15, 2002 of an archived application filed almost eight (8) years ago. Thus, the documentary
evidence and the allegations of respondent Bayantel in this application are all
BAYAN TELECOMMUNICATIONS (Bayantel), INC., petitioner, outdated and should no longer be used as basis of the necessity for the proposed
vs. CMTS service. Moreover, Extelcom alleged that there was no public need for the
EXPRESS TELECOMMUNICATION CO., INC. (Extelcom), respondent. service applied for by Bayantel as the present five CMTS operators --- Extelcom,
YNARES-SANTIAGO, J.: Globe Telecom, Inc., Smart Communication, Inc., Pilipino Telephone Corporation, and
Isla Communication Corporation, Inc. --- more than adequately addressed the market
On December 29, 1992, International Communications Corporation (now Bayan demand, and all are in the process of enhancing and expanding their respective
Telecommunications, Inc. or Bayantel) filed an application with the National networks based on recent technological developments. 1wphi1.nt
Telecommunications Commission (NTC) for a Certificate of Public Convenience or
Necessity (CPCN) to install, operate and maintain a digital Cellular Mobile Telephone Extelcom likewise contended that there were no available radio frequencies that
System/Service (CMTS) with prayer for a Provisional Authority (PA). The application could accommodate a new CMTS operator as the frequency bands allocated in NTC
was docketed as NTC Case No. 92-486.1 Memorandum Circular No. 3-3-99 were intended for and had in fact been applied for
by the existing CMTS operators. The NTC, in its Memorandum Circular No. 4-1-93,
Shortly thereafter, or on January 22, 1993, the NTC issued Memorandum Circular No. declared it its policy to defer the acceptance of any application for CMTS. All the
4-1-93 directing all interested applicants for nationwide or regional CMTS to file their frequency bands allocated for CMTS use under the NTC's Memorandum Circular No.
respective applications before the Commission on or before February 15, 1993, and 5-11-88 and Memorandum Circular No. 2-12-92 had already been allocated to the
deferring the acceptance of any application filed after said date until further orders. 2 existing CMTS operators. Finally, Extelcom pointed out that Bayantel is its
substantial stockholder to the extent of about 46% of its outstanding capital stock,
On May 6, 1993, and prior to the issuance of any notice of hearing by the NTC with and Bayantel's application undermines the very operations of Extelcom.
respect to Bayantel's original application, Bayantel filed an urgent ex-parte motion
to admit an amended application.3 On May 17, 1993, the notice of hearing issued by On March 13, 2000, Bayantel filed a Consolidated Reply/Comment, 10 stating that the
the NTC with respect to this amended application was published in the Manila opposition was actually a motion seeking a reconsideration of the NTC Order reviving
Chronicle. Copies of the application as well as the notice of hearing were mailed to the instant application, and thus cannot dwell on the material allegations or the
all affected parties. Subsequently, hearings were conducted on the amended merits of the case. Furthermore, Extelcom cannot claim that frequencies were not
application. But before Bayantel could complete the presentation of its evidence, the available inasmuch as the allocation and assignment thereof rest solely on the
NTC issued an Order dated December 19, 1993 stating: discretion of the NTC.
In view of the recent grant of two (2) separate Provisional Authorities in In the meantime, the NTC issued on March 9, 2000 Memorandum Circular No. 9-3-
favor of ISLACOM and GMCR, Inc., which resulted in the closing out of all 2000, re-allocating the following radio frequency bands for assignment to existing
available frequencies for the service being applied for by herein applicant, CMTS operators and to public telecommunication entities which shall be authorized
and in order that this case may not remain pending for an indefinite period to install, operate and maintain CMTS networks, namely: 1745-1750MHz / 1840-
of time, AS PRAYED FOR, let this case be, as it is, hereby ordered ARCHIVED 1845MHz; 1750-1775MHz / 1845-1850MHz; 1765-1770MHz / 1860-1865MHz; and
without prejudice to its reinstatement if and when the requisite frequency 1770-1775MHz / 1865-1870MHz.11
becomes available.
On May 3, 2000, the NTC issued an Order granting in favor of Bayantel a provisional
SO ORDERED.4 authority to operate CMTS service.12 The Order stated in pertinent part:
On June 18, 1998, the NTC issued Memorandum Circular No. 5-6-98 re-allocating five On the issue of legal capacity on the part of Bayantel, this Commission has
(5) megahertz (MHz) of the radio frequency spectrum for the expansion of CMTS already taken notice of the change in name of International
networks. The re-allocated 5 MHz were taken from the following bands: 1730- Communications Corporation to Bayan Telecommunications, Inc. Thus, in
1732.5 / 1825-1827.5 MHz and 1732.5-1735 / 1827.5-1830 MHz.5 the Decision entered in NTC Case No. 93-284/94-200 dated 19 July 1999, it
was recognized that Bayan Telecommunications, Inc., was formerly named
Likewise, on March 23, 1999, Memorandum Circular No. 3-3-99 was issued by the International Communications Corp. Bayantel and ICC Telecoms, Inc. are
NTC re-allocating an additional five (5) MHz frequencies for CMTS service, namely: one and the same entity, and it necessarily follows that what legal capacity
1735-1737.5 / 1830-1832.5 MHz; 1737.5-1740 / 1832.5-1835 MHz; 1740-1742.5 / ICC Telecoms has or has acquired is also the legal capacity that Bayantel
1835-1837.5 MHz; and 1742.5-1745 / 1837.5-1840 MHz.6 possesses.

148
On the allegation that the Commission has committed an error in allowing 1. To ensure effective competition in the CMTS market considering the
the revival of the instant application, it appears that the Order dated 14 operational merger of some of the CMTS operators, new CMTS operators
December 1993 archiving the same was anchored on the non-availability of must be allowed to provide the service.
frequencies for CMTS. In the same Order, it was expressly stated that the
archival hereof, shall be without prejudice to its reinstatement "if and when 2. The re-allocated frequencies for CMTS of 3 blocks of 5 Mhz x 2 is
the requisite frequency becomes available." Inherent in the said Order is sufficient for the number of applicants should the applicants be qualified.
the prerogative of the Commission in reviving the same, subject to 3. There is a need to provide service to some or all of the remaining cities
prevailing conditions. The Order of 1 February 2001, cited the availability of and municipalities without telephone service.
frequencies for CMTS, and based thereon, the Commission, exercising its
prerogative, revived and reinstated the instant application. The fact that 4. The submitted documents are sufficient to determine compliance to the
the motion for revival hereof was made ex-parte by the applicant is of no technical requirements. The applicant can be directed to submit details
moment, so long as the oppositors are given the opportunity to be later such as channeling plans, exact locations of cell sites, etc. as the project
heard and present the merits of their respective oppositions in the implementation progresses, actual area coverage ascertained and traffic
proceedings. data are made available. Applicant appears to be technically qualified to
undertake the proposed project and offer the proposed service.
On the allegation that the instant application is already obsolete and
overtaken by developments, the issue is whether applicant has the legal, IN VIEW OF THE FOREGOING and considering that there is prima facie
financial and technical capacity to undertake the proposed project. The evidence to show that Applicant is legally, technically and financially
determination of such capacity lies solely within the discretion of the qualified and that the proposed service is technically feasible and
Commission, through its applicable rules and regulations. At any rate, the economically viable, in the interest of public service, and in order to
oppositors are not precluded from showing evidence disputing such facilitate the development of telecommunications services in all areas of
capacity in the proceedings at hand. On the alleged non-availability of the country, as well as to ensure healthy competition among authorized
frequencies for the proposed service in view of the pending applications for CMTS providers, let a PROVISIONAL AUTHORITY (P.A.) be issued to
the same, the Commission takes note that it has issued Memorandum Applicant BAYAN TELECOMMUNICATIONS, INC. authorizing it to
Circular 9-3-2000, allocating additional frequencies for CMTS. The eligibility construct, install, operate and maintain a Nationwide Cellular Mobile
of existing operators who applied for additional frequencies shall be treated Telephone Systems (CMTS), subject to the following terms and
and resolved in their respective applications, and are not in issue in the conditions without prejudice to a final decision after completion of the
case at hand. hearing which shall be called within thirty (30) days from grant of authority,
in accordance with Section 3, Rule 15, Part IV of the Commission's Rules of
Accordingly, the Motions for Reconsideration filed by SMARTCOM and Practice and Procedure. xxx.14
GLOBE TELECOMS/ISLACOM and the Motion to Dismiss filed by EXTELCOM
are hereby DENIED for lack of merit.13 Extelcom filed with the Court of Appeals a petition for certiorari and
prohibition,15 docketed as CA-G.R. SP No. 58893, seeking the annulment of the Order
The grant of the provisional authority was anchored on the following findings: reviving the application of Bayantel, the Order granting Bayantel a provisional
COMMENTS: authority to construct, install, operate and maintain a nationwide CMTS, and
Memorandum Circular No. 9-3-2000 allocating frequency bands to new public
1. Due to the operational mergers between Smart Communications, Inc. telecommunication entities which are authorized to install, operate and maintain
and Pilipino Telephone Corporation (Piltel) and between Globe Telecom, Inc. CMTS.
(Globe) and Isla Communications, Inc. (Islacom), free and effective
competition in the CMTS market is threatened. The fifth operator, Extelcom, On September 13, 2000, the Court of Appeals rendered the assailed Decision, 16 the
cannot provide good competition in as much as it provides service using dispositive portion of which reads:
the analog AMPS. The GSM system dominates the market. WHEREFORE, the writs of certiorari and prohibition prayed for
2. There are at present two applicants for the assignment of the are GRANTED. The Orders of public respondent dated February 1, 2000
frequencies in the 1.7 Ghz and 1.8 Ghz allocated to CMTS, namely Globe and May 3, 2000 in NTC Case No. 92-486 are hereby ANNULLED and SET
and Extelcom. Based on the number of subscribers Extelcom has, there ASIDE and the Amended Application of respondent Bayantel
appears to be no congestion in its network - a condition that is necessary is DISMISSED without prejudice to the filing of a new CMTS application.
for an applicant to be assigned additional frequencies. Globe has yet to The writ of preliminary injunction issued under our Resolution dated August
prove that there is congestion in its network considering its operational 15, 2000, restraining and enjoining the respondents from enforcing the
merger with Islacom. Orders dated February 1, 2000 and May 3, 2000 in the said NTC case is
hereby made permanent. The Motion for Reconsideration of respondent
3. Based on the reports submitted to the Commission, 48% of the total Bayantel dated August 28, 2000 is denied for lack of merit.
number of cities and municipalities are still without telephone service
despite the more than 3 million installed lines waiting to be subscribed. SO ORDERED.17

CONCLUSIONS: Bayantel filed a motion for reconsideration of the above decision. 18 The NTC,
represented by the Office of the Solicitor General (OSG), also filed its own motion for
reconsideration.19 On the other hand, Extelcom filed a Motion for Partial

149
Reconsideration, praying that NTC Memorandum Circular No. 9-3-2000 be also VIII. THE COURT OF APPEALS SERIOUSLY ERRED IN ITS FINDING THAT THE
declared null and void.20 NTC VIOLATED THE PROVISIONS OF THE CONSTITUTION PERTAINING TO
DUE PROCESS OF LAW.
On February 9, 2001, the Court of Appeals issued the assailed Resolution denying all
of the motions for reconsideration of the parties for lack of merit.21 IX. THE COURT OF APPEALS SERIOUSLY ERRED IN DECLARING THAT THE
MAY 3, 2000 ORDER GRANTING BAYANTEL A PROVISIONAL AUTHORITY
Hence, the NTC filed the instant petition for review on certiorari, docketed as G.R. SHOULD BE SET ASIDE AND REVERSED.
No. 147096, raising the following issues for resolution of this Court:
i. Contrary to the finding of the Court of Appeals, there was no violation of
A. Whether or not the Order dated February 1, 2000 of the petitioner which the NTC Rule that the legal, technical, financial and economic
revived the application of respondent Bayantel in NTC Case No. 92-486 documentations in support of the prayer for provisional authority should
violated respondent Extelcom's right to procedural due process of law; first be submitted.
B. Whether or not the Order dated May 3, 2000 of the petitioner granting ii. Contrary to the finding of the Court of Appeals, there was no violation of
respondent Bayantel a provisional authority to operate a CMTS is in Sec. 3, Rule 15 of the NTC Rules of Practice and Procedure that a motion
substantial compliance with NTC Rules of Practice and Procedure and must first be filed before a provisional authority could be issued.
Memorandum Circular No. 9-14-90 dated September 4, 1990. 22
iii. Contrary to the finding of the Court of Appeals that a plea for provisional
Subsequently, Bayantel also filed its petition for review, docketed as G.R. No. authority necessitates a notice and hearing, the very rule cited by the
147210, assigning the following errors: petitioner (Section 5, Rule 4 of the NTC Rules of Practice and Procedure)
I. THE COURT OF APPEALS SERIOUSLY ERRED IN ITS INTERPRETATION OF provides otherwise.
THE PRINCIPLE OF "EXHAUSTION OF ADMINISTRATIVE REMEDIES" WHEN IT iv. Contrary to the finding of the Court of Appeals, urgent public need is not
FAILED TO DISMISS HEREIN RESPONDENT'S PETITION FOR CERTIORARI the only basis for the grant of a provisional authority to an applicant;
DESPITE ITS FAILURE TO FILE A MOTION FOR RECONSIDERATION.
v. Contrary to the finding of the Court of Appeals, there was no violation of
II. THE COURT OF APPEALS SERIOUSLY ERRED IN ITS FINDING THAT THE the constitutional provision on the right of the public to information when
REVIVAL OF NTC CASE NO. 92-486 ANCHORED ON A EX-PARTE MOTION TO the Common Carrier Authorization Department (CCAD) prepared its
REVIVE CASE WAS TANTAMOUNT TO GRAVE ABUSE OF DISCRETION ON THE evaluation report.23
PART OF THE NTC.
Considering the identity of the matters involved, this Court resolved to consolidate
III. THE COURT OF APPEALS SERIOUSLY ERRED WHEN IT DENIED THE the two petitions.24
MANDATE OF THE NTC AS THE AGENCY OF GOVERNMENT WITH THE SOLE
DISCRETION REGARDING ALLOCATION OF FREQUENCY BAND TO At the outset, it is well to discuss the nature and functions of the NTC, and analyze
TELECOMMUNICATIONS ENTITIES. its powers and authority as well as the laws, rules and regulations that govern its
existence and operations.
IV. THE COURT OF APPEALS SERIOUSLY ERRED IN ITS INTERPRETATION OF
THE LEGAL PRINCIPLE THAT JURISDICTION ONCE ACQUIRED CANNOT BE The NTC was created pursuant to Executive Order No. 546, promulgated on July 23,
LOST WHEN IT DECLARED THAT THE ARCHIVED APPLICATION SHOULD BE 1979. It assumed the functions formerly assigned to the Board of Communications
DEEMED AS A NEW APPLICATION IN VIEW OF THE SUBSTANTIAL CHANGE IN and the Telecommunications Control Bureau, which were both abolished under the
THE CIRCUMSTANCES ALLEGED IN ITS AMENDMENT APPLICATION. said Executive Order. Previously, the NTC's functions were merely those of the
defunct Public Service Commission (PSC), created under Commonwealth Act No.
V. CONTRARY TO THE FINDING OF THE COURT OF APPEALS, THE ARCHIVING 146, as amended, otherwise known as the Public Service Act, considering that the
OF THE BAYANTEL APPLICATION WAS A VALID ACT ON THE PART OF THE NTC Board of Communications was the successor-in-interest of the PSC. Under Executive
EVEN IN THE ABSENCE OF A SPECIFIC RULE ON ARCHIVING OF CASES SINCE Order No. 125-A, issued in April 1987, the NTC became an attached agency of the
RULES OF PROCEDURE ARE, AS A MATTER OF COURSE, LIBERALLY Department of Transportation and Communications.
CONSTRUED IN PROCEEDINGS BEFORE ADMINISTRATIVE BODIES AND
SHOULD GIVE WAY TO THE GREATER HIERARCHY OF PUBLIC WELFARE AND In the regulatory telecommunications industry, the NTC has the sole authority to
PUBLIC INTEREST. issue Certificates of Public Convenience and Necessity (CPCN) for the installation,
operation, and maintenance of communications facilities and services, radio
VI. CONTRARY TO THE FINDING OF THE COURT OF APPEALS, THE ARCHIVING communications systems, telephone and telegraph systems. Such power includes
OF BAYANTEL'S APPLICATION WAS NOT VIOLATIVE OF THE SUMMARY the authority to determine the areas of operations of applicants for
NATURE OF THE PROCEEDINGS IN THE NTC UNDER SEC. 3, RULE 1 OF THE telecommunications services. Specifically, Section 16 of the Public Service Act
NTC REVISED RULES OF PROCEDURE. authorizes the then PSC, upon notice and hearing, to issue Certificates of Public
VII. THE COURT OF APPEALS SERIOUSLY ERRED IN ITS FINDING THAT THE Convenience for the operation of public services within the Philippines "whenever
ARCHIVING OF BAYANTEL'S APPLICATION WAS VIOLATIVE OF THE ALLEGED the Commission finds that the operation of the public service proposed and the
DECLARED POLICY OF THE GOVERNMENT ON THE TRANSPARENCY AND authorization to do business will promote the public interests in a proper and
FAIRNESS OF ADMINISTRATIVE PROCESS IN THE NTC AS LAID DOWN IN SEC suitable manner."25 The procedure governing the issuance of such authorizations is
4(1) OF R.A. NO. 7925. set forth in Section 29 of the said Act, the pertinent portion of which states:

150
All hearings and investigations before the Commission shall be governed by "Article 2. Laws shall take effect after fifteen days following the
rules adopted by the Commission, and in the conduct thereof, the completion of their publication in the Official Gazette (or in a
Commission shall not be bound by the technical rules of legal evidence. newspaper of general circulation in the Philippines), unless it is
xxx. otherwise provided. x x x"
In granting Bayantel the provisional authority to operate a CMTS, the NTC applied The fact that the amendments to Administrative Order No. SOCPEC 89-08-
Rule 15, Section 3 of its 1978 Rules of Practice and Procedure, which provides: 01 were filed with, and published by the UP Law Center in the National
Administrative Register, does not cure the defect related to the effectivity
Sec. 3. Provisional Relief. --- Upon the filing of an application, complaint or of the Administrative Order.
petition or at any stage thereafter, the Board may grant on motion of the
pleader or on its own initiative, the relief prayed for, based on the pleading, This Court, in Taada vs. Tuvera (G.R. No. L-63915, December 29, 1986,
together with the affidavits and supporting documents attached thereto, 146 SCRA 446) stated, thus:
without prejudice to a final decision after completion of the hearing which
shall be called within thirty (30) days from grant of authority asked for. "We hold therefore that all statutes, including those of local
(underscoring ours) application and private laws, shall be published as a condition for
their effectivity, which shall begin fifteen days after publication
Respondent Extelcom, however, contends that the NTC should have applied the unless a different effectivity is fixed by the legislature.
Revised Rules which were filed with the Office of the National Administrative Register
on February 3, 1993. These Revised Rules deleted the phrase "on its own initiative;" Covered by this rule are presidential decrees and executive orders
accordingly, a provisional authority may be issued only upon filing of the proper promulgated by the President in the exercise of legislative power
motion before the Commission. or, at present, directly conferred by the Constitution.
Administrative Rules and Regulations must also be published if
In answer to this argument, the NTC, through the Secretary of the Commission, their purpose is to enforce or implement existing law pursuant also
issued a certification to the effect that inasmuch as the 1993 Revised Rules have not to a valid delegation.
been published in a newspaper of general circulation, the NTC has been applying the
1978 Rules. Interpretative regulations and those merely internal in nature, that
is, regulating only the personnel of the administrative agency and
The absence of publication, coupled with the certification by the Commissioner of not the public, need not be published. Neither is publication
the NTC stating that the NTC was still governed by the 1978 Rules, clearly indicate required of the so-called letters of instructions issued by
that the 1993 Revised Rules have not taken effect at the time of the grant of the administrative superiors concerning the rules or guidelines to be
provisional authority to Bayantel. The fact that the 1993 Revised Rules were filed followed by their subordinates in the performance of their duties.
with the UP Law Center on February 3, 1993 is of no moment. There is nothing in the
Administrative Code of 1987 which implies that the filing of the rules with the UP xxx
Law Center is the operative act that gives the rules force and effect. Book VII, We agree that the publication must be in full or it is no publication
Chapter 2, Section 3 thereof merely states: at all since its purpose is to inform the public of the contents of the
Filing. --- (1) Every agency shall file with the University of the Philippines laws."
Law Center three (3) certified copes of every rule adopted by it. Rules in The Administrative Order under consideration is one of those issuances
force on the date of effectivity of this Code which are not filed within three which should be published for its effectivity, since its purpose is to enforce
(3) months from the date shall not thereafter be the basis of any sanction and implement an existing law pursuant to a valid delegation, i.e., P.D.
against any party or persons. 1071, in relation to LOI 444 and EO 133.27
(2) The records officer of the agency, or his equivalent functionary, shall Thus, publication in the Official Gazette or a newspaper of general circulation is a
carry out the requirements of this section under pain or disciplinary action. condition sine qua non before statutes, rules or regulations can take effect. This is
(3) A permanent register of all rules shall be kept by the issuing agency and explicit from Executive Order No. 200, which repealed Article 2 of the Civil Code, and
shall be open to public inspection. which states that:

The National Administrative Register is merely a bulletin of codified rules and it is Laws shall take effect after fifteen days following the completion of their
furnished only to the Office of the President, Congress, all appellate courts, the publication either in the Official Gazette or in a newspaper of general
National Library, other public offices or agencies as the Congress may select, and to circulation in the Philippines, unless it is otherwise provided.28
other persons at a price sufficient to cover publication and mailing or distribution The Rules of Practice and Procedure of the NTC, which implements Section 29 of the
costs.26 In a similar case, we held: Public Service Act (C.A. 146, as amended), fall squarely within the scope of these
This does not imply however, that the subject Administrative Order is a laws, as explicitly mentioned in the case Taada v. Tuvera.29
valid exercise of such quasi-legislative power. The original Administrative Our pronouncement in Taada vs. Tuvera is clear and categorical.
Order issued on August 30, 1989, under which the respondents filed their Administrative rules and regulations must be published if their purpose is to
applications for importations, was not published in the Official Gazette or in enforce or implement existing law pursuant to a valid delegation. The only
a newspaper of general circulation. The questioned Administrative Order, exceptions are interpretative regulations, those merely internal in nature, or
legally, until it is published, is invalid within the context of Article 2 of Civil those so-called letters of instructions issued by administrative superiors
Code, which reads:

151
concerning the rules and guidelines to be followed by their subordinates in The Court of Appeals ruled that there was a violation of the fundamental right of
the performance of their duties.30 Extelcom to due process when it was not afforded the opportunity to question the
motion for the revival of the application. However, it must be noted that said Order
Hence, the 1993 Revised Rules should be published in the Official Gazette or in a referred to a simple revival of the archived application of Bayantel in NTC Case No.
newspaper of general circulation before it can take effect. Even the 1993 Revised 92-426. At this stage, it cannot be said that Extelcom's right to procedural due
Rules itself mandates that said Rules shall take effect only after their publication in a process was prejudiced. It will still have the opportunity to be heard during the full-
newspaper of general circulation.31 In the absence of such publication, therefore, it is blown adversarial hearings that will follow. In fact, the records show that the NTC has
the 1978 Rules that governs. scheduled several hearing dates for this purpose, at which all interested parties shall
In any event, regardless of whether the 1978 Rules or the 1993 Revised Rules should be allowed to register their opposition. We have ruled that there is no denial of due
apply, the records show that the amended application filed by Bayantel in fact process where full-blown adversarial proceedings are conducted before an
included a motion for the issuance of a provisional authority. Hence, it cannot be administrative body.34 With Extelcom having fully participated in the proceedings,
said that the NTC granted the provisional authority motu proprio. The Court of and indeed, given the opportunity to file its opposition to the application, there was
Appeals, therefore, erred when it found that the NTC issued its Order of May 3, 2000 clearly no denial of its right to due process.
on its own initiative. This much is acknowledged in the Decision of the Court of In Zaldivar vs. Sandiganbayan (166 SCRA 316 [1988]), we held that the
Appeals: right to be heard does not only refer to the right to present verbal
As prayer, ICC asked for the immediate grant of provisional authority to arguments in court. A party may also be heard through his pleadings.
construct, install, maintain and operate the subject service and to charge where opportunity to be heard is accorded either through oral arguments or
the proposed rates and after due notice and hearing, approve the instant pleadings, there is no denial of procedural due process. As reiterated
application and grant the corresponding certificate of public convenience in National Semiconductor (HK) Distribution, Ltd. vs. NLRC (G.R. No.
and necessity.32 123520, June 26, 1998), the essence of due process is simply an
opportunity to be heard, or as applied to administrative proceedings, an
The Court of Appeals also erred when it declared that the NTC's Order archiving opportunity to explain one's side. Hence, in Navarro III vs. Damaso (246
Bayantel's application was null and void. The archiving of cases is a widely accepted SCRA 260 [1995]), we held that a formal or trial-type hearing is not at all
measure designed to shelve cases in which no immediate action is expected but times and not in all instances essential. Plainly, petitioner was not denied
where no grounds exist for their outright dismissal, albeit without prejudice. It saves due process.35
the petitioner or applicant from the added trouble and expense of re-filing a
dismissed case. Under this scheme, an inactive case is kept alive but held in Extelcom had already entered its appearance as a party and filed its opposition to
abeyance until the situation obtains wherein action thereon can be taken. the application. It was neither precluded nor barred from participating in the
hearings thereon. Indeed, nothing, not even the Order reviving the application, bars
In the case at bar, the said application was ordered archived because of lack of or prevents Extelcom and the other oppositors from participating in the hearings and
available frequencies at the time, and made subject to reinstatement upon adducing evidence in support of their respective oppositions. The motion to revive
availability of the requisite frequency. To be sure, there was nothing irregular in the could not have possibly caused prejudice to Extelcom since the motion only sought
revival of the application after the condition therefor was fulfilled. the revival of the application. It was merely a preliminary step towards the
resumption of the hearings on the application of Bayantel. The latter will still have to
While, as held by the Court of Appeals, there are no clear provisions in the Rules of prove its capability to undertake the proposed CMTS. Indeed, in its Order dated
the NTC which expressly allow the archiving of any application, this recourse may be
February 1, 2000, the NTC set several hearing dates precisely intended for the
justified under Rule 1, Section 2 of the 1978 Rules, which states: presentation of evidence on Bayantel's capability and qualification. Notice of these
Sec. 2. Scope.--- These rules govern pleadings, practice and procedure hearings were sent to all parties concerned, including Extelcom.
before the Board of Communications (now NTC) in all matters of hearing, As regards the changes in the personal circumstances of Bayantel, the same may be
investigation and proceedings within the jurisdiction of the Board.However,
ventilated at the hearings during Bayantel's presentation of evidence. In fact,
in the broader interest of justice and in order to best serve the public Extelcom was able to raise its arguments on this matter in the Opposition (With
interest, the Board may, in any particular matter, except it from these rules
Motion to Dismiss) anent the re-opening and re-instatement of the application of
and apply such suitable procedure to improve the service in the transaction Bayantel. Extelcom was thus heard on this particular point.
of the public business. (underscoring ours)
Likewise, the requirements of notice and publication of the application is no longer
The Court of Appeals ruled that the NTC committed grave abuse of discretion when it necessary inasmuch as the application is a mere revival of an application which has
revived Bayantel's application based on an ex-parte motion. In this regard, the
already been published earlier. At any rate, the records show that all of the five (5)
pertinent provisions of the NTC Rules: CMTS operators in the country were duly notified and were allowed to raise their
Sec. 5. Ex-parte Motions. --- Except for motions for provisional authorization respective oppositions to Bayantel's application through the NTC's Order dated
of proposed services and increase of rates, ex-parte motions shall be acted February 1, 2000.
upon by the Board only upon showing of urgent necessity therefor and the
It should be borne in mind that among the declared national policies under Republic
right of the opposing party is not substantially impaired. 33 Act No. 7925, otherwise known as the Public Telecommunications Policy Act of the
Thus, in cases which do not involve either an application for rate increase or an Philippines, is the healthy competition among telecommunications carriers, to wit:
application for a provisional authority, the NTC may entertain ex-parte motions only A healthy competitive environment shall be fostered, one in which
where there is an urgent necessity to do so and no rights of the opposing parties are
telecommunications carriers are free to make business decisions and to
impaired.1wphi1.nt

152
interact with one another in providing telecommunications services, with Clearly, Extelcom violated the rule on exhaustion of administrative remedies when it
the end in view of encouraging their financial viability while maintaining went directly to the Court of Appeals on a petition for certiorari and prohibition from
affordable rates.36 the Order of the NTC dated May 3, 2000, without first filing a motion for
reconsideration. It is well-settled that the filing of a motion for reconsideration is a
The NTC is clothed with sufficient discretion to act on matters solely within its prerequisite to the filing of a special civil action for certiorari.
competence. Clearly, the need for a healthy competitive environment in
telecommunications is sufficient impetus for the NTC to consider all those applicants The general rule is that, in order to give the lower court the opportunity to
who are willing to offer competition, develop the market and provide the correct itself, a motion for reconsideration is a prerequisite to certiorari. It
environment necessary for greater public service. This was the intention that came also basic that petitioner must exhaust all other available remedies before
to light with the issuance of Memorandum Circular 9-3-2000, allocating new resorting to certiorari. This rule, however, is subject to certain exceptions
frequency bands for use of CMTS. This memorandum circular enumerated the such as any of the following: (1) the issues raised are purely legal in nature,
conditions prevailing and the reasons which necessitated its issuance as follows: (2) public interest is involved, (3) extreme urgency is obvious or (4) special
circumstances warrant immediate or more direct action.40
- the international accounting rates are rapidly declining, threatening the
subsidy to the local exchange service as mandated in EO 109 and RA 7925; This case does not fall under any of the recognized exceptions to this rule. Although
the Order of the NTC dated May 3, 2000 granting provisional authority to Bayantel
- the public telecommunications entities which were obligated to install, was immediately executory, it did not preclude the filing of a motion for
operate and maintain local exchange network have performed their reconsideration. Under the NTC Rules, a party adversely affected by a decision,
obligations in varying degrees; order, ruling or resolution may within fifteen (15) days file a motion for
- after more than three (3) years from the performance of the obligations reconsideration. That the Order of the NTC became immediately executory does not
only 52% of the total number of cities and municipalities are provided with mean that the remedy of filing a motion for reconsideration is foreclosed to the
local telephone service. petitioner.41

- there are mergers and consolidations among the existing cellular mobile Furthermore, Extelcom does not enjoy the grant of any vested interest on the right
telephone service (CMTS) providers threatening the efficiency of to render a public service. The Constitution is quite emphatic that the operation of a
competition; public utility shall not be exclusive. Thus:

- there is a need to hasten the installation of local exchange lines in No franchise, certificate, or any other form of authorization for the
unserved areas; operation of a public utility shall be granted to citizens of the Philippines or
to corporations organized under the laws of the Philippines at least
- there are existing CMTS operators which are experiencing congestion in sixty per centum of whose capital is owned by such citizens, nor shall such
the network resulting to low grade of service; franchise, certificate or authorization be exclusive in character or for a
longer period than fifty years. Neither shall any such franchise or right be
- the consumers/customers shall be given the freedom to choose CMTS granted except under the condition that it shall be subject to amendment,
operators from which they could get the service. 37
alteraion, or repeal by the Congress when the common good so requires.
Clearly spelled out is the need to provide enhanced competition and the xxx xxx xxx.42
requirement for more landlines and telecommunications facilities in unserved areas
In Radio Communications of the Phils., Inc. v. National Telecommunications
in the country. On both scores, therefore, there was sufficient showing that the NTC Commission,43 we held:
acted well within its jurisdiction and in pursuance of its avowed duties when it
allowed the revival of Bayantel's application. It is well within the powers of the public respondent to authorize the
installation by the private respondent network of radio communications
We now come to the issue of exhaustion of administrative remedies. The rule is well-
systems in Catarman, Samar and San Jose, Mindoro. Under the
entrenched that a party must exhaust all administrative remedies before resorting to circumstances, the mere fact that the petitioner possesses a franchise to
the courts. The premature invocation of the intervention of the court is fatal to one's
put up and operate a radio communications system in certain areas is not
cause of action. This rule would not only give the administrative agency an an insuperable obstacle to the public respondent's issuing the proper
opportunity to decide the matter by itself correctly, but would also prevent the
certificate to an applicant desiring to extend the same services to those
unnecessary and premature resort to courts. 38 In the case of Lopez v. City of areas. The Constitution mandates that a franchise cannot be exclusive in
Manila,39 we held:
nature nor can a franchise be granted except that it must be subject to
As a general rule, where the law provides for the remedies against the amendment, alteration, or even repeal by the legislature when the common
action of an administrative board, body or officer, relief to courts can be good so requires. (Art. XII, sec. 11 of the 1986 Constitution). There is an
sought only after exhausting all remedies provided. The reason rests upon express provision in the petitioner's franchise which provides compliance
the presumption that the administrative body, if given the chance to correct with the above mandate (RA 2036, sec. 15).
its mistake or error, may amend its decision on a given matter and decide it Even in the provisional authority granted to Extelcom, it is expressly stated that
properly. Therefore, where a remedy is available within the administrative
such authority is not exclusive. Thus, the Court of Appeals erred when it gave due
machinery, this should be resorted to before resort can be made to the course to Extelcom's petition and ruled that it constitutes an exception to the rule on
courts, not only to give the administrative agency the opportunity to decide
exhaustion of administrative remedies.
the matter by itself correctly, but also to prevent unnecessary and
premature resort to courts.

153
Also, the Court of Appeals erred in annulling the Order of the NTC dated May 3, that Bayantel was the first company to comply with its obligation to install local
2000, granting Bayantel a provisional authority to install, operate and maintain exchange lines pursuant to E.O. 109 and R.A. 7925. In recognition of the same, the
CMTS. The general rule is that purely administrative and discretionary functions may provisional authority awarded in favor of Bayantel to operate Local Exchange
not be interfered with by the courts. Thus, in Lacuesta v. Herrera,44 it was held: Services in Quezon City, Malabon, Valenzuela and the entire Bicol region was made
permanent and a CPCN for the said service was granted in its favor. Prima facie
xxx (T)he powers granted to the Secretary of Agriculture and Commerce evidence was likewise found showing Bayantel's legal, financial and technical
(natural resources) by law regarding the disposition of public lands such as capacity to undertake the proposed cellular mobile telephone service.
granting of licenses, permits, leases and contracts, or approving, rejecting,
reinstating, or canceling applications, are all executive and administrative Likewise, the May 3, 2000 Order did not violate NTC Memorandum Circular No. 9-14-
in nature. It is a well recognized principle that purely administrative and 90 dated September 4, 1990, contrary to the ruling of the Court of Appeals. The
discretionary functions may not be interfered with by the courts. (Coloso vs. memorandum circular sets forth the procedure for the issuance of provisional
Board of Accountancy, G.R. No. L-5750, April 20, 1953) In general, courts authority thus:
have no supervising power over the proceedings and actions of the
administrative departments of the government. This is generally true with EFFECTIVE THIS DATE, and as part of the Commission's drive to streamline
respect to acts involving the exercise of judgement or discretion and and fast track action on applications/petitions for CPCN other forms of
findings of fact. (54 Am. Jur. 558-559) xxx. authorizations, the Commission shall be evaluating applications/petitions
for immediate issuance of provisional authorizations, pending hearing and
The established exception to the rule is where the issuing authority has gone beyond final authorization of an application on its merit.
its statutory authority, exercised unconstitutional powers or clearly acted arbitrarily
and without regard to his duty or with grave abuse of discretion. 45 None of these For this purpose, it is hereby directed that all applicants/petitioners seeking
obtains in the case at bar. for provisional authorizations, shall submit immediately to the Commission,
either together with their application or in a Motion all their legal, technical,
Moreover, in petitions for certiorari, evidentiary matters or matters of fact raised in financial, economic documentations in support of their prayer for
the court below are not proper grounds nor may such be ruled upon in the provisional authorizations for evaluation. On the basis of their completeness
proceedings. As held in National Federation of Labor v. NLRC:46 and their having complied with requirements, the Commission shall be
issuing provisional authorizations.
At the outset, it should be noted that a petition for certiorari under Rule 65
of the Rules of Court will prosper only if there is a showing of grave abuse Clearly, a provisional authority may be issued even pending hearing and final
of discretion or an act without or in excess of jurisdiction on the part of the determination of an application on its merits.
National Labor Relations Commission. It does not include an inquiry as to
the correctness of the evaluation of evidence which was the basis of the Finally, this Court finds that the Manifestations of Extelcom alleging forum shopping
labor official or officer in determining his conclusion. It is not for this Court on the part of the NTC and Bayantel are not impressed with merit. The divisions of
to re-examine conflicting evidence, re-evaluate the credibility of witnesses the Supreme Court are not to be considered as separate and distinct courts. The
nor substitute the findings of fact of an administrative tribunal which has Supreme Court remains a unit notwithstanding that it works in divisions. Although it
gained expertise in its special field. Considering that the findings of fact of may have three divisions, it is but a single court. Actions considered in any of these
the labor arbiter and the NLRC are supported by evidence on record, the divisions and decisions rendered therein are, in effect, by the same Tribunal. The
same must be accorded due respect and finality. divisions of this Court are not to be considered as separate and distinct courts but as
divisions of one and the same court.52
This Court has consistently held that the courts will not interfere in matters which
are addressed to the sound discretion of the government agency entrusted with the Moreover, the rules on forum shopping should not be literally interpreted. We have
regulation of activities coming under the special and technical training and stated thus:
knowledge of such agency.47 It has also been held that the exercise of administrative It is scarcely necessary to add that Circular No. 28-91 must be so
discretion is a policy decision and a matter that can best be discharged by the interpreted and applied as to achieve the purposes projected by the
government agency concerned, and not by the courts. 48 In Villanueva v. Court of Supreme Court when it promulgated that circular. Circular No. 28-91 was
Appeals,49 it was held that findings of fact which are supported by evidence and the designed to serve as an instrument to promote and facilitate the orderly
conclusion of experts should not be disturbed. This was reiterated in Metro Transit administration of justice and should not be interpreted with such absolute
Organization, Inc. v. National Labor Relations Commission, 50 wherein it was ruled literalness as to subvert its own ultimate and legitimate objection or the
that factual findings of quasi-judicial bodies which have acquired expertise because goal of all rules of procedure which is to achieve substantial justice as
their jurisdiction is confined to specific matters are generally accorded not only expeditiously as possible.53
respect but even finality and are binding even upon the Supreme Court if they are
supported by substantial evidence.1wphi1.nt Even assuming that separate actions have been filed by two different parties
involving essentially the same subject matter, no forum shopping was committed as
Administrative agencies are given a wide latitude in the evaluation of evidence and the parties did not resort to multiple judicial remedies. The Court, therefore, directed
in the exercise of its adjudicative functions. This latitude includes the authority to the consolidation of the two cases because they involve essentially the same issues.
take judicial notice of facts within its special competence. It would also prevent the absurd situation wherein two different divisions of the
In the case at bar, we find no reason to disturb the factual findings of the NTC which same court would render altogether different rulings in the cases at bar.
formed the basis for awarding the provisional authority to Bayantel. As found by the We rule, likewise, that the NTC has legal standing to file and initiate legal action in
NTC, Bayantel has been granted several provisional and permanent authorities cases where it is clear that its inaction would result in an impairment of its ability to
before to operate various telecommunications services. 51 Indeed, it was established

154
execute and perform its functions. Similarly, we have previously held in Civil Service
Commission v. Dacoycoy54 that the Civil Service Commission, as an aggrieved party,
may appeal the decision of the Court of Appeals to this Court.
As correctly stated by the NTC, the rule invoked by Extelcom is Rule 65 of the Rules
of Civil Procedure, which provides that public respondents shall not appear in or file
an answer or comment to the petition or any pleading therein. 55 The instant petition,
on the other hand, was filed under Rule 45 where no similar proscription exists.
WHEREFORE, in view of the foregoing, the consolidated petitions are GRANTED.
The Court of Appeals' Decision dated September 13, 2000 and Resolution dated
February 9, 2001 are REVERSED and SET ASIDE. The permanent injunction issued
by the Court of Appeals is LIFTED. The Orders of the NTC dated February 1, 2000
and May 3, 2000 are REINSTATED. No pronouncement as to costs.
SO ORDERED.
Davide, Jr., C.J., Puno, Kapunan, and Pardo, JJ., concur.

155
SPOUSES CASIMIRO ET AL. VS. CA Ruling on petitioners motion for reconsideration, the Regional Trial Court of Pasay
City, Branch CXI, set aside its earlier decision, and held that the report of the
G.R. No. 136911 February 11, 2003 engineers from the Bureau of Lands were more credible and accurate, and enjoy the
presumption of regularity and accuracy.
SPOUSES LEON CASIMIRO and PILAR PASCUAL, doing business under the
name and style "CASIMIRO VILLAGE SUBDIVISION", substituted by their On July 15, 1987, respondents moved for reconsideration of the above Order, but the
heirs: EMILIO, TEOFILO and GABRIEL, all surnamed CASIMIRO, petitioners, same was denied on January 19, 1988. Respondents appealed to the Court of
vs. Appeals on the sole question of the proper location of the common boundary
COURT OF APPEALS, former Thirteenth Division, NILDA A. PAULIN, separating the adjoining lots of petitioners and respondents. The Court of Appeals
MANOLITO A. PAULIN, SUSAN P. MARTIN, SYLVIA P. FARRES, CYNTHIA P. ordered that a relocation survey be conducted by a team of surveyors composed of
LAZATIN, CELESTINO P. PAULIN and UNIWIDE SALES REALTY AND a surveyor designated by the respondents, a surveyor designated by the petitioners,
RESOURCES CORPORATION, respondents. and a third member-surveyor chosen by the said two surveyors. Petitioners
designated Engr. Nicolas Bernardo, while respondents designated Engr. Manuel P.
RESOLUTION Lopez. Upon agreement of the parties that the third member shall be from the Land
Registration Commission, Engr. Felino Cortez, Chief, Ordinary and Cadastral Division,
YNARES-SANTIAGO, J.: Land Registration Commission, was designated third member and chairman of the
This resolves the Motion for Reconsideration filed by petitioners, seeking to set aside relocation survey.
our Decision dated July 3, 2002, which affirmed the assailed decision of the Court of Petitioners complained of irregularities in the conduct of the relocation survey,
Appeals in CA-G.R. CV No. 16165. namely, (a) the actual field work was conducted by a separate survey team
The facts as set forth in the Decision are as follows: composed of employees of the LRC without the knowledge and presence of Engr.
Bernardo; (b) the relocation plan and computations were done without consultation
Respondents were the registered owners of a 25,000 square meter parcel of land and coordination among the members of the survey team; and (c) the relocation
situated in Pamplona, Las Pias City, covered by Transfer Certificate of Title No. S- plan that was prepared by Engr. Cortez did not conform to the verification plan
74375. Adjoining their property on the northern side was petitioners land, covered earlier approved by the Bureau of Lands in January 1982.
by Original Certificate of Title No. 5975.
However, the Court of Appeals found nothing irregular in the conduct of the
Sometime in 1979, during a relocation survey conducted by Geodetic Engineer relocation survey. Petitioners representative, Engr. Bernardo, admitted that he was
Emilio Paz at the instance of respondents, it was discovered that the Casimiro Village furnished copies of the field notes and data gathered by the LRA team, but did not
Subdivision, owned by petitioners, encroached by 3,110 square meters into enter any objection thereto. If at all, Engr. Bernardos exclusion from the actual field
respondents land. Respondents notified petitioners and demanded that they desist work was rectified by the opportunity given him to comment on the final report
from making further development in the area. Subsequently, on March 13, 1980, prepared by Engr. Cortez, which Engr. Bernardo did not do.1awphi1.nt
respondents demanded that petitioners remove all constructions in the area.
After the survey, the Court of Appeals found that the final relocation survey report
Failing in their efforts to regain possession of the disputed premises, respondents yielded the "indisputable and inevitable conclusion" that petitioners encroached on a
filed with the Court of First Instance of Pasay City an action for recovery of portion of the respondents property comprising an area of 3,235 square meters. On
possession with damages against petitioners and the latters lot buyers, docketed as November 11, 1996, a judgment was rendered as follows:
Civil Case No. LP-8840-P. Respondents alleged that 3,110 square meters of their
property, which has a market value of P640,000.00, computed at the then prevailing The foregoing considered, We hereby REVERSE and SET ASIDE the order of the trial
price of P200.00 per square meter, have been encroached upon and fenced in by court dated June 25 1987 and REINSTATE the decision dated December 29, 1982 as
petitioners as part of the Casimiro Village Subdivision, and subdivided and sold to lot prayed for by the Appellants [spouses Paulin]. SO ORDERED.
buyers. In support of their contention, respondents presented the geodetic engineer Petitioners motion for reconsideration was denied for lack of merit. Hence, the
who conducted the actual ground relocation survey. instant petition for review.1
In their defense, petitioners denied that there was an encroachment in respondents In denying the petition for review, we upheld the factual findings of the Court of
land. They presented Geodetic Engineers Lino C. Reyes and Felipe Venezuela from Appeals, citing the rule that we are not a trier of facts, 2 and that factual findings of
the Bureau of Lands. Meanwhile, defendant-lot buyers interposed a cross-claim the Court of Appeals, when supported by substantial evidence, are conclusive and
against petitioners spouses Casimiro, averring that they were innocent purchasers in binding on the parties and are not reviewable by this Court.3
good faith and for value of their respective lots.
In their Motion for Reconsideration, petitioners argue that this case falls within the
On December 29, 1982, the Court of First Instance, Branch XXVIII, Pasay City, exceptions when review of the factual findings of the Court of Appeals is proper.
rendered a decision in favor of respondents, the dispositive portion of which reads: According to them, the findings of fact of the appellate court were contrary to those
WHEREFORE, judgment is hereby rendered in favor of the plaintiffs and against the of the trial court. Moreover, it was alleged that there was grave abuse of discretion
defendants Casimiros sentencing the latter to pay the former the sum of on the part of the Court of Appeals when it approved the Report of the Relocation
P640,000.00 with interest thereon at the legal rate from March 13, 1980 until the Survey Team without the signature of petitioners representative therein, Engr.
same is fully paid and to pay attorneys fees equivalent to 25% of the total amount Nicolas Bernardo. Likewise, the inference of the Court of Appeals as to the
due and the costs. On the cross-claim, cross defendants Casimiros are ordered to conclusiveness of the survey report was manifestly mistaken because the same
pay cross plaintiffs the sum of P5,000.00 as attorneys fees. SO ORDERED. were arrived at without the participation and conformity of Engr. Bernardo. Finally,

156
the assailed Decision was based on the assumption that Engr. Bernardo was Engr. Bernardo filed a Comment, setting forth the alleged irregularities in the
furnished copies of the field notes and data gathered by the team of surveyors. relocation survey. According to him, he never received notice of the time and exact
date of the field survey, as agreed upon by the team; that the designation of the LRA
In their Comment, respondents countered that this Court is not tasked with the duty engineers who undertook the field work was not authorized by the court; and that
to review findings of fact; that the findings of fact of the Court of Appeals and the the official survey team appointed by the Court of Appeals never met to perform the
Regional Trial Court are not contrary to each other; and that the Court of Appeals did survey.11
not commit grave abuse of discretion.4
On January 17, 1994, the Court of Appeals denied petitioners motion to require the
Considering the seriousness of the allegation of irregularity in the manner of the chairman of the relocation survey team to comply with the resolution of the court
resurvey, we resolved to take a second look at the evidence on record of this case, dated March 20, 1990.12 Subsequently, it rendered judgment in favor of respondents,
particularly those before the Court of Appeals pertaining to the composition of the finding that the Report submitted by the Relocation Survey Team was arrived at after
resurvey team and the conduct of the resurvey field work. a careful and deliberate process of survey, computation and assessment of its
As narrated above, the Court of Appeals, upon agreement of both parties, ordered technical findings. Hence, it sustained the finding that petitioners property
that a relocation survey on the questioned properties be conducted by a team of encroached on respondents property by 3,235 square meters.13
surveyors. The Court of Appeals Resolution reads: The reason for the requirement of representation of both parties in the resurvey
Finding the proposal well-taken as the only issue in this controversy is the team is to ensure that the interests of both sides are protected. If this requirement is
correctness of the relocation survey to determine the true location of the common breached, then serious prejudice can result. This is especially true in this case where
boundary between the lot of the plaintiffs and the lot of the defendants, the Court the purpose of the resurvey is to determine the boundaries of the parties adjacent
pursuant to Section 9 (3) of B.P. 129, hereby directs that a relocation survey of the lots. The placing of boundary lines and demarcation points on the soil must be
strip of land in question in this case, be conducted by a team of surveyors composed precise, and the smallest error in alignment may result in the loss of a large portion
of (1) a surveyor designated by the appellants, (2) a surveyor designated by the of ones property. Hence, it is crucial that each party must have a representative
appellees and (3) a surveyor to be chosen by the said two surveyors. The resurvey present to ensure that the fixing of the metes and bounds on the soil is accurately
shall be conducted in the presence of both parties or their authorized performed.
representatives. In view of the manifestation of defendants-appellees that they are Indeed, the requirement of notice and representation in the proceedings is an
willing to advance the cost of said relocation survey, reimbursable to them essential part of due process of law. In Roxas & Co., Inc. v. Court of Appeals, 14 we
contingently as part of the costs of this action, should they win, the costs of such held:
relocation survey shall be advanced by the defendants appellees. 5
Respondent DAR, on the other hand, avers that surveys on the land covered by the
Subsequently, the Court of Appeals designated the following as members of the four titles were conducted in 1989, and that petitioner, as landowner, was not
survey team: denied participation therein. The results of the survey and the land valuation
(1) Engr. Manuel P. Lopez (for respondents); summary report, however, do not indicate whether notices to attend the same were
actually sent to and received by petitioner or its duly authorized representative. To
(2) Engr. Nicolas R. Bernardo (for petitioners); reiterate, Executive Order No. 229 does not lay down the operating procedure, much
less the notice requirements, before the VOS is accepted by respondent DAR. Notice
(3) Engr. Felino M. Cortez of the Land Registration Commission.6
to the landowner, however, cannot be dispensed with. It is part of
On July 10, 1992, Engr. Cortez submitted a report stating, among others, that the administrative due process and is an essential requisite to enable the landowner
members of the resurvey team have agreed that the actual field work will be himself to exercise, at the very least, his right of retention guaranteed under the
undertaken by five technical personnel, three of whom shall come from the Land CARL. (emphasis ours)15
Registration Authority and the remaining two shall be Engrs. Lopez and Bernardo or
Poring over the records of the Court of Appeals regarding the resurvey of the subject
their respective representatives.7 properties, it appears that the actual field work was performed by engineers from
By February 9, 1993, the field work had already been completed, pursuant to the LRA, without the representatives of petitioners and respondents being present.
Report of Engr. Cortez to the Court of Appeals. 8 On May 10, 1993, petitioners filed a There was no clear showing that notices of the field work were sent to petitioners
"Motion to Require Engineer Cortez to Comply with the Terms and Conditions of this and respondents. Worse, the actual field work was undertaken by only four
Honorable Courts Resolution of March 21, 1990," 9 wherein they complained that the engineers, all of whom were designated from the LRA. This is in violation of the
actual survey field work was done without the knowledge and presence of their agreement of the parties that the actual field work should be done by five technical
representative, Engr. Bernardo, in contravention of the appellate courts directive personnel, three of whom shall come from the Land Registration Authority and the
that the resurvey shall be conducted in the presence of both parties or their remaining two shall be Engrs. Lopez and Bernardo or their respective
authorized representatives. representatives.16

Subsequently, Engr. Cortez submitted his Report which states that the actual field As stated above, the representatives of petitioners and respondents were not
work was undertaken by a survey team created by the Administrator of the LRA notified of and thus failed to participate in the survey. This is evident from the Report
composed of Engrs. Cortez, Ildefonso Padigos, Jr., Porfirio Encisa, Jr., and Alexander submitted by Engr. Cortez himself, stating that the actual field work was undertaken
Montemayor. The team found that petitioners property encroached on respondents by a survey team created by the Administrator of the LRA composed of Engrs.
property by 3,235 square meters. The Report was signed by Engrs. Cortez and Cortez, Ildefonso Padigos, Jr., Porfirio Encisa, Jr., and Alexander Montemayor. 17 It is
Lopez. Engr. Bernardo did not sign above his typewritten name. 10 clear that Engrs. Lopez and Bernardo were not present at the field survey. The failure
of Engr. Cortez, as chairman of the resurvey team, to notify Engr. Bernardo of the

157
actual field work to enable him to participate therein constituted as serious violation which is ordered to forthwith cause the resurvey the boundaries on the parties
of petitioners right to due process, especially considering that it resulted in a respective properties by the team of surveyors agreed upon by the parties, and
deprivation of their property to the extent of 3,235 square meters. The actual survey thereafter to decide the case accordingly.
proceedings must, therefore, be conducted anew, ensuring this time that the
interests of both parties are adequately protected. Hence, this case must be SO ORDERED. Davide, Jr., C.J., (Chairman), Vitug, and Austria-Martinez, JJ., concur.
remanded to the Court of Appeals for the retaking of the survey of the boundaries on
the parties respective properties.
WHEREFORE, based on the foregoing, the Decision dated July 3, 2002 in G.R. No.
136911 is SET ASIDE. The instant petition is REMANDED to the Court of Appeals,

158

Você também pode gostar